Download as pdf or txt
Download as pdf or txt
You are on page 1of 202

*KVS RO DEHRADUN STUDENT SUPPORT MATERIAL MATHEMATICS/ XII/2023-24(VN)*

कें द्रीय विद्यालय संगठन दे हरादून संभाग


KENDRIYA VIDYALAYA SANGATHAN
DEHRADUN REGION

STUDENTSUPPORTMATERIAL
SESSION :2023-24
(An outcome of Two Days Subject Enrichment Workshop
For PGT Mathematics , KVS Dehradun Region)

MATHEMATICS

Class: XII
Based on the latest CBSE Exam Pattern for the Session 2023-
24

KVS RO DDN S SM XII MATHEMATICS/ 2023-24(VN) Page 1


*KVS RO DEHRADUN STUDENT SUPPORT MATERIAL MATHEMATICS/ XII/2023-24(VN)*

INSPIRATION

Dr. (Mrs.) Sukriti Raiwani


Deputy Commissioner
K.V.S Regional Office Dehradun

Mrs Swaty Agarwal Mr. Lalit Mohan Bisht Mr. Surjeet singh
Asst Commissioner Asst Commissioner Asst Commissioner
KVS Regional Office Dehradun KVS Regional Office Dehradun KVS Regional Office Dehradun

COORDINATED BY
SHRI VIJAY NAITHANI
PRINCIPAL
KENDRIYA VIDYALAYA NO.2 SURVEY OF INDIA
HATHIBADKALA DEHRADUN

KVS RO DDN S SM XII MATHEMATICS/ 2023-24(VN) Page 2


*KVS RO DEHRADUN STUDENT SUPPORT MATERIAL MATHEMATICS/ XII/2023-24(VN)*

KENDRIYA VIDYALAYA SANGATHAN


DEHRADUN REGION
TEACHERS WHO HAVE CONTRIBUTED
S.NO
S.NO NAME& KV OF TEACHER NAME& KV OF TEACHER
. PGT (Maths) PGT (Maths)

1 Sh. Chandra Prakash Joshi KV ALMORA 24 Sh Anil Rawat , KV OFD DEHRADUN


2 Sh P C Pant, KV BANBASA CANTT. 25 Sh. K.C.Guniyal, KV OLF DEHRADUN
3 Sh N B Singh, KV BANBASANHPC 26 Sh. B.K Prabhakar, KV ONGC DEHRADUN
4 Sh Pervash Kumar, KV BHEL HARIDWAR 27 Sh Birampal Singh, KV ONGC DEHRADUN
5 Sh. Vimal Kishor Pandey, KV BHIMTAL 28 Sh Pritam Singh, KV PAURI GARHWAL
6 Sh. M.S. Rawat, KV BIRPUR 29 Sh Manoj Singh Khati, KV PITHORAGARH
7 Sh. Rajiv Singh, KV FRI DDN 30 Sh Sudhir Kumar, KV RAIWALA
8 Sh Prabhat Tyagi , KV FRI DDN 31 Mrs. Vivek Bharti, KV RAJGARI
9 Sh. Rakesh Pandey, KV HALDWANI CANTT (I Shift) 32 Mr. Kirti Ballabh Joshi, KV RANIKHET
10 Sh. R.K Arya, KV HALDWANI CANTT (II Shift) 33 Sh Kamlesh Kumar, KV RISHIKESH
11 Sh. Vinay Kumar, KV IIP, DEHRADUN 34 Mr Vikas Mehndiratta , KV ROORKEE NO. 1
12 Ms Garima Jain, ITBP DEHRADUN (I Shift) 35 Mrs Saroj Tomar ROORKEE NO. 1
13 Sh R P Sati, KV ITBP DEHRADUN (I Shift) 36 Mr. Gopal Singh ROORKEE NO.2
14 Sh Hari Shanker Saini, KV KASHIPUR 37 Sh D. Y. Singh Negi, KV SRINAGARH SSB
15 Sh Harish Joshi , KV KASHIPUR 38 Mr Vivek Kakar , KV Upper Camp
16 Sh Deepak, KV KAUSANI 39 Mr.ManishSemwal, KV Uttarkashi
17 Sh Gaurav Sachdeva, KV LANSDOWN
18 Sh. Ram Kishor Jadli, KV LOHAGHAT
19 Sh Abhay Dutt Kaushik, KV MERTHIITBP
20 Sh Balwant Singh Manral , KV MUKTESHWARIVRI
21 Ms Renu , KV MUSSOORIE
22 Sh. Neeraj Kumar Verma, KV NO 1 HBK DEHRADUN
23 Sh. Raj Kishor Gupta NO 2 HBK DEHRADUN

Resource Person:

1. MR. PARVEEN KUMAR, KVNEW TEHRI TOWN


2. MR. NAVEEN WADHERA, KV BHEL HARIDWAR
3. MR. ANUJ KUMAR THAKUR, KV IMA, DEHRADUN

PROGRAM COORDINATOR
SH. VIJAY NAITHANI
Venue Principal

KVS RO DDN S SM XII MATHEMATICS/ 2023-24(VN) Page 3


*KVS RO DEHRADUN STUDENT SUPPORT MATERIAL MATHEMATICS/ XII/2023-24(VN)*
CLASS-XII(2023-24)

One Paper Max Marks: 80


No. Units No. of Periods Marks
I. Relations and Functions 30 08
II. Algebra 50 10
III. Calculus 80 35
IV. Vectors and Three - Dimensional Geometry 30 14
V. Linear Programming 20 05
VI. Probability 30 08
Total 240 80
Internal Assessment 20

Unit-I: Relations and Functions


1. Relations and Functions 15 Periods
Types of relations: reflexive, symmetric, transitive and equivalence relations. One to one and onto functions.

2. Inverse Trigonometric Functions 15 Periods


Definition, range, domain, principal value branch. Graphs of inverse trigonometric functions.

Unit-II: Algebra
1. Matrices 25 Periods
Concept, notation, order, equality, types of matrices, zero and identity matrix, transpose of a matrix, symmetric
and skew symmetric matrices. Operations on matrices: Addition and multiplication and multiplication with a
scalar. Simple properties of addition, multiplication and scalar multiplication. Non- commutativity of
multiplication of matrices and existence of non-zero matrices whose product is the zero matrix (restrict to
square matrices of order 2). Invertible matrices and proof of the uniqueness of inverse, if it exists; (Here all
matrices will have real entries).

2.Determinants 25 Periods
Determinant of a square Matrix (up to 3 x 3 matrices), minors, co-factors and applications of determinants in
finding the area of a triangle. Adjoint and inverse of a square matrix. Consistency, inconsistency and number of
solutions of system of linear equations by examples, solving system of linear equations in two or three
variables (having unique solution) using inverse of a matrix.
Unit-III: Calculus
1. Continuity and Differentiability 20 Periods
Continuity and differentiability, chain rule, derivative of inverse trigonometric functions, 𝑙𝑖𝑘𝑒 sin−1 𝑥 , cos−1 𝑥
and tan−1 𝑥, derivative of implicit functions. Concept of exponential and logarithmic functions. Derivatives of
logarithmic and exponential functions. Logarithmic differentiation, derivative of functions expressed in
parametric forms. Second order derivatives.

2. Applications of Derivatives 10 Periods


Applications of derivatives: rate of change of quantities, increasing/decreasing functions, maxima and minima
(first derivative test motivated geometrically and second derivative test given as a provable tool). Simple
problems (that illustrate basic principles and understanding of the subject as well as reallife situations).

KVS RO DDN S SM XII MATHEMATICS/ 2023-24(VN) Page 4


*KVS RO DEHRADUN STUDENT SUPPORT MATERIAL MATHEMATICS/ XII/2023-24(VN)*

3. Integrals 20 Periods
Integration as inverse process of differentiation. Integration of a variety of functions by substitution, by partial
fractions and by parts, Evaluation of simple integrals of the following types and problems based on them.
dx dx dx dx dx
∫x ∫ ,∫ ,∫ ,∫
2 ± a2, √x2 ± a2 √a2 − x2 ax2 + bx + c √ax2 + 𝑏𝑥 + 𝑐
px + q px + q
∫ dx, ∫ dx, ∫ √ a2 ± x 2 dx, ∫ √ x 2 − a2 dx
ax 2+ bx + c 2 +
√ax bx + c

𝑑𝑥,
Fundamental Theorem of Calculus (without proof). Basic properties of definite integrals and evaluation of
definite integrals.
4. Applications of the Integrals 15 Periods
Applications in finding the area under simple curves, especially lines, circles/ parabolas/ellipses (in standard
form only)
5. Differential Equations 15 Periods
Definition, order and degree, general and particular solutions of a differential equation. Solution of differential
equations by method of separation of variables, solutions of homogeneous differential equations of first order
and first degree. Solutions of linear differential equation of the type:

𝑑𝑦
𝑑𝑥
+ py = q, where p and q are functions of x or constants.
𝑑𝑥
𝑑𝑦
+ px = q, where p and q are functions of y or constants.

Unit-IV: Vecors and Three-Dimensional Geometry


1. Vectors 15 Periods
Vectors and scalars, magnitude and direction of a vector. Direction cosines and direction ratios of a vector.
Types of vectors (equal, unit, zero, parallel and collinear vectors), position vector of a point, negative of a
vector, components of a vector, addition of vectors, multiplication of a vector by a scalar, position vector of a
point dividing a line segment in a given ratio. Definition, Geometrical Interpretation, properties and application
of scalar (dot) product of vectors, vector (cross) product of vectors.

2. Three - dimensional Geometry 15 Periods


Direction cosines and direction ratios of a line joining two points. Cartesian equation and vector equation of a
line, skew lines, shortest distance between two lines. Angle between two lines.

Unit-V: Linear Programming


1. Linear Programming 20 Periods
Introduction, related terminology such as constraints, objective function, optimization, graphical method of
solution for problems in two variables, feasible and infeasible regions (bounded or unbounded), feasible and
infeasible solutions, optimal feasible solutions (up to three non-trivial constraints).

Unit-VI: Probability
1. Probability 30 Periods
Conditional probability, multiplication theorem on probability, independent events, total probability, Bayes’
theorem, Random variable and its probability distribution, mean of random variable.

KVS RO DDN S SM XII MATHEMATICS/ 2023-24(VN) Page 5


*KVS RO DEHRADUN STUDENT SUPPORT MATERIAL MATHEMATICS/ XII/2023-24(VN)*

MATHEMATICS (Code No. - 041)


QUESTION PAPER DESIGN CLASS - XII(2023-24)
Time: 3 hours Max. Marks: 80
%
Total Weightage
S.No. Typology of Questions
Marks

Remembering: Exhibit memory of previously learned material by


recalling facts, terms, basic concepts, and answers.
55
1 Understanding: Demonstrate understanding of facts and ideas by
organizing, comparing, translating, interpreting, giving descriptions,
and stating main ideas 44

Applying: Solve problems to new situations by applying acquired


2 20 25
knowledge, facts, techniques and rules in a different way.

Analysing :
Examine and break information into parts by identifying motives or
causes. Make inferences and find evidence to support generalizations

Evaluating:
Present and defend opinions by making judgments about
3 information, validity of ideas, or quality of work based on a set of 16 20
criteria.

Creating:
Compile information together in a different way by combining
elements in a new pattern or proposing alternative solutions

Total 80 100

1. No chapter wise weightage. Care to be taken to cover all the chapters


2. Suitable internal variations may be made for generating various templates keeping the overall
weightage to different form of questions and typology of questions same.

Choice(s):

There will be no overall choice in the question paper.


However, 33% internal choices will be given in all the sections

INTERNAL ASSESSMENT 20 MARKS


Periodic Tests ( Best 2 out of 3 tests conducted) 10 Marks
Mathematics Activities 10 Marks
Note: For activities NCERT Lab Manual may be referred.

KVS RO DDN S SM XII MATHEMATICS/ 2023-24(VN) Page 6


*KVS RO DEHRADUN STUDENT SUPPORT MATERIAL MATHEMATICS/ XII/2023-24(VN)*

Conduct of Periodic Tests:

Periodic Test is a Pen and Paper assessment which is to be conducted by the respective subject teacher. The
format of periodic test must have questions items with a balance mix, such as, very short answer (VSA), short
answer (SA) and long answer (LA) to effectively assess the knowledge, understanding, application, skills,
analysis, evaluation and synthesis. Depending on the nature of subject, the subject teacher will have the
liberty of incorporating any other types of questions too. The modalities of the PT are as follows:
a) Mode: The periodic test is to be taken in the form of pen-paper test.
b) Schedule: In the entire Academic Year, three Periodic Tests in each subject may be conducted
as follows:
Test Pre Mid-term (PT-I) Mid-Term (PT-II) Post Mid-Term (PT-III)
Tentative Month July-August November December-January
This is only a suggestive schedule and schools may conduct periodic tests as per their
convenience. The winter bound schools would develop their own schedule with similar
time gaps between two consecutive tests.

c) Average of Marks: Once schools complete the conduct of all the three periodic tests, they will
convert the weightage of each of the three tests into ten marks each for identifying best two tests.
The best two will be taken into consideration and the average of the two shall be taken as the
final marks for PT.
d) The school will ensure simple documentation to keep a record of performance as suggested in
detail circular no.Acad-05/2017.
e) Sharing of Feedback/Performance: The students’ achievement in each test must be shared
with the students and their parents to give them an overview of the level of learning that has
taken place during different periods. Feedback will help parents formulate interventions
(conducive ambience, support materials, motivation and morale-boosting) to further enhance
learning. A teacher, while sharing the feedback with student or parent, should be empathetic,
non- judgmental and motivating. It is recommended that the teacher share best
examples/performances of IA with the class to motivate all learners.
Assessment of Activity Work:
Throughout the year any 10 activities shall be performed by the student from the activities given
in the NCERT Laboratory Manual for the respective class (XI or XII) which is available on the
link:http://www.ncert.nic.in/exemplar/labmanuals.htmla record of the same may be kept by the
student. An year end test on the activity may be conducted
The weightage are as under:
• The activities performed by the student throughout the year and record
keeping : 5 marks
• Assessment of the activity performed during the year end test: 3 marks
Viva-voce: 2 marks

Prescribed Books:

1) Mathematics Textbook for Class XI, NCERT Publications


2) Mathematics Part I - Textbook for Class XII, NCERT Publication
3) Mathematics Part II - Textbook for Class XII, NCERT Publication
4) Mathematics Exemplar Problem for Class XI, Published by NCERT
5) Mathematics Exemplar Problem for Class XII, Published by NCERT
6) Mathematics Lab Manual class XI, published by NCERT
7) Mathematics Lab Manual class XII, published by NCERT

KVS RO DDN S SM XII MATHEMATICS/ 2023-24(VN) Page 7


*KVS RO DEHRADUN STUDENT SUPPORT MATERIAL MATHEMATICS/ XII/2023-24(VN)*

CHAPTER 1: RELATIONS AND FUNCTION


MINIMUM LEARNING MATERIALS
RELATIONS AND FUNCTIONS( 2 Marks Each)

1. Let R be the relation in the set Z of integers given by 𝑅 = {(𝑎, 𝑏): 2 𝑑𝑖𝑣𝑖𝑑𝑒𝑠𝑎– 𝑏}.Show that the relation R
transitive? Write the equivalence class [0].
2. Check if the elation R in the set R of real numbers defined as 𝑅 = {(𝑎, 𝑏): 𝑎 < 𝑏} is
(i) Not symmetric (ii) transitive.
3. Show by examples that the relation 𝑅 in, defined by 𝑅 = {(𝑎, 𝑏): 𝑎 ≤ 𝑏 3 } is neither reflexive nor transitive.
4. Prove that the relation R in the set of all real numbers, defined as 𝑅 = {(𝑎, 𝑏): 𝑎 ≤ 𝑏 2 } is neither reflexive nor
transitive.
5. How many equivalence relations on the set {1, 2, 3} containing (1, 2) and (2, 1) are there in all? Justify your
answer.
6. Check whether the relation R defined on the set 𝐴 = {1, 2, 3, 4, 5, 6} as
𝑅 = {(𝑎, 𝑏): 𝑏 = 𝑎 + 1}is reflexive, symmetric or transitive.
7. Show that the relation R on R defined as 𝑅 = {(𝑎, 𝑏): 𝑎 ≤ 𝑏}, is reflexive, and transitive but not
symmetric.
8. Let 𝐴 = {1,2,3,4}. Let R be the equivalence relation on 𝐴 × 𝐴 defined by
(𝑎, 𝑏)𝑅(𝑐, 𝑑)𝑖𝑓𝑓𝑎 + 𝑑 = 𝑏 + 𝑐. Find the equivalence class [(1,3)].
9. State the reason why the relation 𝑅 = {(𝑎, 𝑏): 𝑎 ≤ 𝑏 2 } on the set R of real numbers is not reflexive.

10. If 𝑅 = {(𝑥, 𝑦): 𝑥 + 2𝑦 = 8} is a relation on N, write the range of R.


11. Let 𝑅 = {(𝑎, 𝑎3 ): 𝑎𝑖𝑠𝑝𝑟𝑖𝑚𝑒𝑛𝑢𝑚𝑏𝑒𝑟𝑙𝑒𝑠𝑠𝑡ℎ𝑎𝑛 5} be a relation. Find the range of R.
12. Let R be the equivalence relation in the set 𝐴 = {0,1,2,3,4,5} given by
𝑅 = {(𝑎, 𝑏): 2 𝑑𝑖𝑣𝑖𝑑𝑒𝑠(𝑎 − 𝑏)}. Write the equivalence class [0].
13. State the reason for the relation R in the set {1, 2, 3} given by 𝑅 = {(1, 2), (2, 1)} not to be transitive.

14. For the set A = {1, 2, 3}, define a relation R in the set A as follows: R = {(1, 1), (2, 2), (3, 3), (1, 3)}. Write
the ordered pairs to be added to R to make it the smallest equivalence relation.
15. Let the function f : R → R be defined by f (x) = 4x – 1, ∀ x ∈ R. Then, show that f is one-one.

RELATIONS AND FUNCTIONS( 3 Marks Each)


1 Show that the relation S in the set A  x  Z : 0  x  12given by
S  (a, b) : a, b  Z , a  b is divisible by 4
is an equivalence relation. Find the set of all elements related to .1

2 Show that the relation R in the set of real numbers, defined as R  (a, b) : a  b
2
 is neither reflexive nor
symmetric, nor transitive.
3 Show that the function f : R � R defined by f x  2x  7, for x  R is bijective.
3

4-Let N be the set of all natural numbers and R be the relation in N  N defined by (a, b) R (c, d)  a.d = b.c.
Show that R is an equivalence relation
5.Show that in the set of all triangles in a plane the relation defined by “is similar to’ is an equivalence relation,
6-Show that the function f : R � R defined by f(x) = 3-4x is one one and onto.

7. Check whether the function f : N � N given by f(x) = 𝑥 3 is one one and onto?

KVS RO DDN S SM XII MATHEMATICS/ 2023-24(VN) Page 8


*KVS RO DEHRADUN STUDENT SUPPORT MATERIAL MATHEMATICS/ XII/2023-24(VN)*

RELATIONS AND FUNCTIONS – Solutions for MLL( 2 Marks Each)


1. Let 𝑎, 𝑏, 𝑐. Let (𝑎, 𝑏) ∈ 𝑅𝑎𝑛𝑑(𝑏, 𝑐) ∈ 𝑅.
That is, 2 divides (𝑎– 𝑏) and 2 divides (𝑏– 𝑐).
So, 2 must divide (𝑎– 𝑏) (𝑏– 𝑐) = 𝑎 − 𝑐.
That implies, 2 divides (𝑎– 𝑐).
That is, (𝑎, 𝑐) ∈ 𝑅.
Therefore, R is transitive.
For [0], let (𝑥, 0) ∈ 𝑅∀𝑥 ∈ 𝑍.
That is, 2 𝑑𝑖𝑣𝑖𝑑𝑒𝑠𝑥 − 0 ⇒ 𝑥 = 0, ±2, ±4, ±6, . ...
Hence, [0] = {0, ±2, ±4, ±6, . . . }.
2. We have 𝑅 = {(𝑎, 𝑏): 𝑎 < 𝑏} where 𝑎, 𝑏 ∈ 𝑅
(i) Observe that 1 < 2 is true but 2 < 1 is not true.
i.e., (1, 2) ∈ 𝑅but (2,1) ∉ 𝑅. So, R is not symmetric
(ii) Observe that if 𝑎 < 𝑏 and 𝑏 < 𝑐 are both true then, 𝑎 < 𝑐 is also true, for all real numbers 𝑎, 𝑏, 𝑐
i.e., (𝑎, 𝑏) ∈ 𝑅 and (𝑏, 𝑐) ∈ 𝑅implies (𝑎, 𝑐) ∈ 𝑅. So, R is transitive.
1 1
3. R is not reflexive as (2 , 2) ∉ 𝑅
R is not transitive as (9,4) ∈ 𝑅, (4,2) ∈ 𝑅 but (9,2) ∉ 𝑅
4. 0.1 ∈ 𝑅 with 0.1 ≰ (0.1)2 . Therefore, (0.1,0.1) ∉ 𝑅. Therefore, R is not reflexive.
10, 4, 3 ∈ 𝑅with 10 ≤ 42 , 4 ≤ 32 but 10 ≰ 32
Therefore, (10,4) ∈ 𝑅, (4,3) ∈ 𝑅 but (10,3) ∉ 𝑅. Therefore, R is not transitive.
5. Following equivalence relations can be possible in the given conditions these are,
{(1,1), (2,2), (3,3), (1,2), (2,1)}and {(1,1), (2,2), (3,3), (1,2), (1,3), (2,1), (2,3), (3,1), (3,2)}.
Clearly, only two equivalence relations are there.
6. Here 𝑅 = {(1,2), (2,3), (3,4), (4,5), (5,6)}.
As (1,1) ∉ 𝑅𝑓𝑜𝑟1 ∈ 𝐴, so R isn’t reflexive.
Also (1,2) ∈ 𝑅𝑏𝑢𝑡(2,1) ∉ 𝑅𝑓𝑜𝑟 1,2 ∈ 𝐴, so R isn’t symmetric.
Similarly, (1,2) ∈ 𝑅, (2,3) ∈ 𝑅𝑏𝑢𝑡(1,3) ∉ 𝑅𝑓𝑜𝑟 1,2,3 ∈ 𝐴, so R isn’t transitive.
7. Here (𝑎, 𝑎) ∈ 𝑅∀𝑎 ∈ 𝑅as 𝑎 ≤ 𝑎𝑖𝑠𝑡𝑟𝑢𝑒. So, R is reflexive.
Let 𝑎, 𝑏, 𝑐 ∈ 𝑅. Let (𝑎, 𝑏) ∈ 𝑅𝑎𝑛𝑑(𝑏, 𝑐) ∈ 𝑅.
That is, 𝑎 ≤ 𝑏𝑎𝑛𝑑𝑏 ≤ 𝑐, which clearly, implies 𝑎 ≤ 𝑐. Hence, (𝑎, 𝑐) ∈ 𝑅. So, R is transitive.
Now let 𝑎 = 1, 𝑏 = 2.
We can notice that (1,2) ∈ 𝑅𝑎𝑠1 ≤ 2 𝑖𝑠𝑡𝑟𝑢𝑒 but, (2,1) ∉ 𝑅𝑎𝑠 2 ≤ 1 𝑖𝑠𝑓𝑎𝑙𝑠𝑒.
So, R isn’t symmetric, as (𝑎, 𝑏) ∈ 𝑅𝑑𝑜𝑒𝑠𝑛𝑜𝑡𝑖𝑚𝑝𝑙𝑦 (𝑏, 𝑎) ∈ 𝑅.
8. Let (1, 3) 𝑅 (𝑥, 𝑦) for all (𝑥, 𝑦) ∈ 𝐴 × 𝐴.
That implies, 1 + 𝑦 = 3 + 𝑥𝑖. 𝑒. , 𝑦 − 𝑥 = 2.
So (x, y) may be (1, 3), (2, 4).
Hence [(1,3)] = {(1,3), (2,4)}.
1 1 2 1 1
9. As 2 > (2) ⇒ (2 , 2) ∉ 𝑅. Hence R is not reflexive.
1
10. Since 𝑥 + 2𝑦 = 8 that implies, 𝑦 = 2
(8 – 𝑥).
 𝑅 = {(2, 3), (4, 2), (6, 1)}. So, range of 𝑅 = {1, 2, 3}.
11. Given relation is 𝑅 = {(𝑎, 𝑎3 ) ∶ 𝑎𝑖𝑠𝑎𝑝𝑟𝑖𝑚𝑒𝑛𝑢𝑚𝑏𝑒𝑟𝑙𝑒𝑠𝑠𝑡ℎ𝑎𝑛 5}.
∴ 𝑅 = {(2,8), (3,27)} So, the range of R is {8, 27}.
12. Given, 𝑅 = {(𝑎, 𝑏): 2 𝑑𝑖𝑣𝑖𝑑𝑒𝑠(𝑎 − 𝑏)} and 𝐴 = {0,1,2,3,4,5}
Clearly, [0] = {𝑏 ∈ 𝐴: (0, 𝑏) ∈ 𝑅}
= {𝑏 ∈ 𝐴: 2 𝑑𝑖𝑣𝑖𝑑𝑒𝑠 (0 − 𝑏)} = {𝑏 ∈ 𝐴: 2 𝑑𝑖𝑣𝑖𝑑𝑒𝑠 (−𝑏)} = {0,2,4}
Hence, equivalence class of [0] = {0,2,4}
13. R is not transitive as (1,2) ∈ 𝑅, (2,1) ∈ 𝑅 but (1,1) ∉ 𝑅
# Note: a relation R in a set A is said to be transitive if (𝑎, 𝑏) ∈ 𝑅, (𝑏, 𝑎) ∈ 𝑅 but (𝑎, 𝑐) ∈ 𝑅, ∀𝑎, 𝑏, 𝑐 ∈ 𝑅
14.(3, 1) is the single ordered pair which needs to be added to R to make it the smallest equivalence relation.

15. For any two elements x1 , x2∈ R such that f (x1 ) = f (x2 ), we have 4x1 – 1 = 4x2 – 1 ⇒ 4x1 = 4x2 , i.e., x1 = x2
Hence f is one-one

KVS RO DDN S SM XII MATHEMATICS/ 2023-24(VN) Page 9


*KVS RO DEHRADUN STUDENT SUPPORT MATERIAL MATHEMATICS/ XII/2023-24(VN)*

Relations And Functions: HOTS


Multiple Choice Questions
1. The relation R in the set of natural numbers N defined as R ={ (x,y) : x > y} is
(a) An equivalence relation
(b) Reflexive, symmetric but not transitive
(c) Symmetric, transitive but not reflexive
(d) Transitive but neither symmetric nor reflexive.
2. Let A = {a, b, c}. Then the number of equivalence relations in A containing (a, c) is
(a)1 (b) 2 (c) 3 (d) 4
3. Consider the set A contains n elements. Then the total number of injective functions from A onto itself is
2
(a) 2𝑛 (b)𝑛! (c) 2n –n (d)None of these
4. The number of onto functions from a set A containing 5 elements to set B containing 2 elements is
(a) 10 (b) 32 (c) 30 (d) 0
5. Let A = {x : -1 ≤ x ≤ 1} and f : A → A is a function defined by f(x) = x |x| then f is
(a) a bijective (b) injective but not surjective
(c) surjective but not injective (d) neither injective nor surjective
6. Which of the following functions from Z into Z are bijective?
(a) f(x) = x3 (b) f(x) = x + 2 (c) f(x) = 2x + 1 (d) f(x) = x2 + 1
7. The number of bijective functions from set A to itself when A contains 106 elements is
(a) 106 (b) (106)2 (c) 106! (d) 2106
2 marks Question
8. Prove that the greatest integer function f: R → R, given by f(x) = [x], is neither one-one nor onto.
9. Check the greatest integer function f: R → Z, given by f(x) = [x], is 1-1 or onto.
1 + 𝑥2
10. Prove that the function f: R- {-1} → R defined as f(x) = is not one-one.
1+𝑥
11. Check whether the relation S in R defined by S= {(x, y) : x . y is an irrational number} is transitive or not
?
3 marks Question
12. Show that the relation S in set R of real numbers defined by

S = { (a,b) : 𝑎 ≤ 𝑏 3 , a∈ 𝑹, b∈ 𝑹} is neither reflexive, nor symmetric, nor transitive.


13. Determine whether the relation S defined on the set R of all real numbers as
𝑆 = {(𝑎, 𝑏): 1 + 𝑎𝑏 > 0, 𝑎, 𝑏 ∈ 𝑹}, is reflexive, symmetric or transitive.
5 marks Question

14. Define the relation R in the set 𝑁 × 𝑁 as follows:


For (a, b), (c, d) ∈𝑁 × 𝑁, (a, b) R (c, d) iff ad = bc. Prove that R is an equivalence relation in 𝑁 ×
𝑁.
15. Prove that the function f: [0, ∞) → [-5, ∞ ) defined as f(x) = 9 x2 + 6x -5 is both one-one and onto. Also
find pre image of 43.
1 1 𝑥
16. Check whether a function f: R→ [-2 , 2 ] defined as f(x) = 1+𝑥 2
is bijective or not.
17. Prove that the function f: [-4, 4 ) → [0, 4] defined as f(x) = √16 − 𝑥 2 is an onto function but not a one-
one function. Further, find all possible values of ‘a’ for which f(a) =√7.
18. Define the relation R in the set 𝑁 × 𝑁 as follows:
For (a, b), (c, d) ∈𝑁 × 𝑁, (a, b) R (c, d) iff ad (b +c) = b c (a +d). Prove that R is an equivalence
relation in 𝑁 × 𝑁.
𝑥
19. Prove that the function f: R → (-1, 1) defined as f(x) = 1+|𝑥| is both one-one and onto.
20. Determine whether the relation R defined on the set R of all real numbers as
𝑅 = {(𝑎, 𝑏): 𝑎, 𝑏 ∈ 𝑅𝑎𝑛𝑑𝑎 − 𝑏 + √2𝑖𝑠𝑖𝑟𝑟𝑎𝑡𝑖𝑜𝑛𝑎𝑙𝑛𝑢𝑚𝑏𝑒𝑟}, is reflexive, symmetric or transitive.

KVS RO DDN S SM XII MATHEMATICS/ 2023-24(VN) Page 10


*KVS RO DEHRADUN STUDENT SUPPORT MATERIAL MATHEMATICS/ XII/2023-24(VN)*

Relations And Functions : HOTS : Solutions:


1. (d) Transitive but neither symmetric nor reflexive.
2. (b) 2
3. (b) 𝑛!
4. (c)30

5. (a) a bijective
6. .(b) f(x) = x + 2

7.(c) 106!

8. f(2.1)=2 , f(2.5)=2. So not 1-1


f is not onto as 2.1 has no pre image
9. f is 1-1 but not onto
10. f(0)=1 , f(1) =1 . So f(0)= f(1) but 0  1
11. Not transitive as (2, √2 ) and ( √2, 3) ∈ 𝑆 but (2,3) ∉ S
1 1
12. S is not reflexive as (2 , 2) ∉ 𝑆
1 1
S is not symmetric as (2 , 1) ∈ 𝑆 but (1, 2) ∉ 𝑆
S is not transitive as (9,4) ∈ 𝑆, (4,2) ∈ 𝑆 but (9,2) ∉ 𝑆

13. reflexive ,symmetric but not transitive


14. Showing ab=ba ,hence R is reflexive
Showing cb= da⇒ (𝑐, 𝑑)R(𝑎, 𝑏),hence R is symmetric
(𝑎, 𝑏)R(𝑐, 𝑑)⇒ad=bc,(𝑐, 𝑑)R(𝑒, 𝑓)⇒cf=de
af =be⇒(𝑎, 𝑏)𝑅 (𝑒, 𝑓),hence R is transitive
R is reflexive, symmetric, and transitive hence R is equivalence relation
15. For one –one
f(x1)= f(x2)
(x1-x2)(9x1 + 9x2 +6)=0
x1=x2 as (9x1 + 9x2 +6) > 0
so f is 1-1
For onto
f(x) = y
x=( -1 + √6 + 𝑦) /3 ∈ [0, ∞ ) for all y ∈ [-5, ∞ )
So f is onto
Pre image of 43 is 2
16. not 1-1 but onto
17. a=  3
20. Reflexivity: Let a be any real number. ∵ 𝑎 − 𝑎 + √2 = √2 is irrational number ∴ (a,𝑎) ∈ 𝑅
So, R is reflexive.
Symmetry: Let (√2, 2) ∈ 𝑅 ⇔ √2 − 2 + √2 = 2√2 − 2 is irrational number
But if (2, √2) ∈ 𝑅 ⇔ 2 − √2 + √2 = 2 is not irrational number
That is, (𝑎, 𝑏) ∈ 𝑅 doesn’t necessarily imply (𝑏, 𝑎) ∈ 𝑅∀𝑎, 𝑏 ∈ 𝑅. So, R isn’t symmetric.
Transitivity: Let (2, √5), (√5, √2) ∈ 𝑅 ⇔ 2 − √5 + √2𝑖𝑠𝑖𝑟𝑟𝑎𝑡𝑖𝑜𝑛𝑎𝑙𝑛𝑢𝑚𝑏𝑒𝑟𝑎𝑛𝑑√5 − √2 + √2 =
√5 is irrational number
But if (2, √2) ∈ 𝑅 ⇔ 2 − √2 + √2 = 2 𝑖𝑠𝑛𝑜𝑡𝑖𝑟𝑟𝑎𝑡𝑖𝑜𝑛𝑎𝑙𝑛𝑢𝑚𝑏𝑒𝑟
That is, (𝑎, 𝑏)𝑎𝑛𝑑 (𝑏, 𝑐) ∈ 𝑅 doesn’t necessarily imply (𝑎, 𝑐) ∈ 𝑅∀𝑎, 𝑏, 𝑐 ∈ 𝑅.
So, R isn’t transitive as well.

KVS RO DDN S SM XII MATHEMATICS/ 2023-24(VN) Page 11


*KVS RO DEHRADUN STUDENT SUPPORT MATERIAL MATHEMATICS/ XII/2023-24(VN)*

Relation and Function


Case Study Based Questions: MLL

Case Study 1 –
Mr Pradeep take a lecture in class – XII , in this class students divided into two groups A and B group A has 22
Students and group B has 23 students. The relation R: A → B between students are defined as 𝑅 = {(𝑥 , 𝑦): 𝑥 ∈
𝐴𝑎𝑛𝑑𝑦 ∈ 𝐵}

Based of the above information answer the following questions-


(i) Find the number of elements in co-domain of R: A → B.
(ii) The set of all identity elements
(iii) Find the total number of relations that can be formed from set A to B.
(iv) Suppose the relation defined from set A to set B is 𝑅1 = {(𝑎1 , 𝑏1 ), (𝑎2 , 𝑏2 ), (𝑎3 , 𝑏3 )} then find the
inverse relation.

Case Study 2 –
Sherlin and Danju are playing Ludo at home during Covid-19. While rolling the dice, Sherlin’s sister Raji
observed and noted the possible outcomes of the throw every time belongs to set {1, 2, 3, 4, 5, 6}. Let A be the
set of players while B be set of all possible outcomes. A = {S, D}, B = {1, 2, 3, 4, 5, 6}
Based on the above information answer the following:
(i) Let R : B → B be defined by R = {(x, y) : y is divisible by x} is
(a) Reflexive and transitive but not symmetric (b) Reflexive and symmetric but not transitive
(c) Not reflexive but symmetric and transitive (d) Equivalence
(ii) Raji wants to know the number of functions from A to B. How much number of functions are possible?
(a) 62 (b) 26 (c) 6! (d) 212
(iii) Let R be a relation on B defined by R = {(1, 2), (2, 2), (1, 3), (3, 4), (3, 1), (4, 3), (5, 5)}. Then R is
(a) Symmetric (b) Reflexive (c) Transitive (d) None of these three
(iv) Raji wants to know the number of relations possible from A to B. How many numbers of relations are
possible?
(a) 62 (b) 26 (c) 6! (d) 212
(v) Let R : B –> B be defined by R = {(1, 1), (1, 2), (2, 2)(3, 3), (4, 4), (5, 5), (6, 6)}, then R is
(a) Symmetric (b) Reflexive and Transitive
(c) Transitive and symmetric (d) Equivalence

KVS RO DDN S SM XII MATHEMATICS/ 2023-24(VN) Page 12


*KVS RO DEHRADUN STUDENT SUPPORT MATERIAL MATHEMATICS/ XII/2023-24(VN)*

Case Study 3 –
In two different societies, there are some school going students – including girl as well as boys. Satish forms two
sets with these students , as his college project.
Let 𝐴 = {𝑎1 , 𝑎2 , 𝑎3 , 𝑎4 , 𝑎5 } and 𝐵 = {𝑏1 , 𝑏2 , 𝑏3 , 𝑏4 } where 𝑎𝑖 ’s and 𝑏𝑖 ’s are the school going students of first
and second society respectively. Satish decides to explore these sets for various types of relations and functions.
Using the above information given above, answer the following
(i) Satish wishes to know the number of reflexive relations defined on set A. How many such relations
are possible?
(a) 0, (b) 25 (c) 211 (d) 220
(ii) Let : 𝐴 → 𝐴, 𝑅 = {(𝑥, 𝑦): 𝑥𝑎𝑛𝑑𝑦𝑎𝑟𝑒𝑠𝑡𝑢𝑑𝑒𝑛𝑡𝑠𝑜𝑓𝑠𝑎𝑚𝑒𝑠𝑒𝑥} . Then relation R is
(a) Reflexive only (b) reflexive and symmetric but not transitive (c) reflexive and
transitive but not symmetric (d) an equivalence relation
(iii) Satish and his friend Rajat are interested to know the number of symmetric relations defined on the
both the set A and B, separately. Satish decides to find the symmetric relation on A. while Rajat
decides to find the symmetric relation on set B. What is difference between their results?
(a) 1024 (b) 210 (15) (c) 210 (31) (d) 210 (63)
(iv) Let 𝑅: 𝐴 → 𝐵, 𝑅 = {(𝑎1 , 𝑏1 ), (𝑎1 , 𝑏2 ), (𝑎2 , 𝑏1 ), (𝑎3 , 𝑏3 ), (𝑎4 , 𝑏2 ), (𝑎5 , 𝑏2 )} , then R is
(a) Neither one-one nor onto (b) one-one but not onto (c) only onto but not one-one
(d) one-one and onto both
(v) To help Satish in his project Rajat decides to form onto function from A to B. How many such
functions are possible?
(a) 342 (b) 243 (c) 729 (d) 1024

Case study 4
AgeneralelectionofLokSabhaisagiganticexercise.About911millionpeoplew
ereeligibletovoteandvoter turnoutwas about67%,thehighest ever
LetIbethesetofallcitizensofIndiawhowereeligibletoexercisetheirvotingrighti
ngeneralelectionheld in2019.Arelation ‘R’is definedonI asfollows:
R= {(𝑉1,𝑉2)∶𝑉1,𝑉2∈𝐼andboth usetheirvotingrightin generalelection–
2019}

1. TwoneighborsXandY∈I.XexercisedhisvotingrightwhileYdid
notcasthervoteingeneral election–2019.Whichofthefollowingis true?
a. (X,Y)∈R b. (Y,X)∈R c. (X,X)∉R d. (X,Y)∉R
2. Mr.’𝑋’andhiswife‘𝑊’bothexercisedtheirvotingrightingeneralelection-2019,Whichofthefollowingis true?
a. both(X,W)and(W,X)∈R b. (X,W)∈R but(W,X)∉R
c. both(X,W)and(W,X)∉R d. (W,X)∈R but(X,W)∉R
3. ThreefriendsF1,F2andF3exercisedtheirvotingrightingeneralelection-2019,thenwhichof thefollowingis
true?
a.(F1,F2 ) ∈R, (F2,F3) ∈R and(F1,F3)∈R b.(F1,F2) ∈R, (F2,F3) ∈R and(F1,F3)∉R
c.(F1,F2 )∈R, (F2,F2)∈Rbut (F3,F3) ∉R d.(F1,F2 ) ∉R,(F2,F3)∉R and(F1,F3)∉R
4. TheabovedefinedrelationRis
a. Symmetricandtransitivebutnot reflexive b. Universalrelation
c. Equivalencerelation d. Reflexivebutnotsymmetricandtransitive
5. Mr.ShyamexercisedhisvotingrightinGeneralElection–2019,thenMr.Shyamisrelatedtowhich of
thefollowing?
a. Allthoseeligiblevoterswhocasttheirvotes
b. Familymembersof Mr.Shyam
c. AllcitizensofIndia
d. EligiblevotersofIndia

KVS RO DDN S SM XII MATHEMATICS/ 2023-24(VN) Page 13


*KVS RO DEHRADUN STUDENT SUPPORT MATERIAL MATHEMATICS/ XII/2023-24(VN)*

Case Study Based Questions: HOTS : Solutions

Case Study – 1
Answer: (i) 23 (ii) ∅ (iii) 2506 (iv) {(𝑏1 , 𝑎1 ), (𝑏2 , 𝑎2 ), (𝑏3 , 𝑎3 )}
Case Study – 2
Answer: (i) (a) (ii) (a), (iii) (d) , (iv) (d), (v) (b)
Case Study – 3
Answer (i) d (ii) d (iii) c (iv) a (v) b
Case Study – 4
Answer: 1. D 2. A 3. A 4. C 5. A

KVS RO DDN S SM XII MATHEMATICS/ 2023-24(VN) Page 14


*KVS RO DEHRADUN STUDENT SUPPORT MATERIAL MATHEMATICS/ XII/2023-24(VN)*
Relation and Function
Case Study Based Questions: HOTS
Each of the following questions are of 4 marks.
1. Read the following passage and answer the following questions:
In a family, there are seven members, a grandfather (GF), a grandmother
(GM), a couple (ie.a husband (H) and a wife (W)), and their three
children (a boy (B), a girl (G)and a baby boy(BB)).
Let M = { GF, H, B, BB} and F = { GM, W, G}, where M represents the
set of male and F represent the set of Female.
(i) How many relations are possible from M to F? (1)
(ii) Among all possible relations from M to F, how many function can be
formed from M to F? (1)
(iii) (a) Let 𝑅: 𝐹 → 𝐹 be defined by 𝑅 = {(𝑥, 𝑦): 𝑥 𝑎𝑛𝑑 𝑦 𝑎𝑟𝑒 𝑜𝑓 𝑠𝑎𝑚𝑒 𝑠𝑒𝑥} Check if R is an equivalence
relation in F or not. (2)
OR
(iii) (b) Can you define a bijective function from M to F?
If Yes, give an example of an bijective function. If No, give reason why it is not possible. (2)

2. Read the following passage and answer the following questions:


4𝑥+9
A teacher plotted a graph of a function defined by𝑓(𝑥)=
𝑥+2

Based on the above details, answer the following question:


(i) Based on the observation of the graph, the domain of the function 𝑓(𝑥) is
(a) ℝ − {2} (b)ℝ − {−2} (c)ℝ − {4} (d)ℝ − {−4}
(ii) Based on the observation of the graph, the range of the function 𝑓(𝑥) is
(a) ℝ − {2} (b)ℝ − {−2} (c)ℝ − {4} (d) ℝ − {−4}
(iii) Check whether the function 𝑓(𝑥) is one - one or Many - one, from its domain to range (2)

KVS RO DDN S SM XII MATHEMATICS/ 2023-24(VN) Page 15


*KVS RO DEHRADUN STUDENT SUPPORT MATERIAL MATHEMATICS/ XII/2023-24(VN)*

3. Read the following passage and answer the following


questions:
A teacher plotted two graphs on the blackboard,
𝑓: ℝ → ℝ, 𝑔: ℝ → ℝsuch that
𝑓(𝑥) = 2𝑥 + 3 𝑎𝑛𝑑 𝑔(𝑥) = 𝑥 2 + 2𝑥 − 3

(i) Check whether 𝑓(𝑥) is bijective or not. (2)


(ii) Check whether 𝑔(𝑥) is bijective or not. (2)

KVS RO DDN S SM XII MATHEMATICS/ 2023-24(VN) Page 16


*KVS RO DEHRADUN STUDENT SUPPORT MATERIAL MATHEMATICS/ XII/2023-24(VN)*

Case Study Based Questions: HOTS : Solutions

1(i) Number of Relations = 2𝑛(𝑀)×𝑛(𝐹) = 24×3 = 212 = 4096


1(ii) Number of Functions = (𝑛(𝐹))𝑛(𝑀) = 34 = 81
1(iii) (a) 𝑅: 𝐹 → 𝐹, 𝑅 = {(𝑥, 𝑦): 𝑥 𝑎𝑛𝑑 𝑦 𝑎𝑟𝑒 𝑜𝑓 𝑠𝑎𝑚𝑒 𝑠𝑒𝑥}
Reflexive: ∵ (𝑥, 𝑥) ∈ 𝑅 ∀ 𝑥 ∈ 𝐹 ⟹ 𝑅 𝑖𝑠 𝑟𝑒𝑓𝑙𝑒𝑥𝑖𝑣𝑒
Symmetric: Let (𝑥, 𝑦) ∈ 𝑅 ⟹ 𝑥 𝑎𝑛𝑑 𝑦 𝑎𝑟𝑒 𝑜𝑓 𝑠𝑎𝑚𝑒 𝑠𝑒𝑥
⟹ 𝑦 𝑎𝑛𝑑 𝑥 𝑎𝑟𝑒 𝑜𝑓 𝑠𝑎𝑚𝑒 𝑠𝑒𝑥 ⟹ (𝑦, 𝑥) ∈ 𝑅
Transitive: Let (𝑥, 𝑦)𝑎𝑛𝑑 (𝑦, 𝑧) ∈ 𝑅
⟹ 𝑥 𝑎𝑛𝑑 𝑦 𝑎𝑟𝑒 𝑜𝑓 𝑠𝑎𝑚𝑒 𝑠𝑒𝑥 𝑎𝑛𝑑 𝑦 𝑎𝑛𝑑 𝑧 𝑎𝑟𝑒 𝑜𝑓𝑠𝑎𝑚𝑒 𝑠𝑒𝑥.
⟹ 𝑥 𝑎𝑛𝑑 𝑧 𝑎𝑟𝑒 𝑜𝑓 𝑠𝑎𝑚𝑒 𝑠𝑒𝑥 ⟹ (𝑥, 𝑧) ∈ 𝑅 ⟹ 𝑅 𝑖𝑠 𝑡𝑟𝑎𝑛𝑠𝑖𝑡𝑖𝑣𝑒
Hence, R is reflexive, symmetric and transitive. ∴ R is an equivalence relation in F.
1(iii) (b) No, we can’t define a bijective function from M to F.
For a bijective function 𝑛(𝑀) = 𝑛(𝐹) 𝑏𝑢𝑡 ℎ𝑒𝑟𝑒 𝑛(𝑀) = 4 ≠ 3 = 𝑛(𝐹).

2(i) (b)ℝ − {−2}


2(ii) (c)ℝ − {4}
4𝑥1+9 4𝑥2 +9
2(iii) For𝑓(𝑥1 ) = 𝑓(𝑥2 ) ⟹ =
𝑥1 +2 𝑥2 +2
⟹ 4𝑥1 𝑥2 + 8𝑥1 + 9𝑥2 + 18 = 4𝑥1 𝑥2 + 8𝑥2 + 9𝑥1 + 18
⟹ 8𝑥1 + 9𝑥2 = 8𝑥2 + 9𝑥1 ⟹ −𝑥1 = −𝑥2 ⟹ 𝑥1 = 𝑥2
∴ 𝑓(𝑥) 𝑖𝑠 1 − 1.
3(i) For 𝑓(𝑥1 ) = 𝑓(𝑥2 ) ⟹ 2𝑥1 + 3 = 2𝑥2 + 3 ⟹ 2𝑥1 = 2𝑥2 ⟹ 𝑥1 = 𝑥2 ∴ 𝑓(𝑥) 𝑖𝑠 1 − 1
𝑦−3 𝑦−3
Let 𝑦 = 2𝑥 + 3 ⟹ 𝑥= 2
ie. 𝑓𝑜𝑟 𝑒𝑣𝑒𝑟𝑦 𝑦 ∈ ℝin Co-domain ∃ 2
∈ ℝin domain. Hence 𝑓 is bijective.
3(ii) 𝑔(−3) = 0 = 𝑔(1)𝑏𝑢𝑡 − 3 ≠ 1, 𝐻𝑒𝑛𝑐𝑒 𝑔(𝑥)𝑖𝑠 𝑚𝑎𝑛𝑦 − 1 𝑓𝑢𝑛𝑐𝑡𝑖𝑜𝑛
Further, 𝑅𝑎𝑛𝑔𝑒 𝑜𝑓 𝑔(𝑥)𝑖𝑠[−4, ∞)𝑚𝑒𝑎𝑛𝑠 𝑛𝑜 𝑒𝑙𝑒𝑚𝑒𝑛𝑡 𝑖𝑛 (−∞, −4)ℎ𝑎𝑠 𝑎 𝑝𝑟𝑒 − 𝑖𝑚𝑎𝑔𝑒.
Hence 𝑔(𝑥) is many – 1, into function ie. not a bijective function.

KVS RO DDN S SM XII MATHEMATICS/ 2023-24(VN) Page 17


*KVS RO DEHRADUN STUDENT SUPPORT MATERIAL MATHEMATICS/ XII/2023-24(VN)*
Relation and Function : Class Test (MLL)
Duration: 1 hour Max.Marks:
30 .
Q1.Let f : R → R be defined by f (x) = 1/x ∀ x ∈ R. Then f is
(A) one-one (B) onto (C) bijective (D) f is not defined

2. Let A = {1, 2, 3, 4}, B = {1, 5, 9, 11, 15, 16} and f = {(1, 5), (2, 9), (3, 1), (4, 5), (2, 11)}.
Then,
(a) f is a relation from A to B (b) f is a function from A to B
(c) Both (a) and (b) (d) None of these

3. If A = {2, 3, 4, 5} and B = {3, 6, 7, 10}. R is a relation defined by R = {(a, b) : a is relatively


prime to b, a ∈ A and b ∈ B}, then domain of R is
(a) {2, 3, 5} (b) {3, 5} (c) {2, 3, 4} (d) {2, 3, 4, 5}

5 If g = {(1, 1), (2, 3), (3, 5), (4, 7)} is a function described by the formula, g (x) = αx + β then
what values should be assigned to α and , β ?
(a) α=2, β= 1 (b) α=2, β= -1 (c) α=1, β= -1 (d) α=3, β= -1

6. Show that the relation R defined in the set A of all polygons as R = {(P1, P2) : P1 and P2 have same number
of sides}, is an equivalence relation. What is the set of all elements in A related to the right angle triangle T with
s ides 3, 4 and 5?

7 Let f : R → R be the function defined by f (x) = 1/2−cosx∀x ∈R. Then, find the range of f.
8 Show that the relation R in the set A={ x∈ Z: 0≤ x ≤ 12} given by
R={ a − b is divisible by 4} is an equivalence relation. And the equivalence class of 3 i.e.[3]

9 Is g = {(1, 1), (2, 3), (3, 5), (4, 7)} a function? If g is described by g (x) = αx + β, then what value should be
assigned to α and β.
10. If 𝑓 ∶ 𝑅 → 𝑅 is the function defined by 𝑓(𝑥) = 4𝑥 3 + 7, then show that 𝑓 is a bijection ? Show that the

relation S in the set A  x  Z : 0  x  12given by



S  (a, b) : a, b  Z , a  b is divisible by 4 
is an
equivalence relation. Find the set of all elements related to 1

KVS RO DDN S SM XII MATHEMATICS/ 2023-24(VN) Page 18


*KVS RO DEHRADUN STUDENT SUPPORT MATERIAL MATHEMATICS/ XII/2023-24(VN)*

Relation and Function : Class Test (HOTS)


Duration: 1 hour Max.Marks: 30 .
GENERAL INSTRUCTIONS
(a) The question paper consists of 10 questions divided into Four sections A, B, C & D.
(b) Each question in section A (Q. No. 1 – 2) MCQ/AR carry 1 Mark.
(c) Each question in section B (Q. No. 3 – 4) carries 2 Marks
(d) Each question in section C (Q. No. 5 – 7) carries 3 Marks.
(e) Each question in section D (Q. No. 8 – 10) carries 5 Marks.
SECTION – A
Q1. Let us define a relation R in R as aRb if a  b. Then R is
(a) an equivalence relation (b) reflexive, transitive but not symmetric
(c) symmetric, transitive but not reflexive (d) neither transitive nor reflexive but symmetric.
Q2. Given below are two statements: one is labeled as Assertion A and other is labeled as Reason R.
Assertion A: The number of bijective functions from the set containing 10 elements to itself is 210 .
Reason R: The total number of bijections from a set containing n elements to itself is n!.
In the light of the above statements, choose the most appropriate answer from the options given below
a. Both A and R are correct and R is the correct explanation of A
b. Both A and R are correct but R is NOT the correct explanation of A
c. A is correct but R is not correct
d. A is not correct but R is correct

SECTION – B
Q3. State whether the function is one – one, onto or bijective f: R → R defined by f(x) = 1+ x2
Q4. Checkwhether the relation R defined in the set {1, 2, 3, 4, 5, 6} as R = {(a, b): b = a+1} is reflexive,
symmetric or transitive.
SECTION – C
Q5. Let A and B be two sets. Show that f: A B → B A such that f(a, b) = (b, a) is a bijective function.
𝑥2
Q6. Check whether the function f: R → R defined as f(x) = 1+𝑥2 is bijective function
Q7. Check whether the relation R in the set R of real numbers, defined by R = {(a, b) : 1 + ab > 0)}, is reflexive,
symmetric or transitive.
SECTION – D
Q8..Show that the relation S in set R of real numbers defined by
S = { (a,b) : 𝑎 ≤ 𝑏 3 , a∈ 𝑹, b∈ 𝑹} is neither reflexive, nor symmetric, nor transitive.
Q9. Prove that the function f: [0, ∞) → [-5, ∞ ) defined as f(x) = 4 x2 + 4x -5 is both one-one and onto.
Q10. Define the relation R in the set 𝑁 × 𝑁 as follows:
For (a, b), (c, d) ∈𝑁 × 𝑁, (a, b) R (c, d) iff ad (b +c) = b c (a +d). Prove that R is an equivalence relation in 𝑁 ×
𝑁.

KVS RO DDN S SM XII MATHEMATICS/ 2023-24(VN) Page 19


*KVS RO DEHRADUN STUDENT SUPPORT MATERIAL MATHEMATICS/ XII/2023-24(VN)*

Relation and Function : Class Test (HOTS): Solutions:


1. b

2. d

3. Ans not 1-1, not onto

4. Ans not reflexive, not symmetric, not transitive

5. Ans Let (a1, b1) and (a2, b2) A B


(i) f(a1, b1) = f(a2, b2)
b1 = b2 and a1 = a2
(a1, b1) = (a2, b2)
(ii) f is injective,
Let (b, a) be an arbitrary
Element of B A. then b B and a A
(a, b) ) (A B)
Thus for all (b, a) B A their exists (a, b) ) (A B)
Hence that
f(a, b) = (b, a)
So f is an onto function.
Hence bijective
6. Not 1-1 ,not onto
7. reflexive ,symmetric but not transitive

KVS RO DDN S SM XII MATHEMATICS/ 2023-24(VN) Page 20


*KVS RO DEHRADUN STUDENT SUPPORT MATERIAL MATHEMATICS/ XII/2023-24(VN)*
CHAPTER 2: INVERSE TRIGONOMETRIC FUNCTIONS
MINIMUM LEARNING LEVEL
Section - A (MCQs ) ( 1 Marks each)
Q1. The principal value of sec (2/√3) is
-1

(a) π/3 (b) π/6 (c) –π/3 (d) 2π/3

Q2. The principal value of sin-1 (1/2) + tan-1(1/√3) is


(a) π/3 (b) π/6 (c) –π/3 (d) 2π/3

Q3. The value of sin- { π/3 + sin-1(1/2) } is


(a) 1 (b) 0 (c) 2 (d) 4

Q4. The principal value of cos-1(1/2) + 2 sin-1 (1/2) is


(a) π/3 (b) π/6 (c) 0 (d) 2π/3

Q5. The value of tan-1 [2sin(2cos-1√3/2) is


(a) π/3 (b) π/6 (c) –π/3 (d) 2π/3

Q6. For What vale of x : sin-1 ½ = tan-1 x


(a) 1/√3 (b) √3 (c) 2 (d) 0

Q7. The domain of cos-1(2x-1) is


(a) [0,1] (b) [-1,1] (c) (-1,1) (d) [ 0, π]

Q8. The domain of the function f(x) = sin-1 (√x-1) is


(a) [1,2] (b) [-1,1] (c) (2,1) (d) [ 0, 1]
𝜋 1
Q9. Find the value of sin ( - sin-1(− ))
3 2
(a) 1/√3 (b) 1 (c) 2 (d) 0
1
Q10. Write the principal value of tan-1(1) + cos-1(- )
2
(a) π/3 (b) π/6 (c) 7π/12 (d) 2π/3

Section –B (2 marks questions)


Q1. The value of tan-1 √3 + cot-1 √3 + tan-1(cos (0)) is ……………..
Q2. The value of tan-1 [2sin(2cos-1 √3/2) is ………..
Q3. Find the value of tan--1 1 + cos--1 (-1/2) + sin--1(-1/2).
Q4 . Find the value of tan--1(tan5π/6) + cos1 cos(13π/6)
Q5. Find the value of (i) tan-1(-1) + cot-1(-1/√3)
Section –B ( 3 marks questions)
Q6. Simplify (i) tan-1 (x/(√1-x2 ) (ii) tan-1 (sinx / 1+cosx)
Q7. Find principal values of the following : (i) sin-1(sin 3π/5) (ii) cosec-1(cosec 13π/4)
Q8. Find principal values of sec-1( -2/√3) + cosec-1(-√2)
Q9. Write in simplest form : (i) tan-1 (cosx-sinx)/cosx+sinx)
(ii) tan-1(√(1+x2) -1 /x )
Q10. . Prove that tan (√1+x - √1-x)/√1+x + √1-x) = π/4 – ½ cos-1x
-1

More Practice Questions


1
1. Write the principal value of tan-1(1) + cos-1(-2)
𝜋 1
2. Find the value of sin (3 - sin-1(− 2))
1
3. If sin(sin-1 + cos-1x) =1 then find the value of x.
2
1
4. Write the value of tan (2tan-15)
3𝜋
5. Using principal value evaluate sin-1(sin 5 )

KVS RO DDN S SM XII MATHEMATICS/ 2023-24(VN) Page 21


*KVS RO DEHRADUN STUDENT SUPPORT MATERIAL MATHEMATICS/ XII/2023-24(VN)*

Answers

Ans (MCQ ). 1. (b) 2.(a) 3. (a) 4. 5. (a) 6. (a) 7. (a) 8. (a) 9. (b) 10. (c )

Ans. (S.A.Q.) 1. 3 π/4 2. π/3 3. . π/4 4. 0 5. π/12 6. (i) sin-1x (ii) x/2
7.(i) 2 π/5 (ii) . - π/4 8. . 7 π/12 9. . (i) π/4 -x (ii) ½ tan-1x
More Practice Questions:
11𝜋 1 5 2𝜋
Ans 1: 2. 1 3. 4. 5.
12 2 12 5

KVS RO DDN S SM XII MATHEMATICS/ 2023-24(VN) Page 22


*KVS RO DEHRADUN STUDENT SUPPORT MATERIAL MATHEMATICS/ XII/2023-24(VN)*

QUESTION BANK FOR INVERSE TRIGONOMETRIC FUNCTIONS


HOTS
MULTIPLE CHOICE QUESTIONS
1 1
1. The value of cos−1 (− ) + 2 sin−1 ( )
2 2
(a) 𝜋 (b) −𝜋/2 (c) 0 (d) None of these

13𝜋
2.The principal value of cos−1 (cos 6
) is
13𝜋 𝜋 𝜋 𝜋
(A) 6
(b) 2
(C) 3 (d) 6

3. The value of 𝑡𝑎𝑛−1 √3 − 𝑐𝑜𝑠 −1 (−1) is


(a) 0 (b) −𝜋/2 (c) 𝜋 (d) −2𝜋/3

4. The value of tan−1 (−1)


(a) 𝜋/4 (b) −𝜋/4 (c) 3𝜋/4 (d) 𝑁𝑜𝑛𝑒 𝑜𝑓 𝑡ℎ𝑒𝑠𝑒

1
5. The principal value of cos−1 (− 2) is
(a) 𝜋 (b) −𝜋/3 (c) 0 (d) 2𝜋/3

6. The principal value of cot −1(−√3) is


(a) 𝜋 (b) −𝜋/6 (c) 5𝜋/6 (d) 5𝜋/3

𝜋 1
7. The value of sin ( − sin−1 (− )).
3 2
(a) 1 (b) −𝜋/2 (c) -1 (d) None of these

8. The value of 𝑡𝑎𝑛−1 √3 − 𝑐𝑜𝑡 −1 ( − √3).


(a) 𝜋 (b) −𝜋/2 (c) 0 (d) None of these

9. The value of tan−1 √3 − sec −1(−2).


(a) 𝜋 (b) −2𝜋/3 (c) -𝜋/3 (d) 2𝜋/3

𝜋
10. The value of the following: 𝑐𝑜𝑡 [ 2 − 2 cot −1 √3]
(a) −1 (b) √3 (c) 1 (d) None of these

√3
11. The value of tan−1 {2 sin (2 cos−1 2
)}
(a) 𝜋 (b) 𝜋/3 (c) -𝜋/3 (d) 2𝜋/3

1
12. The principal value of tan−1 (1) + cos −1 (− )
2
(a) 𝜋 (b) 11𝜋/12 (c) -11𝜋/12 (d) 𝜋/12

1
13. The value of 𝑡𝑎𝑛 {2 tan−1 }
5
(a) 1/5 (b) 2/5 (c) 5/12 (d) none of these
3
14. The value of 𝑠𝑖𝑛 {2 sin−1 5}
(a) 24/25 (b) 3/5 (c) 6/5 (d) none of these

15. The principal value of sec −1(−2).


(a) 𝜋 (b) 𝜋/3 (c) -𝜋/3 (d) 2𝜋/3

KVS RO DDN S SM XII MATHEMATICS/ 2023-24(VN) Page 23


*KVS RO DEHRADUN STUDENT SUPPORT MATERIAL MATHEMATICS/ XII/2023-24(VN)*

2 Marks Questions
1
1. Prove that sin−1 (2𝑥√1 − 𝑥 2 ) = 2 cos −1 𝑥, ≤ 𝑥 ≤ 1.
√2
1 1
2. Prove that 3 sin−1 𝑥 = sin−1(3𝑥 − 4𝑥 3 ), 𝑥 ∈ [− 2 , 2]
1
3. Prove that 3 cos−1 𝑥 = cos −1 (4𝑥 3 − 3𝑥), 𝑥 ∈ [2 , 1]
1 3
4−√7
4. Show that: tan (2 sin−1 4) = 3
−1 1 −1 1−𝑥
5. Prove that tan √𝑥 = 2 cos (1+𝑥), 𝑥 ∈ [0,1]
√3 1
6. Write the principal value of cos −1 ( 2 ) + cos−1 (− 2)
1 1
7. Write the principal value of cos −1 (2) − 2 sin−1 (− 2).
17𝜋
8. Find the value of sin−1 (sin (− 8 )).
1
9. Evaluate: 𝑡𝑎𝑛−1 [2 𝑐𝑜𝑠 (2 𝑠𝑖𝑛−1 2)].
2𝜋 2𝜋
10. What is the principal value of cos −1 (cos 3 ) + sin−1 (sin 3 )
−1 (−𝑥) −1
11.Prove that: cot = 𝜋 − cot 𝑥, 𝑥 ∈ ℝ.
−1 1
12. Write cot ( ), |𝑥| > 1 in simplest form.
√𝑥 2 −1
−1 sin 𝑥+cos 𝑥 𝜋 𝜋
13. Express sin ( ); where − 4 < 𝑥 < 4 , in the simplest form.
√2
14. Prove that sin (2𝑥√1 − 𝑥 ) = 2 sin−1 𝑥
−1 2
1 1
15. Using principal values, write the value of 2 𝑐𝑜𝑠 −1 + 3 𝑠𝑖𝑛−1 .
2 2

3 Marks Questions
√1+𝑥−√1−𝑥 𝜋 1 1
1. Prove thattan−1 ( ) = − cos −1 𝑥, − ≤ 𝑥 ≤ 1.
√1+𝑥+√1−𝑥 4 2 √2

√1+𝑥+√1−𝑥 𝜋 1 1
2. Prove thattan−1 ( ) = 4 + 2 cos −1 𝑥, − ≤ 𝑥 ≤ 1.
√1+𝑥−√1−𝑥 √2

√1+𝑥 2 +√1−𝑥 2 𝜋 1
3. Prove that tan−1 ( ) = 4 + 2 cos−1 𝑥 2 , −1 < 𝑥 < 1
√1+𝑥 2 −√1−𝑥 2

√1+sin 𝑥+√1−sin 𝑥 𝑥 𝜋 𝜋
4. Prove that cot −1 ( ) = 2, 𝑥 ∈ (0, 2 ) or 𝑥 ∈ (0, 4 )
√1+sin 𝑥−√1−sin 𝑥

√1+cos 𝑥+√1−cos 𝑥 𝜋 𝑥 3𝜋
5. Prove that tan−1 ( ) = 4 − 2, 𝑥 ∈ (𝜋, ).
√1+cos 𝑥−√1−cos 𝑥 2

√1+cos 𝑥+√1−cos 𝑥 𝜋 𝑥 𝜋
6. Prove that tan−1 ( ) = 4 + 2 , 𝑖𝑓0 < 𝑥 < 2 .
√1+cos 𝑥−√1−cos 𝑥
1+𝑥 2
7. Prove the following: cos[tan−1 (sin(cot −1 𝑥))] = √2+𝑥2
1
8. Find the value of sin (2 tan−1 4) + cos(tan−1 2√2).
√1−cos 𝑥
9. Write in to the simplest form: tan−1 ( )
√1+cos 𝑥

√3
10. Write the value of 𝑡𝑎𝑛−1 [2 𝑠𝑖𝑛 (2 𝑐𝑜𝑠 −1 2
)].

KVS RO DDN S SM XII MATHEMATICS/ 2023-24(VN) Page 24


*KVS RO DEHRADUN STUDENT SUPPORT MATERIAL MATHEMATICS/ XII/2023-24(VN)*

CASE STUDY BASED QUESTIONS

1. The Government of India is planning to fix a hoarding board at the face of a building onthe road of a busy
market for awareness on COVID -19 protocol. Ram , Robert andRahim are the three engineers who are working
on this project . If“A” is considered tobe a personviewingthehoardingboard20
metresawayfromthebuilding,standingattheedgeofapathwaynearby.Ram ,Robert andRahim suggestedtothe filmto
placethe hoarding board at three different locations namely C,D and E .“C “ is at the height of 10metres from the
ground level. For the viewer A, the angle of elevation of “D “is doublethe angle of elevation of “C” .The angle of
elevation of “E” is triple the angle of elevationof “C”for the same viewer .Look at the figure given and based on
the above informationanswerthe following

Based on above information answer the following questions


1. The measure of ∠𝐶𝐴𝐵 is
2
(a) tan−1(1/3) (b) tan−1 ( ) (𝑐) tan−1 (1/2) (d) none of these
3
2. The measure of ∠𝐷𝐴𝐵 is
3
(a) tan−1(4/3) (b) tan−1 (4) (𝑐) tan−1 (3) (d) tan−1 (4)
3. The measure of ∠𝐸𝐴𝐵 is
2 11
(a) tan−1(11) (b) tan−1 (11) (𝑐) tan−1 (3) (d) tan−1 ( 2 )
4. If A’is anotherviewer standing on the same line of observation across the road. If thewidth of theroad is
5meters, then the ∠𝐶𝐴′𝐵is
2
(a) tan−1(1/5) (b) tan−1 (5) (𝑐) tan−1 (2) (d) none of these

3. A satellite flying at height h is watching the top of the two tallest mountains inUttarakhandand
Karnataka,as Nanda Devi (height 7816 meters ) and Mullayanagiri(height1937 meters ).The angles
ofdepressionfrom thesatellite, to thetop ofNanda Devi and Mullayanagiriare
cot−1√3andtan−1√3respectively. If the distance between the peaks of the twomountainsis 1937kmand the
satelliteis vertically above the midpoint of the distance betweenthe twomountains .Look attheFiguregiven
belowand answerthe Questions.

KVS RO DDN S SM XII MATHEMATICS/ 2023-24(VN) Page 25


*KVS RO DEHRADUN STUDENT SUPPORT MATERIAL MATHEMATICS/ XII/2023-24(VN)*

1. The distance of the satellite from the top of Nanda Devi hill is
(a)1139.4km (b)577.52km (c) 1937 km (d)1025. 36 km

2. The distance of the satellite from the top of Mullayangiriis


(a)1139.4 km (b )577.52 km (c) 1937km (d)1025.36 km

3. Thedistanceof thesatellitefromthe groundis


(a) 1139.4km (b)577.52 km (c) 1937km (d)1025. 36 km

4 . What is the angle of elevation of the top of Nanda Devi if a man is standing atadistanceof 7816 meters
from NandaDevi
(a)sec−1(2) (b)cot−1(1) (𝑐)sin−1(√3/2)(d)cos−1(1/2)

KVS RO DDN S SM XII MATHEMATICS/ 2023-24(VN) Page 26


*KVS RO DEHRADUN STUDENT SUPPORT MATERIAL MATHEMATICS/ XII/2023-24(VN)*

Solution of Question Bank


Solution of MCQ 1 Marks Questions

1. a 5. d 9. c 13. c
2. d 6. c 10. a 14. a
3. d 7. a 11. b 15. d
4. b 8. b 12. b

Solution of 2 Marks Questions


1. Let 𝑥 = cos 𝜃 then cos −1 𝑥 = 𝜃
Now, sin−1(2𝑥√1 − 𝑥 2 ) = sin−1 (2 cos 𝜃 √1 − (cos 𝜃)2 )
= sin−1 (2 cos 𝜃 √1 − 𝑐𝑜𝑠 2 𝜃)
= sin−1 (2 cos 𝜃 sin 𝜃)
= sin−1 (sin 2𝜃)
= 2𝜃
= 2 cos−1 𝑥

2. 𝑅𝐻𝑆: 𝐿𝑒𝑡𝑦 = 𝑠𝑖𝑛−1 [ 3𝑥 − 4𝑥 3 ]


𝑃𝑢𝑡𝑥 = 𝑠𝑖𝑛 𝜃 ⇒ 𝜃 = 𝑠𝑖𝑛−1 𝑥 . . . (𝑖)
∴ 𝑦 = 𝑠𝑖𝑛−1[ 3 𝑠𝑖𝑛 𝜃 − 4 𝑠𝑖𝑛3 𝜃] = 𝑠𝑖𝑛−1( 𝑠𝑖𝑛 3 𝜃)
1 1 1 1 1 1
As 𝑥 ∈ [− , ] ⇒ − ≤ 𝑥 ≤ ⇒ − ≤ 𝑠𝑖𝑛 𝜃 ≤
2 2 2 2 2 2
𝜋 𝜋 𝜋 𝜋
⇒ − ≤ 𝜃 ≤ ⇒ − ≤ 3𝜃 ≤ ⇒ −1 ≤ 𝑠𝑖𝑛 3 𝜃 ≤ 1
6 6 2 2
∴ 𝑦 = 3𝜃 = 3 𝑠𝑖𝑛−1 𝑥 = 𝐿𝐻𝑆
3. In RHS, put 𝑥 = 𝑐𝑜𝑠 𝜃 ⇒ 𝜃 = 𝑐𝑜𝑠 −1 𝑥.
1 1 𝜋
Also, as ≤ 𝑥 ≤ 1 ⇒ ≤ 𝑐𝑜𝑠 𝜃 ≤ 1 ⇒ 0 ≤ 𝜃 ≤ ⇒ 0 ≤ 3𝜃 ≤ 𝜋.
2 2 3
Now, 𝑅𝐻𝑆: 𝑐𝑜𝑠 −1 ( 4𝑥 3 − 3𝑥) = 𝑐𝑜𝑠 −1( 4 𝑐𝑜𝑠 3 𝜃 − 3 𝑐𝑜𝑠 𝜃) = 𝑐𝑜𝑠 −1 𝑐𝑜𝑠 3 𝜃 = 3𝜃 = 3 𝑐𝑜𝑠 −1 𝑥 = 𝐿𝐻𝑆.

1 3
4. LHS: Let Y = 𝑡𝑎𝑛 (2 𝑠𝑖𝑛−1 4)
3 3 9 √7
Put 𝑠𝑖𝑛−1 4 = 𝑥 ⇒ 𝑠𝑖𝑛 𝑥 = 4 ⇒ 𝑐𝑜𝑠 𝑥 = √1 − 𝑠𝑖𝑛2 𝑥 = √1 − 16 = 4
…(i)
𝑥 𝑥 𝑥 𝑥
𝑥 𝑠𝑖𝑛( ) 2 𝑠𝑖𝑛( ) 𝑠𝑖𝑛( ) 2 𝑠𝑖𝑛2 ( )
2 2 2 2
Now Y = 𝑡𝑎𝑛 ( ) = 𝑥 = 𝑥 𝑥 =
2 𝑐𝑜𝑠( ) 2 𝑐𝑜𝑠( ) 𝑠𝑖𝑛( ) 𝑠𝑖𝑛 𝑥
2 2 2
√7
1−𝑐𝑜𝑠 𝑥 1−
4
⇒= = 3 [By (i)
𝑠𝑖𝑛 𝑥
4
4−√7
⇒= 3
 RHS.
1 1−𝑥
5. RHS = cos −1 ( )
2 1+𝑥
2
1 1−(√𝑥)
= 2 cos −1 ( 2 )
1+(√𝑥)
On substituting √𝑥 = 𝑡𝑎𝑛 𝜃, we get
1 1−(𝑡𝑎𝑛 𝜃)2 1 1−𝑡𝑎𝑛2 𝜃
= 2 cos −1 (1+(𝑡𝑎𝑛 𝜃)2 ) or 2 cos−1 (1+𝑡𝑎𝑛2 𝜃)
1
= 2 cos −1 (cos 2𝜃)
1
= 2 . 2𝜃
=𝜃
= tan−1 √𝑥 = RHS
√3 1 𝜋 2𝜋
6. We have [cos−1 + 𝑐𝑜𝑠 −1 (− )] = [cos−1 𝑐𝑜𝑠 + 𝑐𝑜𝑠 −1 𝑐𝑜𝑠 ]
2 2 6 3
𝜋 2𝜋 5𝜋
⇒= + = {∵ Range of principal value branch of cos−1 x is [0,𝜋]
6 3 6

KVS RO DDN S SM XII MATHEMATICS/ 2023-24(VN) Page 27


*KVS RO DEHRADUN STUDENT SUPPORT MATERIAL MATHEMATICS/ XII/2023-24(VN)*

1 1 π 𝜋 π 𝜋
8. Given 𝑐𝑜𝑠 −1 (2) − 2 𝑠𝑖𝑛−1 (− 2) = 𝑐𝑜𝑠 −1 (𝑐𝑜𝑠 3) + 2 𝑠𝑖𝑛−1 (𝑠𝑖𝑛 6 ) = 3 + 2 𝑠𝑖𝑛−1 (𝑠𝑖𝑛 ( 6 ))
𝜋 𝜋 𝜋
=( )+( )=
3 6 2
𝜋 𝜋
8.Sol: we know that, the principal value branch of sin−1 𝑥 is [− 2 , 2 ].
17𝜋 𝜋 𝜋 17𝜋
17𝜋 𝜋
Since, ∉ [− , ]. So, write = 2𝜋 + as
8 2 2 88 8
17𝜋 17𝜋 𝜋 𝜋
Now, sin−1 (sin (− 8
)) = sin (− sin ( 8 )) = sin−1 (− sin (2𝜋
−1
+ 8 )) = sin−1 (− sin ( 8 )) =
𝜋 𝜋
sin−1 (sin (− 8 )) = − 8

1 𝜋
9.Sol: We have 𝑡𝑎𝑛−1 [2 𝑐𝑜𝑠 (2 𝑠𝑖𝑛−1 2)] = 𝑡𝑎𝑛−1 [2 𝑐𝑜𝑠 (2 𝑠𝑖𝑛−1 𝑠𝑖𝑛 6 )]
𝜋 𝜋 1
⇒= 𝑡𝑎𝑛−1 [2 𝑐𝑜𝑠 (2 × 6 )] = 𝑡𝑎𝑛−1 [2 𝑐𝑜𝑠 3 ] = 𝑡𝑎𝑛−1 (2 × 2)
𝜋 𝜋
⇒= 𝑡𝑎𝑛−1 ( 1) = 𝑡𝑎𝑛−1 (𝑡𝑎𝑛 4 ) = 4 .

2𝜋 2𝜋 2𝜋 𝜋 2𝜋 𝜋 2𝜋 𝜋
10. Sol:cos −1 (cos 3
)+ sin−1 (sin 3
) = 3
+ sin−1 (sin (𝜋 − 3 )) = 3
+ sin−1 (sin 3 ) = 3
+3=𝜋

2𝜋 2𝜋 2𝜋 𝜋 2𝜋 𝜋 2𝜋 𝜋
Sol:cos−1 (cos 3
)+ sin−1 (sin 3
) = 3
+ sin−1 (sin (𝜋 − 3 )) = 3
+ sin−1 (sin 3 ) = 3
+3 =𝜋

11.Sol: Let cot −1 (−𝑥) = 𝜃 ⇒ cot 𝜃 = −𝑥 so that 𝑥 = − cot 𝜃 ⇒ 𝑥 = cot(𝜋 − 𝜃) ⇒ cot −1 𝑥 = 𝜋 − 𝜃


⇒ cot −1 𝑥 = 𝜋 − cot −1(−𝑥) ⇒ cot −1(−𝑥) = 𝜋 − cot −1 𝑥

1
12. Sol: Let 𝑦 = 𝑐𝑜𝑡 −1 , 𝑥 < −1.
√𝑥 2 −1
𝜋
Put 𝑥 = 𝑠𝑒𝑐 𝜃 ⇒ 𝜃 = 𝑠𝑒𝑐 −1 𝑥. Then 𝑠𝑒𝑐 𝜃 < −1 ⇒ 2 < 𝜃 < 𝜋 [1/2]
1 1 1
∴ 𝑦 = 𝑐𝑜𝑡 −1 = 𝑐𝑜𝑡 −1 (|𝑡𝑎𝑛 ) ⇒𝑦= 𝑐𝑜𝑡 −1 ( ) = 𝑐𝑜𝑡 −1 (− 𝑐𝑜𝑡 𝜃)
√𝑠𝑒𝑐 2 𝜃−1 𝜃| − 𝑡𝑎𝑛 𝜃
[1/2]
⇒ 𝑦 = 𝑐𝑜𝑡 −1 (𝑐𝑜𝑡( 𝜋 − 𝜃)) = 𝜋 − 𝜃 ∴ 𝑦 = 𝜋 − 𝑠𝑒𝑐 −1 𝑥.

𝑠𝑖𝑛 𝑥+𝑐𝑜𝑠 𝑥
13. Sol: Let 𝑦 = 𝑠𝑖𝑛−1 ( )
√2
1 1
⇒ 𝑦 = 𝑠𝑖𝑛−1 ( 2 𝑠𝑖𝑛 𝑥 + 2 𝑐𝑜𝑠 𝑥)
√ √
𝜋 𝜋
⇒ 𝑦 = 𝑠𝑖𝑛−1 (𝑠𝑖𝑛 𝑥 𝑐𝑜𝑠 + 𝑐𝑜𝑠 𝑥 𝑠𝑖𝑛 )
4 4
𝜋
⇒ 𝑦 = 𝑠𝑖𝑛−1 𝑠𝑖𝑛 (𝑥 + 4 )
𝜋
∴𝑦=𝑥+
4

1 1 1 1 1
15.Sol: We have 2 𝑐𝑜𝑠 −1 + 3 𝑠𝑖𝑛−1 = 2 (𝑐𝑜𝑠 −1 + 𝑠𝑖𝑛−1 ) + 𝑠𝑖𝑛−1
2 2 2 2 2
𝜋 𝜋 𝜋
⇒= 2 ( 2 ) + 𝑠𝑖𝑛−1 (𝑠𝑖𝑛 6 ) [∵ 𝑠𝑖𝑛−1 𝑥 + 𝑐𝑜𝑠 −1
𝑥= 2
𝜋 7𝜋
⇒= 𝜋 + 6 = 6
.

KVS RO DDN S SM XII MATHEMATICS/ 2023-24(VN) Page 28


*KVS RO DEHRADUN STUDENT SUPPORT MATERIAL MATHEMATICS/ XII/2023-24(VN)*

Solution of 3 Marks Questions


√1+𝑥−√1−𝑥 √1+𝑐𝑜𝑠 2𝜃−√1−𝑐𝑜𝑠 2𝜃
1. LHS : Let 𝑦 = 𝑡𝑎𝑛−1 ( 1+𝑥+ 1−𝑥) = 𝑡𝑎𝑛−1 ( ) [𝑃𝑢𝑡𝑥 = 𝑐𝑜𝑠 2 𝜃. . . (𝑖)
√ √ √1+𝑐𝑜𝑠 2𝜃+√1−𝑐𝑜𝑠 2𝜃
√2|𝑐𝑜𝑠 𝜃|−√2|𝑠𝑖𝑛 𝜃| |𝑐𝑜𝑠 𝜃|−|𝑠𝑖𝑛 𝜃|
∴ 𝑦 = 𝑡𝑎𝑛−1 ( ) = 𝑡𝑎𝑛−1 (|𝑐𝑜𝑠 )
√2|𝑐𝑜𝑠 𝜃|+√2|𝑠𝑖𝑛 𝜃| 𝜃|+|𝑠𝑖𝑛 𝜃|
1 1 3𝜋 3𝜋
𝑁𝑜𝑡𝑒 𝑡ℎ𝑎𝑡 − 2 ≤ 𝑥 ≤ 1 ⇒ − 2 ≤ 𝑐𝑜𝑠 2 𝜃 ≤ 1 ⇒ 0 ≤ 2𝜃 ≤ 4 ⇒ 0 ≤ 𝜃≤ 8
i.e., 𝜃 ∈ 𝐼𝑞𝑢𝑎𝑑𝑟𝑎𝑛𝑡.
√ √
𝑐𝑜𝑠 𝜃−𝑠𝑖𝑛 𝜃 1−𝑡𝑎𝑛 𝜃 𝜋
So 𝑦 = 𝑡𝑎𝑛−1 ( ) = 𝑡𝑎𝑛−1 ( ) = 𝑡𝑎𝑛−1 𝑡𝑎𝑛 ( − 𝜃)
𝑐𝑜𝑠 𝜃+𝑠𝑖𝑛 𝜃 1+1.𝑡𝑎𝑛 𝜃 4
𝜋 𝜋 1
⇒ 𝑦 = 4 − 𝜃 = 4 − 2 𝑐𝑜𝑠 −1 𝑥 =RHS. [𝐵𝑦(𝑖).

√1+𝑥+√1−𝑥 √1+𝑐𝑜𝑠 2𝜃+√1−𝑐𝑜𝑠 2𝜃


2. LHS : Let 𝑦 = 𝑡𝑎𝑛−1 ( ) = 𝑡𝑎𝑛−1 ( ) [𝑃𝑢𝑡𝑥 = 𝑐𝑜𝑠 2 𝜃. . . (𝑖)
√1+𝑥−√1−𝑥 √1+𝑐𝑜𝑠 2𝜃−√1−𝑐𝑜𝑠 2𝜃
√2|𝑐𝑜𝑠 𝜃|+√2|𝑠𝑖𝑛 𝜃| |𝑐𝑜𝑠 𝜃|+|𝑠𝑖𝑛 𝜃|
∴ 𝑦 = 𝑡𝑎𝑛−1 ( 2|𝑐𝑜𝑠 𝜃|− 2|𝑠𝑖𝑛 𝜃|) = 𝑡𝑎𝑛−1 (|𝑐𝑜𝑠 𝜃|−|𝑠𝑖𝑛 𝜃|)
√ √
1 1 3𝜋 3𝜋
𝑁𝑜𝑡𝑒 𝑡ℎ𝑎𝑡 − 2 ≤ 𝑥 ≤ 1 ⇒ − 2 ≤ 𝑐𝑜𝑠 2 𝜃 ≤ 1 ⇒ 0 ≤ 2𝜃 ≤ 4 ⇒ 0 ≤ 𝜃≤ 8
i.e., 𝜃 ∈ 𝐼𝑞𝑢𝑎𝑑𝑟𝑎𝑛𝑡.
√ √
𝑐𝑜𝑠 𝜃+𝑠𝑖𝑛 𝜃 1+𝑡𝑎𝑛 𝜃 𝜋
So 𝑦 = 𝑡𝑎𝑛−1 (𝑐𝑜𝑠 𝜃−𝑠𝑖𝑛 𝜃) = 𝑡𝑎𝑛−1 (1−1.𝑡𝑎𝑛 𝜃) = 𝑡𝑎𝑛−1 𝑡𝑎𝑛 ( 4 + 𝜃)
𝜋 𝜋 1
⇒ 𝑦 = + 𝜃 = + 𝑐𝑜𝑠 −1 𝑥 =RHS.
4 4 2

3. Put 𝑥 2 = 𝑐𝑜𝑠 2 𝜃𝑖𝑛 𝐿𝐻𝑆.


√1+𝑥 2 +√1−𝑥 2 √1+𝑐𝑜𝑠 2𝜃+√1−𝑐𝑜𝑠 2𝜃
LHS : 𝑡𝑎𝑛−1 ( ) = 𝑡𝑎𝑛−1 ( )
√1+𝑥 2 −√1−𝑥 2 √1+𝑐𝑜𝑠 2𝜃−√1−𝑐𝑜𝑠 2𝜃
∵ −1 < 𝑥 < 1 ⇒ 0 ≤ 𝑥 2 < 1
√2 𝑐𝑜𝑠2 𝜃+√2 𝑠𝑖𝑛2 𝜃 |𝑐𝑜𝑠 𝜃|+|𝑠𝑖𝑛 𝜃|
⇒= 𝑡𝑎𝑛−1 ( ) = 𝑡𝑎𝑛−1 (|𝑐𝑜𝑠 ) [⇒ 0 ≤ 𝑐𝑜𝑠 2 𝜃 < 1
√2 𝑐𝑜𝑠2 𝜃−√2 𝑠𝑖𝑛2 𝜃 𝜃|−|𝑠𝑖𝑛 𝜃| 𝜋 𝜋
⇒ 0 < 2𝜃 ≤ 2 ∴ 0 < 𝜃 ≤ 4
𝑐𝑜𝑠 𝜃+𝑠𝑖𝑛 𝜃 1+𝑡𝑎𝑛 𝜃 𝜋
⇒= 𝑡𝑎𝑛−1 (𝑐𝑜𝑠 𝜃−𝑠𝑖𝑛 𝜃 ) = 𝑡𝑎𝑛−1 (1−𝑡𝑎𝑛 𝜃) = 𝑡𝑎𝑛−1 𝑡𝑎𝑛 ( 4 + 𝜃)
𝜋 1
⇒= + 𝑐𝑜𝑠 −1 𝑥 2 = 𝑅𝐻𝑆.
4 2

2 2
√(𝑐𝑜𝑠𝑥+𝑠𝑖𝑛𝑥) +√(𝑐𝑜𝑠𝑥−𝑠𝑖𝑛𝑥)
√1+𝑠𝑖𝑛 𝑥+√1−𝑠𝑖𝑛 𝑥
4. 𝑳𝑯𝑺: 𝑐𝑜𝑡 −1 = 𝑐𝑜𝑡 −1 2 2
2
2 2
2
√1+𝑠𝑖𝑛 𝑥−√1−𝑠𝑖𝑛 𝑥
√(𝑐𝑜𝑠𝑥+𝑠𝑖𝑛𝑥) −√(𝑐𝑜𝑠𝑥−𝑠𝑖𝑛𝑥)
2 2 2 2

𝑥 𝑥 𝑥 𝑥
|𝑐𝑜𝑠 +𝑠𝑖𝑛 |+|𝑐𝑜𝑠 −𝑠𝑖𝑛 | 𝜋 𝑥 𝜋
⇒= 𝑐𝑜𝑡 −1 2
𝑥
2
𝑥
2
𝑥
2
𝑥 [∵ 0 < 𝑥 < 2
∴0<2< 4
|𝑐𝑜𝑠 +𝑠𝑖𝑛 |−|𝑐𝑜𝑠 −𝑠𝑖𝑛 |
2 2 2 2
𝑥 𝑥 𝑥 𝑥 𝑥
(𝑐𝑜𝑠 +𝑠𝑖𝑛 )+(𝑐𝑜𝑠 −𝑠𝑖𝑛 ) 2 𝑐𝑜𝑠 𝑥 𝑥
⇒= 𝑐𝑜𝑡 −1 2
𝑥
2
𝑥
2
𝑥
2
𝑥 = 𝑐𝑜𝑡 −1 ( 2
𝑥 ) = 𝑐𝑜𝑡 −1 𝑐𝑜𝑡 2 = 2 = 𝑹𝑯𝑺.
(𝑐𝑜𝑠 +𝑠𝑖𝑛 )−(𝑐𝑜𝑠 −𝑠𝑖𝑛 ) 2 𝑠𝑖𝑛
2 2 2 2 2

𝑥 𝑥
√2 𝑐𝑜𝑠2 2+√2 𝑠𝑖𝑛2 2
√1+𝑐𝑜𝑠 𝑥+√1−𝑐𝑜𝑠 𝑥
5. LHS : 𝑡𝑎𝑛−1 ( 1+𝑐𝑜𝑠 𝑥− 1−𝑐𝑜𝑠 𝑥) = 𝑡𝑎𝑛 −1
( )
√ √ 𝑥
√2 𝑐𝑜𝑠2 2−√2 𝑠𝑖𝑛2 2
𝑥

3𝜋 𝜋 𝑥 3𝜋
𝑥 𝑥
|𝑐𝑜𝑠 |+|𝑠𝑖𝑛 |
𝑥
− 𝑐𝑜𝑠 +𝑠𝑖𝑛
𝑥 ∵𝜋<𝑥< 2
⇒ 2
<2< 4
−1 2 2 −1 2 2
⇒= 𝑡𝑎𝑛 ( 𝑥 𝑥 ) = 𝑡𝑎𝑛 ( 𝑥 𝑥 )[ 𝑥
|𝑐𝑜𝑠 |−|𝑠𝑖𝑛 | − 𝑐𝑜𝑠 −𝑠𝑖𝑛 ∴ ∈ 𝐼𝐼𝑞𝑢𝑎𝑑.
2 2 2 2
2
𝑥
1−𝑡𝑎𝑛 𝜋 𝑥 𝜋 𝑥
⇒= 𝑡𝑎𝑛−1 ( 2
𝑥 ) = 𝑡𝑎𝑛−1 𝑡𝑎𝑛 ( 4 − 2) = 4 − 2 = 𝑅𝐻𝑆.
1+𝑡𝑎𝑛
2

𝜋
2|𝑐𝑜𝑠 |+√2|𝑠𝑖𝑛 | ∵ 𝑥 ∈ (0, 2 )
𝑥 𝑥
√1+𝑐𝑜𝑠 𝑥+√1−𝑐𝑜𝑠 𝑥 −1 √
6. LHS: 𝑡𝑎𝑛−1 { 1+𝑐𝑜𝑠 𝑥− 1−𝑐𝑜𝑠 𝑥}
= 𝑡𝑎𝑛 { 2
𝑥
2
𝑥 }[
√ √ √2|𝑐𝑜𝑠2|−√2|𝑠𝑖𝑛2| ∴ 𝑥 ∈ (0, 𝜋)
2 4
𝑥 𝑥 𝑥 𝑥
𝑐𝑜𝑠 +𝑠𝑖𝑛 1+𝑡𝑎𝑛 1+𝑡𝑎𝑛
⇒= 𝑡𝑎𝑛−1 { 𝑥2 2
𝑥} = 𝑡𝑎𝑛
−1
{ 2
𝑥} = 𝑡𝑎𝑛
−1
{ 2
𝑥}
𝑐𝑜𝑠 −𝑠𝑖𝑛 1−𝑡𝑎𝑛 1−1×𝑡𝑎𝑛
2 2 2 2
𝑥 𝜋 𝑥
⇒= 𝑡𝑎𝑛−1 ( 1) + 𝑡𝑎𝑛−1 𝑡𝑎𝑛 2 = 4 + 2 = 𝑅𝐻𝑆.

KVS RO DDN S SM XII MATHEMATICS/ 2023-24(VN) Page 29


*KVS RO DEHRADUN STUDENT SUPPORT MATERIAL MATHEMATICS/ XII/2023-24(VN)*

1 1
6. Sol Let 𝑐𝑜𝑡 −1 𝑥 = 𝛼 ⇒ cot 𝛼 = 𝑥 ⇒ 𝑠𝑖𝑛 𝛼 = ⇒ 𝛼 = 𝑠𝑖𝑛−1 ( )
√𝑥 2 +1 √𝑥 2 +1

cos[tan−1(sin(cot −1 𝑥))]
= cos[tan−1 (sin 𝛼)]
1
= cos [tan−1 (sin 𝑠𝑖𝑛−1 ( ))]
√𝑥 2 +1
1
= cos [tan−1 ( )]
√𝑥 2 +1
1 1 √𝑥 2 +1 𝑥 2 +1
Also let 𝑡𝑎𝑛−1 = 𝛽 ⇒ tan 𝛽 = ⇒ 𝑐𝑜𝑠 𝛽 = ⇒ 𝛽 = 𝑐𝑜𝑠 −1 √𝑥 2 +2
√𝑥 2 +1 √𝑥 2 +1 √𝑥 2 +2
= cos 𝛽
𝑥 2 +1
= cos (𝑐𝑜𝑠 −1 √ )
𝑥 2 +2

𝑥 2 +1
= √𝑥 2 +2
1+𝑥 2
∴ cos[tan−1(sin(cot −1 𝑥))] = √2+𝑥 2

1
8.Sol: Let 𝑦 = 𝑠𝑖𝑛 (2 𝑡𝑎𝑛−1 4) + 𝑐𝑜𝑠( 𝑡𝑎𝑛−1 2 √2) = 𝑢 + 𝑣,
1
𝑤ℎ𝑒𝑟𝑒 𝑢 = 𝑠𝑖𝑛 2 𝑡𝑎𝑛−1 𝑎𝑛𝑑 𝑣 = 𝑐𝑜𝑠𝑡𝑎𝑛−1 2 √2
4
1 2
1 2. 8
∴ 𝑢 = 𝑠𝑖𝑛 (2 𝑡𝑎𝑛−1 ) = 𝑠𝑖𝑛𝑠𝑖𝑛−1 ( 4
1 2
)= 4
17 = [1½]
4 1+( ) 17
4 16
1 1
Also, 𝑣 = 𝑐𝑜𝑠𝑡𝑎𝑛−1 2 √2 [𝑃𝑢𝑡 𝑡𝑎𝑛−1 2 √2 = 𝜃 ⇒ 𝑡𝑎𝑛 𝜃 = 2√2 ⇒ 𝑐𝑜𝑠 𝜃 = 3 ∴ 𝜃 = 𝑐𝑜𝑠 −1 3
1 1
∴ 𝑣 = 𝑐𝑜𝑠 (𝑐𝑜𝑠 −1 3) = 3 [1½]
8 1 41
Now, 𝑦 = 𝑢 + 𝑣 = + = .
17 3 51
𝑥
9. 2

√3 𝜋
10. Sol: Let 𝑌 = 𝑡𝑎𝑛−1 [2 𝑠𝑖𝑛 (2 𝑐𝑜𝑠 −1 2
)] = 𝑡𝑎𝑛−1 [2 𝑠𝑖𝑛 (2 𝑐𝑜𝑠 −1 𝑐𝑜𝑠 6 )]
𝜋 𝜋 √3
= 𝑡𝑎𝑛−1 [2 𝑠𝑖𝑛 (2 × 6 )] = 𝑡𝑎𝑛−1 [2 𝑠𝑖𝑛 ( 3 )] = 𝑡𝑎𝑛−1 [2 × 2
]
𝜋 𝜋
= 𝑡𝑎𝑛−1 √3 = 𝑡𝑎𝑛−1 (𝑡𝑎𝑛 3 ) ∴ 𝑌 = 3 .

Case Study Based Question

Case Study 1
1.(a) tan−1 (1/2) 2.(b) tan−1 (4/3) 3. (𝑐) tan−1 (11/2) 4.(d) tan−1(2/5)

Case Study 2
1.(a)1139.4km, 2.(c)1937km3. (b) 577.52km 4.(b)cot−1(1)

KVS RO DDN S SM XII MATHEMATICS/ 2023-24(VN) Page 30


*KVS RO DEHRADUN STUDENT SUPPORT MATERIAL MATHEMATICS/ XII/2023-24(VN)*

CLASS TEST
GENERAL INSTRUCTION
1-All Questions are compulsory.
SECTION-A ( 8×1=8)
−1 1 1
1. The value of cos (− 2) + 2 sin−1 (2)
(a) 𝜋 (b) −𝜋/2 (c) 0 (d) None of these

13𝜋
2.The principal value of cos−1 (cos ) is
6
13𝜋 𝜋 𝜋 𝜋
(A) 6
(b) 2
(C) 3 (d) 6

3. The value of 𝑡𝑎𝑛−1 √3 − 𝑐𝑜𝑠 −1 (−1) is


(a) 0 (b) −𝜋/2 (c) 𝜋 (d) −2𝜋/3

4. The value of tan−1 (−1)


(a) 𝜋/4 (b) −𝜋/4 (c) 3𝜋/4 (d) 𝑁𝑜𝑛𝑒 𝑜𝑓 𝑡ℎ𝑒𝑠𝑒

1
5. The principal value of cos−1 (− 2) is
(a) 𝜋 (b) −𝜋/3 (c) 0 (d) 2𝜋/3

6. The principal value of cot −1(−√3) is


(a) 𝜋 (b) −𝜋/6 (c) 5𝜋/6 (d) 5𝜋/3

𝜋 1
7. The value of sin ( 3 − sin−1 (− 2)).
(a) 1 (b) −𝜋/2 (c) -1 (d) None of these

8. The value of 𝑡𝑎𝑛−1 √3 − 𝑐𝑜𝑡 −1 ( − √3).


(a) 𝜋 (b) −𝜋/2 (c) 0 (d) None of these

SECTION-B ( 4×2=8)
−1 −1 1
9. Prove that sin (2𝑥√1 − 𝑥2) = 2 cos 𝑥, ≤ 𝑥 ≤ 1.
√2

1
10. Prove that 3 cos−1 𝑥 = cos −1(4𝑥 3 − 3𝑥), 𝑥 ∈ [2 , 1]

1 1−𝑥
11. Prove that tan−1 √𝑥 = 2 cos −1 (1+𝑥), 𝑥 ∈ [0,1]
√3 1
12. Write the principal value of cos −1 ( 2 ) + cos −1 (− 2)

SECTION-C ( 3×3=9)
√1+𝑥−√1−𝑥 𝜋 1 1
13.Prove thattan−1 ( 1+𝑥+ 1−𝑥) = 4
− 2 cos −1 𝑥, − ≤ 𝑥 ≤ 1.
√ √ √2
√1+𝑥 2 +√1−𝑥 2 𝜋 1
14. Prove that tan−1 ( ) = 4 + 2 cos −1 𝑥 2 , −1 < 𝑥 < 1
√1+𝑥 2 −√1−𝑥 2
1+𝑥 2
15. Prove the following: cos[tan−1(sin(cot −1 𝑥))] = √2+𝑥 2

SECTION-B ( 1×5=5)
16. Ram , Robert andRahim are the three engineers who are working on this project . If“ A“ is considered tobe a
personviewingthehoardingboard20 metersawayfromthebuilding,standingattheedgeofapathwaynearby. Ram
,Robert andRahim suggestedtothe filmto placethe hoarding board at three different locations namely C,D and E
.“C “ is at the height of 10metres from the ground level. For the viewer A , the angle of elevation of “D “is
doublethe angle of

KVS RO DDN S SM XII MATHEMATICS/ 2023-24(VN) Page 31


*KVS RO DEHRADUN STUDENT SUPPORT MATERIAL MATHEMATICS/ XII/2023-24(VN)*

elevation of “C” .The angle of elevation of “E” is triple the angle of elevationof “C”for the same viewer .Look at
the figure given and based on the above informationanswerthe following

Based on above information answer the following questions


1. The measure of ∠𝐶𝐴𝐵 is (1)
2
(a) tan−1(1/3) (b) tan−1 (3) (𝑐) tan−1 (1/2) (d) none of these
2. The measure of ∠𝐷𝐴𝐵 is (1)
−1 3
(a) tan (4/3) (b) tan−1 (4) (𝑐) tan−1 (3) −1
(d) tan (4)
3. Find the measure of ∠𝐸𝐴𝐵 is (1.5)

4. If A’is anotherviewer standing on the same line of observation across the road. If thewidth of theroad is
5meters, then find the ∠𝐶𝐴′𝐵is (1.5)

KVS RO DDN S SM XII MATHEMATICS/ 2023-24(VN) Page 32


*KVS RO DEHRADUN STUDENT SUPPORT MATERIAL MATHEMATICS/ XII/2023-24(VN)*

TEST PAPER

Class- XII ( MLL )

Topic: Inverse Trigonometric Functions

Section - A (MCQs ) ( 1 Marks each)

Q1. The principal value of sec-1 (2/√3) is

(a) π/3 (b) π/6 (c) –π/3 (d) 2π/3

Q2. The principal value of sin-1 (1/2) + tan-1(1/√3) is

(a) π/3 (b) π/6 (c) –π/3 (d) 2π/3

Q3. The value of sin- { π/3 + sin-1(1/2) } is

(a) 1 (b) 0 (c) 2 (d) 4

Q4. The principal value of cos-1(1/2) + 2 sin-1 (1/2) is

(a) π/3 (b) π/6 (c) 0 (d) 2π/3

Q5. The value of tan-1 [2sin(2cos-1√3/2) is

(a) π/3 (b) π/6 (c) –π/3 (d) 2π/3

Q6. For What vale of x : sin-1 ½ = tan-1 x

(a) 1/√3 (b) √3 (c) 2 (d) 0

Q7. The domain of cos-1(2x-1) is

(a) [0,1] (b) [-1,1] (c) (-1,1) (d) [ 0, π]

Q8. The domain of the function f(x) = sin-1 (√x-1) is

(a) [1,2] (b) [-1,1] (c) (2,1) (d) [ 0, 1]


𝜋 1
Q9. Find the value of sin (3 - sin-1(− 2))

(a) 1/√3 (b) 1 (c) 2 (d) 0


1
Q10. Write the principal value of tan-1(1) + cos-1(-2)

(a) π/3 (b) π/6 (c) 7π/12 (d) 2π/3

Section –B (2 marks questios)

Q1. The value of tan-1 √3 + cot-1 √3 + tan-1(cos (0)) is ……………..

Q2. The value of tan-1 [2sin(2cos-1 √3/2) is ………..

KVS RO DDN S SM XII MATHEMATICS/ 2023-24(VN) Page 33


*KVS RO DEHRADUN STUDENT SUPPORT MATERIAL MATHEMATICS/ XII/2023-24(VN)*

CHAPTER – 3
MATRICES & DETERMINANTS
MINIMUM LEARNING LEVEL
Q.No. Question
1. If A and B are square matrices of the same order, then(A + B) (A – B) is equal to :
(a) A (b) B (c) 𝐴2 − 𝐵2 (d) 𝐴2 − 𝐵𝐴 + 𝐴𝐵 + 𝐵2

2. If A and B are two matrices of the order 3 × m and 3 × n, respectively, and m = n, then write the
order of matrix (5A – 2B) :
(a) 3x 3 (b) m x n (c) 3 x m (d) none of these

3 0 𝑎 3
If the matrix [2 𝑏 −1 is a skew-symmetric matrix, then the value of 𝑎, 𝑏 𝑎𝑛𝑑 𝑐 are :
𝑐 1 0

(a) A=1 , b=2 ,c=3 (b) a =-1 b = 2 , c=-3


(c ) a= -2 , b=0 , c=-3 (d) none of these
4. A matrix which is both symmetric and skew symmetric is :

(a) Row matrix (b) column matrix (c) null matrix (d) None of these
5. 𝑖
For a 2 X 2 matrix A=[𝑎𝑖𝑗 ] , whose elements are given by 𝑎𝑖𝑗 = 𝑡ℎ𝑒𝑛 𝑡ℎ𝑒 𝑣𝑎𝑙𝑢𝑒 𝑜𝑓 𝑎12 is :
𝑗
(a) 1/2 (b) 2 (c) 1/3
(d) None of these
6. 𝑐𝑜𝑠𝛼 −𝑠𝑖𝑛𝛼
The value of 𝛼 for which the matrix A= [ ] is an identity matrix will be :
𝑠𝑖𝑛𝛼 𝑐𝑜𝑠𝛼
𝜋 𝜋
(a) (b) 0 (c) (d) 𝜋
2 3
7. 2 3 𝑦 𝑧
If [ ]= [ ] then the value of ( x-y+z) is:
7 𝑥 7 −5

(a) 0 (b) -4 (c) 4 (d) 3


8. 0 2 −1
The matrix 𝐴 = [−2 0 −4 is :
1 4 0

(a) Symmetric (b) skew symmetric (c) both symmetric and skew symmetric (d)
None of these
Short Answer Type ( 2 Marks )
Q.No. Question
1 3 2
If A= [ ] then prove that 𝐴𝑇 + 𝐴 is symmetric.
5 1

2 0 1 −2
For what value of 𝑥 , is A=[−1 0 3 is a skew symmetric matrix.
𝑥 3 0

3. 2 3 1 −3 −4 6
If [ ][ ]=[ ] find the value of x .
5 7 −2 4 −9 𝑥

4. 1
If A is a 3 X 3 matrix whose elements are given by 𝑎𝑖𝑗 = 3 |−2𝑖 + 3𝑗| then find
the value of 𝑎23 + 𝑎32 − 𝑎11 .
5. 2 1
1 0 1
If A= [ 3 2 𝑎𝑛𝑑 𝐵 = [ ] then find AB .
−1 2 1
−1 1

KVS RO DDN S SM XII MATHEMATICS/ 2023-24(VN) Page 34


*KVS RO DEHRADUN STUDENT SUPPORT MATERIAL MATHEMATICS/ XII/2023-24(VN)*

Short Answer Type ( 3 Marks)

Q.No. Question Answer


1. 2 1 −2
If A = [−1 1 3 then find 𝐴2 .
2 3 0

2. 5 2 𝑎 a=4 , b=2 ,
If A= [𝑏 𝑐 −3 is a symmetric matrix find the value of a , b, c and d . c= any real
4 𝑑 −7 no. d= -3
3. 2 3
If A= [ ] , find the value of 𝐴2 − 4𝐴 + 7𝐼
−1 2
4. 0 6 7 0 1 1 2 10
Evaluate: ([−6 0 8 + [1 0 2 ) [−2 [20
7 −8 0 1 2 0 3 28

ASSERTION AND REASON BASED QUESTION:


1. Assertion (A) : Let A and B are matrices of order 3 X2 and 2 X4 respectively, then the order of matrix
AB is 3 X 4 .
5 𝑥
Reason (R ): If A= [ ] and A=𝐴′ then x=y .
𝑦 0
a) Both A and R are true and R is correct explanation of A
b) Both A and R are true and R is not correct explanation of A.
c) A is true but R is false.
d) A is false but R is true.
Select correct option . [ Ans. b]
2. Assertion (A) : if A is a skew symmetric matrix, then 𝐴2 is a symmetric matrix.
Reason (R): if a matrix A is both symmetric and skew-symmetric then matrix A is a zero matrix of order
nXn.
e) Both A and R are true and R is correct explanation of A
f) Both A and R are true and R is not correct explanation of A.
g) A is true but R is false.
h) A is false but R is true. [ Ans. b ]

KVS RO DDN S SM XII MATHEMATICS/ 2023-24(VN) Page 35


*KVS RO DEHRADUN STUDENT SUPPORT MATERIAL MATHEMATICS/ XII/2023-24(VN)*

Answers :
1Marks Questions :
(1). 𝐴2 − 𝐵𝐴 + 𝐴𝐵 + 𝐵 2 (2). 3Xm (3) a=-2 , b=0 , c=-3 (4) Null Matrix (5) ½
(6). 0 (7). -4 (8). Skew symmetric
2 Marks Questions

1 2 3
1) (2). 2 (3). 13 (4). 2 (5). [1 4 5 (6)
−2 2 0
3 Marks Questions
−1 −3 −1 10
1). [ 3 9 5 (2) a=4 , b= 2 c c= any real no. (d) -3 (3) Null Matrix (4) [20
1 5 5 28

Assertion and Reasoning :


1). B (2). B

KVS RO DDN S SM XII MATHEMATICS/ 2023-24(VN) Page 36


*KVS RO DEHRADUN STUDENT SUPPORT MATERIAL MATHEMATICS/ XII/2023-24(VN)*

MATRICES
HOTS
MULTIPLE CHOICE QUESTIONS

Q1: The number of all possible matrices of order 2x2 with each entry 5, 8 or 9 is:
(a):9 (b): 6 (c): 81 (d): 27
0 2𝑏 −2
Q2: If the matrix A=[ 3 1 3 is symmetric, the value of 3a + 2b is
3𝑎 3 −1
5
(a): 0 (b): −1 (c): (d): 1
6
Q3: If a matrix A is both symmetric and skew symmetric then matrix A necessarily a
(a) Scalar matrix (b) Diagonal matrix
(c) Null matrix (d) Identity matrix
Q4: The order of matrices P, W and Y are p× k, n× 3 and 3× k respectively. If PY + WY will be defined, then
(a): k =3, p = n (b): k is arbitrary, p =2
(c): p is arbitrary, k = 3 (d): k =2, p =3
𝛼 𝛽
Q5: Given that A = [ ] and A2 =3I,then
𝛾 −𝛼
(a). 1+ 𝛼 2+ 𝛽 𝛾 = 0 (b). 1- 𝛼 2- 𝛽 𝛾 = 0
(c). 3- 𝛼2 - 𝛽 𝛾 = 0 (d). 3 + 𝛼 2 - 𝛽 𝛾 = 0
Q6: Which of the given values of x and y make following pair of matrices equal?
3𝑥 + 7 5 0 𝑦−2
[ ] and [ ]
𝑦+1 2 − 3𝑥 8 4
7 2
(a) x = − 3 , y = 7 (b) ) x = − 3 , y = 7
7 2
(c) x = − , y = − (d) None of these
3 3
1 0 2 4
Q7: If [4 −5 −1] [0 2 1 [4 = 𝐴, then the order of A is
2 0 3 1
(a): 1× 1 (b): 3× 1 (c): 1× 3 (d): 3× 3
Q8: If A =[𝑎𝑖𝑗 ] is a square matrix of order 2 such that
1, 𝑤ℎ𝑒𝑛 𝑖 = 𝑗
𝑎𝑖𝑗 = { , Then find the value of A2
0, 𝑤ℎ𝑒𝑛 𝑖 ≠ 𝑗
1 0 1 0 1 1 0 1
(a): [ ] (b): [ ] (c): [ ] (d): [ ]
1 0 0 1 1 0 1 0
Q9: A and B square matrices of same order,
Assertion (A): (A + B)2≠A2+ 2AB + B2
Reason (R): Generally, AB ≠ BA
Choose appropriate option
(a). Both A & R are true & R is the correct explanation of A
(b). Both A & R are true but R is not the correct explanation of A
(c). A is true but R is false
(d). A is false but R is true

2 3 −2 3
Q10: Assertion (A): If A=[ ] and B=[ ], then B is the inverse of A
1 2 1 −2
Reason(R): If A is a square matrix of order 𝑚 and if there exists another square matrix B of the same order 𝑚
,such that

KVS RO DDN S SM XII MATHEMATICS/ 2023-24(VN) Page 37


*KVS RO DEHRADUN STUDENT SUPPORT MATERIAL MATHEMATICS/ XII/2023-24(VN)*

AB=BA=I, then B is called the inverse of A

Choose appropriate option


(a). Both A & R are true & R is the correct explanation of A
(b). Both A & R are true but R is not the correct explanation of A
(c. A is true but R is false
(d). A is false but R is true

VERY SHORT ANSWER TYPE QUESTIONS


0 2 0 3𝛼
Q1: If A=[ ] and kA = [ ], then find the values of k, 𝛼, and 𝛽
3 −4 2𝛽 24

Q2: For what values of x and y are the following matrices equal?
2𝑥 + 1 3𝑦 2
A= [ 2 ] and A = [𝑥 + 3 𝑦 + 2]
0 𝑦 − 5𝑦 0 −6
Q3: If A is 2 × 3 matrix such that AB and AB′ both are defined, then find the order of the matrix B

Q4: If A is a square matrix such that A2 = A, find the value of (I + A)3− 7A.

𝑐𝑜𝑠𝛼 𝑠𝑖𝑛𝛼 𝜋
Q5: If A = [ ],then find 𝛼 satisfying 0 < 𝛼 < 2 when A + AT = √2 I, where AT is transpose of A.
−𝑠𝑖𝑛𝛼 𝑐𝑜𝑠𝛼
1 −1
2 1 5
Q6: If AT= [2 3 and B=[ ] , then find A-B
0 1 −1
2 0
Q7: If A is a skew symmetric matrix, then show that A2 is a symmetric matrix
3 𝑥−1
Q8: If A =[ ] is a symmetric matrix. Find the value of x
2𝑥 + 3 𝑥+2

SHORT ANSWER TYPE QUESTIONS


3 1
Q1: If A = [ ] show that A2−5A + 7I = O. Hence find 𝐴−1
−1 2

2 1 −3 2 1 0
Q2:Find the matrix A satisfying [ ]𝐴 [ ]=[ ]
3 2 5 −3 0 1
0 1
Q3: If A = [ ], then find the real values of x and y such that (𝑥 𝐼 + 𝑦 𝐴)2 = 𝐴
−1 0
2 3
Q4: If A is a symmetric matrix and B is a skew symmetric matrix such that A + B = [ ]. Find A, B and AB
5 −1
0 2𝑦 𝑧
𝑇
Q5: Find the values of x, y ,z if 𝐴 𝐴 = 𝐼, for the matrix A = [𝑥 𝑦 −𝑧
𝑥 −𝑦 𝑧
1 0
Q6: If A = [ ], find k such that A2 – 8A + kI =O
−1 7

LONG ANSWER TYPE QUESTIONS

cosx −sinx 0
Q1: If F(x) = [ sinx cosx 0 , prove that F(x) F(y) = F(x+y)
0 0 1

KVS RO DDN S SM XII MATHEMATICS/ 2023-24(VN) Page 38


*KVS RO DEHRADUN STUDENT SUPPORT MATERIAL MATHEMATICS/ XII/2023-24(VN)*

𝑎
0 −𝑡𝑎𝑛
2
Q2. Let A=[ 𝑎 and I be identity matrix of order 2. Show that
𝑡𝑎𝑛 0
2
𝑐𝑜𝑠𝑎 −𝑠𝑖𝑛𝑎
I+A= (I-A)[ ]
𝑠𝑖𝑛𝑎 𝑐𝑜𝑠𝑎
Q3: A trust fund has Rs 30,000 that is to be invested in two different types of bonds. The first bond pays 5%
interest per year which will be given to orphanage and second bond pays 7% interest per year which will be
given to financial benefits of the trust. Using matrix multiplication, determine how to divide Rs 30,000 among
two types of bonds, if the trust fund obtains an annual total interest of (a) Rs 1800. (b) Rs 2000
0 1
Q4: If A =[ ] , show that: (𝑎 𝐼 + 𝑏𝐴)3 = 𝑎3 + 3𝑎2 𝑏 𝐴, where I is an Identity matrix of order 2.
0 0
2 3
Q5: If A = [ ], then show that A2– 4A +7I =O. Hence find A5
−1 2

CASE BASED QUESTION

Q1: Three schools A, B and C organized a mela for collecting funds for helping the rehabilitation of flood
victims. They sold handmade fans, mats and plates from recycled material at a cost of Rs. 25, Rs.100 and Rs.50
each. The number of articles sold by schools A, B and C are given below.

Article School A B C

Fans 40 25 35

Mats 50 40 50

Plates 20 30 40

Based on the above information, answer the following questions:

(i) Write a matrix P be a 3× 3 matrix represent the sale of handmade fans, mats and plates by schools.

(ii) Write a matrix Q of 3× 1 represents the sale price of given products.

(iii) Find the collected funds by all the three schools for all products.

KVS RO DDN S SM XII MATHEMATICS/ 2023-24(VN) Page 39


*KVS RO DEHRADUN STUDENT SUPPORT MATERIAL MATHEMATICS/ XII/2023-24(VN)*

ANSWER KEY
MULTIPLE CHOICE QUESTIONS
Ans1 :(c): 81
Ans2 :(d): 1
Ans3 :(c) Null matrix
Ans4 :(a): k =3, p = n
Ans5 :(c). 3- 𝛼 2 - 𝛽 𝛾 = 0
Ans6 :(d) None of these
Ans7 :(a): 1× 1
1 0
Ans8 :(b) [ ]
0 1
Ans9 :(a). Both A & R are true & R is the correct explanation of A
Ans10 :(d). A is false but R is true

VERY SHORT ANSWER TYPE QUESTIONS


Ans1: k= −6,𝛼 = −4, 𝛽 = −9
0 2 0 2𝑘
Hint: kA = k[ ] ⇒ kA = [ ]
3 −4 3𝑘 −4𝑘
0 3𝛼 0 2𝑘
ATQ: [ ]=[ ]
2𝛽 24 3𝑘 −4𝑘

Ans2: x =2 and y =3
Ans3: Order of B = 3 × 3
Hint: Let order of B be m × n
 AB is defined,
∴ No. of columns in A = No. of rows in B
⇒3=m
Order of B′ = n × m
 AB′ is defined,
∴ No. of columns in A = No. of rows in B′
⇒3=n
So, order of B = m × n = 3 × 3.

Ans4: I
Hint: (I + A)3 − 7A = I3 + A3 + 3I2A + 3IA2− 7A
= I + A2.A + 3IA + 3IA2= I + A.A + 3A + 3IA− 7A
= I + A2 + 3A + 3A− 7A
= I + A + 3A + 3A− 7A
= I + 7A− 7A = I

Ans5:𝛼 = 𝜋/4
−1 1 −3
Ans6: [ ]
−1 2 1
Ans7: Hint. AT = - A (A is skew symmetric matrix)
(A2)T = (A A)T = AT .AT = (-A)(-A) = A2
(A2)T = A2
Hence A2 is symmetric matrix.
Ans8: x = − 4

Hint: 𝑎12 = 𝑎21 ⇒ x−1 = 2x +3 ⇒ x = − 4

KVS RO DDN S SM XII MATHEMATICS/ 2023-24(VN) Page 40


*KVS RO DEHRADUN STUDENT SUPPORT MATERIAL MATHEMATICS/ XII/2023-24(VN)*

SHORT ANSWER TYPE QUESTIONS


2⁄ − 1⁄
Ans1: 𝐴−1 = [ 7 7
1⁄ 3⁄
7 7
Hint: A2−5A + 7I = O
⇒ 𝐴−1 (A2−5A + 7I) =𝐴−1 O ( Pre multiplying both sides by 𝐴−1 )
⇒ 𝐴−1 A2−5𝐴−1 A + 7𝐴−1 I) =O
⇒ A – 5I + 7𝐴−1 = O ( A 𝐴−1 = 𝐴−1 A = I )
2⁄ − 1⁄
1 0 3 1
⇒ 7𝐴−1 = 5𝐼 − 𝐴 ⇒ 7𝐴−1 = 5[ ]−[ ] ⇒ 𝐴−1 = [ 7 7
0 1 −1 2 1⁄ 3⁄
7 7
1 1
Ans2: [ ]
1 0
Let order of A be m × n
2 1 2 1
[ ] 𝐴 is defined ⇒ Number of columns of [ ] = Number of rows of A ⇒ m =2
3 2 3 2
−3 2 −3 2
𝐴[ ] is defined ⇒ Number of columns of A = Number of rows of [ ] ⇒ n =2
5 −3 5 −3
∴ Order of A is 2 × 2
𝑎 𝑏
Let A =[ ]
𝑐 𝑑
2 1 𝑎 𝑏 −3 2 1 0
Now solve [ ][ ][ ]=[ ] to get a =1, b=1,c=1 and d =0
3 2 𝑐 𝑑 5 −3 0 1
1 1 1 1
Ans3: x = , y= or x =− , y=−
√2 √2 √2 √2

Ans4: A = [2 4
], B = [
0 −1
], AB = [
4 −2
]
4 −1 1 0 −1 −4
1 1 1
Ans5: x = ± , : y = ± , : z = ±
√2 √6 √3
Ans6: k = 7
LONG ANSWER TYPE QUESTIONS
cosx −sinx 0
Ans1: Hint. Given F(x) = [ sinx cosx 0
0 0 1
cosy −siny 0 cos(x + y) −sin(x + y) 0
so F(y) = [ siny cosy 0 and F(x+y) = [ sin(x + y) cos(x + y) 0
0 0 1 0 0 1

cosx −sinx 0 cosy −siny 0


LHS: F(x). F(y) = [ sinx cosx 0 [ siny cosy 0
0 0 1 0 0 1

cosxcosy − sinxsiny −cosxsiny − sinxcosy 0


= [sinxcosy + cosxsiny −sinxsiny + cosxcosy 0
0 0 1
cos(x + y) −sin(x + y) 0
= [ sin(x + y) cos(x + y) 0 = F(x + y) = RHS
0 0 1

Ans3: (a) Rs 15000 is invested in both bonds.


(b) Investment in first bond Rs 5000 and in second bond Rs 25000

KVS RO DDN S SM XII MATHEMATICS/ 2023-24(VN) Page 41


*KVS RO DEHRADUN STUDENT SUPPORT MATERIAL MATHEMATICS/ XII/2023-24(VN)*

−118 −93
Ans5: [ ]
31 −118
Hint: A2–4A +7I =O ⇒A2= 4A − 7I (i)
A3 = AA2
A3= 𝐴(4A − 7I) = 4A2− 7A = 4(4A − 7I) − 7A ( AI =A)
A3=9A − 28 I (ii)
A5 = A3A2⇒A5 = (9A − 28I)(4A − 7I) (Substituting values of A3 and A2)
2 3 1 0 −118 −93
A5 = − 31A − 56I ⇒ A5 = −31 [ ] − 56 [ ]=[ ]
−1 2 0 1 31 −118

CASE BASED QUESTION

40 25 35 25 5250
Ans1: (i) [50 40 50 (ii) [100 (iii)[7750
20 30 40 50 5500

KVS RO DDN S SM XII MATHEMATICS/ 2023-24(VN) Page 42


*KVS RO DEHRADUN STUDENT SUPPORT MATERIAL MATHEMATICS/ XII/2023-24(VN)*

CLASS TEST -12


MATRICES
SECTION – A ( ONE MARKS QUESTION)

1 2 4 1+𝑥 1−𝑦
If [ ] =[ ] , then value of x and y is
5 7 5 𝑥+6
(a) 1,2 (b)1,3 (c) 1, -3 (d) 2 ,3
(b)
2 If A is a square matrix of order 3 ,such that |𝐴| =9, then value of |𝐴 𝐴𝑑𝑗𝐴| is
(a) 648 (b) 234 (c) 436 (d) 285

SECTION- B (2 MARKS )
3 6−𝑥 4
For what value of x , the matrix [ ] is singular ?
3−𝑥 1

4 3 1
For the matrix A =[ ] , find x and y so that 𝐴2 +xI =yA .
7 5

5 If 𝐴, 𝐵 are square matrix of same order and B is a skew-symmetric matrix, show that
𝐴′ 𝐵𝐴 is a skew-symmetric matrix

SECTION- C (3 MARKS )
6. If A and B are two symmetric matrices of same order then show that AB-BA is skew
symmetric.

7 2 −1
If A=[ ] then find the value of 𝐴2 + 2𝐴 − 3𝐼.
1 2
8 1 5 9 1
If 𝐴 = [ ] and = [ ] , find a matrix C such that 3𝐴 + 5𝐵 + 2𝐶 is a null
7 12 7 8
matrix.

9 2 1 3 2 1 0
Find the matrix A satisfying the matrix equation: [ ]𝐴[ ]=[ ].
3 2 5 3 0 1

SECTION- D ( 5 MARKS)

10 A trust fund has 30,000 that must be invested in two different types of bonds. The first
bond pays 5% interest per year, and the second bond pays 7% interest per year. Using
matrix multiplication, determine how to divide 30,000 among the two types of bonds. If
the trust fund must obtain an annual total interest (i) 1800 and (ii) 2000 .
11 3 5
If 𝐴 = [ ], then find 𝐴2 − 5𝐴 − 14𝐼. Hence, obtain 𝐴3 .
4 2

KVS RO DDN S SM XII MATHEMATICS/ 2023-24(VN) Page 43


*KVS RO DEHRADUN STUDENT SUPPORT MATERIAL MATHEMATICS/ XII/2023-24(VN)*

CLASS TEST (MM:30)

Instructions: Questions no 1 to 6 are of 2 marks, question 7 and 8 are of 4 marks and question 9 &10 are of 5
marks.
Q1: If A and B are square matrices of same order and AB = 3I, then 𝐴−1 is equal to
1 1
(a) 3B (b) B (c) 3𝐵−1 (d) 𝐵−1
3 3

Q2: If A and B are symmetric matrices of same order, then AB – BA is a


(a):Symmetric Matrix (b): Identity Matrix (c): Skew Symmetric matrix (d): Null Matrix

2 0
Q3: If [ ] = P + Q, where P is symmetric and Q is a skew symmetric matrix, then Q is equal to
5 4
2 5/2 0 −5 0 −5/2 1 −5/2
(a) [ ] (b) [ ] (c)[ ] (d) [ ]
5/2 4 5 0 5/2 0 5/2 1

2x + y 4x 7 7y − 13
Q4: .If[ ]=[ ], then the value of x and y is
5x − 7 4x y x+6
(a).x = 3, y = 1 (b) x = 2, y = 3 (c) x = 2, y = 4 (d) x = 3, y = 3

Q5: If A is a square matrix such that A2 = I, find the value of (A + I)3+(A - I)3− 7A.

3 4
−1 2 1
Q6: If AT= [−1 2 and B = [ ] , then find AT - BT
1 2 3
0 1

Q7: Express the following matrix as the sum of a symmetric and skew-symmetric matrix, and verify the result:
3 −2 −4
A = [ 3 −2 −5 .
−1 1 2
2 −1 5 2 2 5
Q8: Let A = [ ], B = [ ] and C = [ ], then find a matrix D such that 𝐶𝐷 − 𝐴𝐵 = 𝑂
3 4 7 4 3 8
1 3 2
3 2
Q9: If A = [2 0 −1 , then show that A - 4A -3A + 11 I3 = O
1 2 3
𝑎
0 −𝑡𝑎𝑛
2
Q10.Let A=[ 𝑎 and I be identity matrix of order 2. Show that
𝑡𝑎𝑛 2 0
𝑐𝑜𝑠𝑎 −𝑠𝑖𝑛𝑎
I+A= (I−A)[ ]
𝑠𝑖𝑛𝑎 𝑐𝑜𝑠𝑎

KVS RO DDN S SM XII MATHEMATICS/ 2023-24(VN) Page 44


*KVS RO DEHRADUN STUDENT SUPPORT MATERIAL MATHEMATICS/ XII/2023-24(VN)*

CHAPTER 4:DETERMINANTS
(MINIMUM LEARNING LEVEL)

Q:- Evaluate the following determinants


𝑥 −7
i) | | = x(5x+1) –[(-7)x]= 5x2+x+7x = 5x2+8x
𝑥 5𝑥 + 1
0 0
ii) 𝑠ℎ𝑜𝑤 𝑡ℎ𝑎𝑡 |𝑠𝑖𝑛100 −𝑐𝑜𝑠100 | = 1
𝑠𝑖𝑛80 𝑐𝑜𝑠80

0
|𝑠𝑖𝑛100 −𝑐𝑜𝑠100 |= 𝑠𝑖𝑛100 𝑐𝑜𝑠800 + 𝑐𝑜𝑠100 𝑠𝑖𝑛800= sin(10+80)0= sin900 = 1
𝑠𝑖𝑛80 𝑐𝑜𝑠800

iii) Find the values of x, if


2 4 2𝑥 4
| |= | |
5 1 6 𝑥
2
2-20 = 2x -24
-18 = 2x2-24
2x2= 6
X2= 3 x =±√3
Q :- solve the following and find value of x
3 𝑥 3 2
i) | |=| | ans= ±2√2
𝑥 1 4 1
3𝑥 7
ii) | | = 10 ans = 2
2 4
𝑥 + 1 𝑥 − 1 4 −1
iii) | |=| | ans= 2
𝑥+3 𝑥+2 1 3
2𝑥 5 6 5
iv) | |=| | ans = ±3
8 𝑥 8 3

Q :- for what value of x the matrix A is singular ?


1+𝑥 7
A=[ ]
3−𝑥 8
A is singular matrix so |𝐴| = 0
1+𝑥 7
| |= (1+x) 8- 7( 3-x) = 8+8x-21+7x = 15x-13 =0
3−𝑥 8
13
X= 15
Q:- if matrix A is singular matrix then find the values of unknown
2𝑥 − 1 3𝑥
i) [ ] answer:- x=2
2 4
4𝑥 − 1 6𝑥
ii) [ ] answer :- x=1
2 4

KVS RO DDN S SM XII MATHEMATICS/ 2023-24(VN) Page 45


*KVS RO DEHRADUN STUDENT SUPPORT MATERIAL MATHEMATICS/ XII/2023-24(VN)*

HOTS DETERMINANT
1. Given a square matrix A of order 3x3 such that |A| = 12. Find the value of |A adjA|.
2. If A =[𝑎 2] and |A|3 = 125, then find a
2 𝑎
3. What is the value of a11A11+ a12A12+ a13A13 with respect to determinant A of order 3
4. A and B are square matrix of order 3 such that |A| =2 and |B|= 3, then find |3AB|.
5. If for any 2 x 2 square matrix A, A(adjA) = [7 0] then write the value of |A|.
0 7
6. If A is a square matrix of order 3 and |3𝐴| = k|𝐴|, then write the value of k
2 𝜆 −3
7. Let A =[0 2 5 , then find the value of λ if A-1 exists
1 1 3
2 3 4 −6
8. Let A=| |, B=| | Then compute AB.
1 2 −2 4
9. Using matrices, solve the following system of linear equations: 2x − y + z = 3 , −x + 2y −z = − 4 and
x− y + 2z = 1
3 −1 1 1 2 2
10. If A-1 =[15 6 −5 and B =[1 3 0 find (AB)-1
5 −2 2 0 1 21
11. If a, b, c are all positive and the pth, qth, rth terms of a G.P respectively, then prove that
𝑙𝑜𝑔 𝑎 𝑝 1
|𝑙𝑜𝑔 𝑏 𝑞 1| = 0
𝑙𝑜𝑔 𝑐 𝑥 1
11. Using matrix method, solve the following system of linear equations:

𝑥 + 2𝑦 + 𝑧 = 4, 𝑥 − 𝑦 − 𝑧 = 0, 𝑥 − 3𝑦 + 𝑧 = 2. Write the system of equations in matrix


form (Ax = b), where A is the coefficient matrix, x is the column vector of
variables (x, y, z), and b is the column vector of constants on the right-hand side:
3 2 1
12. If A = [4 1 2 then find A-1 and hence solve the following system of
7 3 −3

equations 3x + 4y + 7z = 14 ,2x – y + 3z = 4, x + 2y – 3z = 0
13. Two schools A and B decided to award prizes to their students for three values Honesty, Punctuality
and Obedience. School A decided to award a total of Rs 2200 for the three values to 3, 2 and 1
students respectively, while school B decided to award Rs 3100 for the three values to 4,1 and
3 students respectively. If the three prizes per person amount to Rs 1200,using matrices, find
the award prizes for each value
14. Two institutions decided to award their employees for the three values of resourcefulness,
competence and determination in the form of prices at the rate of ₹x, ₹y, and ₹z respectively
per person. The first institution decided to award respectively 4,3 and 2 employees with a total
prize money of ₹37000 and the second institution decided to award respectively 5,3 and 4
employees with a total prize money of ₹47000.If all the three prizes per person together amount
to ₹ 12000 , then using matrix method find the values of x, y and z. What values are described in
this question?

KVS RO DDN S SM XII MATHEMATICS/ 2023-24(VN) Page 46


*KVS RO DEHRADUN STUDENT SUPPORT MATERIAL MATHEMATICS/ XII/2023-24(VN)*
SOLUTIONS

1 Given a square matrix A of order 3x3 such that |A| = 12. Find the value of |A adjA|.
SOLUTION:- Find the (adjA) of matrix A.
Compute the product of A and adjA.
Calculate the determinant of the resulting matrix.

𝑎 2
2 If A =[ ] and |A|3 = 125, then find a
2 𝑎
SOLUTION
IA|3 = 125

We can calculate the determinant of A directly:

|A| = aa - 22 = a2 - 4

Now, we can use the fact that |A|^3 = 125:

(a2 - 4)3 = 125

Take the cube root of both sides:

a2 - 4 = 5

Now, add 4 to both sides:

a2 = 9

Take the square root of both sides:

a = ±3

So, the values of 'a' can be either 3 or -3.

3 What is the value of a11A11+ a12A12+ a13A13 with respect to determinant A of order 3

a11A11 + a12A12 + a13A13 = |A|

4 A and B are square matrix of order 3 such that |A| =2 and |B|= 3, then find |3AB|.

SOLUTION
In this case:
|3AB| = (33) |A| |B|
|3AB| = 27 x 2 x 3
|3AB| = 162
So, |3AB| = 162.

7 0
5 If for any 2 x 2 square matrix A, A(adjA) = [ ] then write the value of |A|.
0 7
The product of a matrix and its (adjA) is equal to the determinant of the matrix times the identity
matrix of the same size. In this case,
A(adjA) isequal to 7 times the 2x2 identity matrix:

A(adjA) = 7 I

KVS RO DDN S SM XII MATHEMATICS/ 2023-24(VN) Page 47


*KVS RO DEHRADUN STUDENT SUPPORT MATERIAL MATHEMATICS/ XII/2023-24(VN)*

6 If A is a square matrix of order 3 and |3𝐴| = k|𝐴|, then write the value of k
SOLUTION
The determinant of a scalar multiple of a matrix is equal to the scalar raised to the power of the matrix's
order times the determinant of the original matrix. In this case:
|3A| = (33) |A|
|3A| = 27 |A|
|3A| = k|A|, so you can equate the two expressions:
27 |A| = k|A|
27 = k
So, the value of k is 27.

2 𝜆 −3
7 Let A =[0 2 5 , then find the value of λ if A-1 exists.
1 1 3
SOLUTION
|A| = 2(2X3 – 5X1) - λ(0 – 5X1) - 3(0 – 2X1)
|A| = 2 + 5λ + 6
|A| = 8 - 5λ
For the inverse of A to exist, |A| must be nonzero. Therefore:
8 - 5λ ≠ 0
Now, solve for λ:
8 - 5λ ≠ 0
5λ ≠ 8
λ ≠ 8/5
So, the value of λ for which the inverse of matrix A exists is λ ≠ 8/5

2 3 4 −6
8 Let A=| |, B=| | Then compute AB.
1 2 −2 4

Hence solve the following system of Equations : 2x+y = 4, 3x+2y =1.


SOLUTION
To find the product AB, you simply multiply the matrices as follows:
AB = A * B
AB = | 2x4 + 3(-2) 2x(-6) + 3x4 |
| 1x4 + 2(-2) 1(-6) + 2x4 |
2 0
AB = [ ]
0 2
Now, let's use the result of AB to solve the system of equations:
1. 2x + y = 4
2. 3x + 2y = 1
We can represent this system of equations using matrix notation as AX = B, where:
AX = B:
2x + y = 4 3x + 2y = 1
x=2y=0

KVS RO DDN S SM XII MATHEMATICS/ 2023-24(VN) Page 48


*KVS RO DEHRADUN STUDENT SUPPORT MATERIAL MATHEMATICS/ XII/2023-24(VN)*

9 Using matrices, solve the following system of linear equations: 2x − y + z = 3 , −x + 2y −z = − 4 and x−


y + 2z = 1

A X=B
We can solve for X by multiplying both sides by the inverse of A (if it exists). If the determinant of A is nonzero,
then the inverse exists.
First, calculate the determinant of matrix A:
|A| = 6
Since |A| ≠ 0, the inverse of A exists. Now, let's find the inverse of A. The inverse, A⁻¹, is given by:
A⁻¹ = (1/|A|) * adj(A)
To find the adjoint, we need to find the cofactor matrix of A and then transpose it.
Cofactor matrix of A: C11 = 2x2 - (-1)(-1) = 4 - 1 = 3
C12 = (-1)(2 - (-1)(-1)) = -1(2 + 1) = -3
C13 = 1(2 - (-1)(-1)) = 1(2 + 1) = 3
C21 = -1(1 - (-1)(-1)) = -1(1 + 1) = -2
C22 = 2x2 - (-1)(1) = 4 + 1 = 5
C23 = -1(1 - (-1)(-1)) = -1(1 + 1) = -2
C31 = 1(1 - 2(-1)(-1)) = 1(1 - 2) = -1
C32 = -1(2 - (-1)(-1)) = -1(2 + 1) = -3
C33 = 2x2 - (-1)(-1) = 4 - 1 = 3
Now, transpose the cofactor matrix to get adj(A):
3 −2 −1
adj(A) =[−3 5 −3
3 −3 3
Now, we can find the inverse of A:
3 −2 −1
A⁻¹ = (1/|A|) * adj(A) = (1/6) [−3 5 −3
3 −3 3
Now, we can use A⁻¹ to solve for X:
x = 8/3 y = -16/3 z = 4

3 −1 1 1 2 2
10. If A-1 =[15 6 −5 and B =[1 3 0 find (AB)-1
5 −2 2 0 1 21
Matrix B-1 is simply the inverse of matrix B:
(AB)-1 = B-1 A-1
multiply two matrices:
perform the multiplications and additions:
Simplify further:

12. If a, b, c are all positive and the pth, qth, rth terms of a G.P respectively, then prove that
𝑙𝑜𝑔 𝑎 𝑝 1
|𝑙𝑜𝑔 𝑏 𝑞 1| = 0
𝑙𝑜𝑔 𝑐 𝑥 1

KVS RO DDN S SM XII MATHEMATICS/ 2023-24(VN) Page 49


*KVS RO DEHRADUN STUDENT SUPPORT MATERIAL MATHEMATICS/ XII/2023-24(VN)*

Given that the pth, qth, and rth terms are a, b, and c respectively, we have:
a = ar(p-1) ...(1)
b = ar(q-1) ...(2)
c = ar(r-1) ...(3)
Now, take the logarithm of both sides of equations (1), (2), and (3):
log(a) = (p-1) log(r) ...(4)
log(b) = (q-1) log(r) ...(5)
log(c) = (r-1) log(r) ...(6)

13. Using matrix method, solve the following system of linear equations:

𝑥 + 2𝑦 + 𝑧 = 4, 𝑥 − 𝑦 − 𝑧 = 0, 𝑥 − 3𝑦 + 𝑧 = 2. Write the system of equations in matrix form (Ax = b), where A is


the coefficient matrix, x is the column vector of variables (x, y, z), and b is the column vector of constants on the
right-hand side:
1 2 1
A = [1 −1 −1
1 −3 1
𝑥
x = [𝑦 ]
2
4
B = [0
2

Now, we want to solve for x. To do this, we'll use the equation Ax = b and find the inverse of matrix A. The
solution is given by x = A-1B.
Calculate the inverse of matrix A, A-1.
First, calculate the determinant of A:
det(A) = 1[(-1)(1) - (-1)(-3)] - 2[(1)(1) - (-1)(1)] + 1[(1)(-3) – (1)(-1)]
det(A) = (-1 - 3) -2 (1 + 1) + (-3 +1)
det(A) = 0
Since the determinant of A is 0, the matrix A is singular, and its inverse does not exist. This means that the
system of equations does not have a unique solution.
This is the unique solution to the system of equations.

3 2 1
12. If A = [4 1 2 then find A-1 and hence solve the following system of
7 3 −3

equations 3x + 4y + 7z = 14 ,2x – y + 3z = 4, x + 2y – 3z = 0

Step 1: Calculate the inverse of matrix A


. Step 2: Use A-1 to solve the system of equations.
To find A-1, we'll first calculate the determinant of A.
det(A) = 24
Step 3: Calculate the matrix of minors (Cofactor matrix).
Step 4: Calculate the matrix of cofactors (Adjoint matrix).
The matrix of cofactors (adjoint matrix) is obtained by changing the sign of the elements in the matrix of minors
Step 5: Calculate the inverse of A using the formula A-1 = (1/det(A)) * Adj(A).

KVS RO DDN S SM XII MATHEMATICS/ 2023-24(VN) Page 50


*KVS RO DEHRADUN STUDENT SUPPORT MATERIAL MATHEMATICS/ XII/2023-24(VN)*

let's proceed to solve the system of equations using it (Step 2):


The system of equations is:
1. 3x + 4y + 7z = 14

2. 2x - y + 3z = 4

3. x + 2y - 3z = 0

We can write this system of equations in matrix form as AX = B, where:


3 4 7
A = [2 −1 3
1 2 −3
𝑥
X = [𝑦]
2
4
B = [4
0
Now, to solve for X, we can use the formula X = A-1 B:
x = -25/12 y = -55/12 z = -1/6
14. Two schools A and B decided to award prizes to their students for three values Honesty, Punctuality and
Obedience. School A decided to award a total of Rs 2200 for the three values to 3, 2 and 1 students
respectively, while school B decided to award Rs 3100 for the three values to 4,1 and 3 students
respectively. If the three prizes per person amount to Rs 1200,using matrices, find the award prizes for each
value

Let: x = Award for Honesty y = Award for Punctuality z = Award for Obedience
According to the information given:
For School A: 3x + 2y + z = 2200
For School B: 4x + y + 3z = 3100
Additionally, it is given that the three prizes per person amount to Rs 1200, so we have:
x + y + z = 1200
Now, we can represent this system of equations as a matrix equation AX = B, where:
3 2 1
A = [4 2 3
1 1 1
𝑥
X = [𝑦]
2
2200
B = [3100
1200

To solve for X (the award amounts for each value),


we can use the matrix equation X = A-1 B,
where A-1 is the inverse of matrix A.
First, calculate the inverse of matrix A
Honesty (x) = Rs 300 Punctuality (y) = Rs 2800 Obedience (z) = Rs -740

KVS RO DDN S SM XII MATHEMATICS/ 2023-24(VN) Page 51


*KVS RO DEHRADUN STUDENT SUPPORT MATERIAL MATHEMATICS/ XII/2023-24(VN)*

Note: The negative value for Obedience (-740) suggests that School A awarded a higher amount for Obedience
compared to School B. You may want to check the data for any inconsistencies.

15. Two institutions decided to award their employees for the three values of resourcefulness, competence and
determination in the form of prices at the rate of ₹x, ₹y, and ₹z respectively per person. The first institution
decided to award respectively 4,3 and 2 employees with a total prize money of ₹37000 and the second
institution decided to award respectively 5,3 and 4 employees with a total prize money of ₹47000.If all the
three prizes per person together amount to ₹ 12000 , then using matrix method find the values of x, y and z.
What values are described in this question?

Let's represent the given information using a system of equations and solve for x, y, and z using the matrix
method.
Let:
x = Rate per person for resourcefulness (in ₹)
y = Rate per person for competence (in ₹)
z = Rate per person for determination (in ₹)

According to the information given:

For the first institution:


4x + 3y + 2z = 37000 (Total prize money)

For the second institution:


5x + 3y + 4z = 47000 (Total prize money)

Additionally, it is given that the three prizes per person together amount to ₹12,000:

x + y + z = 12000 (Total prize money per person)

Now, we can represent this system of equations as a matrix equation AX = B, where:


To solve for X (the values of x, y, and z), we can use the matrix equation
X = A-1 B,
First, calculate the inverse of matrix A
x = ₹24,000 (Rate per person for resourcefulness)
y = ₹26,000 (Rate per person for competence)
z = ₹3,500 (Rate per person for determination)

KVS RO DDN S SM XII MATHEMATICS/ 2023-24(VN) Page 52


*KVS RO DEHRADUN STUDENT SUPPORT MATERIAL MATHEMATICS/ XII/2023-24(VN)*

Case Study Based Question


12. Two farmers Ramakishan and Gurucharan Singh cultivate only three varieties of rice
namely Basmati, Permal and Naura. The sale (in Rs) of these varieties of rice by both the
farmers in the month of September and October are given by the following matrices A and
B
10000 20000 30000 Ramakishan
September sale (in Rs)A = [ ] 1
50000 30000 10000 Gurucharan
5000 10000 6000 Ramakishan
October sale (in Rs)B = [ ]
20000 10000 10000 Gurucharan 1
Answer the following questions using above information: 1
(i) Compute the total sales in September and October and write answer in terms of 1
A and B.
(ii) What is the value of A23 ?
(iii) Determine the decrease in sales from September to October in terms of
A and B.
(iv) If Ramakishan receives 2% profit on gross sales, compute his profit (in Rs) for each
variety sold in October using matrix method.

ANSWERS
12.

(i) A + B

(ii) A23 = 10000

(iii) A – B

(iv) Rs 100, 200, 120.

KVS RO DDN S SM XII MATHEMATICS/ 2023-24(VN) Page 53


*KVS RO DEHRADUN STUDENT SUPPORT MATERIAL MATHEMATICS/ XII/2023-24(VN)*

CLASS TEST

Max Marks: 20 Time: 40 Min

1. If A is a 2×3 matrix such that AB and AB’ both are defined, then find the order of the 1
matrix B.

2. 0 a 3 1
If the matrix [2 b −1 is a skew symmetric matrix, find the values a, b and c.
c 1 0

3. 1
Prove that AA’ is always a symmetric matrix for any square matrix of A.

4. If A and B are square matrices, each of order 2 such that |A|=3 and |B|= - 2, then write the 1
value of |3AB|.

5. If A is a square matrix of order 3 such that |adj A| = 225, find |Aʹ|. 1

6. 2x 5 6 −2 1
If | |=| |, then find the possible value(s) of x.
8 x 7 3

7. Find the equation of the line joining A(1,3) and B(0,0) using determinants and find k if 2
D(k,0) is a point such that area of triangle ABD is 3 sq units.

8. 4 −4 8 4 2
Find A, if [1 A = [−1 2 1
3 −3 6 3

9. 1 −1 0 2 2 −4 5
Given A = [2 3 4 and B = [−4 2 −4 , find BA and use it to find the values
0 1 2 2 −1 5
of x, y, z from given equations:
x − y = 3, 2x + 3y + 4z = 17, y + 2z = 17

10. cos x − sin x 0 5


If f(x) = [ sin x cos x 0 , prove that: f(x)f(−y) = f(x − y)
0 0 1

KVS RO DDN S SM XII MATHEMATICS/ 2023-24(VN) Page 54


*KVS RO DEHRADUN STUDENT SUPPORT MATERIAL MATHEMATICS/ XII/2023-24(VN)*
CLASS TEST
Max Marks: 30 Time: 60 Min

1. If A is a square matrix of order 3 such that A2 = 2A, then find the value of |A|. 1

2. If A is a square matrix of order 3 and |A| = - 4, find |adj A|. 1

3. 2 4 2x 4 1
If | |=| |, then find the possible value(s) of x.
5 1 6 x

4. 0 1 1
If A = [ ], then find A2024 .
0 0

5. 2 0 1
If [ ] = P + Q, where P is a symmetric and Q is a skew symmetric matrix, then find Q.
5 4

6.   1 0 2
0  tan 
Let A=  2

and I    . Prove that cos 
I  A  ( I  A) 
 sin  .
 0 1  sin  cos  
 tan 0 
 2 

7. Find the equation of the line joining P(4,0) and Q(0,2) using determinants and find λ if 2
R(λ,0) is a point such that area of triangle PQR is 4 sq units.

8. Show that A =  5 3  satisfies the equation x2 – 3x – 7 = 0. Thus find A 1 . 2


  1  2

9. 3  1 0 5
Express A = 2 0 3 as the sum of a symmetric and a skew-symmetric matrix. And

1  1 2
verify it.

10. 3 1 2 5
If A = [3 2 −3 , Find A−1 . Hence, solve the system of equations :
2 0 −1
3x + 3y + 2z = 1, x + 2y = 4, 2x − 3y − z = 5

11. −5 1 3 5
Find the product of two matrices A and B, where A = [ 7 1 −5 ,
1 −1 1
1 1 2
B = [3 2 1 and use it for solving the equations:
2 1 3
x + y + 2z = 1, 3x + 2y + z = 7, 2x + y + 3z = 2

KVS RO DDN S SM XII MATHEMATICS/ 2023-24(VN) Page 55


*KVS RO DEHRADUN STUDENT SUPPORT MATERIAL MATHEMATICS/ XII/2023-24(VN)*

CHAPTER 5: CONTINUITY AND DIFFERENTIABILITY

MINIMUM LEARNING LEVEL


ONE MARK QUESTIONS

1.Examine the continuity of the function f(x) = x2 +5 at x= -1.


2.Give an example of a function which is continuous at x= 1 but not differentiable
at x= 1.

3.Differentiate, sin(logx) ,w.r.t.x.


−1 𝑥
4. Differentiate,𝑒 sin , 𝑤. 𝑟. 𝑡. 𝑥.
5. Differentiate, log(logx).

𝑐𝑜𝑠√𝑥
6.Differentiate with respect to x.
√𝑥

7. Differentiate with respect to x if 𝑦 = 3𝑥 .

8. If y = xx , find dy/dx.
9. Is sin x continuous at x = π/2 or not?
10. Differentiate y = sin (x2 + 5) w. r. t. x.
Two Marks Qs
 kx  1, x   
11.Find the unknown constant if the function is continuous f ( x)   
cos x , if x   

 k ( x 2  2), if x  0 
12. Find the unknown constant if the function is continuous f ( x)   
 3 x  1, if x  0 
𝑑𝑦
13.If 𝑥 𝑥 = 𝑦 𝑦 𝑡ℎ𝑒𝑛 𝑑𝑥

14 Find the derivative of 𝑥 𝑠𝑖𝑛𝑥+𝑐𝑜𝑠𝑥


 sin 5 x
 3x x  0

15 Find k for which the function f(x) = .  is continuous at x = 0
 k x0

1  cos 4 x
 ,x  0
16.For what value of k the function f(x) = 1 -  8 x 2 is continuous at x =0.

 k , x  0

17.Prove that y’’ + y = 0 , if y = 5 cos x – 3 Sinx .

18.Prove that the greatest integer function defined by f(x) = [ x ] is not differential at x = 1.

KVS RO DDN S SM XII MATHEMATICS/ 2023-24(VN) Page 56


*KVS RO DEHRADUN STUDENT SUPPORT MATERIAL MATHEMATICS/ XII/2023-24(VN)*

THREE Marks QUESTIONS

2 x  1, if x  2 
 
19. Find the unknown constant if the function is continuous f ( x )   a, if x  2 
 x  1, if x  2 
 
20 .Find the unknown constant if the function is continuous
2 x  1, if x  2 
 
f ( x)  k , if x  2 
3x  1, if x  2 
 
3ax  b, if x  1 
 
21. Find the unknown constant if the function is continuous f ( x)  11, if x 1 
5ax  2b, if x  1 
 
𝑑𝑦
22.If find y = a( Sin  -  Cos  ) and x= a ( Cos  +  Sin  )find 𝑑𝑥

𝑑𝑦
23. If , find 𝑑𝑥
1
24.If sin x  y, show that (1  x 2 ) y ''  xy'  0

25. y = xcosx + (cosx)x find


𝑑𝑦 𝑠𝑖𝑛2 (𝑎+𝑦)
26. If siny = x sin(a+y) ,prove that 𝑑𝑥 = 𝑠𝑖𝑛𝑎

FIVE MARKS QUESTIONS


27. Show that function f(x) defined by
𝑠𝑖𝑛𝑥
+ 𝑐𝑜𝑠𝑥, 𝑥>0
𝑥
𝑓(𝑥) = 2, 𝑥 = 0, is continuous at x=0
4(1−√1−𝑥)
{ 𝑥
, 𝑥<0
dy 1 y2
28. If 1  x 2  1  y 2 = a ( x – y ) , prove that  .
dx 1 x2

d2y
29 . Find dx 2 , if x = a { Cos t + log (tant/2)} , y = a { Sin t}.

KVS RO DDN S SM XII MATHEMATICS/ 2023-24(VN) Page 57


*KVS RO DEHRADUN STUDENT SUPPORT MATERIAL MATHEMATICS/ XII/2023-24(VN)*

Solutions
One Mark QUESTIONS
𝟏 . lim 𝑓(𝑥) = lim 𝑥 2 + 5 = 6
𝑥→−1 𝑥→−1

f(-1) = 6∴ lim 𝑓(𝑥) = 𝑓(−1) ∴ 𝑓(𝑥)𝑖𝑠 𝑐𝑜𝑛𝑡𝑖𝑛𝑢𝑜𝑢𝑠 𝑎𝑡 𝑥 = −1


𝑥→−1

2. .
f(x) = |𝑥 − 1|it iscontinuous at x = 1 but not differentiable at x= 1.
.𝑑{sin(𝑙𝑜𝑔𝑥)} 𝑑 1
3.. 𝑑𝑥
= cos(logx) 𝑑𝑥 (𝑙𝑜𝑔𝑥) = 𝑥 cos(𝑙𝑜𝑔𝑥).
−1 𝑥
𝑑 −1 𝑒 sin
4. (𝑒 sin 𝑥 ) =
𝑑𝑥 √1−𝑥 2
𝑑 1 𝑑 1
5. {log(𝑙𝑜𝑔𝑥)} = (𝑙𝑜𝑔𝑥) = 𝑥𝑙𝑜𝑔𝑥
𝑑𝑥 𝑙𝑜𝑔𝑥 𝑑𝑥

 x sin x  cos x
6 .
2x x
7. 3x log3
8. xx ( 1+ logx)
9. . Yes , being a trigonometric function sin x is continuous in its domain.
𝑑𝑦
10 . 𝑑𝑥 =2xcos(x2+5)

( 2 Marks Questions )
11. Lim 𝑓(𝑥)=Lim 𝑓(𝜋 − ℎ)=Lm 𝑘(𝜋 − ℎ) + 1 = 𝑘𝜋 + 1
𝑥→𝜋− ℎ→0 ℎ→0

Lim 𝑓(𝑥)=Lim 𝑓(𝜋 + ℎ)=Lm 𝑐𝑜𝑠(𝜋 + ℎ) = 𝑐𝑜𝑠𝜋 = −1


𝑥→+ ℎ→0 ℎ→0

𝑓(𝜋) = 𝜋𝑘 + 1
∴ 𝜋𝑘 + 1 = 𝜋𝑘 + 1 = −1
2
𝜋𝑘 = −2 ∴ 𝑘 = −
𝜋
12. Lim 𝑓(𝑥)=Lim 𝑓(0 − ℎ)=Lm 𝑘(ℎ2 + 2) = 2𝑘
𝑥→0− ℎ→0 ℎ→0

Lim 𝑓(𝑥)=Lim 𝑓(0 + ℎ)=Lm 3(0 + ℎ) + 1 = 1 𝑓(0) = 2𝑘


𝑥→+ ℎ→0 ℎ→0
1
∴ 2𝑘 = 2𝑘 = 1 ∴𝑘=
2

13. x logx = ylog y


Diff. both sides w.r.t x
1 + log x = (log y )y/ + y/
1  log x
y/ =
1  log y
14. log y = ( Sinx + Cosx)logx
Diff. both sides w.r.t x
1 dy sin x  cos x
 (cos x  sin x) log x 
y dx x
dy sin x  cos x
 y(cos x  sin x) log x 
dx x

KVS RO DDN S SM XII MATHEMATICS/ 2023-24(VN) Page 58


*KVS RO DEHRADUN STUDENT SUPPORT MATERIAL MATHEMATICS/ XII/2023-24(VN)*

15. here f (x) is a continuous function


5
lim x0 f ( x)  k
3

1  cos 4 x 2Sin 2 2 x
16. lim x0  lim x0 1
8x 2 8x2
Therefore , k =1
17. y/ = -5 Sinx – 3Cosx , y// = -5 Cosx + 3 Sin x
y// = - y and hence y// + y =0
18. As we know that [1+h] = 1 and [1-h] = 0
So LHD  RHD , so it is not differential at x =1

( 3 Marks Questions )

19. Lim 𝑓(𝑥)=Lim 𝑓(2 − ℎ)=Lm (2 − ℎ)2 = 3


𝑥→2− ℎ→0 ℎ→0

Lim 𝑓(𝑥)=Lim 𝑓(2 + ℎ)=Lm 2 + ℎ + 1 = 3


𝑥→+ ℎ→0 ℎ→0

𝑓(2) = 𝑎
∴3=3=𝑎 ∴𝑎=3

20. Lim 𝑓(𝑥)=Lim 𝑓(2 − ℎ)=Lm (4 + 2ℎ) + 1 = 5


𝑥→2+ ℎ→0 ℎ→0

Lim 𝑓(𝑥)=Lim 𝑓(2 + ℎ)=Lm (6 + 3ℎ − 1) = 5


𝑥→2+ ℎ→0 ℎ→0

𝑓(2) = 𝑘 ∴𝑘=5
21. Lim 𝑓(𝑥)=Lim 𝑓(1 − ℎ)=Lm 5𝑎(1 − ℎ) − 2𝑏 = 5𝑎 − 2𝑏
𝑥→2− ℎ→0 ℎ→0

Lim 𝑓(𝑥)=Lim 𝑓(1 + ℎ)=Lm 3𝑎(1 + ℎ) + 𝑏 = 3𝑎 + 𝑏


𝑥→+ ℎ→0 ℎ→0

5a – 2b = 3a + b = 11
a = 3 , b= 2
𝑑𝑦 𝑑𝑥 𝑑𝑦
22. .𝑑𝜃 = a𝜃 sin 𝜃 , 𝑑𝜃
= a θcos 𝜃 , 𝑑𝑥
= tan 𝜃

23. Let u = xy , v = yx
𝑑𝑢 𝑑𝑦
= y.𝑥 𝑦−1 + 𝑥 𝑦 log x ,
𝑑𝑥 𝑑𝑥

𝑑𝑣 𝑑𝑦
= 𝑥𝑦 𝑥−1 + 𝑦 𝑥 log y
𝑑𝑥 𝑑𝑥

𝑑𝑦 𝑦 𝑥 𝑦−1 + 𝑦 𝑥 log 𝑥
𝑑𝑥
= 𝑥 𝑦 .𝑙𝑜𝑔𝑥+ 𝑥𝑦𝑥−1
.

KVS RO DDN S SM XII MATHEMATICS/ 2023-24(VN) Page 59


*KVS RO DEHRADUN STUDENT SUPPORT MATERIAL MATHEMATICS/ XII/2023-24(VN)*

24. sin 1 x  y,
1 1(−2𝑥)
𝑦′ = => 𝑦 ′ √1 − 𝑥 2 = 1 => 𝑦 ′′ √1 − 𝑥 2 + 𝑦 ′ =0
√1 − 𝑥 2 2√1 − 𝑥 2
=> (1  x 2 ) y ''  xy'  0

25. y =𝑒 𝑐𝑜𝑠𝑥 𝑙𝑜𝑔𝑥 + 𝑒 𝑥.log(𝑐𝑜𝑠𝑥)


𝑑𝑦 1 1
= ecosx .iogx{𝑐𝑜𝑠𝑥 . + 𝑙𝑜𝑔𝑥(−𝑠𝑖𝑛𝑥)} + 𝑒 𝑥 𝑖𝑜𝑔(𝑐𝑜𝑠𝑥) {𝑥. (−𝑠𝑖𝑛𝑥) +
𝑑𝑥 𝑥 𝑐𝑜𝑠𝑥

log(𝑐𝑜𝑠𝑥)}
1
= 𝑥 𝑐𝑜𝑠𝑥 {𝑐𝑜𝑠𝑥 . + 𝑙𝑜𝑔𝑥(−𝑠𝑖𝑛𝑥)} + (𝑐𝑜𝑠𝑥)𝑥 {𝑥. (−𝑡𝑎𝑛𝑥) + log(𝑐𝑜𝑠𝑥)}
𝑥
𝑠𝑖𝑛𝑦
26.. Consider siny = x sin(a+y) => 𝑥 =
sin(𝑎+𝑦)

𝑑𝑥 sin(𝑎 + 𝑦) 𝑐𝑜𝑠𝑦 − 𝑠𝑖𝑛𝑦. cos(𝑎 + 𝑦)


=> =
𝑑𝑦 𝑠𝑖𝑛2 (𝑎 + 𝑦)
sin(𝑎 + 𝑦 − 𝑦)
=
𝑠𝑖𝑛2 (𝑎 + 𝑦)
𝑑𝑦 𝑠𝑖𝑛2 (𝑎+𝑦)
=> =
𝑑𝑥 𝑠𝑖𝑛𝑎
(05 MARKS QUESTIONSS)

27 .Solving LHL = 2.
Solving RHL = 2
Also f(0) = 2
LHL = RHL = f(0)
f(x) is continuous at x = 2.
28 .Put x = sin α, y = sin β
Cosα + cos β = a ( sin α – sin β )
  
 = a  sin x – sin y = 2 cot a
-1 -1 -1
cot 
 2 

dy 1 y2
 
dx 1  x2
29.

dx sec 2 t / 2 dy
 a(S int  .1/ 2 ) = a ( Cos2t/Sint) annd  aCost
dt tan t / 2 dt
𝑑2𝑦 𝑑𝑡
Hence dy/dx = tant = 𝑠𝑒𝑐 2 𝑡 =
𝑑𝑥 2 𝑑𝑥

KVS RO DDN S SM XII MATHEMATICS/ 2023-24(VN) Page 60


*KVS RO DEHRADUN STUDENT SUPPORT MATERIAL MATHEMATICS/ XII/2023-24(VN)*

HOTS
Q 1.The set of points where the function f given by f (x) =| 2x – 1| sin x is differentiable is
1
(a) R (b) R -
2
(c) (0, ∞) (d) None of these

1−𝑥 2 𝑑𝑦
Q 2.If y = log , then is equal to
1+𝑥 2 𝑑𝑥
4𝑥 3 −4𝑥 1 −4𝑥 3
(a) (b) (c) (d)
1−𝑥 4 1−𝑥 4 4−𝑥 4 1−𝑥 4

Q 3.Let f(x) = |sin x| Then

(a) f is everywhere differentiable


(b) f is everywhere continuous but not differentiable at x = nπ, n ∈ Z
𝜋
(c) f is everywhere continuous but no differentiable at x = (2n + 1) n ∈ Z
2
(d) None of these

𝑑𝑦
Q 4.If y = √𝑠𝑖𝑛𝑥 + 𝑦 𝑡ℎ𝑒𝑛 is equal to
𝑑𝑥

𝑐𝑜𝑠𝑥 𝑐𝑜𝑠𝑥 𝑠𝑖𝑛𝑥 𝑠𝑖𝑛𝑥


(a) (b) (c) (d)
2𝑦−1 1−2𝑦 1−𝑥𝑦 2𝑦−1

Q 5.The derivative of cos-1 (2x² – 1) w.r.t cos-1 x is


−1 2
(a) 2 (b) ( 2
) (c) ( ) (d) 1 – x²
2√1−𝑥 𝑥

𝑑2𝑦
Q 6.If x = t², y = t³, then ( )
𝑑𝑥 2
3 3 3 3
(a (b) ( ) (c) ( ) (d) ( )
2 4𝑡 2𝑡 4𝑡
√4+𝑥−2
Q 7.If 𝑓 (𝑥) = ( ) 𝑥 ≠ 0 be continuous at 𝑥 = 0, 𝑡ℎ𝑒𝑛𝑓 (0) =
𝑥
1 1 3
(a) (b) ( ) (c) 2 (d) ( )
2 4 2

𝑙𝑖𝑚 𝑥𝑓(2)−2𝑓(𝑥)
Q 8.let 𝑓 (2) = 4 𝑡ℎ𝑒𝑛 𝑓”(2) = 4 𝑡ℎ𝑒𝑛 𝑥→2 ( )is given by
𝑥−2

(a) 2 (b) -2 (c) -4 (d) 3

𝑙𝑖𝑚 𝑓(𝑥)−𝑓(1)
Q 9.If 𝑓 (𝑥) = (√25 − 𝑥 2 ), 𝑡ℎ𝑒𝑛 𝑥→2 ( )is equal to
𝑥−1
1 1 1
(a) (b) (c) –√24 (d)
24 5 √24

𝑑𝑦
Q 10.If y = x tan y, then =
𝑑𝑥
𝑡𝑎𝑛𝑥 𝑦 𝑡𝑎𝑛𝑦 𝑡𝑎𝑛𝑥
(a) (b) (c) (d)
𝑥−𝑥 2 −𝑦 2 𝑥−𝑥 2 −𝑦 2 𝑦−𝑥 𝑥−𝑦 2

KVS RO DDN S SM XII MATHEMATICS/ 2023-24(VN) Page 61


*KVS RO DEHRADUN STUDENT SUPPORT MATERIAL MATHEMATICS/ XII/2023-24(VN)*

𝑑𝑦
Q 11 .If y = (1 + x) (1 + x²) (1 + x4) …….. (1 + x2n), then the value of 𝑎𝑡𝑥 = 0 is
𝑑𝑥
(a) 0 (b) -1 (c) 1 (d) None of these
𝑥−𝑦 𝑑𝑦
Q 12 .If 𝑠𝑒𝑐 ( ) = 𝑎𝑡ℎ𝑒𝑛 is
𝑥+𝑦 𝑑𝑥
𝑦 𝑥 𝑥 𝑦
(a) – (b) (c) – (d)
𝑥 𝑦 𝑦 𝑥

𝑠𝑖𝑛𝑥+𝑐𝑜𝑠𝑥 𝑑𝑦
Q 13 .𝐼𝑓𝑦 = 𝑡𝑎𝑛−1 ( ) 𝑡ℎ𝑒𝑛 is equal to
𝑐𝑜𝑥−𝑠𝑖𝑛𝑥 𝑑𝑥
1 𝜋
(a) (b) (c) 0 (d) 1
2 4

𝑑𝑦
Q 14 .If 𝑠𝑖𝑛𝑦 + 𝑒 −𝑥𝑐𝑜𝑠𝑦 = 𝑒, 𝑡ℎ𝑒𝑛 𝑎𝑡 (1, 𝜋) is equal to
𝑑𝑥
(a) sin y (b) -x cos y (c) e (d) sin y – x cos y

1
x 𝑥−1 2 𝑑𝑦
Q 15 .𝐼𝑓𝑦 = 𝑙𝑜𝑔 [𝑒 ( ) ] , 𝑡ℎ𝑒𝑛 is equal to
𝑥−2 𝑑𝑥
3 3
(a) 7 (b) (c) (𝑥−1) (d) None of these
𝑥−2

KVS RO DDN S SM XII MATHEMATICS/ 2023-24(VN) Page 62


*KVS RO DEHRADUN STUDENT SUPPORT MATERIAL MATHEMATICS/ XII/2023-24(VN)*

Long Answer Type Qs

√1+𝑘𝑥−√1−𝑘𝑥
, 𝑖𝑓 − 1 ≤ 𝑥 < 0
16. Find the value of k, for which𝑓(𝑥) = { 𝑥 is continuous at x = 0.
2𝑥+1
, 𝑖𝑓 0 ≤ 𝑥 < 1
𝑥−1

17. Discuss the continuity of the following function at x = 0:

𝑥 4 +2𝑥 3 +𝑥 2
, 𝑖𝑓𝑥 ≠ 0,
f(x) = { 𝑡𝑎𝑛−1 𝑥
0, 𝑖𝑓𝑥 = 0

18. Find all points of discontinuity of f, where f is defined as

|𝑥 | + 3, 𝑖𝑓 𝑥 ≤ −3
f(x) = {−2𝑥, 𝑖𝑓 − 3 < 𝑥 < 3
6𝑥 + 2, 𝑖𝑓 𝑥 ≥ 3

3𝑥 − 2 , 0 < 𝑥 ≤ 1
19. Show that f(x) = 2x2 − x , 1 < x ≤ 2
{
5x − 4 , 𝑥 > 2

is continuous at x = 2 but not differentiable.

20. Find the value of ‘a’ for which the function f defined as

𝜋
𝑎 𝑠𝑖𝑛 (𝑥 + 1), 𝑖𝑓𝑥 ≤ 0
2
f(x) = { 𝑡𝑎𝑛 𝑥−𝑠𝑖𝑛 𝑥 is continuous at x = 0.
, 𝑖𝑓𝑥 > 0
𝑥3

𝑦 𝑑𝑦 𝑥+𝑦
21.If 𝑙𝑜𝑔( 𝑥 2 + 𝑦 2 ) = 2 𝑡𝑎𝑛−1 ( ), then show that =
𝑥 𝑑𝑥 𝑥−𝑦

𝑑𝑦 𝑙𝑜𝑔𝑥
22.𝐼𝑓 𝑥 𝑦 = 𝑒 𝑥−𝑦 , 𝑠ℎ𝑜𝑤𝑡ℎ𝑎𝑡 = {𝑙𝑜𝑔(𝑥𝑒)}2
𝑑𝑥

𝑑𝑦 1
23.𝐼𝑓𝑥 √1 + 𝑦 + 𝑦√1 + 𝑥 = 0, −1 < 𝑥 < 1, 𝑝rove 𝑡ℎ𝑎𝑡 =−
𝑑𝑥 (1+𝑥)2

24. Differentiate the following with respect to x:

√1+𝑠𝑖𝑛 𝑥+√1−𝑠𝑖𝑛 𝑥 𝜋
𝑡𝑎𝑛−1 ( ),0 < 𝑥 < .
√1+𝑠𝑖𝑛 𝑥−√1−𝑠𝑖𝑛 𝑥 2

KVS RO DDN S SM XII MATHEMATICS/ 2023-24(VN) Page 63


*KVS RO DEHRADUN STUDENT SUPPORT MATERIAL MATHEMATICS/ XII/2023-24(VN)*

Hints and Solutions


1. (b) R – ½
−4𝑥
2. (b)
1−𝑥 4

3. (b) f is everywhere continuous but not differentiable at x = nπ, n ∈ Z


𝑐𝑜𝑠𝑥
4. (a) ( )
2𝑦−1

5. (a) 2
3
6. (b) ( )
4𝑡
1
7. (b)( )
4

8. (c) -4
1
9. (d)
√24
𝑦
10. (b)
𝑥−𝑥 2 −𝑦 2

11. (c) 1
𝑦
12. (d)
𝑥

13. (d) 1
14. (c) e
15. (d) None of these
Long Answer Type

16 .Hint ForLHL multiply Nr and Dr by the conjugate of Nr


Answer: k=-1
17. Function is continuous at x=0
tan−1 𝑥
Hint Use lim =1
𝑥→𝑜 𝑥

18.Function is continuous everywhere except x=3


19 . at x=2
LHD=7, RHD=5 hence its not differentiable at x=2
−1
20. a=
2

21 . Differentiating w.r.t x and simplifying we get


𝑑𝑦 𝑥 + 𝑦
=
𝑑𝑥 𝑥 − 𝑦
22. Taking log on both the sides and differentiating we get the result

KVS RO DDN S SM XII MATHEMATICS/ 2023-24(VN) Page 64


*KVS RO DEHRADUN STUDENT SUPPORT MATERIAL MATHEMATICS/ XII/2023-24(VN)*

23. Given .𝑥 √1 + 𝑦 + 𝑦√1 + 𝑥 = 0

𝑥 √1 + 𝑦 = 𝑦√1 + 𝑥
Squaring and finding factors (x-y)(x+y+xy)=0
This gives x=y or (x+y+xy)=0
Differentiating (x+y+xy)=0 wrt x we get the result
𝑥 𝑥
24. write 1+sin x as (sin + cos )2
2 2
𝑥 𝑥
And write 1-sin x as (cos − sin )2
2 2
𝜋 𝑥
Answer : −
2 2

KVS RO DDN S SM XII MATHEMATICS/ 2023-24(VN) Page 65


*KVS RO DEHRADUN STUDENT SUPPORT MATERIAL MATHEMATICS/ XII/2023-24(VN)*

CLASS TEST

1. If y  sin x   sin 1
x dy
x find .
dx
2. Find
dy
if cos x  y  cos y  .
x

dx
1 dy y 1
3. If x  a sin t
,y  .
a cos t
, show that
dx x
4. If x  a (cos t  t sin t) and y  a (sin t – t cos t),
d2 y
find .
dx 2
5. If y  (tan –1 x) 2 , show that
(x 2  1) 2 y 2  2x (x 2  1) y 1  2

 y dy x  y
7.. If log e x 2  y 2  tan 1  , prove that 
 x dx x  y

8.. If x m . y n  x  y 
m n dy y
, prove that 
dx x

9.. If x 1  y  y 1  x  0 , for , – 1  x  1, prove that


dy 1

dx 1  x2

10. If y = log e x x  x log x dy


e
find .
dx

KVS RO DDN S SM XII MATHEMATICS/ 2023-24(VN) Page 66


*KVS RO DEHRADUN STUDENT SUPPORT MATERIAL MATHEMATICS/ XII/2023-24(VN)*

CLASS TEST
.1. The function f(x) = [x]is continuous at 1M
(a)4 ( b ) -2 ( c) 1 ( d ) 1.5
d2 y
2. If x = t2 and y = t3 then dx2 is equal to 1M
3 3 3 3t
(a)2 (b) (c) (d)
4t 2t 2
2 3
. 3. Derivative of x w.r.t x is 1M
1 2 2 3x
(a)x (b) (c) (d)
3x 3 2

3x − 5 x ≤ 3
.4. If f(x) ={ is continuous at x=3 then k is 1M
2k x >3
−2
( a )2 ( b ) 4 ( c ) 15 (d)
7

5. Assertion : f(x) = |x| not differentiable at x = 0 1M


Reason : f(x) = |x| not continuous at x = 0
( a ) Both A and R are true and R is the correct explanation of A
( b ) Both A and R are true and R is not the correct explanation of A
( c ) A is true but R is false
( d ) A is false but R is true
d2y π
6. If x  a θ  sinθ , y  a 1  cosθ , find 2
at θ  2M
dx 2
  dy π
7. If X = a cos   log tan  and y  a sin  find at θ = 4 2M
 2 dx

3ax  b, if x  1
8. If f(x) = 11 if x  1 , continuous at x = 1,find the values of a and b. 2M
5ax - 2b , if x  1

dy
9. (cos x) y  (sin y) x , then find . 3M
dx
dy π
10. Find dx at x =1, y = 4 if sin2 y + cos(xy) = k 3M

dy
11. Find dx if y = etan−1 √x 3M

KVS RO DDN S SM XII MATHEMATICS/ 2023-24(VN) Page 67


*KVS RO DEHRADUN STUDENT SUPPORT MATERIAL MATHEMATICS/ XII/2023-24(VN)*
Chapter 6-
APPLICATION OF DERIVATIVE
MINIMUM LEARNING LEVEL

Q1.An edge of a variable cube is increasing at the rate of 3cm/s. How fast is the Volume of the
cube increasing ,when the edge is 10cm long?
Q2.The radius of spherical balloon is increasing at the rate of 5cm/s.At what rate is the
surface the surface of the balloon increasing, when the radius is 10cm.
Q3.The length x of the rectangle is decreasing at the rate of 3cm/m and the width y is increasing
at the rate of 2cm/m.Find the rate of change of the perimeter and the area of the rectangle, when
x=8cm and y=6cm.
Q.4.The total cost C(x) associated with product of x units of an item is given by:
C(x) = 0.005x3 – 0.02x2 + 30x + 5000.
Find the marginal cost when 3 units are produced.
Q. 5.Find the intervals in which the function f(x) = 2x3 – 9 x2 +12x+15,is increasing and
decreasing.
Q.6.Find the intervals in which the function f(x) = 2x3+ 9 x2 +12x+20, is increasing and
decreasing.
Q.7.Find the intervals in which the function f(x) = 2x3- 9 x2 +12x- 15,is increasing and
decreasing.
Q.8.Find the intervals in which the function f(x) =2x3- 15 x2 +36x+15 ,is increasing and
decreasing.
Q.9.Find the intervals in which the function f(x) = 20 – 9x+6x2 – x3, is increasing and
decreasing.
Q.10.Find the intervals in which the function f(x) = (x – 1)3 (x – 2)2,is increasing and
decreasing.
Q.11.Find the intervals in which the function f(x) = (x – 1) (x – 2)2,is increasing and
decreasing.
Q.12.Find the intervals in which the function f(x) = sin x + cos x, for 0≤ 𝑥 ≤ 2 𝜋 ,is
increasing and decreasing.
Q.13.Find all the points of local maxima and local minima of the function
f(x)= 2x3 – 9x2 + 12x + 5.
Q.14.Determine the absolute maximum and absolute minimum values of each of the following
in stated domains.
(i)f(x) = ½ x2+ 5x + 3/2; [-6,-2].
(ii)f(x) = 3x4 – 8x3 +12x2 -48x +25; [1,3].
Q.15. Find the positive numbers whose sum is 16 and the product is maximum.

KVS RO DDN S SM XII MATHEMATICS/ 2023-24(VN) Page 68


*KVS RO DEHRADUN STUDENT SUPPORT MATERIAL MATHEMATICS/ XII/2023-24(VN)*

SOLUTIONS
Q.No.1. Solution. dx/dt=3cm/s,
Required dv/dt=dx3/dt =3x2 dx/dt.
dv/dt=3 x 100 x 3 =900 cubic cm /s.
Q.No.2. Solution. dr/dt =5cm/s,
dA/dt =d (4𝜋r2)/dt =8πr dr/dt =8𝜋 x 10 x 5 =400 sq cm/s.
Q.No.3. Solution. dx/dt = -3cm/s, dy/dt = 2cm/s.
dP/dt = d 2(x + y)/dt = 2(dx/dt +dy/dt) = 2( -3 + 2) =-2 cm/s.
dA/dt=d(xy)/dt=x dy/dt +y dx/dt = x 2 + y (-3) = 16 -18 = -2sq.cm/s.
Q.No.4. Solution: C(x) = 0.005x3 – 0.02x2 + 30x + 5000.
Find the marginal cost when 3 units are produced.
dc/dt =0.015 x2 -0.04 x +30.
=0.015 x 9 -0.04 x 3 +30
=0.135 – 0.12 +30 =30.135 – 0.12 =30.015.
Q.No.5. Solution. f’(x)= 6 x2 -18x +12 =6(x2 -3x + 2) =6(x-2) (x -1).
x= 1,2 and the intervals are (-∞, 1),(1,2), (2,∞)
f’(x)>0,in intervals (-∞, 1) &(2,∞),f(x)is st increasing in these intervals. f’(x)<0,in
(1,2). It is St decreasing in (1,2).
Q.No.6. Solution.f’(x) =6(x+1) (x+2), gives turning points x = -2,-1,and
Intervals are (-∞,-2),( -2,-1) and ( -1,∞)
f(x)is St increasing in (-∞,-2) &( -1,∞),and St decreasing in (-2,-1).
Q.No.7. Solution.f’(x)=6(x-1)(x-2) and intervals are (-∞, 1),(1,2) (2,∞).
f(x)is St increasing in (-∞, 1)&(2,∞), decreasing in (1,2).
Q.No.8. Solution.f’(x) =6(x-3) (x-2) and intervals are (-∞, 2),(2,3) (3,∞)
The f(x) is St increasing in (-∞, 2),(3,∞) and St decreasing in (2,3).
Q.No.9. Solution. f’(x) = -3 (x-1) (x-3) and the intervals are(-∞, 1),(1,3) (3,∞).
The f(x) is St increasing in (1,3) and St decreasing in (-∞,1),(3,∞).
Q.No. 10. Solution. f’(x)=(x-1)2 (x-2)(5x-8)and Intevals are (−∞,1),(1,8/5),(8/5,2) and (2,∞).
The f(x) is St increasing in R.
Q.No.11. Solution.f’(x)=( x-2) (3x – 4) and the intevals are (-∞,4/3) , (4/3,2) and (2,∞). So f(x) is St
increasing in R.
Q.No.12. Solution.f’(x) =cos x – sin x and the intervals are (0,𝜋/4), (π/4, 5π/4 ) and ( 5π/4, 2π). The
f(x) is St increasing in (0,𝜋/4) and ( 5π/4, 2π) and St deceasing in ((π/4, 5π/4 ).
Q.No.13. Solution.f’(x) = 6 (x-1 )(x-2). Turning points x=1 and 2.
f’’(x) =12x-18. & f’’(1) = -6.f’’(2) =6.
So the x=1 is the point of maxima & x=2 is the point of minima.

KVS RO DDN S SM XII MATHEMATICS/ 2023-24(VN) Page 69


*KVS RO DEHRADUN STUDENT SUPPORT MATERIAL MATHEMATICS/ XII/2023-24(VN)*

Q.No.14(i). Solution. f’(x) = x + 5,gives x=-5, at f’(x) =0.


f(-6) =99/2, f(-2) = 27/2, and f(-5) =39.
Ans. Absolute maximum value =99/2 & Minimum is 27/2.
Q.No.14(ii). Solution. f’(x) = 12(x-2) ( x2 +2), gives x=2 as critical point.
Now f(2)= -39. f(1)= -16 and f(3) = 16.
Absolute maximum value =16. And Minimum value= - 39
Q.No.15. Solution. We have, x +y = 16 & P = xy,
P(x) = x (16 – x) =16x – x 2
P’(x) =16 – 2x =2 (8 - x), gives critical point x =8,

KVS RO DDN S SM XII MATHEMATICS/ 2023-24(VN) Page 70


*KVS RO DEHRADUN STUDENT SUPPORT MATERIAL MATHEMATICS/ XII/2023-24(VN)*

HOTS
3
Q1: Diameter of a sphere is 2 (2𝑥 + 5), the rate of change of its surface area with respect to x is
3 3
(a) 18𝜋(2𝑥 + 5)(b) 2 (c)9𝜋(2𝑥 + 5) (d) 4

Q2: The edgeof a cube is increasing at the rate of 0.3 cm/s, the rate of change of its surface area when
edge is 3 cm is
(a) 10.8 cm (b) 10.8c𝑚2 (c) 10.8c𝑚2 /s (d) 10.8c𝑚 /𝑠

Q3 The total revenue in Rs. received from the sale of x units of an article is given by 𝑅(𝑥) = 3𝑥 2 +
36𝑥 + 5. The marginal revenue when x=15 is (in Rs.)
(a) 126 (b) 116 (c) 96 (d) 90

Q4 The function 𝑓(𝑥) = 𝑥 − 𝑡𝑎𝑛, 𝑥 ∈ 𝑅 is


(a) Always increasing
(b) Always decreasing
(c) Neither increasing nor decreasing
(d) None of these

𝜋 3𝜋 5𝜋
Q5 The function 𝑓(𝑥) = cos (2𝑥 + 4 ) , 𝑥 ∈ [ 8 , ] is
8

(a) Increasing
(b) Decreasing
(c) Neither increasing nor decreasing
(d) None of these

Q6: The interval in which the function defined by 𝑓(𝑥) = cot −1 𝑥 + 𝑥increasing is
𝜋 𝜋
(a) [− 2 , 2 ]

(b) ( −𝜋, 𝜋)
(c) (−∞, ∞)
(d) None of these

Q7 The absolute maximum value of 𝑦 = 𝑥 3 − 3𝑥 + 2 in [0,2] is


(a) 4 (b) 6 (c) 2 (d) 0

Q8 If at x=1, the function 𝑓(𝑥) = 𝑥 4 + 𝑝𝑥 2 + 120𝑥 + 9 attains its maximum value on the
interval[0,2].
(a) 62 (b) -62 (c) 74 (d) -74

Q9 The minimum value of 𝑓(𝑥) = 𝑒 2𝑥 + 𝑒 −2𝑥 is


(a) 2 (b) 0 (c) 2 (d) 3

KVS RO DDN S SM XII MATHEMATICS/ 2023-24(VN) Page 71


*KVS RO DEHRADUN STUDENT SUPPORT MATERIAL MATHEMATICS/ XII/2023-24(VN)*

𝜋
Q10 Maximum value of y=sinx.cosx , 𝑥 ∈ [0, ] is
2

(a) 1/3 (b) ½ (c) 4/√3 (d) √2

Q11The side of an equilateral triangle is is increasing ate the rate of 0.5 cm/s . Find the rate of
increase of its perimeter.
Q12 The volume of a cube is increasing at the rate of 9 c𝑚3 /𝑠. How fast is its surface area increasing
when the length of an edge is 10cm?
Q13 Show that the function 𝑓(𝑥) = 4𝑥 3 − 18𝑥 2 + 27𝑥 − 7 is always increasing on R.
1
Q14 Find the intervals in which the function f given by 𝑓(𝑥) = 𝑥 3 + 𝑥 3 , 𝑥 ≠ 0 is

(i) Increasing (ii) decreasing

Q15 If the sum of a side and the hypotenuse of a right angled triangle be given, show that the area of
the triangle will be maximum if the angle between the given side and the hypotenuse be 60°.
Q 16 Find the area of the greatest rectangle that can be inscribed in an ellipse 𝑥
𝑥2 𝑦2
+ = 1.
𝑎2 𝑏 2
Q17 A window is in the form of a rectangle surmounted by a semicircular opening. The total perimeter
of the window is 10m. Find the dimensions of the window to admit maximum light through the whole
opening.
Assertion and Reason Question
In the following questions, a statement of assertion (A) is followed by a statement of reason (R ).
Choose the correct answer out of the following choices.
(a) Both A and R are true and R is the correct explanation of A.
(b) Both A and R are true but R is not the correct explanation of A.
(c) A is true but R is false.
(d) A is false but R is true.

Q18 Assertion A: The function f(x) = 𝑥 3 + 5𝑥 + 10, 𝑥 ∈ 𝑅 is always increasing.


Reason R: f’(x)>0 for 𝑥 ∈ 𝑅, for increasing function.

𝜋 𝜋
Q19 Assertion A: the function 𝑦 = −𝑐𝑜𝑠2𝑥𝑥 ∈ ( 4 , 2 ) is decreasing function.

Reason R: If dy/dx is negative in the given interval then function is decreasing function.

KVS RO DDN S SM XII MATHEMATICS/ 2023-24(VN) Page 72


*KVS RO DEHRADUN STUDENT SUPPORT MATERIAL MATHEMATICS/ XII/2023-24(VN)*

Q20 Case Study Based MCQ

A toy manufacturer wants to cut a 28 metres long wire into two pieces. One of the two pieces is to be
bent into the form of a square of side x and other into the form of a circle of radius y. Based on the
above information answer the following questions:
(i) The relation between x and y is
(a) 4x+𝜋y=28 (b) 2x+𝜋y=14(c) 4x+𝜋y=28 (d) x+𝜋y=14
(ii) If A is the total area of the circle and square, then A=
(a) (𝜋 + 4)𝑥 2 − 56𝑥 + 196
1
(b) 𝜋 [(𝜋 + 4)𝑥 2 + 56𝑥 + 196]
1
(c) [(𝜋 + 4)𝑥 2 − 56𝑥 + 196]
𝜋
1
(d) 𝜋 [(𝜋 − 4)𝑥 2 − 56𝑥 + 196]

(iii) If dA/dx=0, then y=


28 28𝜋 14𝜋 14
(a) (b) 𝜋+4 (c) 𝜋+4 (d)
𝜋+4 𝜋+4

(iv) Area A is minimum when x=


28 14 28𝜋 14𝜋
(a) (b) 𝜋+4(c) 𝜋+4 (d)𝜋+4
𝜋+4

(v) The maximum value of A is


196 196 196 196𝜋
(a) (𝜋+4)2
(b) 𝜋+4(c) 𝜋(𝜋+4) (d) 𝜋+4

KVS RO DDN S SM XII MATHEMATICS/ 2023-24(VN) Page 73


*KVS RO DEHRADUN STUDENT SUPPORT MATERIAL MATHEMATICS/ XII/2023-24(VN)*

SOLUTIONS

Sol 1: (c) 9𝜋(2𝑥 + 5)


Sol 2 (a) 10.8 cm
Sol 3 (a) 126
Sol 4: (b) Always decreasing
Sol 5 (a) Increasing
Sol 6 (c) (−∞, ∞)
Sol 7: (a) 4
Sol 8: (b) -62
Sol 9 (c) 2
Sol 10: (b) ½
Sol 11 Let the side of triangle be x, then dx/dt =0.5cm/s Perimeter=3x
dP/dt=3dx/dt=3 (0.5) =1.5cm/s
Sol 12 V=x^3 dV/dt=3x^2.dx/dt= 9 cm^3/s dx/dt=3/x^2
S=6x^2 dS/dt=12x.dx/dt = 36/x
dS/dt(x=10)=3.6cm^2/s
3 2
Sol 13 f’(x)=12𝑥 2 − 36𝑥 + 27=12 (𝑥 − 2) ≥ 0

So f(x) is always increasing


1
Sol 14 𝑓(𝑥) = 𝑥 3 + 𝑥 3 differentiating both sides w.r.t. x we get
3 3(𝑥 6 −1) 3(𝑥 2 −1)(𝑥 4 +𝑥 2 +1)
f’(x) =3𝑥 2 − 𝑥 4 = =
𝑥4 𝑥4

For critical point f’(x)=0 x=∓ 1


Case I When 𝑥 < −1, 𝑓 ′ (𝑥) > 0 Increasing on (−∞, −1)
Case II When −1 < 𝑥 < 1, 𝑓 ′ (𝑥) < 0 Decreasing on (−1,1)0
Case III When𝑥 > 1, 𝑓 ′ (𝑥) > 0 Increasing on (−∞, −1)
Hence function is increasing in (1,∞) and function is decreasing on (-∞, −1) ∪ (1, ∞).
Sol 15 𝐴 =
1
𝑥. √ℎ2 − 𝑥 2
2

A.T.Q.x+h=y

KVS RO DDN S SM XII MATHEMATICS/ 2023-24(VN) Page 74


*KVS RO DEHRADUN STUDENT SUPPORT MATERIAL MATHEMATICS/ XII/2023-24(VN)*

1 1
A=2 𝑥√(𝑦 − 𝑥)2 − 𝑥 2 = 2 𝑥 √𝑦 2 − 2𝑥 𝑦

B=A^2
𝑑𝐵 1
= (2𝑥𝑦 2 − 6𝑦𝑥 2 )
𝑑𝑥 4
𝑑𝐵
= 0 → 𝑦 = 3𝑥
𝑑𝑥
𝑑2 𝐵 1
= (2𝑦 2 − 12𝑦𝑥) < 0
𝑑𝑥 2 4
Area is maximum for y=3x
𝑥
x+h=3x h=2x ½ = ℎ = 𝑐𝑜𝑠𝜃 → 𝜃 = 60°

Sol 16 . Let ABCD is a rectangle inscribed inside ellipse. AB=2x, BC=2y.


Area of rectangle =4xy

2
16𝑏 2 𝑏 2 (𝑎2 − 𝑥 2 )
2 2
𝐵 = 𝐴 = 16𝑥 𝑦 = 2 [ ]
𝑎 𝑎2
𝑑𝐵 𝑎
=0 implies x=
𝑑𝑥 √2
𝑑2 𝐵 𝑎
< 0forx= , area is maximum.
𝑑𝑥 2 √2

Maximum area is A=2ab sq units

Sol 17 Let x be side of a rectangle and r be radius of semicircle.


∴ 2𝑥 + 2𝑦 + 𝜋𝑟 = 10
1
Area of the window A=2xr+2 𝜋𝑟 2
10
dA/dr=0 implies 𝑟 = 4+𝜋
𝑑2 𝐴 10
< 0for𝑟 = 4+𝜋m area is maximum.
𝑑𝑥 2

∴dimensions of the rectangle are 20/(4+𝜋)m, 10/(4+𝜋)m


Sol 18. (a)
Sol 19 (d)
Solution 20
1 14 28
(i) (b) (ii) (c) 𝜋 [(𝜋 + 4)𝑥 2 − 56𝑥 + 196] (iii) (d) (iv) (a) 𝜋+4
𝜋+4
196
(v) (b)𝜋+4

KVS RO DDN S SM XII MATHEMATICS/ 2023-24(VN) Page 75


*KVS RO DEHRADUN STUDENT SUPPORT MATERIAL MATHEMATICS/ XII/2023-24(VN)*

CASE STUDY

1. A rectangular hall is to be developed for a meeting of farmers in an agriculture college to


aware them for new technique in cultivation. It is given that the rectangular hall has a fixed
perimeter P as shown below.

(i) If x and y represents the length and breadth of the rectangular region, then the relation
between the variables is
2 2
(A) x + y = P (B) x + y = P (C) 2(x + y) =P (D) None of these
(ii) The area of the rectangular region expressed as a function of x as
1 1 1
(A) (P + x2) (B) (P x - 2x2) (C) (P x + 2x2) (D) None of these
2 2 2
(iii)Principal of Agriculture College is interested in maximizing the area of rectangular region.
For this to happen the value of x should be
(A) P/2 (B) P/3 (C) 2P/3 (D) P/4
(iv) To maximizing the area of rectangular region, value of y should be
(A) P/2 (B) P/3 (C) 2P/3 (D) P/4
(v) The maximum value of area of rectangular region is (A) P2/6 (B) P2/12 (C) P2/16 (D)
P2/36

2. Er. Shreshtha residing in Mumbai went to see an apartment of 3 BHK in Mumbai. The
window of the house was in the form of a rectangle surmounted by a semicircular opening
having a perimeter of the window 10 m as shown in the figure

(i) If x and y represents the length and breadth of the rectangular region, then the relation
between the variables is (A) x + y + x/2 = 10 (B) x + 2y + x/2 = 10 (C ) x + 2y + (π x)/2 = 10
(D) 2x + 2y = 10 (ii) The area of the window(A) expressed as a function of x is (A) A = x –
πx3/8– x2/2 (B) A = 5x – x2/2 – 𝜋x2/8 (C ) A = 5x – x2/2 – 3x2/8 (D) None of these
(iii) For maximum value of A, the breadth of rectangular part of window is (A) 20/(4+π)(B)
20/π (C ) 10/(π +4)(D) 5/(2 + π)(iv) The maximum area of the window is (A) 100/(4 + π )2 (B) (
200 + 50𝜋)/(4 + π )2(C ) ( 200 + 50𝜋)/(4 + π )2 (D) None of these

KVS RO DDN S SM XII MATHEMATICS/ 2023-24(VN) Page 76


*KVS RO DEHRADUN STUDENT SUPPORT MATERIAL MATHEMATICS/ XII/2023-24(VN)*

3. In order to set up a rain water harvesting system, a tank to collect rain water is to be dug. The tank
should have a square base and a capacity of 250 cu.m. The cost of land is ₹ 50 per sqm & cost of
digging increases with depth & for the whole tank , it is ₹ 400 h2, where h is the depth of tank in metres.
& x is the side of the square base of the tank in metres.
Answer the following from the above passage:
A) Find the total cost C of digging the tank in terms of x.
B) Find dC/dx.
C) Find the value of x for which cost C is minimum
OR
Check whether the Cost Function is increasing or not.

45cm
1. . 24cm

A packaging company got the orders to make open boxes of maximum volume from rectangular sheets
of dimensions 45cm x 24cm. The execution department of company suggested to cut squares of equal
side from all corners of rectangular sheet and folding up the flaps as shown.
If square of side x cm is cut from each corner, then answer the following:
(a) Volume function of the box is
(i) x(45 - 2x)(24 - 2x) cm3 (ii) x(45 - x)(24 - x) cm3
ii) (45 - 2x)(24 - 2x) cm3 (iv) (45 - x)(24 - x) cm3
(b) Find the side of the square for which volume is maximum.
(c) Find the Area of base of box. OR Find the Volume of the box.

4. Second order derivative of a function is the derivative of the first order derivative of the function.

Let y = (x) ...(i)


𝑑𝑦
Then, 𝑑𝑥 = ' (x) is called the first derivative of y or (x). It can also be written as y' or y1.

If ' (x) is differentiable, then derivative of dy/dx exists.

Again, differentiating Eq. (i) both sides w.r.t. x, we get


𝑑 𝑑𝑦 𝑑 𝑑2 𝑦 𝑑 𝑑𝑦
( ) = 𝑑𝑥 [' (x)]  𝑑𝑥 2 = '' (x) where, ( ) is called the second order derivative of y w.r.t. x
𝑑𝑥 𝑑𝑥 𝑑𝑥 𝑑𝑥
𝑑2 𝑦
and is denoted by 𝑑𝑥 2 or f" (x) or y" or D2 y or y2.

𝑑 𝑑2 𝑦 𝑑
Similarly, 𝑑𝑥 ( 𝑑𝑥 2 ) = 𝑑𝑥 {'' (x)}

𝑑3 𝑦 𝑑3 𝑦
 = "' (x) is called the third order derivative of y w.r.t. x and is denoted by or "' (x)
𝑑𝑥 3 𝑑𝑥 3

or y"' or D3 y or y3.

KVS RO DDN S SM XII MATHEMATICS/ 2023-24(VN) Page 77


*KVS RO DEHRADUN STUDENT SUPPORT MATERIAL MATHEMATICS/ XII/2023-24(VN)*

𝑑𝑛 𝑦 𝑑𝑛 𝑦
In general, = n(x) is called the nth de order derivative of y w.r.t. x and is denoted by or n (x)
𝑑𝑥 𝑛 𝑑𝑥 𝑛

or y"'... n or Dn y or yn.

On the basis of above information answer the following questions.


𝑑2 𝑦
(i) If x = at2 and y = at, then 𝑑𝑥 2 is equal to

−1 1 −1 1
(a) 4𝑎𝑡 3 (b) 4𝑎𝑡 3 (c) 𝑎𝑡 3 (d) 𝑎𝑡 3

𝑑2 𝑦
(ii) If y = x2emx, where m is a constant, then 𝑑𝑥 2 is equal to____________ OR

𝑑2 𝑦
If y = t10 + 1 and x = t8 + 1, then 𝑑𝑥 2 is equal to_____________

(iii) If y = em sin-1x, then show that (1 - x2) d2y/dx2-xdy/dx-m2y=0.

5. Let (x, y) = 0 be a function in the form of x and y. If it is not possible to express y as a function of x
in the form of y =  (x), then y is said to be an implicit function of x.

e.g. x2 + y2 = xy
𝑑𝑦
Following steps are used to find 𝑑𝑥 of implicit functions.

I. Write the given equation in x and y and then differentiate both sides of the given equation w.r.t. x.
𝑑𝑦
II. Take all terms involving on LHS and transfer the remaining terms on RHS to get equation in
𝑑𝑥
𝑑𝑦
theform of 𝑑𝑥 (x, y) = g(x, y).

𝑑𝑦 𝑑𝑦 𝑔(𝑥,𝑦)
III. Now, find 𝑑𝑥 , i.e. 𝑑𝑥 = 𝑓(𝑥,𝑦)

On the basis of above information answer the following questions.


𝑑𝑦
(i) If y = 5x2 - 9ey, then find .
𝑑𝑥
𝑑𝑦
(ii) If x4 + y3 - 3x2y = 0, then find 𝑑𝑥 .

Or
𝑑𝑦
If y = x2 + 3y2 + xy, then find 𝑑𝑥
𝑑𝑦 𝑠𝑖𝑛2 (𝑎+𝑦)
iii) 𝑠𝑖𝑛𝑦 = 𝑥 sin(𝑎 + 𝑦), prove that 𝑑𝑥 = 𝑠𝑖𝑛𝑎

6. The government of a state, which has mostly hilly area decided to have adventurous playground on
the top of hill having plane area and space for 10000 persons to sit at a time. After survey it was
decided to have rectangular play ground with a semicircular
parking at one end of play ground only as space is less. The
total perimeter of the field is measured as 1000 m as shown

KVS RO DDN S SM XII MATHEMATICS/ 2023-24(VN) Page 78


*KVS RO DEHRADUN STUDENT SUPPORT MATERIAL MATHEMATICS/ XII/2023-24(VN)*

Based on the above information answer the following.

(i) Looking at the figure (plan), find the relation between x and y

(ii) Find the Area of sports ground in terms of x

(iii) The government wants to maximise the area including parking area for this to happen, value of y is

(a) 10004+πm (b) 20004+πm(c) 5004+πm (d) 7504+πm

(iv) What is the maximum area of the sports field alone?

ANSWER KEY OF CASE STUDY

1(i) C (ii) B (iii) D (iv) D (v) D

2 (i) C (ii) B (iii) C (iv) C

3 a) C = 400h2 + 50 x2

b) Correct diff

c) C is minimum at h = 2.5 & x = 10

4. a) i) b) x = 5cm c) 490sq cm OR 2450 cucm

5i) -1/4at3

ii) emx (m2x2 + 2mx + 2m2 +2) or 5/16 t16

6i) dy/dx = 10x / 1+ 9 ey

ii) dy/dx = 6xy -4x3 / 3(y2 – x2 )

KVS RO DDN S SM XII MATHEMATICS/ 2023-24(VN) Page 79


*KVS RO DEHRADUN STUDENT SUPPORT MATERIAL MATHEMATICS/ XII/2023-24(VN)*

Class Test

Q1 If the function f(x)=𝑥 2 + 5𝑥 − 15 is increasing on ___. [1]


Q2 The critical point of 𝑓(𝑥) = 𝑠𝑖𝑛𝑥 + 𝑐𝑜𝑠𝑥 in first quadrant ____. [1]
Q3 Find the rate of change of the total surface area of a cylinder of radius r and height h, where
the radius varies. [2]
Q4: Find the local maximum or minimum value of the function𝑓 (𝑥) = 2𝑥 3 − 3𝑥 2 − 12𝑥 + 5.
[2]
Q5A ladder 10 meters long is leaning against a vertical wall. If the bottom of the ladder is pulled
horizontally away from the wall at the rate of 1.2 ,meters per second, find how fast the top of the
ladder is sliding down the wall, when the bottom is 6 meters away from the wall. [3]
Q6: Determine the intervals where the function f(x) =𝑥 3 − 6𝑥 2 + 9𝑥is decreasing. [3]
Q7 Find the maximum and minimum values of the function f(x) =4𝑥 3 − 12𝑥 2 + 6𝑥 + 2 in the
interval [0,3]. [3]
Q8 Show that the right circular cylinder of given surface and maximum volume is such that its
height is equals to diameter of the base. [5]
Q9 A rocket is launched vertically into the air. Its height (in meters above the ground) at time
t (in seconds) is given by the function ℎ(𝑡 ) = 10 + 80𝑡 − 16𝑡 2 . Determine the maximum
height reached by the rocket and the time it takes to reach that height.
[5]
Q10 A wire of length 20m is to be cut into two pieces. One of the pieces will be bent into shape of
a square and the other into shape of an equilateral triangle. Where the wire should be cut so that
the sum of the areas of the square and triangle is minimum? [5]

KVS RO DDN S SM XII MATHEMATICS/ 2023-24(VN) Page 80


*KVS RO DEHRADUN STUDENT SUPPORT MATERIAL MATHEMATICS/ XII/2023-24(VN)*

Class Test
Q.1.The value of x for which function Sin2x attains its maximum value is:
(a)π/4 (b) π/3 (c) π/6 (d) π/2.

Q.2.The function f(x) = log(sin x),is strictly increasing in the interval:


(a) ( 0,𝜋/2) (b) ( π/2, π ) (c) ( 3π/2, 2π) (d) None of these.

Q.3. The total revenue received from the sale of x units of a product is given by
R(x) = 3x2 + 36x+ 5.The marginal revenue, when x=15 is
(a) 116 (b) 96 (c) 90 (d) 126.

Q.4. Find the maximum vulue, of the function f(x) = |sin 4x + 3|.
(a) 4 (b) 3 (c) 2 (d) None of these.

Q.5. The interval in which y =x2 e -x is increasing is


(a) ( -∞ , ∞ ) (b) ( -2, 0) (c) ( 2, ∞ ) (d) (0, 2 ).

Q.6. Find the intervals in which the function f(x) = x2 - 4x +6.is


(a) strictly increasing (b) strictly decreasing.

Q.7. The radius of a circle is increasing at the rate of 0.7 cm/s. What is the rate of increase of its
circumference?

Q.8. Find the maximum value of the function 2x3 -24x +107 in the interval [1,3].

Q.9.Find the intervals in which the function f(x) = sin x + cos x, 0 ≤ x ≤ 2π, is strictly increasing or
strictly decreasing .

Q.10.A ladder 5 m long is leaning against a wall.The bottom of the ladder is pulled along the
ground,away from the wall,at the rate of 2 m/sec.How fast is its height on hall decreasing when the foot
of the ladder is 4 cm away from the wall?

Q.11.If the sum of the lengths of the hypotenuse and a side of a right-angled is given.Show that the area
of the triangle is maximum when the angle between them is π/3.

Q.No.12.A window of fixed perimeter is in the form of a rectangle surrounded by a semi-circle. The
semi-circular portion is filled with coloured glass while the rectangular part is filled with clear glass.
The clear glass transmits three times as much light per square metre as the coloured glass does. What
is the ratio of the sides of the rectangle so that the window transmits the maximum light.

Q.No.13.The cost of fuel for running bus is proportional to the square of the speed generated in km/hr.It
costs Rs 48/h when the bus is moving with a speed of 20 km/h.What is the most economical speed if the
fixed charges are Rs 108 for one hour ,over and above the running charges?

KVS RO DDN S SM XII MATHEMATICS/ 2023-24(VN) Page 81


*KVS RO DEHRADUN STUDENT SUPPORT MATERIAL MATHEMATICS/ XII/2023-24(VN)*

CHAPTER 7
INTEGRATION
( MINIMUM LEARNING LEVEL)
SECTION-A
2𝑥+5
1. ∫ 𝑑𝑥
𝑥 2 +5𝑥−7
3𝑙𝑜𝑔𝑥
2. ∫ 𝑒 4
(𝑥 + 1)−1 𝑑𝑥
𝑆𝑖𝑛2𝑥
3. ∫ 𝑑𝑥
𝑎2 𝐶𝑜𝑠 2 𝑥+𝑏2 𝑆𝑖𝑛2 𝑥
4 (sin−1 𝑥)3
4. ∫ 𝑑𝑥
√1−𝑥 2
(1+𝑙𝑜𝑔𝑥)2
5. ∫ 𝑑𝑥
𝑥
𝑑𝑥
6. ∫
𝑒 𝑥 +1
1−𝑆𝑖𝑛2𝑥
7. ∫ 𝑑𝑥
𝑥+𝐶𝑜𝑠 2 𝑥
𝐶𝑜𝑠2𝑥
8. ∫ 𝑑𝑥
(𝐶𝑜𝑠𝑥+𝑆𝑖𝑛𝑥)2
SECTION-B
4𝑥+1
1. ∫ 𝑑𝑥
𝑥 2 +3𝑥+2

(3𝑆𝑖𝑛𝑥−2)𝐶𝑜𝑠𝑥
2. ∫ 𝑑𝑥
5−𝐶𝑜𝑠 2 𝑥−4𝑆𝑖𝑛𝑥
2𝑥+5
3. ∫ 𝑑𝑥
√𝑥 2 +2𝑥+5
3𝑥+1
4. ∫ 𝑑𝑥
√5−2𝑥−𝑥 2
6𝑥−5
5. ∫ 𝑑𝑥
√3𝑥 2 −5𝑥+1
5𝑥+3
6. ∫ 𝑑𝑥
√𝑥 2 +4𝑥+10

SECTION-C
2𝑥−1
1. Evaluate. ∫ 𝑑𝑥
(𝑥−1)(𝑥+2)(𝑥−3)
𝐶𝑜𝑠𝑥
2. Evaluate. ∫ 𝑑𝑥
(2+𝑆𝑖𝑛𝑥)(3+4𝑆𝑖𝑛𝑥)
𝑥 2 +1
3. Evaluate. ∫ 𝑑𝑥
(𝑥−1)2 (𝑥+3)
𝑥
4. Evaluate. ∫ 𝑑𝑥
(𝑥−1)(𝑥 2 +4)
𝑥2
5. Evaluate. ∫ 𝑑𝑥
(𝑥 2 +1)(𝑥 2 +4)
(𝑥−1)(𝑥−2)(𝑥−3)
6. Evaluate. ∫ 𝑑𝑥
(𝑥−4)(𝑥−5)(𝑥−6)

KVS RO DDN S SM XII MATHEMATICS/ 2023-24(VN) Page 82


*KVS RO DEHRADUN STUDENT SUPPORT MATERIAL MATHEMATICS/ XII/2023-24(VN)*

SECTION-D
1 1 𝑥−4
1. ∫ 𝑒 𝑥 ( − ) 𝑑𝑥 6. ∫ 𝑒 𝑥 𝑑𝑥
𝑥 𝑥2 (𝑥−2)3

2. ∫ 𝑒 𝑥 (𝑡𝑎𝑛𝑥 − 𝑙𝑜𝑔𝑐𝑜𝑠𝑥)𝑑𝑥 7. ∫ 𝑒 𝑥
𝑥−1
𝑑𝑥
(𝑥+1)3
1 2
3. ∫ 𝑒 𝑥 ( − ) 𝑑𝑥 1 + 𝑠𝑖𝑛2𝑥
𝑥2 𝑥3
8. ∫ 𝑒 2𝑥 { }𝑑𝑥
1 + 𝑐𝑜𝑠2𝑥
𝑥
4. ∫ 𝑒 𝑥 𝑑𝑥
(1+𝑥)2 1−𝑠𝑖𝑛2𝑥
9. ∫ 𝑒 2𝑥 { }𝑑𝑥
1−𝑐𝑜𝑠2𝑥
𝑥−3
5. ∫ 𝑒 𝑥 𝑑𝑥
(𝑥−1)3 1
10. ∫ 𝑒 𝑥 (𝑙𝑜𝑔𝑥 + ) 𝑑𝑥
𝑥2

SECTION-E
𝜋 𝜋 𝜋
𝑆𝑖𝑛𝑛 𝑥 𝐶𝑜𝑠 𝑛 𝑥 𝑇𝑎𝑛𝑛 𝑥
∫02 𝑑𝑥 ∫0
2 𝑑𝑥 ∫0 2 𝑑𝑥
𝑆𝑖𝑛𝑛 𝑥+𝐶𝑜𝑠 𝑛 𝑥 𝑆𝑖𝑛𝑛 𝑥+𝐶𝑜𝑠 𝑛 𝑥 𝑇𝑎𝑛𝑛 𝑥+𝐶𝑜𝑡 𝑛 𝑥

𝜋 𝜋 𝜋
𝐶𝑜𝑡 𝑛 𝑥 𝐶𝑜𝑠𝑒𝑐 𝑛 𝑥 𝑆𝑒𝑐 𝑛 𝑥
∫02 𝑑𝑥 ∫0
2 𝑑𝑥 ∫0 2 𝑑𝑥
𝑇𝑎𝑛𝑛 𝑥+𝐶𝑜𝑡 𝑛 𝑥 𝐶𝑜𝑠𝑒𝑐 𝑛 𝑥+𝐶𝑜𝑡 𝑛 𝑥 𝑇𝑎𝑛𝑛 𝑥+𝑆𝑒𝑐 𝑛 𝑥

𝜋 𝜋
𝑑𝑥 𝑑𝑥
∫02
1+𝑇𝑎𝑛𝑛 𝑥
∫02
1+𝐶𝑜𝑡 𝑛 𝑥

𝑏 𝑆𝑖𝑛𝑛 𝑥 𝑏 𝐶𝑜𝑠 𝑛 𝑥 𝑏 𝑇𝑎𝑛𝑛 𝑥


∫𝑎 𝑆𝑖𝑛𝑛 𝑥+𝐶𝑜𝑠𝑛 𝑥 𝑑𝑥 ∫𝑎 𝑆𝑖𝑛𝑛 𝑥+𝐶𝑜𝑠𝑛 𝑥 𝑑𝑥 ∫𝑎 𝑇𝑎𝑛𝑛 𝑥+𝐶𝑜𝑡 𝑛 𝑥 𝑑𝑥

𝑏 𝐶𝑜𝑡 𝑛 𝑥 𝑏 𝐶𝑜𝑠𝑒𝑐 𝑛 𝑥 𝑏 𝑆𝑒𝑐 𝑛 𝑥


∫𝑎 𝑑𝑥 ∫𝑎 𝑑𝑥 ∫𝑎 𝑑𝑥
𝑇𝑎𝑛𝑛 𝑥+𝐶𝑜𝑡 𝑛 𝑥 𝐶𝑜𝑠𝑒𝑐 𝑛 𝑥+𝐶𝑜𝑡 𝑛 𝑥 𝑇𝑎𝑛𝑛 𝑥+𝑆𝑒𝑐 𝑛 𝑥

𝑏 𝑑𝑥 𝑏 𝑑𝑥 𝝅
∫𝑎 ∫𝑎 provided 𝒂 + 𝒃 =
1+𝑇𝑎𝑛𝑛 𝑥 1+𝐶𝑜𝑡 𝑛 𝑥 𝟐

𝑛
𝑎 √𝑥 𝑎 𝑛√𝑎−𝑥
∫0 𝑛 𝑥+ 𝑛 𝑎−𝑥 𝑑𝑥 ∫0 𝑛 𝑥+ 𝑛 𝑎−𝑥 𝑑𝑥
√ √ √ √

𝑛 𝑛
𝑏 √𝑥 𝑏 √𝑎+𝑏−𝑥
∫𝑎 𝑛 𝑥+ 𝑛√𝑎+𝑏−𝑥 𝑑𝑥 ∫𝑎 𝑛 𝑥+ 𝑛√𝑎+𝑏−𝑥 𝑑𝑥
√ √

KVS RO DDN S SM XII MATHEMATICS/ 2023-24(VN) Page 83


*KVS RO DEHRADUN STUDENT SUPPORT MATERIAL MATHEMATICS/ XII/2023-24(VN)*

𝜋 3 𝜋 3𝜋
4
𝑆𝑖𝑛2 𝑥 √𝐶𝑜𝑠𝑥 8 𝑇𝑎𝑛7 𝑥
1. ∫0 2
3 3 𝑑𝑥 2. ∫0 2
4 4 𝑑𝑥 3. ∫ 𝜋 𝑑𝑥
√𝐶𝑜𝑠𝑥 + √𝑆𝑖𝑛𝑥 𝑇𝑎𝑛7 𝑥+𝐶𝑜𝑡 7 𝑥
𝑆𝑖𝑛2 𝑥+𝐶𝑜𝑠 2 𝑥 8

𝜋 𝜋 3
3 𝑑𝑥 𝑑𝑥 5 √5−𝑥
4. ∫ 𝜋 3
1+ √𝑡𝑎𝑛𝑥
5. ∫0 2
1+𝐶𝑜𝑡 4 𝑥
6. ∫0 3 𝑥+ 3 5−𝑥 𝑑𝑥
6 √ √

𝑛 𝜋 4
8 √10−𝑥 3 𝑑𝑥 4 √5−𝑥
7. ∫2 𝑛 𝑥+ 𝑛√10−𝑥 𝑑𝑥 8. ∫ 𝜋 4
1+ √𝐶𝑜𝑡𝑥
9. ∫1 4 𝑥+ 4 5−𝑥 𝑑𝑥
√ 6 √ √

4 5𝜋
1 𝑑𝑥 5 √𝑥+4 12 𝐶𝑜𝑡 2 𝑥
10. ∫0 𝑥(1−𝑥)𝑛 11. ∫0 4 𝑥+4+ 4 9−𝑥 𝑑𝑥 12. ∫ 𝜋
𝑇𝑎𝑛2 𝑥+𝐶𝑜𝑡 2 𝑥
𝑑𝑥
√ √ 12

SECTION-F
𝜋 𝜋 𝜋
7 5 4
2
1. ∫ 𝑆𝑖𝑛 𝑥𝑑𝑥
𝜋
2
2. ∫ 𝑥 𝑆𝑖𝑛 𝑥𝑑𝑥
𝜋 3. ∫ 𝑥 7 𝐶𝑜𝑠 3 𝑥𝑑𝑥
2
𝜋
− − −
2 2 2

𝜋
2−𝑆𝑖𝑛𝑥 1 4−𝑥 𝑎
2
4. ∫ log(
𝜋 )𝑑𝑥 5. .∫−1 log( )𝑑𝑥 6. ∫−𝑎 𝑥 3 √𝑎2 − 𝑥 2 𝑑𝑥
− 2+𝑆𝑖𝑛𝑥 4+𝑥
2

SECTION-G

8 2
1. ∫2 |𝑥 − 5|𝑑𝑥 2. ∫−2 |𝑥 + 1|𝑑𝑥

4 1
3. ∫0 |𝑥 − 1|𝑑𝑥 4. ∫−1 |2𝑥 − 1|𝑑𝑥

4
5. ∫1 {|𝑥 − 1| + |𝑥 − 2| + |𝑥 − 3|}𝑑𝑥

5
6. ∫2 {|𝑥 − 2| + |𝑥 − 3| + |𝑥 − 5|}𝑑𝑥

4
7. ∫0 {|𝑥| + |𝑥 − 2| + |𝑥 − 4|}𝑑𝑥

2
8. ∫−1{|𝑥 + 1| + |𝑥| + |𝑥 − 1|}𝑑𝑥

KVS RO DDN S SM XII MATHEMATICS/ 2023-24(VN) Page 84


*KVS RO DEHRADUN STUDENT SUPPORT MATERIAL MATHEMATICS/ XII/2023-24(VN)*

ANSWERS
SECTION-A

1. Hint Put 𝑥 2 + 5𝑥 − 7 = 𝑡 Ans. 𝑙𝑜𝑔|𝑥 2 + 5𝑥 − 7| + 𝑐


1
2. Hint Put 𝑥 4 + 1 = 𝑡 Ans. 𝑙𝑜𝑔|𝑥 4 + 1| + 𝐶
4

3. Hint Put 𝑎2 𝐶𝑜𝑠 2 + 𝑏 2 𝑆𝑖𝑛2 𝑥 = 𝑡 Ans. Log|𝑎2 𝐶𝑜𝑠 2 + 𝑏 2 𝑆𝑖𝑛2 𝑥| + 𝐶


(sin−1 𝑥)4
4. Hint Put sin−1 𝑥 = 𝑡 Ans. +C
4
(1+𝑙𝑜𝑔𝑥)3
5. Hint Put (1 + 𝑙𝑜𝑔𝑥) = 𝑡 Ans +𝐶
3

6. Hint. Dividing Numerator and Denominator by 𝑒 𝑥 then put 𝑒 −𝑥 + 1 = 𝑡


Ans. − log|𝑒 −𝑥 + 1| + 𝐶
7. Hint Put 𝑥 + 𝐶𝑜𝑠 2 𝑥 = 𝑡 Ans. Log| 𝑥 + 𝐶𝑜𝑠 2 𝑥| + 𝐶
8. Hint Apply formula 𝐶𝑜𝑠2𝑥 = 𝐶𝑜𝑠 2 𝑥 − 𝑆𝑖𝑛2 𝑥 and put 𝐶𝑜𝑠𝑥 + 𝑆𝑖𝑛𝑥 = 𝑡
Ans. log|𝐶𝑜𝑠𝑥 + 𝑆𝑖𝑛𝑥| + 𝐶

SECTION-B
𝑥+1
1. Ans. 2Log|𝑥 2 + 3𝑥 + 2|−5 log | |+𝐶
𝑥+2

4
2. Ans. 3 log|2 − 𝑆𝑖𝑛𝑥| + +𝐶
2−𝑆𝑖𝑛𝑥

3. Ans. 2√𝑥 2 + 2𝑥 + 5 + 3log |(𝑥 + 1) + √𝑥 2 + 2𝑥 + 5|+C


𝑥+1
4. Ans. −3√5 − 2𝑥 − 𝑥 2 − 2 sin−1 +𝐶
√6

5. Ans. 2√3𝑥 2 − 5𝑥 + 1 + 𝐶
6. Ans. 5√𝑥 2 + 4𝑥 + 10 − 7log |𝑥 + 2 + √𝑥 2 + 4𝑥 + 10 + 𝐶

SECTION-C
1 1 1
1. Ans. − log|𝑥 − 1| − log|𝑥 + 2| + log|𝑥 − 3| + 𝐶
6 3 2

1 1
2. Ans. − log|2 + 𝑆𝑖𝑛𝑥| + log|3 + 4𝑆𝑖𝑛𝑥| + 𝐶
5 5

3 1 5
3. Ans. log|𝑥 − 1| − + log|𝑥 + 3| + 𝐶
8 2(𝑥−1) 8

1 1 2 𝑥
4. Ans. log|𝑥 − 1| − log|𝑥 2 + 4| + tan−1 + 𝐶
5 10 5 2

KVS RO DDN S SM XII MATHEMATICS/ 2023-24(VN) Page 85


*KVS RO DEHRADUN STUDENT SUPPORT MATERIAL MATHEMATICS/ XII/2023-24(VN)*

1 2 𝑥
5. Ans. − tan−1 𝑥 tan−1 + 𝐶
3 3 2

6. Ans. 𝑥 + 3 log|𝑥 − 4| − 24 log|𝑥 − 5| + 30 log|𝑥 − 6| + 𝐶

SECTION-D
1 1
1. Ans. 𝑒 𝑥 + 𝐶 2. Ans. 𝑒 𝑥 (𝑙𝑜𝑔𝑐𝑜𝑠𝑥) + 𝐶 3. Ans. 𝑒 𝑥 ( 2) + 𝐶
𝑥 𝑥
𝑒𝑥 𝑒𝑥 𝑒𝑥
4. Ans. +𝐶 5. Ans. 2
+𝐶 6. Ans. +𝐶
1+𝑥 (𝑥−1) (𝑥−2)2

𝑒𝑥 𝑒 2𝑥 𝑡𝑎𝑛𝑥 −𝑒 2𝑥 𝐶𝑜𝑡𝑥
7. 𝐴𝑛𝑠. 2
+𝐶 8. 𝐴𝑛𝑠. +𝐶 9. 𝐴𝑛𝑠. +𝐶
(𝑥+1) 2 2
1
10. Ans. 𝑒 𝑥 (𝑙𝑜𝑔𝑥 − ) + 𝐶
𝑥

SECTION-E
𝜋 𝜋 𝜋
1. Ans. 2. Ans. . 3. Ans.
4 4 8
𝜋 𝜋 5
4. Ans 5. Ans. 6. Ans.
12 4 2
𝜋 3
7. 𝐴𝑛𝑠. 3 8. 𝐴𝑛𝑠 9. 𝐴𝑛𝑠.
12 2
5 𝜋
10. Ans. 11. Ans. 12. Ans.
2 6

SECTION-F
Ans. Q.N.1 to Q.N.6 is 0

SECTION-G
1. Ans. 9 2. Ans. 5 3.Ans. 5
5 19 23
4. Ans. 5. Ans. 6. Ans.
2 2 2
19
7. Ans. 20 8. Ans.
2

KVS RO DDN S SM XII MATHEMATICS/ 2023-24(VN) Page 86


*KVS RO DEHRADUN STUDENT SUPPORT MATERIAL MATHEMATICS/ XII/2023-24(VN)*

CASE STUDY :CALCULUS


3. Read the following text and answer the following questions on the basis of the same:

The bridge connects two places 100 feet apart. The arch on the bridge is in a parabolic form.
The highest point on the bridge is 10 feet above the road at the middle of the bridge as shown in
the figure.
(i) The equation of the parabola designed on the bridge is
50 𝑥 2
(ii) The value of the integral ∫−50 250 𝑑𝑥 𝑖𝑠 is
50
(iii) The integrand of the integral∫−50 𝑥 2 𝑑𝑥 𝑖𝑠
(iv) The area formed by the curve x2 = 250y, X- axis, y = 0 and y = 10 is
4. Three Friends Ami, Harshul and Rahul living in a colony. The location of their houses in the
colony forms a triangular shape. The location of three houses of that colony is represented
by the points A(-1,0), B(2,3) and C(3,2) as shown in the given figure.

(i) The equation of the line AB is

(ii) The equation of the line BC is


(iii) Area of the triangle ABC is
1
(iv)Value of ∫02 √1 − (𝑥 − 1)2 𝑑𝑥 is

KVS RO DDN S SM XII MATHEMATICS/ 2023-24(VN) Page 87


*KVS RO DEHRADUN STUDENT SUPPORT MATERIAL MATHEMATICS/ XII/2023-24(VN)*

CLASS XII (CALCULUS)

ANSWER KEY OF CASE STUDY

CASE STUDY 1 :

(i) x2 =- 250y (ii) 1000/3 (iii) Even (iv) 1000/3

CASE STUDY 2:

3 −𝑥 7 𝜋 √3
(i) y = 2(x + 1) (ii) y = 2 + (iii) 3sq. units (iv) −
2 6 8

KVS RO DDN S SM XII MATHEMATICS/ 2023-24(VN) Page 88


*KVS RO DEHRADUN STUDENT SUPPORT MATERIAL MATHEMATICS/ XII/2023-24(VN)*

CLASS TEST
Section-A MCQ (1X8=8)
𝑑𝑥
1. ∫ 1
𝑆𝑖𝑛2 𝑥𝐶𝑜𝑠 2 𝑥
(a) sin² x – cos² x + C (b) -1

(c)tan x + cot x + C (d) tan x – cot x + C


2. If ∫ sec² (7 – 4x) dx = a tan (7 – 4x) + C, then value of a is 1
1
(a) 7 (b) -4 (c) 3 (d) −
4
𝑑𝑥
3. Evaluate ∫
√16−25𝑥 2

5𝑥 1 5𝑥 1 4−5𝑥 1 4+5𝑥
(a) sin−1 + C (b) sin−1 +C (c) log + 𝑐 (d) log +𝑐 1
4 5 4 8 4+5𝑥 8 4−5𝑥

3 √4−𝑥
4. ∫1 𝑑𝑥 1
√𝑥+√4−𝑥
(a) 4 (b) -4 (c) 2 (d) 1
𝑑𝑥
5. Evaluate ∫ 1
𝑥𝐶𝑜𝑠 2 (1+𝑙𝑜𝑔𝑥)
(a) tan(1 + 𝑙𝑜𝑔𝑥) + n (b) sin(1 + 𝑙𝑜𝑔𝑥) + 𝐶
(c) sin(1 + 𝑙𝑜𝑔𝑥) + 𝐶 (d) 𝑁𝑂 𝑆𝑜𝑙𝑢𝑡𝑖𝑜𝑛

6. If ∫ 𝑒 −2𝑙𝑜𝑔𝑥 𝑑𝑥 = 𝑓(𝑥) + 𝑘, 𝑡ℎ𝑒𝑛 𝑓(𝑥) 𝑖𝑠 1


𝑥3 1 2 1
(a) (b) − (c) − (d)
3 𝑥 𝑥 𝑥

2
7. 𝑉𝑎𝑙𝑢𝑒 𝑜𝑓 ∫1 𝑙𝑜𝑔𝑥𝑑𝑥
𝑒 2 𝑒 4
(a) log (b) 𝑙𝑜𝑔 (c) 2𝑙𝑜𝑔 (d) 𝑙𝑜𝑔
2 𝑒 2 𝑒
𝜋

8. 𝐸𝑣𝑎𝑙𝑢𝑎𝑡𝑒 ∫ (𝑥𝑠𝑖𝑛4 𝑥 + 𝑡𝑎𝑛5 𝑥 + 𝑥)𝑑𝑥


2
𝜋

2
(a) 0 (b) -3 (c) -1 (d) 1

(𝑥+3)𝑑𝑥
9. Evaluate ∫
√5−4𝑥−𝑥 2

𝜋
10. Evaluate ∫0 𝑙𝑜𝑔𝑆𝑖𝑛𝑥𝑑𝑥
2

KVS RO DDN S SM XII MATHEMATICS/ 2023-24(VN) Page 89


*KVS RO DEHRADUN STUDENT SUPPORT MATERIAL MATHEMATICS/ XII/2023-24(VN)*
CHAPTER: 8
APPLICATION OF INTEGRALS
MINIMUM LEARNING LEVEL

Q.1) The area bounded by parabola y2 = 4ax by its latus rectum lying in first quadrant is...
4𝑎 2 2𝑎 2 𝑎2
A) 0 B) 3 C) 3
D) 3
Q.2) The area bounded by curve y = sin 𝑥 between ordinates x = 0, x = π and x-axis is...
A) 2 sq units B) 4 sq units C) 3 sq units D) 1 sq units
2
Q.3) The area (in sq. units) between curve 𝑦 = x & line 2y = x is...
4 2 1
A) 3 B) 1 C) 3
D) 3
Q.4) The area bounded by region (in sq. units) bounded by curve y = x + 1 and x = 2 and x = 3, is
7 9 11 13
A) 2 B) 2 C) 2
D) 3
Q.5) The area (in sq. units) having the curve x = √2 − 𝑦 2 is...
A) 4𝜋 B) 2√2𝜋 C) 2𝜋 D) 4𝜋 2

SHORT ANSWER TYPE QUESTIONS (3 MARKS)

Q.6) Find the area of the region bounded by the curve 𝑦 2 = 4𝑥, y-axis and y = 3.
Q.7) Find the area enclosed by curve y = 2√1 − 𝑥 2 , using integration –
1
Q.8) Find the area of region enclosed by the curves y2 = x, x = , y = 0 and x = 1, using integration –
4

Q.9) Find the area (region) bounded by line y = 3x + 2, x-axis, and x = -2 & x = 1, using integration.
Q.10) Calculate the area under the curve y = 2√𝑥 included between x = 0 & x = 1.
Q.11) Find the area of region {(x, y): x 2 + y 2 ≤ 1 ≤ x + y }.
𝑥2 𝑦2
Q.12) Evaluate the area of curve + = 1 above X axis.
4 9

Q.13) Using integration, find area of region bounded by


y = |x - 1| and y = 1.
Q.14) Find the area of region bounded by the line 2y – 8 = x and x-axis, x= +2 & x = 4.
Q.15) Using integration, find area of curve 2x2 + 2y2 = 1
Q.16) Calculate the area under the curve y = 2√𝑥 included between the lines x = 0 & x = 1.
𝑥2 𝑦2
Q.17) Find area of region bounded by the ellipse, 4
+ 9
= 1, in fourth quadrant

Q.18) Find the area of region bounded by y2 = x and line 2y = x.


Q.19) Find the area of the region {(x, y): 0 ≤ y ≤ x2 + 1, 0 ≤ y ≤ x + 1 and 0 ≤ x ≤ 2 }
Q.20) Find the area bounded by the curve x = 4 – y2 and y-axis.

KVS RO DDN S SM XII MATHEMATICS/ 2023-24(VN) Page 90


*KVS RO DEHRADUN STUDENT SUPPORT MATERIAL MATHEMATICS/ XII/2023-24(VN)*

ANSWERS

MCQ TYPE QUESTIONS

Ans.1) Option B
Ans.2) Option A
Ans.3) Option A
Ans. 4) Option A
Ans. 5) Option C

SHORT ANSWER TYPE QUESTIONS(3 MARKS)

3 9
Ans.6) Required area = ∫0 𝑥𝑑𝑦 = sq. units
4
1
Ans.7) Required area = 4 × ∫0 2√1 − 𝑥 2 𝑑𝑥 = 2𝜋sq. units
1 7
Ans. 8) Required area = ∫1 𝑦𝑑𝑥 = sq. units
4 12
2
− 1 41
Ans. 9) Required area = |∫−23 (3𝑥 + 2)𝑑𝑥 | + |∫−2 (3𝑥 + 2)𝑑𝑥 |= sq. units
3 6

1 4
Ans. 10) Required area = ∫0 2√𝑥𝑑𝑥 = sq. units
3
1 𝜋 1
Ans.11) Required area = ∫0 [√(1 − 𝑥 2 ) − (1 − 𝑥)]𝑑𝑥 = ( − ) sq. units
4 2
3 2
Ans. 12) Required area = [ ∫0 √(2)2 − (𝑥 2 )𝑑𝑥 ] × 2 = 3π sq. units
2
2 1 2
Ans. 13) Required area = ∫0 𝑦𝑑𝑥 - ∫0 (−𝑥 + 1) 𝑑𝑥 - ∫0 (𝑥 − 1) 𝑑𝑥 = 1 sq. units
4 1 4
Ans. 14) Required area = ∫2 𝑦𝑑𝑥 = ∫2 (−𝑥 + 8) 𝑑𝑥 = 5 sq. units
2
1
√2 𝜋
Ans. 15) Required area = 4 × ∫0 (√12 − x 2 ) 𝑑𝑥 = sq. units
4

1 4
Ans. 16) Required area = ∫0 2√𝑥𝑑𝑥 = sq. units
3
3 3 3𝜋
Ans. 17) Required area = |∫0 𝑦𝑑𝑥 | = |23 ∫0 √9 − 𝑥 2 𝑑𝑥 | = sq. units
2
4 4 4
Ans. 18) Required area = ∫0 √𝑥𝑑𝑥 - 12 ∫0 𝑥𝑑𝑥 = sq. units
3
1 2 23
Ans. 19) Required area = ∫0 𝑥 2 + 1 𝑑𝑥 +∫1 𝑥 + 1 𝑑𝑥 = sq. units
6
4 4 32
Ans. 20) Required area = 2 × ∫0 𝑦𝑑𝑥 = 2 × ∫0 √4 − 𝑥𝑑𝑥 = sq. units
3

KVS RO DDN S SM XII MATHEMATICS/ 2023-24(VN) Page 91


*KVS RO DEHRADUN STUDENT SUPPORT MATERIAL MATHEMATICS/ XII/2023-24(VN)*

HOTS
1) Using integration find the area of the region bounded between the line X= 4 and the parabola
y2=16x.
2) Sketch the region bounded by y = 2x - x2 and X-axis and find its area using integration.
𝑥2 𝑦2
3) Find the area of the region bounded by the ellipse𝑎2 + 𝑏2 = 1.

4) Using integration, find the area of the region bounded by the line 2y=-x+8, X-axis and the lines
X=2 and x=4.
5) Find the area of the region bounded by the line y= 3x+2, the X- axis and the coordinates X= 1
and x= -1.
6) Using integration find the area of the triangle formed by the positive X-axis and the tangent and
normal to the circle 𝑥 2 + 𝑦 2 = 4𝑎𝑡(1, √3).
7) Find the area bounded by the curve Y= x |x|, X-axis and the coordinates X= -3 and x= 3.
8) Determine the area under the curve 𝑦 = √𝑎2 − 𝑥 2 included between the lines X= 0 and X= a.
𝑎
9) Find the area of the minor segment of the circle𝑥 2 + 𝑦 2 = 𝑎2 cut off by the line 𝑥 = 2.

10) Find the area of the region bounded by the curve x=at2, y= 2at between the coordinates
corresponding t=1 and t=2.
11) Find the area bounded by the curve Y2 = 4ax and the lines Y = 2a and y-axis.
3𝑥 2
12) Find the area of the region included between parabola 𝑦 = and the line 3x-2y+12=0.
4

13) Find the area of the region bounded by the curves y=x2 +2, y= x, x= 0, and x= 3.
14) Compute the area bounded by the lines x+2y= 2, y-x=1 and 2x+y=7.
15) Find the area bounded by the curve Y= 2x – x2 and the straight line y= -x.
16) Find the area bounded by the parabola y2= 4x and the line y= 2x-4.
 By using horizontal strips
17) Find the area of the region bounded by the curve 𝑦 = 𝑥 3 and the lines y= x+6 and y=0.
18) Find the area bounded by the curve y2= 4a2(x-1) and the lines x=1 and y=4a.

19) Find the area bounded by the lines y= 4x+5, y=5-x and 4y=x+5.
𝑎2
20) If the area bounded by the parabola Y2= 4ax and the line Y= mx is 𝑠𝑞. 𝑢𝑛𝑖𝑡𝑠, then using
12

integration find the value of m.

KVS RO DDN S SM XII MATHEMATICS/ 2023-24(VN) Page 92


*KVS RO DEHRADUN STUDENT SUPPORT MATERIAL MATHEMATICS/ XII/2023-24(VN)*

ANSWERS
128
1. 𝑠𝑞. 𝑢𝑛𝑖𝑡𝑠
3

4
2. 𝑠𝑞. 𝑢𝑛𝑖𝑡𝑠
3

3. 𝜋𝑎𝑏 𝑠𝑞. 𝑢𝑛𝑖𝑡𝑠

4. 5 sq. units
13
5. 𝑠𝑞. 𝑢𝑛𝑖𝑡𝑠
3

6. 2√3 𝑠𝑞. 𝑢𝑛𝑖𝑡𝑠

7. 18 sq. units

𝜋𝑎2
8. 𝑠𝑞. 𝑢𝑛𝑖𝑡𝑠
4

𝑎2
9. (4𝜋 − 3√3)𝑠𝑞. 𝑢𝑛𝑖𝑡𝑠
12

56𝑎2
10. 𝑠𝑞, 𝑢𝑛𝑖𝑡𝑠
3

2𝑎2
11. A 𝑠𝑞. 𝑢𝑛𝑖𝑡𝑠
3

12. 27 sq. units


21
13. 𝑠𝑞. 𝑢𝑛𝑖𝑡𝑠
2

14. 6 sq. unit


9
15. 𝑠𝑞. 𝑢𝑛𝑖𝑡𝑠
2

16. 9 sq. units

17. 28 sq. units


16𝑎
18. 𝑠𝑞. 𝑢𝑛𝑖𝑡𝑠
3

15
19. 𝑠𝑞, 𝑢𝑛𝑖𝑡𝑠
2

20. m = 2

KVS RO DDN S SM XII MATHEMATICS/ 2023-24(VN) Page 93


*KVS RO DEHRADUN STUDENT SUPPORT MATERIAL MATHEMATICS/ XII/2023-24(VN)*

CLASS TEST

SECTION – A (Each question carries 1 mark)

𝑥2 𝑦2
Q.1) The area (in sq. units) enclosed by ellipse 2
+ = 1 is
𝑎 𝑏2

a) 4πab, b) πab, c) 4πa2b , d) πab2


Q.2) The area (in sq. units) enclosed by circle 𝑦 = √2 − 𝑥 2 is
a) 4π, b) 2√2π, c) 2π, d) 4π2

SECTION – B (Each question carries 3 marks)

Q.3) Find the area under the following curves and lines: -
y = x2, x = 1, x = 2, and x-axis
Q.4) Find the area lying in first quadrant and bounded by y = 4x2
Q.5) Find the area enclosed by the curve y = x3, and x-axis and x = -2 and x = 1
Q.6) Using integration, find area of the region bounded by the lines
y – 1 = x, x-axis, x = -2, and x = -3
Q.7) Find the area of the region bounded by parabola y = x2 and y = |x|.
Q.8) Find the area lying between the curve y2 = 4x and line y = 2x.

SECTION – C (Each question carries 5 marks)

Q.9) Find the smaller area cut from the circle x2 + y2 = 25 by line x = 3.
2
Q.10) Sketch the graph for y = |x + 1| and thus evaluate ∫−4|𝑥 + 1|𝑑𝑥 .

KVS RO DDN S SM XII MATHEMATICS/ 2023-24(VN) Page 94


*KVS RO DEHRADUN STUDENT SUPPORT MATERIAL MATHEMATICS/ XII/2023-24(VN)*

CHAPTER: 9
DIFFERENTIAL EQUATION
MINIMUM LEARNING LEVEL
SECTION- A- OBJECTIVE QUESTION
QUESTION
1 𝑑𝑦
Order and degree of the differential equation , 𝑑𝑥 - cos x = 0 is…………………..
2 Order and degree of differential equation , xy2 + x (y1)2- yy1 = 0 is …………………
3 Order and degree of the differential equation , (y3) y2 - y1 = 0
4 𝑑𝑦
Degree of the differential equation sins + cos ( 𝑑𝑥 ) = y2is ……………………………
5 Find the category of the differential equation
𝑑𝑦
(i) 𝑑𝑥 = 2xy …………………………
(ii) (x + y) dy + (x – y) dx = 0 ……………………………
𝑑𝑦
(iii) (x+ 2y2) = y ……………………………………
𝑑𝑥
6 𝑑2 𝑦 𝑑𝑦 3
The sum of the order and degree of the differential equation + [𝑑𝑥 ] = sin y …….
𝑑𝑥 2
7 𝑑𝑦
Integrating factor of the differential equation (1-x2) 𝑑𝑥 - xy = 1 is
𝑥 1
(a) -x (b) 1+ 𝑥 2 (c) √1 − 𝑥 2 (d)2 log( 1- 𝑥 2 )
8 The general solution of the differential equation x dy – (1 + 𝑥 2 ) dx= dx is
𝑥3 𝑥3
( a) y = 2x + +C (b) y = 2 log x + +C
3 3
𝑥2 𝑥2
(c) y = +C (d) y = 2 log x + +𝐶
2 2

ASSERTION –REASON TYPE QUESTION


(a) A and R both are correct and R is the correct explanation of A.
(b) Both A and R are correct but R is not correct explanation of A.
(c) A is correct but R is false.
(d) Both A and R are false
𝑑𝑦
9
A: Solution of the differential equation 𝑒 𝑑𝑥 = 𝑥 2 is y = 2 (x log x – x) + C
𝑑𝑦 −1
R :The integrating factor of the D.E ( 1 + 𝑥 2 )𝑑𝑥 +y = 𝑡𝑎𝑛−1 x is 𝑒 𝑡𝑎𝑛 𝑥
10 𝑑𝑥
A : The integrating factor of differential equation + ( tan y ) .x = sec 2y is sec y.
𝑑𝑦
R . Linear differnetial equation of the form
𝑑𝑦
+ 𝑃𝑥 + 𝑞 , P , Q = f(y) or constant has integrating factor IF = 𝑒 ∫ 𝑃𝑑𝑦
𝑑𝑥

SECTION- B ( Three marks question)


1 Find the general solution of 𝑦 2 dx+ (𝑥 2 – xy + 𝑦 2 )dy = 0
2 𝑑𝑦 𝑋+𝑌
Find the particular solution of the D.E 𝑑𝑥 = 𝑋 , y(1) =0
3 𝑑𝑦 𝑌
Solve differential equation 𝑑𝑥
+ 𝑋 = 𝑥2
4 Find general solution of 𝑦1 = 1 – x + y –xy
5 Solve differential equation 𝑠𝑒𝑐 2x tan y dx- 𝑠𝑒𝑐 2 y tan x dy = 0
6 𝑑𝑥
Solve differential equation ( y + 3𝑥 2 ) 𝑑𝑦 = x
7 𝑑𝑦
Solve differential equation 𝑑𝑥
= 𝑦 tan 𝑥 ; 𝑦 = 1 𝑤ℎ𝑒𝑛 𝑥 = 0
8 𝑑𝑦
Solve differential equation cos ( 𝑑𝑥
) = a , y= 1 when x= 0
9 𝑑𝑦
Solve differential equation ( x +1 )𝑑𝑥 = 2xy
10 𝑑𝑦
Find particular solution of differential equation sin2x 𝑑𝑥 - y = tan x

KVS RO DDN S SM XII MATHEMATICS/ 2023-24(VN) Page 95


*KVS RO DEHRADUN STUDENT SUPPORT MATERIAL MATHEMATICS/ XII/2023-24(VN)*

Answer SECTION A

1. 1
2 ,1
3, 2
4. Not defined
5. (i)separable variable

(ii)homogeneous

(iii)Linear ifferential Equation


6. 3
7. (c)
8. (d)
9. (b)
10. (a)

Answer SECTION B

1. y + √ (𝑥 2 + 𝑦 2 ) = 𝐶 𝑥 2
𝑦
2. 𝑒𝑥 = x
3. 𝑥3
y= + c 𝑥 −1
4
4. 𝑥2
log|1 + 𝑦|= x- +c
2
5. tan x = c tan y

6. y- 3𝑥 2 + Cx=0

7. y = sec x
8. 𝑦−1
a = cos ( )
𝑥
9. logy = 2 [ x- log (x- 1) ]+ c
10. Y = tan x – √tan 𝑥

KVS RO DDN S SM XII MATHEMATICS/ 2023-24(VN) Page 96


*KVS RO DEHRADUN STUDENT SUPPORT MATERIAL MATHEMATICS/ XII/2023-24(VN)*

HOTS

1- If m and n are the order and degree of the differential equation


5 𝑑2 𝑦 3
( ) 𝑑3 𝑦
𝑑𝑥2
𝑑2𝑦 +4 𝑑3 𝑦
+ = 𝑥 2 − 1, then
( 2
) 𝑑𝑥 3
𝑑𝑥 𝑑𝑥3
(a) m = 3 , n = 3 ( b) m = 3 , n= 2 (c) m =3 , n= 5 (d) m= 3 , n= 1
𝑑2 𝑦 𝑑𝑦
2- The sum of the order and the degree of the differential equation + ( )3 = sin 𝑦 is
𝑑𝑥 2 𝑑𝑥
(a) 5 (b) 2 (c) 3 (d) 4
𝑑𝑦 𝑑3 𝑦
3- The order and the degree of the differential equation 1 + 3 =4 respectively are
𝑑𝑥 𝑑𝑥 3
2
(a)1 , (b) 3 , 1 (c) 3 , 3 (d) 1, 2
3
𝑑2𝑦
4- What is the product of the order and degree of the differential equation , sin 𝑦 +
𝑑𝑥 2
𝑑𝑦
( )3 cos 𝑦 = √𝑦 ?
𝑑𝑥
(a) 3 (b) 2 (c) 6 (d) Not defined
𝑑𝑦
5- The solution of − 𝑦 = 1 , 𝑦(0) = 1 𝑖𝑠 𝑔𝑖𝑣𝑒𝑛 𝑏𝑦
𝑑𝑥
𝑥
(a) 𝑥𝑦 = − 𝑒 (b) xy = −𝑒 −𝑥 (c) 𝑥𝑦 = −1(d) 𝑦 = 2𝑒 𝑥 − 1
𝑑𝑦 1+𝑦
6- The integrating factor of differential equation +𝑦= 𝑖𝑠
𝑑𝑥 𝑥
𝑥 𝑒𝑥
(a) (b) (c) 𝑥𝑒 𝑥 (d) 𝑒 𝑥
𝑒𝑥 𝑥
𝑑𝑦 −𝑥
7- The solution of +y= 𝑒 , y (0) =0 is
𝑑𝑥
(a) y = 𝑒 𝑥 ( x − 1 ) (b) y = x 𝑒 −𝑥 (c) ) y = x 𝑒 −𝑥 + 1 (d) ) y = (x + 1 )𝑒 −𝑥
𝑑𝑥 𝑑𝑦
8- The solution of the differential equation + = 0 is
𝑥 𝑦
1 1
(a) + =C (b) log x – log y = c (c) xy = c (d) x + y = c
𝑥 𝑦
𝑑𝑦
9- The Integrating factor for solving the differential equation x − 𝑦 = 2 𝑥 2 is :
𝑑𝑥
1
(a) 𝑒 −𝑦 (b) 𝑒 −𝑥 (c) x (d)
𝑥

10-The general solution of the differential equation 𝑥 𝑑𝑦 − ( 1 + 𝑥 2 )𝑑𝑥 = 𝑑𝑥 is


𝑥3 𝑥3 𝑥2 𝑥2
(a) y = 2 x + +c (b) y = 2 log x + + c (c) y = + 𝑐(d) y = 2 log x + +c
3 3 2 2

ASSERTION – REASON BASED QUESTIONS


Directions ( Q. No. 11-13 ) In the questions given below are two statements labeled as
Assertion ( A ) and Reason ( R ). In the context of the two statements, which one of the
following is correct?
(i) Both A and R is true and R is the correct explanation of A.
(ii) Both A and R is true but R is not the correct explanation of A.
(iii) A is true but R is false.
(iii) A is false but R is true.
𝑑𝑦
11- Assertion (A) : The general solution of the differential equation 𝑥 + 2y = 𝑥 2 ( x ≠
𝑑𝑥
𝑥2
0 ) is 𝑦 = + k 𝑥 −2 .
4

KVS RO DDN S SM XII MATHEMATICS/ 2023-24(VN) Page 97


*KVS RO DEHRADUN STUDENT SUPPORT MATERIAL MATHEMATICS/ XII/2023-24(VN)*

Reason ( R ) : The number of arbitrary constants in the general solution is equal to the
order of the differential equation.
12- Assertion (A):The order of the differential equation whose solution is y = 𝑐1 𝑒 2𝑥+𝑐2 +
𝑐3 𝑒 2𝑥+𝑐4 is 4.
Reason (R): Order of the differential equation is the order of the highest order derivative
occurring in the differential equation.
𝑑𝑦 𝑦 1
13- Assertion (A): Integrating factor of + = 3𝑥 2 is 𝑥 2 .
𝑑𝑥 2𝑥

𝑑𝑦
Reason (R): Integrating factor of + P y = Q is 𝑒 ∫ 𝑃 𝑑𝑥
𝑑𝑥

CASE STUDY BASED QUESTIONS

𝑑𝑦
14- Reshma a Mathematics teacher of Class XII writes a differential equation +
𝑑𝑥
(sec 𝑥 ) 𝑦 = tan 𝑥 on the blackboard and asks some questions which are based on above
differential equation to check the preparation of the topic, then answer the following
questions.
(i) What is the form of the given equation?
(ii) Write the value of ( 𝑃2 − 𝑄2 ).
𝑑𝑦
(iii) Find the integrating factor of equation + (sec 𝑥 ) 𝑦 = tan 𝑥.
𝑑𝑥
OR
𝑑𝑦
What is the solution of +Py=Q
𝑑𝑥

SHORT ANSWER ( S.A.)


𝑑𝑦 y
15- Find the general solution of the differential equation = .
𝑑𝑥 x
𝑑𝑦 y
16- Solve the differential equation + = x2 .
𝑑𝑥 x
𝑑𝑦
17- Find the solution of = 2y−x .
𝑑𝑥
𝑑𝑦 −2y
18- Given that = e and y = 0 when x = 5. Find the value of x when y = 3.
𝑑𝑥
19- Solve : y dx – x dy = 𝑥 2 y dx
Long Answer (L.A.)
𝜋
20- Find the equation of a curve passing through (1, ) , if the slope of the tangent to the
4
𝑦 y
curve at any point P ( x, y ) is − cos2 .
𝑥 x
dy y 𝜋
21- Solve 𝑥 2 − xy = 1 + cos ( ) , x ≠ 0 and x = 1 , y =
dx x 2
22- State the type of the differential equation for the equation x dy – y dx = √𝑥 2 + 𝑦 2 dx
and solve it .

KVS RO DDN S SM XII MATHEMATICS/ 2023-24(VN) Page 98


*KVS RO DEHRADUN STUDENT SUPPORT MATERIAL MATHEMATICS/ XII/2023-24(VN)*

Answer :

1- Ans : b 6- Ans : (b)

2- Ans : c 7- Ans : b

3- Ans : b 8- Ans : c

4- Ans : b 9- Ans : d

5- Ans : d 10- Ans:d ,

11- Ans: ii ,

12- Ans :iv

13- Ans : i

14- Ans : Linear form , Ans : 1 ,

Ans: sec x + tan x OR y (IF) = ∫ 𝑄(𝐼𝐹 )𝑑𝑥 + C)

15- Ans : y = cx

𝑥3 c
16- Ans : y = +
4 x

17- Ans : 2−𝑥 − 2−y = k

𝑒6+ 9
18- Ans :
2

𝑦 1
19- log ( ) = − 𝑥 2 + 𝑐
𝑥 2

𝑦
20- Ans : tan ( ) + log 𝑥 = 1 )
𝑥

𝑦 1 3
21- Ans : tan ( ) = − +
2𝑥 2𝑥 2 2

22- Ans : Homogeneous differential equation, y + √𝑥 2 + 𝑦 2 = c 𝑥 2

KVS RO DDN S SM XII MATHEMATICS/ 2023-24(VN) Page 99


*KVS RO DEHRADUN STUDENT SUPPORT MATERIAL MATHEMATICS/ XII/2023-24(VN)*

CLASS TEST - I

1 𝑑2𝑦 3 𝑑𝑦 1
Write 5the degree of the D.E x ( ) +y ( )4 + 𝑥 3 = 0
𝑑𝑥 2 𝑑𝑥
2 𝑑𝑦 𝑑𝑦 1
Write the order and degree of the D.E , + sin ( )=0
𝑑𝑥 𝑑𝑥
3 𝑑 𝑑𝑦 2
Write the sum of the order and degree of the D.E { ( )3 }= 0
𝑑𝑥 𝑑𝑥
4 𝑑𝑦 2
Write the solution D.E = 2−𝑦
𝑑𝑥
5 𝑑𝑦 3
Find particular solution of x ( 𝑥 2 - 1 ) = 1 ; y= 0 when x= 2
𝑑𝑥
6 𝑑𝑦 3
Solve D.E = y tan x
𝑑𝑥
7 Solve the D.E ,xdy – ydx = √( 𝑥 2 + 𝑦 2 ) dx 4
8 Solve the equation xdy + (y - 𝑥 3 ) = 𝑑𝑥 = 0 4
9 𝑑𝑦 1 5
Solve the differential equation ( 1+ 𝑥 2 ) + 2xy = , given y=0 when x = 1
𝑑𝑥 1 + 𝑥2
10 Solve the differential equation ( 1+ 𝑒 2𝑥 )dy + ( 1 + 𝑦 ) 𝑒 𝑥 dx = 0 given that 2 5
y = 1, when x = 0

CLASS TEST- II

𝑑𝑦 1+𝑦
1. The integrating factor of differential equation +𝑦 = 𝑖𝑠
𝑑𝑥 𝑥
𝑥 𝑒𝑥
(b) (b) (c) 𝑥𝑒 𝑥 (d) 𝑒 𝑥
𝑒𝑥 𝑥

𝑑𝑦
2. The solution of + y = 𝑒 −𝑥 , y (0) =0 is
𝑑𝑥
(b) y = 𝑒 𝑥 ( x − 1 ) (b) y = x 𝑒 −𝑥 (c) ) y = x 𝑒 −𝑥 + 1 (d) ) y = (x + 1 )𝑒 −𝑥
𝑑𝑥 𝑑𝑦
3. The solution of the differential equation + = 0 is
𝑥 𝑦
1 1
(b) + =C (b) log x – log y = c (c) xy = c (d) x + y = c
𝑥 𝑦
Section – B (04 x 03 = 12)
𝑑𝑦
4- State whether y = 𝑒 −𝑥 ( x + a ) is a solution of differential equation + y = 𝑒 −𝑥 .
𝑑𝑥
𝑑𝑦
5- If = y 𝑒 𝑥 and x = 0, y = e, then find the value of y , when x = 1.
𝑑𝑥
𝑑𝑦 𝑥+𝑦
6- Show that the given differential equation is homogeneous and solve it = .
𝑑𝑥 𝑥
dy
7- Find the particular solution of the differential equation2𝑥𝑦 + 𝑦 2 − 2𝑥 2 = 0 ,y =
dx
2, when x = 1
Section – C (03 x 05 = 15)
𝑑𝑦 𝑒 𝑥 ( 𝑆𝑖𝑛2 𝑥+sin 2𝑥 )
8- Solve the differential equation =
𝑑𝑥 𝑦 ( 2 log 𝑦+1 )
𝑑𝑦
9- Solve : 𝑥2 = 𝑥 2 + 𝑥𝑦 + 𝑦 2
𝑑𝑥

10. Solve the differential equation ( 1 + tan y ) ( dx – dy )+ 2x dy = 0

KVS RO DDN S SM XII MATHEMATICS/ 2023-24(VN) Page 100


*KVS RO DEHRADUN STUDENT SUPPORT MATERIAL MATHEMATICS/ XII/2023-24(VN)*

Answer :

1-b

2- b

3- c

4 – Yes

5- y = 𝑒 𝑒

6. y = x log | x | + c x
2𝑥
7. -y = ( x ≠ 0, x ≠ e )
1−log|𝑥|

8- 𝑦 2 (log 𝑦 ) = 𝑒 𝑥 𝑠𝑖𝑛2 𝑥 + C

𝑦
9. tan−1 ( ) = log|𝑥| + 𝐶
𝑥

10- x (siny + cos y) = sin y + C 𝑒 −𝑦

KVS RO DDN S SM XII MATHEMATICS/ 2023-24(VN) Page 101


*KVS RO DEHRADUN STUDENT SUPPORT MATERIAL MATHEMATICS/ XII/2023-24(VN)*

CHAPTER 10:
VECTOR ALGEBRA
MINIMUM LEARNING LEVEL

1.Write two different vectors having same magnitude.


2. Write two different vectors having same direction.
3. Find the scalar and vector components of the vector with initial point (2, 1) and terminal point (- 5, 7).
4. Find the unit vector in the direction of the vector 𝑎⃗= 𝑖̂ + 𝑗̂ + 2𝑘̂.
⃗⃗⃗⃗⃗⃗
⃗⃗⃗⃗⃗⃗ , Where P and Q are the points (1, 2, 3) and (4, 5, 6).
5. Find the unit vector in the direction of vector𝑃𝑄
Respectively.
6. Show that the vectors 2𝑖̂ −3𝑗̂ + 4𝑘̂ and −4𝑖̂ +6𝑗̂ −8𝑘̂ are collinear.
7.Find the direction cosines of the vector 𝑖̂ +2𝑗̂ + 3𝑘̂.
8.Find a vector in the direction of vector 5𝑖̂ − 𝑗̂ + 2𝑘̂ which has magnitude 8 units.
⃗⃗
9. Show that the points A, B and C with position vectors, 𝑎⃗ = 3𝑖̂ −4𝑗̂ − 4𝑘̂, 𝑏⃗⃗ = 2𝑖̂-𝑗̂ + 𝑘̂ and𝑐⃗ = 𝑖̂-3𝑗̂ -
5𝑘̂, respectively from the vertices of a right-angled triangle.
⃗⃗
10. Find the sum of the following vectors ⃗⃗⃗𝑎⃗ = 𝑖̂ −2𝑗̂, 𝑏⃗⃗ =2𝑖̂-3𝑗̂ and 𝑐⃗ =2𝑖̂+3𝑘̂.
11.Find the projection of the vector 𝑖̂ + 3𝑗̂ + 7𝑘̂ on the vector 7𝑖̂ − 𝑗̂ + 8𝑘̂.
12. Show that the points A (−2𝑖̂ + 3𝑗̂ + 5𝑘̂), B (𝑖̂ + 2𝑗̂ + 3𝑘̂) and C (7𝑖̂ − 𝑘̂) are collinear.
13.Show that the vectors 2𝑖̂ − 𝑗̂ + 𝑘̂, 𝑖̂ −3𝑗̂ −5𝑘̂and 3 𝑖̂ −4𝑗̂ −4𝑘̂from the vertices of a right-angled
triangle.
14. If the vertices A, B, C of a triangle ABC are (1,2,3), (-1,0,0) and (0,1,2) respectively, then find
∠ABC.

⃗⃗
15. If 𝑎⃗ = 2𝑖̂ +2𝑗̂ + 3𝑘̂, 𝑏⃗⃗ = −𝑖̂ + 2𝑗̂ + 𝑘̂ and 𝑐⃗⃗ =3𝑖̂ + 𝑗̂ are such that 𝑎⃗+𝜆𝑏
⃗⃗⃗⃗⃗ is perpendicular to𝑐⃗, then
find the value of 𝜆.
𝑎, ⃗⃗⃗⃗
16. If ⃗⃗⃗⃗ 𝑏, ⃗⃗⃗𝑐 are unit vectors such that 𝑎⃗ +𝑏⃗⃗ +𝑐⃗ =0
⃗⃗ , find the value of 𝑎⃗. 𝑏⃗⃗ +𝑏⃗⃗.𝑐⃗ +𝑐⃗. 𝑎⃗.
17. Find the area of a triangle having the points A (1,1,1), B (1,2,3) and C (2,3,1) as its vertices.
18. Find |𝑎⃗ × 𝑏⃗⃗|, 𝑎⃗ = 𝑖̂ −7𝑗̂ + 7𝑘̂ and 𝑏⃗⃗ = 3 𝑖̂ −2𝑗̂ + 2𝑘̂.

19. Find the area of the parallelogram whose adjacent sides are determined by the vector 𝑎⃗ = 𝑖̂ − 𝑗̂ + 3𝑘̂ and 𝑏⃗⃗
= 2 𝑖̂ −7𝑗̂ + 𝑘̂.
⃗⃗
20. Let 𝑎⃗ = 𝑖̂ +4𝑗̂ + 2𝑘̂ , 𝑏⃗⃗ = 3𝑖̂ − 2𝑗̂ +7𝑘̂ and 𝑐⃗⃗ =2𝑖̂ − 𝑗̂+ 4𝑘
⃗⃗ , Find a vector 𝑑⃗ which is perpendicular to

both 𝑎⃗ and 𝑏⃗⃗, and 𝑐⃗. 𝑑⃗=15.


21. If 𝑎⃗ = 5𝑖̂ − 𝑗̂ −3𝑘̂ and 𝑏⃗⃗ = 𝑖̂ +3𝑗̂ −5𝑘̂ , then show that the vectors 𝑎⃗ + 𝑏⃗⃗ and 𝑎⃗ − 𝑏⃗⃗ are orthogonal.

KVS RO DDN S SM XII MATHEMATICS/ 2023-24(VN) Page 102


*KVS RO DEHRADUN STUDENT SUPPORT MATERIAL MATHEMATICS/ XII/2023-24(VN)*

𝑨𝒏𝒔𝒘𝒆𝒓𝒔

1.𝐴𝑛 𝑖𝑛𝑓𝑖𝑛𝑖𝑡𝑒 𝑛𝑢𝑚𝑏𝑒𝑟 𝑜𝑓 𝑝𝑜𝑠𝑠𝑖𝑏𝑙𝑒 𝑎𝑛𝑠𝑤𝑒𝑟𝑠.

2. 𝐴𝑛 𝑖𝑛𝑓𝑖𝑛𝑖𝑡𝑒 𝑛𝑢𝑚𝑏𝑒𝑟 𝑜𝑓 𝑝𝑜𝑠𝑠𝑖𝑏𝑙𝑒 𝑎𝑛𝑠𝑤𝑒𝑟𝑠.

3. -7 and 6; −7𝑖̂and 6𝑗̂

4.𝑖̂/√6+ 𝑗̂̂
/√6 + 2𝑘̂/√6

5.𝑖̂/√3+ 𝑗̂̂
/√3 + 𝑘̂/√3

7.[Ans: 1/√14,2/√14,3/√14 ]

8.40𝑖̂/√30 − 8𝑗̂/̂
√30+ 16 𝑘̂ /√30

10.5𝑖̂ −5𝑗̂ + 3𝑘̂

11. 60/√114

14.cos −1 (10/√102)
⃗⃗⃗⃗⃗⃗
⃗⃗⃗⃗⃗⃗
⃗⃗⃗⃗⃗⃗ and 𝐵𝐶
[ ∠ABC is the angle between the vectors𝐵𝐴 ⃗⃗⃗⃗⃗⃗ ]

15.𝜆=8

16.𝑎⃗. 𝑏⃗⃗ +𝑏⃗⃗.𝑐⃗ +𝑐⃗. 𝑎⃗= −3/2

17. the required area is √21 / 2

18. |𝑎⃗ × 𝑏⃗⃗| = 19√2

19. area of the given parallelogram =|𝑎⃗ × 𝑏⃗⃗| = 15√2

20.𝑑⃗ =( 160𝑖̂ − 5𝑗̂ +70𝑘̂) / 3

KVS RO DDN S SM XII MATHEMATICS/ 2023-24(VN) Page 103


*KVS RO DEHRADUN STUDENT SUPPORT MATERIAL MATHEMATICS/ XII/2023-24(VN)*

HOTS
2
1. If (𝑎⃗ × 𝑏⃗⃗) + (𝑎⃗ . 𝑏⃗⃗)2 = 225 𝑎𝑛𝑑 ⃗⃗⃗⃗
|𝑎 | = 5, then find the value of |𝑏⃗⃗|.
2. If 𝑎⃗, 𝑏⃗⃗, 𝑐⃗ 𝑏𝑒 𝑡ℎ𝑒 𝑝𝑜𝑠𝑡𝑖𝑜𝑛 𝑣𝑒𝑐𝑡𝑜𝑟 𝑜𝑓 𝑣𝑒𝑟𝑡𝑖𝑐𝑒𝑠 𝐴, 𝐵, 𝐶, 𝐷 𝑜𝑓 𝑎 𝑝𝑎𝑟𝑎𝑙𝑙𝑒𝑙𝑙𝑜𝑔𝑟𝑎𝑚 𝐴𝐵𝐶𝐷, then
find the position vector of D
3. If 𝑎⃗, 𝑏⃗⃗, 𝑐⃗ are three mutually perpendicular vectors of equal magnitude, then show that

𝑎⃗+𝑏⃗⃗ + 𝑐⃗ is equally inclined to the vectors 𝑎⃗, 𝑏⃗⃗, 𝑐⃗


4. If 𝑎⃗ = 2𝑖̂ + 𝑗̂ − 𝑘̂ , 𝑏⃗⃗ = 4𝑖̂ − 7𝑗̂ + 𝑘̂ find a vector 𝑐⃗ such that 𝑎⃗ x 𝑐⃗ = 𝑏⃗⃗ and⃗⃗⃗⃗.𝑐
𝑎 ⃗=6
5. If 𝑎⃗ = 𝑖̂ − 𝑗̂ + 7𝑘̂ and 𝑏⃗⃗ = 5𝑖̂ − 𝑗̂ + 𝑘̂ then find the value of  𝑠𝑜 𝑡ℎ𝑎𝑡 𝑡ℎ𝑒 𝑣𝑒𝑐𝑡𝑜𝑟𝑠 ⃗⃗⃗⃗⃗
(𝑎 +
𝑏⃗⃗) 𝑎𝑛𝑑 ⃗⃗⃗⃗⃗⃗
(𝑎 − 𝑏⃗⃗ ) are orthogonal
⃗⃗⃗⃗ ≠ 0, 𝑎⃗. 𝑏⃗⃗ = 𝑎⃗. 𝑐⃗ , 𝑎⃗ × 𝑏⃗⃗ = 𝑎⃗ × 𝑐⃗, then show that 𝑏⃗⃗ = 𝑐⃗
6. If 𝑎

7. The vectors 𝑏⃗⃗ = 3𝑖̂ + 4𝑘̂ is to be written as the sum of two vectors 𝛼⃗ 𝑎𝑛𝑑 𝛽⃗where 𝛼⃗ is

parallel to 𝑎⃗ = 𝑖̂ + 𝑗̂ and 𝛽⃗ 𝑖𝑠 𝑝𝑒𝑟𝑝𝑒𝑛𝑑𝑖𝑐𝑢𝑙𝑎𝑟 𝑡𝑜 𝑎⃗. Find 𝛼⃗ 𝑎𝑛𝑑 𝛽⃗


𝜋
8. 𝑎⃗, 𝑏⃗⃗, 𝑐⃗ 𝑏𝑒 𝑡ℎ𝑒 𝑢𝑛𝑖𝑡 𝑣𝑒𝑐𝑡𝑜𝑟𝑠, suppose 𝑎⃗.𝑏⃗⃗ = 𝑎⃗. 𝑐⃗ = 0 and angle between 𝑏⃗⃗ 𝑎𝑛𝑑 𝑐⃗ is . Prove
6

that 𝑎⃗ = ±2(𝑏⃗⃗ × 𝑐⃗)


9. Assertion (A): The vectors 𝑎⃗ = 2𝑖̂ − 𝑗̂ + 5𝑘̂ 𝑎𝑛𝑑 , 𝑏⃗⃗ = 5𝑖̂ + 𝑗̂ − 3𝑘̂ are perpendicular to
each other.
Reason (R ) : 𝑎⃗ × 𝑏⃗⃗ is a vector perpendicular to both 𝑎⃗ 𝑎𝑛𝑑 𝑏⃗⃗
10. Case Study: Team P,Q,R went for playing a tug of war game. Teams P,Q,R have attached a
rope to a metal ring and is trying to pull the ring their own areas (team areas when in the
R
⃗⃗⃗⃗⃗⃗=4𝑖̂ + 0𝑗̂ KN
given figure below). Team P pulls with force 𝐹1
P
⃗⃗⃗⃗⃗⃗=2𝑖̂ + 4𝑗̂KN
Team Q pulls with force 𝐹2
⃗⃗⃗⃗⃗⃗=3𝑖̂ − 3𝑗̂ KN
Team R pulls with force 𝐹3
Q
(i) What is the magnitude of the teams combined
Force ? also, find which team will win the game ?
(ii) In what direction is the ring getting pulled?
11. If two vectors 𝑎⃗ and 𝑏⃗⃗ are such that |𝑎⃗| = 2, |𝑏⃗⃗| = 3 𝑎𝑛𝑑 𝑎⃗ .𝑏⃗⃗ =

4 𝑡ℎ𝑒𝑛 𝑓𝑖𝑛𝑑 𝑡ℎ𝑒 𝑣𝑎𝑙𝑢𝑒 𝑜𝑓 |𝑎⃗ − 2𝑏⃗⃗|.


12. Three vectors A,B,C,D of a parallelogram ABCD are given by A(0,-3,3), B9-5,m-3,0) and
D(1,-3,4). The area of the parallelogram ABCD is 6 sq units. Using vector method, find the
values of m.

KVS RO DDN S SM XII MATHEMATICS/ 2023-24(VN) Page 104


*KVS RO DEHRADUN STUDENT SUPPORT MATERIAL MATHEMATICS/ XII/2023-24(VN)*

ANSWERS : HOTS

1. |𝑏⃗⃗| =3
2. 𝑟⃗ = 𝑎⃗ + 𝑐⃗ − 𝑏⃗⃗
𝑎⃗⃗ ⃗⃗
𝑏 𝑐⃗
3. cos 𝛼 = ⃗⃗⃗⃗
⃗⃗+𝑐|
cos 𝛽 = ⃗⃗⃗⃗
⃗⃗+𝑐|
cos 𝛾 = ⃗⃗⃗⃗
⃗⃗+𝑐|
|𝑎⃗⃗+𝑏 |𝑎⃗⃗+𝑏 |𝑎⃗⃗+𝑏

4. 𝑐⃗ = 3𝑖̂ + 2𝑗̂ + 2𝑘̂


5.  = ±5
3 3 3
7. 𝛼⃗ = (𝑖̂ + 𝑗̂)𝑎𝑛𝑑 𝛽⃗ = 𝑖̂ − 𝑗̂ + 4𝑘̂
2 2 2

9. A is true R is false.
⃗⃗⃗⃗⃗⃗ = 4 𝑘𝑁, 𝐹2
10. (i) 𝐹1 ⃗⃗⃗⃗⃗⃗=√20 kN 𝐹3
⃗⃗⃗⃗⃗⃗=√18 kN
Team Q will win the game.
3𝜋
(ii) 𝐹⃗ = −𝑖̂ + 𝑗̂ angle = radians
4

11. 2√6
12. m=±4

KVS RO DDN S SM XII MATHEMATICS/ 2023-24(VN) Page 105


*KVS RO DEHRADUN STUDENT SUPPORT MATERIAL MATHEMATICS/ XII/2023-24(VN)*

CASE STUDY BASED


PROBLEM NO 1

Anju purchased a air plant holder which is in the shape of a tetrahedron . Let A , B , C ,D are the coordinates of
the air plant holder where 𝐴( 1,1,1) , 𝐵( 2,1,3) , 𝐶(3,2,2) 𝑎𝑛𝑑 𝐷 (3,3,4) .

Based on above information answer the following questions


(i) Find the position vector of 𝐴𝐵⃗⃗⃗⃗⃗⃗⃗ . (1 MARK)
⃗⃗⃗⃗⃗⃗⃗⃗
(ii) Find the length of 𝐴𝐷 . (1 MARK)
(iii) Find the unit vector along ⃗⃗⃗⃗⃗⃗⃗
𝐴𝐶 . (2 MARKS)
PROBLEM NO – 2

For any two vectors𝑎 ⃗⃗⃗⃗ 𝑎𝑛𝑑 ⃗⃗⃗⃗ 𝑎 . ⃗⃗⃗⃗


𝑏 , ⃗⃗⃗⃗ 𝑏 = |𝑎 ⃗⃗⃗⃗||𝑏⃗⃗⃗⃗| 𝑐𝑜𝑠𝜃 , 𝑤ℎ𝑒𝑟𝑒 𝜃 𝑖𝑠 𝑡ℎ𝑒 𝑎𝑛𝑔𝑙𝑒 𝑏𝑒𝑡𝑤𝑒𝑒𝑛 𝑡𝑤𝑜 𝑣𝑒𝑐𝑡𝑜𝑟𝑠 .
⃗⃗⃗⃗ × ⃗⃗⃗⃗
𝑎 𝑏 = |𝑎 ⃗⃗⃗⃗||𝑏⃗⃗⃗⃗| 𝑠𝑖𝑛𝜃 𝑛̂ , 𝑤ℎ𝑒𝑟𝑒 𝑛̂ 𝑖𝑠 𝑎 𝑢𝑛𝑖𝑡 𝑣𝑒𝑐𝑡𝑜𝑟 𝑝𝑒𝑟𝑝𝑒𝑛𝑑𝑖𝑐𝑢𝑙𝑎𝑟 𝑡𝑜 𝑏𝑜𝑡ℎ ⃗⃗⃗⃗ 𝑎 𝑎𝑛𝑑 ⃗⃗⃗⃗
𝑏 .
Based on above information answer the following questions
(i) Find the angle between two vectors ⃗⃗⃗⃗ 𝑎 𝑎𝑛𝑑 ⃗⃗⃗⃗
𝑏 ℎ𝑎𝑣𝑖𝑛𝑔 𝑠𝑎𝑚𝑒 𝑙𝑒𝑛𝑔𝑡ℎ √2and their vector product is −𝑖̂ −
̂
𝑗̂ + 𝑘 .
(1 MARK)
(ii) If the projection of 𝑎 ⃗⃗⃗⃗ = 𝑖̂ − 2𝑗̂ + 3𝑘̂ 𝑜𝑛 𝑏 ⃗⃗⃗⃗ = 2𝑖̂ + 𝜆𝑘̂ is zero then find the value of . (1 MARK)
(iii) If |𝑎
⃗⃗⃗⃗| = 10 , |𝑏⃗⃗⃗⃗| = 2 , ⃗⃗⃗⃗ ⃗⃗⃗⃗ = 12 , 𝑡ℎ𝑒𝑛 𝑓𝑖𝑛𝑑 𝑡ℎ𝑒 𝑣𝑎𝑙𝑢𝑒 𝑜𝑓 |𝑎
𝑎 .𝑏 ⃗⃗⃗⃗ × ⃗⃗⃗⃗
𝑏| . (2 MARKS)

PROBLEM NO – 3
Shakti left from his village (A) on weekend . First he travelled upto the temple(B) . After this he left for the zoo
(C) . After this he left for shopping in a mall(D) . The position of Shakti at different positions is given in the
following graph. (Distance is in kms)

KVS RO DDN S SM XII MATHEMATICS/ 2023-24(VN) Page 106


*KVS RO DEHRADUN STUDENT SUPPORT MATERIAL MATHEMATICS/ XII/2023-24(VN)*
Based on above information answer the following questions
(i) Find the position vector of 𝐵𝐶⃗⃗⃗⃗⃗⃗⃗ . (1 MARK)
(ii) Find the distance between the temple and the zoo . (1 MARK)
⃗⃗⃗⃗⃗⃗⃗⃗
(iii) Find the unit vector along the vector 𝐴𝐷 (2 MARKS)
PROBLEM NO – 4
A building is to be constructed in the form of a triangular pyramid ABCD as shown in the figure .

Let angular points are 𝐴( 0,1,2) , 𝐵(3,0,1), 𝐶( 4,3,6 )𝑎𝑛𝑑 𝐷(2,3,2). G ne the point of intersection of the
medians of Δ BCD .
Based on above information answer the following questions
(i) Find the coordinates of point G . (1 MARK)
⃗⃗⃗⃗⃗⃗⃗
(ii) Find the position vector of 𝐴𝐺 . (1 MARK)
(iii) Using vectors find the area of ΔABC . (2 MARKS)
PROBLEM NO – 5
A barge is a long narrow boat with a flat bottom is used for carrying goods or people on a canal or a river . A
barge is pulled into harbor by two tug boats as shown in the figure .

Based on above information answer the following questions


⃗⃗⃗⃗⃗⃗⃗.
(i) Find the position vector of 𝐴𝐶 (1 MARK)
(ii) Find the unit vector along the vector ⃗⃗⃗⃗⃗⃗⃗
𝐵𝐶 (1 MARK)
(iii) Find a vector which is perpendicular to both the vectors 𝑎𝑛𝑑 ⃗⃗⃗⃗⃗⃗
𝑂𝐵 . (2 MARKS)

KVS RO DDN S SM XII MATHEMATICS/ 2023-24(VN) Page 107


*KVS RO DEHRADUN STUDENT SUPPORT MATERIAL MATHEMATICS/ XII/2023-24(VN)*

ANSWERS

VECTORS
Problem No 1
(i) 𝑖̂ + 2𝑘̂
(ii) √17
2𝑖̂ + 𝑗̂+ 𝑘̂
(iii)
√6

Problem No 2
𝜋
(i)
3
2
(ii) −
3

(iii) 16

Problem No 3
(i) 𝑖̂ + 2𝑗̂
(ii) √5
7𝑖̂ +6 𝑗̂
(iii)
√85

Problem No 4
(i) (3,2,3)
(ii) 3𝑖̂ + 𝑗̂ + 𝑘̂
(iii) 3√10 sq units .

Problem No 5
(i) −8𝑗̂
−5𝑖̂−5𝑗̂
(ii)
5 √2

(iii) − 62𝑘̂

KVS RO DDN S SM XII MATHEMATICS/ 2023-24(VN) Page 108


*KVS RO DEHRADUN STUDENT SUPPORT MATERIAL MATHEMATICS/ XII/2023-24(VN)*

Class Test

1 Find a vector in the direction of vector 4𝑖̂ − 5𝑗̂ + 3𝑘̂ which has magnitude 9 units.

2. Find the projection of the vector 7𝑖̂ + 𝑗̂ − 4𝑘̂ on the vector 2𝑖̂ + 6𝑗̂ + 3𝑘̂ .

3. Show that the points A (−2 𝑖̂ + 3𝑗̂ + 5𝑘̂ ), B (𝑖̂ + 2𝑗̂ + 3𝑘̂ ) and C (7 𝑖̂ − 𝑘̂) are collinear.

4. ⃗⃗
.If 𝑎⃗ = 2𝑖̂ +2𝑗̂ + 3𝑘̂, 𝑏⃗⃗ = −𝑖̂ + 2𝑗̂ + 𝑘̂ and 𝑐⃗⃗ =3𝑖̂ + 𝑗̂ are such that ⃗⃗⃗⃗⃗ is
𝑎⃗+𝜆𝑏

perpendicular to𝑐⃗, then find the value of 𝜆

5. Find the area of a triangle having the points A (1,1,2), B (2,3,5) and C (1,5,5) as its vertices

6. ⃗⃗
Let 𝑎⃗ = 𝑖̂ +4𝑗̂ + 2𝑘̂, 𝑏⃗⃗ = 3𝑖̂ − 2𝑗̂ +7𝑘̂ and 𝑐⃗⃗ =2𝑖̂ − 𝑗̂+ 4𝑘⃗⃗, Find a vector 𝑑⃗ which is

perpendicular to both 𝑎⃗ and 𝑏⃗⃗, and 𝑐⃗. 𝑑⃗=15.

7. If the vertices A, B, C of a triangle ABC are (1,2,3), (-1,0,0) and (0,1,2) respectively,

then find ∠ABC

8. If 𝑎⃗ = 5𝑖̂ − 𝑗̂ −3𝑘̂ and 𝑏⃗⃗ = 𝑖̂ +3𝑗̂ −5𝑘̂ , then show that the vectors 𝑎⃗ + 𝑏⃗⃗ and 𝑎⃗ − 𝑏⃗⃗

are orthogonal

9. ⃗⃗
For what value of 𝜆 are the vectors 𝑎⃗ =2𝑖̂ + 𝜆𝑗̂ + 𝑘̂, and 𝑏⃗⃗ =𝑖̂ − 2𝑗̂ +3𝑘̂ perpendicular

to each other?

10. Show that the vectors 2𝑖̂ − 𝑗̂ + 𝑘̂ , 𝑖̂ −3𝑗̂ −5𝑘̂ and 3 𝑖̂ −4𝑗̂ −4𝑘̂ from the vertices of a

right-angled triangle.

KVS RO DDN S SM XII MATHEMATICS/ 2023-24(VN) Page 109


*KVS RO DEHRADUN STUDENT SUPPORT MATERIAL MATHEMATICS/ XII/2023-24(VN)*

CLASS TEST
SECTION - A ( EACH QUESTION CARRY 1 MARK EACH)

1. If 4î + ĵ – k̂ and 3𝑖̂ + 𝑚𝑗̂ + 2𝑘̂ 𝑎𝑟𝑒 𝑎𝑡 𝑟𝑖𝑔ℎ𝑡 𝑎𝑛𝑔𝑙𝑒𝑠 𝑡ℎ𝑒𝑛 𝑚 𝑖𝑠 𝑒𝑞𝑢𝑎𝑙 𝑡𝑜

(𝑎) − 6 (𝑏 ) − 8 (𝑐 ) − 10 (𝑑) − 12

𝑎 2î + ĵ +2k̂ and ⃗⃗⃗⃗


2. If⃗⃗⃗⃗⃗⃗⃗⃗⃗⃗⃗⃗⃗= 𝑏 = 5𝑖̂ − 3𝑗̂ + 𝑘̂ , 𝑡ℎ𝑒𝑛 𝑡ℎ𝑒 𝑝𝑟𝑜𝑗𝑒𝑐𝑡𝑖𝑜𝑛 𝑜𝑓 ⃗⃗⃗⃗
𝑏 𝑜𝑛 𝑎
⃗⃗⃗⃗
(𝑎) 3 (𝑏) 4 (𝑐 ) 5 (𝑑) 6

SECTION - B ( EACH QUESTION CARRY 2 MARKS EACH)


⃗⃗⃗⃗ = 5î − ĵ − 3k̂ and 𝑏
3. If 𝑎 ⃗⃗⃗⃗ = 𝑖̂ + 3𝑗̂ − 5𝑘̂ then show that ⃗⃗⃗⃗ and
⃗⃗⃗⃗+ 𝑏
𝑎 𝑎 ⃗⃗⃗⃗ are
⃗⃗⃗⃗ − 𝑏
perpendicular .
4.𝐹𝑖𝑛𝑑 |𝑎 ⃗⃗⃗⃗| if two vectors 𝑎
⃗⃗⃗⃗ − 𝑏 ⃗⃗⃗⃗ are such that |𝑎
⃗⃗⃗⃗ 𝑎𝑛𝑑 𝑏 ⃗⃗⃗⃗| = 3 𝑎𝑛𝑑
⃗⃗⃗⃗| = 2 , |𝑏 ⃗⃗⃗⃗ . ⃗⃗⃗⃗
𝑎 𝑏 =4.

SECTION - C ( EACH QUESTION CARRY 3 MARKS EACH)


2 2
5. If |𝑎 ⃗⃗⃗⃗| + |𝑎
⃗⃗⃗⃗ × 𝑏 ⃗⃗⃗⃗ . ⃗⃗⃗⃗
𝑏 | = 400 𝑎𝑛𝑑 |𝑎 ⃗⃗⃗⃗| .
⃗⃗⃗⃗| = 5 , 𝑡ℎ𝑒𝑛 𝑓𝑖𝑛𝑑 𝑡ℎ𝑒 𝑣𝑎𝑙𝑢𝑒 𝑜𝑓 |𝑏
6. Find the area of the parallelogram whose adjacent sides are determined by the vectors ⃗⃗⃗⃗ =
𝑎
î − ĵ + 3k̂ and ⃗⃗⃗⃗⃗⃗⃗⃗⃗⃗⃗⃗⃗⃗
𝑏 = 2𝑖̂ − 7𝑗̂ + 𝑘̂ .
7. Using vectors find the area of the triangle with vertices 𝐴 ( 1,1,2) , 𝐵 (2,3,5) 𝑎𝑛𝑑 𝐶 (1,5. ,5) .

SECTION - D ( EACH QUESTION CARRY 5 MARKS EACH)


8. Show that the vectors2 î − ĵ + k̂ , = î − 3 ĵ − 5k̂ 𝑎𝑛𝑑 = 3 î − 4 ĵ − 4 k̂ form the
vertices of a right angled triangle .
9-The two adjacent sides of a parallelogram are 2 î − 4 ĵ + 5k̂ and î − 2 ĵ − 3k̂ . Find the
unit vector parallel to its diagonal . Also find its area .
10. The scalar product of the vector î + ĵ + k̂ 𝑤𝑖𝑡ℎ a unit vector along the sum of vectors
2 î + 4 ĵ − 5k̂ and λ î + 2 ĵ + 3k̂ is equal to one. Find the value of ,

KVS RO DDN S SM XII MATHEMATICS/ 2023-24(VN) Page 110


*KVS RO DEHRADUN STUDENT SUPPORT MATERIAL MATHEMATICS/ XII/2023-24(VN)*

CHAPTER 11:
THREE-DIMENSIONAL GEOMETRY
MINIMUM LEARNING LEVEL

1. The sum of direction cosines of z – axis is a) 0 b) 1 c) 2 d) 3

2. The angle between a line whose direction ratios are in the ratio 2 : 2 : 1 and a line joining (3, 1, 4) to (7, 2,
𝑧−2
12) .= 5 is
a) parallel to x-axis b) perpendicular to x – axis c) parallel to y-axis d) perpendicular to y-axis
Short Answer Types Questions
1. Find the direction cosines of a line which makes equal angles with the axes. How many such lines are
there?
2. Find the direction cosines of x, y and z-axis.
1 1
3. Find the acute angle which the line with direction-cosines , , n makes with positive direction
3 6
of z-axis.
4. Find the length of the perpendicular drawn from the point (4, –7, 3) on the y-axis.
5. Find the equation of the line passing through (1, –1, 1) and perpendicular to the lines joining the
points(4, 3, 2), (1, –1, 0) and (1, 2, –1), (2, 2, 1).
Long Answer Types Questions
1. The two lines x =ay +b, z =cy +d; and x =a' y +b’, z =c' y +d ' are perpendicular to each other,
find the relation involving a, a', c and c'.
y z y z
2. If the two lines L1: x  5,  , L2: x  2,  are perpendicular, then find value of α.
3 2 1 2  
1 x y  2 z  3 x 1 y 1 6  z
. Find the value of  so that the lines   and   are perpendicular to
3. 3 2 2 3 1 7
each other.
x 1 y  3 z  5 x 2 y4 z6
4. Show that the lines   and   intersect. Find their point of
3 5 7 1 3 5
intersection.
x y 1 z  2
5. Find the image of the point (1, 6, 3) in the line   .
1 2 3
x  2 y 1 z  3
6. Find the point on the line   at a distance 5 units from the point P (1, 3, 3).
3 2 2
7. Find the shortest distance between the following lines and hence write whether the lines are intersecting or
x 1 y 1 x 1 y  2
not.   z,  , z  2.
2 3 5 1
8. A line makes angles A, B, C and D with the diagonal of the cube prove that
Cos2A+ Cos2B+ Cos2C+ Cos2D = 4/3

KVS RO DDN S SM XII MATHEMATICS/ 2023-24(VN) Page 111


*KVS RO DEHRADUN STUDENT SUPPORT MATERIAL MATHEMATICS/ XII/2023-24(VN)*

Answers

1. b 2. a 3. a 4. c 5. d

Short Answer types questions

6. Let 𝛼 be the angle which the line makes with all axes.

∴Its direction cosines are cos𝛼 ,cos𝛼, cos𝛼

1
∴ cos2𝛼 + cos2𝛼 + cos2𝛼 = 1 ∴ cos2𝛼 =3

1
∴ cos𝛼 = ±
√3

1 1 1
∴The required direction cosines are (± ,± ,± ) There are four distinct
√3 √3 √3

lines.

7. (1, 0, 0); (0, 1, 0) and (0, 0, 1)

8. 𝜋
4

9. 5 Units

10. 𝒙−𝟏 𝒚+𝟏 𝒛−𝟏


= = −𝟏
−𝟐 𝟏

Long Answer Types Questions

1. aa' +cc' = -1

2. 7/3

3.   2

4. (1/2,-1/2,-3/2)

5. (1,0,7)

6. (-2,-1,3) or (4,3,7)

7. 19
, not intersecting
√195

KVS RO DDN S SM XII MATHEMATICS/ 2023-24(VN) Page 112


*KVS RO DEHRADUN STUDENT SUPPORT MATERIAL MATHEMATICS/ XII/2023-24(VN)*

HOTS

SECTION –A(MCQ)
1.Find the equation of the plane passing through the points P(1, 1, 1), Q(3, -1, 2), R(-3, 5, -4).

(A) x + 2y = 0 (B). x – y – 2 = 0 (C). -x + 2y – 2 = 0 (D). x + y – 2 = 0

2.The direction ratios of the normal to the plane 7x + 4y – 2z + 5 = 0 are:

(A). 7, 4,-2 (B) 7, 4, 5 (C) . 7, 4, 2 (D). 4, -2, 5

3.Direction ratio of line joining (2, 3, 4) and (−1, −2, 1), are:

(A). (−3, −5, −3) (B). (−3, 1, −3) (C) . (−1, −5, −3) (D). (−3, −5, 5)

4.The vector equation for the line passing through the points (–1, 0, 2) and

(3, 4, 6) is:

A. i + 2k + λ(4i + 4j + 4k) B. i – 2k + λ(4i + 4j + 4k)

C. -i+2k+ λ(4i + 4j + 4k) D. -i+2k+ λ(4i – 4j – 4k)

5.The direction cosines of the y-axis are:

A. (9, 0, 0) B. (1, 0, 0) C. (0, 1, 0) D. (0, 0, 1)

6.The vector equation of the symmetrical form of equation of straight line


𝒙−𝟓 𝒚+𝟒 𝒛−𝟔
= = is
𝟑 𝟕 𝟐

⃗⃗ = (𝟑𝒊 + 𝟕𝒋 + 𝟐𝒌) + 𝝁(𝟓𝒊 + 𝟒𝒋 − 𝟔𝒌)


(a) 𝒓 ⃗⃗ = (𝟓𝒊 + 𝟒𝒋 − 𝟔𝒌)+𝝁(𝟑𝒊 + 𝟕𝒋 + 𝟐𝒌)
(b)𝒓
⃗⃗ = (𝟓𝒊 − 𝟒𝐣 − 𝟔𝒌)+𝝁(𝟑𝒊 − 𝟕𝒋 − 𝟐𝒌)
(c) 𝒓 ⃗⃗ = (𝟓𝒊 − 𝟒𝒋 + 𝟔𝒌)+𝝁(𝟑𝒊 + 𝟕𝒋 + 𝟐𝒌)
(d)𝒓

7. The angle between a line whose direction ratios are in the ratio 2 : 2 : 1 and a line joining (3, 1,
4) to (7, 2, 12) is
(a) 𝒄𝒐𝒔−𝟏 (𝟐/𝟑) (b)𝒄𝒐𝒔−𝟏 (−𝟐/𝟑) (c) 𝒕𝒂𝒏−𝟏 (𝟐/𝟑) (d) None of these

8.If a line makes 600 and 450 angles with the positive direction of x-axis and z-axis respectively ,then
find the angle that it makes with positive direction of y-axis .Hence ,write the direction cosines of the
line.

KVS RO DDN S SM XII MATHEMATICS/ 2023-24(VN) Page 113


*KVS RO DEHRADUN STUDENT SUPPORT MATERIAL MATHEMATICS/ XII/2023-24(VN)*

SHORT ANSWER TYPE QUESTION


1. Find the direction cosines of the line passing through the two points (1,– 2, 4) and (– 1, 1, – 2).
2. Find the direction cosines of x, y and z-axis.
3. If a line makes angles 90o, 135 o, 45owith the x, y and z axes respectively, find its direction cosines.
4. If a line makes 600 and 450 angles with the positive direction of x-axis and z-axis respectively ,then find
the angle that it makes with positive direction of y-axis .Hence ,write the direction cosines of the line.
5. Find the length of the perpendicular drawn from the point (4, –7, 3) on the y-axis.
6. Find the coordinates of the foot of the perpendicular drawn from the point (2, –3, 4) on the y-axis.
7. Find the coordinates of the foot of the perpendicular drawn from the point (–2, 8, 7) on the XZ-plane.
8. Find the image of the point (2, –1, 4) in the YZ-plane.
9. Find the vector and Cartesian equations for the line passing through the points (1, 2, –1) and (2, 1, 1).
10. Find the vector equation of a line passing through the point (-2, 3, 2) and parallel to the line

r  (2î  3 ĵ)  (2î  3 ĵ  6k̂ ) .

11. Find the angle between the lines r  (2 ĵ  3k̂ )  (î  2 ĵ  2k̂ ) and

r  (2î  6 ĵ  3k̂ )  (2î  3 ĵ  6k̂) .
12. The two lines x =ay +b, z =cy +d ; and x =a' y +b' , z =c' y +d ' are perpendicular to each other,
find the relation involving a, a', c and c'.
y z y z
13. If the two lines L1 : x  5,  , L2 : x  2,  are perpendicular, then find value of
3 2 1 2  
α.
14. Find the vector equation of the line passing through the point (–1, 5, 4) and perpendicular to the
plane z = 0.
15.Find the direction cosines of a line which makes equal angles with the coordinate axes.
16.Find the vector and the Cartesian equations of the line through the point (5, 2, – 4) and which is parallel
to the vector (3 î +2 ˆj – 8 k̂ ) .
17. Show that the points (2,3,4) ,(-!,-2,1) and (5,8,7) are collinear
x 1 y  3 z  5 x 2 y4 z6
18. Show that the lines   and   intersect. Find their point of
3 5 7 1 3 5
intersection
19.Show that the line through the points (1, –1, 2), (3, 4, –2) is perpendicular to the line through the
points
(0, 3, 2) and (3, 5, 6).
20.Find the equation of the line which passes through the point (1, 2, 3) and is parallel to the vector

3 î + 2 ˆj -2 k̂ ).

1−𝑋 𝑌+2 𝑍 𝑋 𝑌 𝑍
20. Find the values of p so that the lines 3
= −5
= 1 and 1 = 𝑃 = 3
𝑎𝑟𝑒 𝑝𝑒𝑟𝑝𝑒𝑛𝑑𝑖𝑐𝑢𝑙𝑎𝑟 𝑡𝑜 𝑒𝑎𝑐ℎ 𝑜𝑡ℎ𝑒𝑟

KVS RO DDN S SM XII MATHEMATICS/ 2023-24(VN) Page 114


*KVS RO DEHRADUN STUDENT SUPPORT MATERIAL MATHEMATICS/ XII/2023-24(VN)*

ANSWERS

SECTION-A

1. Answer: D. x + y – 2 = 0
2. Answer: A. 7, 4,-2
3. Answer: A. (−3, −5, −3)
4. Answer: C. -i+2k+ λ(4i + 4j + 4k)
5. Answer: C. (0, 1, 0)
SECTION-B

1. (-2/7,3/7,-6/7) 2. (1, 0, 0); ( 0, 1, 0) and (0, 0, 1)


1 1 1,1, 1 1 1, 1
3. (0,− 2 , 2) 4.( )or (2,− )
2 2 √2 2 √2
5. 5 units 6. (0, –3, 0)

7. (–2, 0, 7) 8. (–2, –1, 4)


 x 1 x  2 z 1
9. r  (î  2 ĵ  k̂ )  (î  ĵ  2k̂ );  
1 1 2

𝟒
10. r  (2î  3 ĵ  2k̂)  (2î  3 ĵ  6k̂) 11𝒄𝒐𝒔−𝟏 𝟐𝟏
7 
12. aa' +cc' = -1 13. 14. r  î  5 ĵ  (4  )k̂
3
1 1 1
15. (± ,± ,± )
√3 √3 √3

Vector equation, r = (5 î +2 ˆj – 4 k̂ ) +  (3 î + 2 ˆj -8 k̂ ).Cartesian equation,
𝑋−5 𝑌−2 𝑍+4
16. = = .
3 2 −8
17.Sol: Let P(2,3,4), Q (-!,-2,1) and R(5,8,7) be the given points.
He direction ratios of PQ are(-1-2,-2-3,1-4) direction ratios of QR are
−3 −5 −3
(5+1,8+2,7-1) ie(-3,-5,-3) and (6,10,6) so 6 = 10 = 6
the lines are collinear.

x 1 y  3 z  5
18.Sol. Any point on     is 3  1, 5  3, 7  5
3 5 7
x 2 y4 z6
Any point on     is   2, 3  4, 5  6
1 3 5
If the lines intersect than for some  & 
3  1    2  3    3 .......i 
5  3  3  4  5  3  7.......ii 
7  5  5  6  7  5  11.....(iii )
1 3
From (i) & (ii )   ,    which satisfies (iii )
2 2
1 1 3
 given lines intersect and point of intersecti on is  ,  , 
2 2 2

KVS RO DDN S SM XII MATHEMATICS/ 2023-24(VN) Page 115


*KVS RO DEHRADUN STUDENT SUPPORT MATERIAL MATHEMATICS/ XII/2023-24(VN)*

19.Solution:

Given:
The points (1, –1, 2), (3, 4, –2) and (0, 3, 2), (3, 5, 6).

Let us consider AB be the line joining the points, (1, -1, 2) and (3, 4, -2), and CD be the line through the
points (0, 3, 2) and (3, 5, 6).

Now,

The direction ratios, a1, b1, c1 of AB are

(3 – 1), (4 – (-1)), (-2 – 2) = 2, 5, -4.

Similarly,

The direction ratios, a2, b2, c2 of CD are

(3 – 0), (5 – 3), (6 – 2) = 3, 2, 4.

Then, AB and CD will be perpendicular to each other, if a1a2 + b1b2 + c1c2 = 0

a1a2 + b1b2 + c1c2 = 2(3) + 5(2) + 4(-4)

= 6 + 10 – 16

=0

∴ AB and CD are perpendicular to each other.

20.P=2

KVS RO DDN S SM XII MATHEMATICS/ 2023-24(VN) Page 116


*KVS RO DEHRADUN STUDENT SUPPORT MATERIAL MATHEMATICS/ XII/2023-24(VN)*
CASE STUDY BASED PROBLEMS
THREE DIMENSIONALGEOMETRY – XII
PROBLEM NO - 1
The equation of a motion of a rocket is : 𝑥 = 2𝑡 , 𝑦 = −4𝑡 , 𝑧 = 4𝑡 , 𝑤ℎ𝑒𝑟𝑒 𝑡ℎ𝑒 𝑡𝑖𝑚𝑒 ′𝑡′ is given in seconds
and the distance is measured in kilometres .

Based on above information answer the following questions


(i) Find the equation of the path of the rocket . (1 MARK)
(ii) If the position of rocket at a certain instant of time is (3, −6, 6) , then what will be the height of the rocket
from the ground , which is along the 𝑥𝑦 − 𝑝𝑙𝑎𝑛𝑒 . (1 MARK)
(iii) At what distance will the rocket be from the starting point (0,0,0) in 10 seconds. (2 MARKS)

PROBLEM NO - 2
If 𝑎1 , 𝑏1 , 𝑐1 𝑎𝑛𝑑 𝑎2 , 𝑏2 , 𝑐2 are direction rations of two lines 𝐿1 𝑎𝑛𝑑 𝐿2 respectively then
𝑎1 𝑏1 𝑐1
𝐿1 ‖ 𝐿2 𝑖𝑓𝑓 = = and 𝐿1 ⏊ 𝐿2 𝑖𝑓 𝑎1 𝑎2 + 𝑏1 𝑏2 + 𝑐1 𝑐2 = 0 .
𝑎2 𝑏2 𝑐2

Based on above information answer the following questions

−𝑥+2 𝑦−1 𝑧+3 𝑥+2 𝑦−4 𝑧−5


(i) Find the value of 𝜆 such that the lines = = 𝑎𝑛𝑑 = = are perpendicular .
−2 7 −3 𝜆 2 4
(1
MARK)
𝑥+3 𝑦−4 𝑧+8
(ii) Find the equation of line passing through (−2 , 4 , −5) parallel to the line 3
= −5 = 6 . (1
MARK)
(iii) Find the coordinates of the foot of perpendicular drawnfrom the point 𝐴(1 , 2, 1) to the line
𝑥−1 𝑦−4 𝑧−6
= = . (2
4 0 −2
MARKS)
PROBLEM NO - 3
The Indian coast guard while patrolling ,saw a suspicious boat with four men. They were nowhere looking like
fisherman . The soldiers were closely observing the movement of the boat for an opportunity to seize the boat .
They observe that the boat is moving along a planar surface. At an instant of time , the coordinates of the

KVS RO DDN S SM XII MATHEMATICS/ 2023-24(VN) Page 117


*KVS RO DEHRADUN STUDENT SUPPORT MATERIAL MATHEMATICS/ XII/2023-24(VN)*

position of coast guard helicopter and boat are ( 4 , 3, 7) 𝑎𝑛𝑑 (1 , −1 , −5) 𝑟𝑒𝑠𝑝𝑒𝑐𝑡𝑖𝑣𝑒𝑙𝑦 .

Based on above information answer the following questions


(i) The soldier decides to shoot the boat at given instant of time , where the distance is measured in
metres, then find the distance travelled by the bullet . (1 MARK)
(ii) If the speed of the bullet is 30 m/s find the time taken to hit the target . (1 MARK)
(iii) At the given instant find the path of the bullet so that it hits the target ( assuming the path followed
by a bullet to be a straight line) (2 MARKS)
PROBLEM NO - 4
Shortest distance between two skew lines.
⃗⃗⃗⃗⃗
1 × ⃗⃗⃗⃗⃗
For lines 𝑟⃗⃗⃗ = ⃗⃗⃗⃗⃗ ⃗⃗⃗⃗1 𝑎𝑛𝑑 𝑟⃗⃗⃗ = ⃗⃗⃗⃗⃗
𝑎1 + 𝜆𝑏 ⃗⃗⃗⃗⃗2 then shortest distance = |(𝑎⃗⃗⃗⃗⃗2
𝑎2 + 𝜇𝑏
− ⃗⃗⃗⃗⃗)
𝑎1 .(𝑏 𝑏2 )
| .
⃗⃗⃗⃗⃗
|𝑏 1 × ⃗⃗⃗⃗⃗
𝑏 2 |

Two lines will intersect if shortest distance is zero i.e. (𝑎


⃗⃗⃗⃗⃗2 − ⃗⃗⃗⃗⃗) ⃗⃗⃗⃗1 × 𝑏
𝑎1 . (𝑏 ⃗⃗⃗⃗⃗2 ) = 0
Based on above information answer the following questions
(i) Find the shortest distance between the following lines 𝑟⃗⃗⃗ = (î − 2 ĵ + 3k̂ ) + λ ( −î + ĵ − 2 𝑘̂ ) and
𝑟⃗⃗⃗ = (î − ĵ − k̂ ) + μ ( î + 2 ĵ − 2 𝑘̂ ) . (2 MARKS)
(ii) Find the value of 𝑥 𝑠𝑢𝑐ℎ 𝑡ℎ𝑎𝑡 lines 𝑟⃗⃗⃗ = (3 î + 2 ĵ − 4k̂ ) + λ ( î + 2 ĵ + 2 𝑘̂ ) and
𝑟⃗⃗⃗ = (5 î − 2ĵ ) + μ ( 𝑥 î + 2 ĵ + 6 𝑘̂ ) intersects . (2 MARKS)

KVS RO DDN S SM XII MATHEMATICS/ 2023-24(VN) Page 118


*KVS RO DEHRADUN STUDENT SUPPORT MATERIAL MATHEMATICS/ XII/2023-24(VN)*

PROBLEM NO - 5
The direction of a line OP is determined bt the angles 𝛼 , 𝛽 , 𝛾 , 𝑤ℎ𝑖𝑐ℎ 𝑖𝑡 makes with OX , OY and OZ . These
angles𝛼 , 𝛽 𝑎𝑛𝑑 𝛾 are called direction angles and cosines of these angles namely 𝑐𝑜𝑠𝛼 , 𝑐𝑜𝑠𝛽 𝑎𝑛𝑑 𝑐𝑜𝑠𝛾 are
called direction cosines .
Direction cosines of a line are denoted by 𝑙 , 𝑚 𝑎𝑛𝑑 𝑛 , 𝑙 = 𝑐𝑜𝑠𝛼 , 𝑚 = 𝑐𝑜𝑠𝛽 𝑎𝑛𝑑 𝑛 = 𝑐𝑜𝑠𝛾 .
Sum of squares of direction cosines is always 1 .𝑙 2 + 𝑚2 + 𝑛2 = 1 .
Numbers proportional to the direction cosines of a line are called direction ratios. If 𝑎, 𝑏 𝑎𝑛𝑑 𝑐 𝑎𝑟𝑒 direction
𝑙 𝑚 𝑛
ratios of a line then ,𝑎 = 𝑏
= 𝑐
.

Based on above information answer the following questions

(i) The direction ratio of a line are 2 , 3 𝑎𝑛𝑑 − 6 , 𝑡ℎ𝑒𝑛 𝑓𝑖𝑛𝑑 𝑖𝑡𝑠 𝑑𝑖𝑟𝑒𝑐𝑡𝑖𝑜𝑛 𝑐𝑜𝑠𝑖𝑛𝑒𝑠 . (1 MARK)
(ii) Find the direction cosines of a line which makes equal angles with the coordinate axes . (1 MARK)
(iii) If a line makes angle 𝛼 , 𝛽 , 𝛾 with 𝑥 − 𝑎𝑥𝑖𝑠 , 𝑦 − 𝑎𝑥𝑖𝑠 𝑎𝑛𝑑 𝑧 − 𝑎𝑥𝑖𝑠 𝑟𝑒𝑠𝑝𝑒𝑐𝑡𝑖𝑣𝑒𝑙𝑦 then fin the value of
𝑠𝑖𝑛2 𝛼 + 𝑠𝑖𝑛2 𝛽 + 𝑠𝑖𝑛2 𝛾 . (2 MARKS)

KVS RO DDN S SM XII MATHEMATICS/ 2023-24(VN) Page 119


*KVS RO DEHRADUN STUDENT SUPPORT MATERIAL MATHEMATICS/ XII/2023-24(VN)*

ANSWERS CASE STUDY BASED PROBLEM XII


Three dimensional geometry

Problem No 1
𝑥 𝑦 𝑧
(i) = =
2 −4 4

(ii) 6 km
(iii) 60 km
Problem No 2
(i) 𝜆 = 1
𝑥+2 𝑦−4 𝑧+5
(ii) = =
3 −5 6

(iii) (3,4,5)
Problem No 3

(i) 13 m
13
(ii) 𝑠𝑒𝑐
30
𝑥−4 𝑦−3 𝑧−7
(iii) = =
−3 −4 −12

Problem No 4
8
(i)
√29

(ii) 𝑥 = 3

Problem No 5
2 3 −6
(i) , ,
7 7 7
1 1 1
(ii) ± ,± ,±
√3 √3 √3

(iii) 2

KVS RO DDN S SM XII MATHEMATICS/ 2023-24(VN) Page 120


*KVS RO DEHRADUN STUDENT SUPPORT MATERIAL MATHEMATICS/ XII/2023-24(VN)*

CLASS TEST
SECTION-A(EACH 2 MARKS)
Q1. Find the direction cosines of the line passing through the following points:
P (-2, 4, -5) and Q (1, 2,3).
Q2.Find the coordinates of the foot of the perpendicular drawn from the point A(1, 8,4) to the line
joining the points B(0, -1,3) and C(2, -3,-1).
𝑥+2 𝑦+1 𝑧−3
Q3.Find the points on the line = = at a distance of 5 units from the point P (1, 3, 3)
3 2 2

Q4.Find the angle between the lines


 
r  (2 ĵ  3k̂ )  (î  2 ĵ  2k̂ ) and r  (2î  6 ĵ  3k̂ )  (2î  3 ĵ  6k̂) .
2𝑥−1 𝑦+2 𝑧−3
Q5.The Cartesian equation of a line AB is = = . Find the direction cosines of a line
√3 2 3

parallel to AB.
SECTION –B(EACH 4 MARKS)
1. Find the equation of the line passing through (1, –1, 1) and perpendicular to the lines joining the
points (4, 3, 2), (1, –1, 0) and (1, 2, –1), (2, 2, 1).
1−𝑋 𝑌−2 𝑍−3 𝑥−1 𝑌−1 6−𝑍
2. Find the value of Also that the lines = = and = =
3 2𝐴 2 3 1 7

are perpendicular to each other.


3. Find the shortest distance between the following lines :
1−𝑋 𝑌−2 𝑍−3 𝑥−1 𝑌−1 6−𝑍
= = and = =
3 2 2 3 1 7
4. Find the shortest distance b/w the following two lines:
 
r = (1 +  ) î + (2 –  ) ˆj + (  + 1) k̂ and r = (2 î – ˆj – k̂ ) +  (2 î + ˆj + 2 k̂ ).

x 1 y  2 z  3 x  4 y 1
5. Show that the lines   and   z intersect. Find their point of intersection.
2 3 4 5 2

KVS RO DDN S SM XII MATHEMATICS/ 2023-24(VN) Page 121


*KVS RO DEHRADUN STUDENT SUPPORT MATERIAL MATHEMATICS/ XII/2023-24(VN)*

CLASS TEST

SECTIONA
1 The line which passes through the origin and intersect the two lines 2
𝑥−1 𝑦+3 𝑧−5 𝑥−4 𝑦+3 𝑧−14
= = , and = = is,
2 4 3 2 3 4
𝑥 𝑦 𝑧 𝑥 𝑦 𝑧 𝑥 𝑦 𝑧 𝑥 𝑦 𝑧
(a) = = (b) = = (c) = = (d) = =
1 −3 5 −1 3 5 1 3 −5 1 4 −5

2 The vector equation of the symmetrical form of equation of straight line 2


𝑥−5 𝑦+4 𝑧−6
= = is
3 7 2
(a) 𝑟⃗ = (3𝑖 + 7𝑗 + 2𝑘) + 𝜇(5𝑖 + 4𝑗 − 6𝑘) (b)𝑟⃗ = (5𝑖 + 4𝑗 − 6𝑘)+𝜇(3𝑖 + 7𝑗 + 2𝑘)
(c) 𝑟⃗ = (5𝑖 − 4𝑗 − 6𝑘)+𝜇(3𝑖 − 7𝑗 − 2𝑘) (d)𝑟⃗ = (5𝑖 − 4𝑗 + 6𝑘)+𝜇(3𝑖 + 7𝑗 + 2𝑘)

3. Study the two statements labeled as assertion (A) reason (R). 1


Point out if : (A) Both Assertion and reason are true and reason is correct explanation of
assertion. (B) Assertion and reason both are true but reason is not the correct explanation
of assertion. (C) Assertion is true, reason is false. (D) Assertion is false, reason is true.
Assertion: Equation of line Passing through the point (1,2,3) and ( 2,-1,5) is (x – 1) /1 = (y
− 2 /(−3) = (z – 3)/ 2
Reason : Equation of line passing through the point (x1,y1,z1) and (x2,y2,z2) is
(x − x 1)/( x2 − x 1 )= (y − y 1)/( y2 − y 1) =( z − z 1)/( z 2 − z 1 )

4. y z y z 3
If the two lines L1 : x  2,  x  5,  , L2 : are perpendicular, then find
1 2   3 2
value of α.
5. Find the equation of the line passing through (1, –1, 1) and perpendicular to the lines 5
joining the points (4, 3, 2), (1, –1, 0) and (1, 2, –1), (2, 2, 1).
x 1 y 1 6  z
so that the lines 1  x  y  2  z  3
6. 3
Find the value of  and   are
3 2 2 3 1 7
perpendicular to each other.
7. Find the shortest distance between the following lines : 3
x 3 y5 z7 x 1 y 1 z 1
  and  
1 2 1 7 6 1
8.  3
Find the angle between the lines r  (2 ĵ  3k̂ )  (î  2 ĵ  2k̂ ) and

r  (2î  6 ĵ  3k̂ )  (2î  3 ĵ  6k̂) .

9. 𝑥−1 𝑦−2 𝑧−3 5


Find the image of the point P (5, 9, 3) in the line = = .
2 3 4

10. Based on the above information ,answer the following questions 5


What is the Cartesian equation of line along EA ?
𝐹𝑖𝑛𝑑 𝑡ℎ𝑒 𝑣𝑒𝑐𝑡𝑜𝑟 𝑒𝑞𝑢𝑎𝑡𝑖𝑜𝑛 𝑜𝑓 the vector ⃗⃗⃗⃗⃗⃗
𝐸𝐷

KVS RO DDN S SM XII MATHEMATICS/ 2023-24(VN) Page 122


*KVS RO DEHRADUN STUDENT SUPPORT MATERIAL MATHEMATICS/ XII/2023-24(VN)*

ANSWERS/HINTS

1 A 6 -2
2 D 7 2 29
3 A 8 𝟒
𝐜𝐨𝐬 −𝟏 ( )
√𝟐𝟏
4 7/3 9 (1,1,11)
5 𝑥−1 𝑦+1 𝑧−1 10 𝑥 𝑦 𝑧−24
(i) −4 =3 = 12 (ii) -8𝑖̂-6𝑗̂-24𝑘̂
= =
−2 1 1

KVS RO DDN S SM XII MATHEMATICS/ 2023-24(VN) Page 123


*KVS RO DEHRADUN STUDENT SUPPORT MATERIAL MATHEMATICS/ XII/2023-24(VN)*

CHAPTER 12
LINEAR PROGRAMMING PROBLEMS
MINIMUM LEARNING LEVEL

Q.1The linear inequalities or equations or restrictions on the variables of a linear


programming problem are called:
A. a constraint B. Decision variables
C. Objective function D. None of the above

Q.2A set of values of decision variables that satisfies the linear constraints and non-negativity
conditions of an L.P.P. is called its:
A. Unbounded solution B. Optimum solution
C. Feasible solution D. None of these

Q.3Solve the following LPP graphically:


Maximise Z = 2x + 3y, subject to x + y ≤ 4, x ≥ 0, y ≥ 0.
Q.4What is the maximum value of Z = 3x + 4y subjected to constraints x + y ≤ 4, x ≥ 0 and y ≥
0?
Q5. Solve the following LPP graphically:
Maximise Z = 510x + 675y, subject to x + y ≤ 300
2x + 3y ≤ 720, x ≥ 0, y ≥ 0.
Q.6Solve the following linear programming problem graphically:
Minimise Z = 200 x + 500 y subject to the constraints:
x + 2y ≥ 10; 3x + 4y ≤ 24 ; x ≥ 0; y ≥ 0

Q.7 Determine the minimum value of Z = 3x + 2y (if any), if the feasible region for an LPP is
shown in Fig.

KVS RO DDN S SM XII MATHEMATICS/ 2023-24(VN) Page 124


*KVS RO DEHRADUN STUDENT SUPPORT MATERIAL MATHEMATICS/ XII/2023-24(VN)*

Q.8 Solve the following LPP graphically:


Minimise Z = 5x + 10y subject to the constraints
x + 2y ≤ 120
x + y ≥ 60,
x – 2y > 0 and x, y ≥ 0

Q.9Solve the following LPPgraphically :


Maximise and minimise Z = x + 2y subject to the constraints
x + 2 y ≥ 100
2x – y ≤ 0
2x+ y ≤ 200
x, y ≥ 0

Q.10 Solve the following LPP graphically


Minimise Z = 50x + 70y , subject to the constraints:
2x + y ≥ 8 , x + 2y ≥ 10 , x, y ≥ 0

Q.11 Minimise Z=-3x+4y


subject to x+2y≤8,3x+2y≤12,x≥0,y≥0

Q.12 Maximise Z=5x+3y


subject to 3x+5y≤15,5x+2y≤10,x≥0,y≥0.

Q.13 Minimise Z=3x+5y


subject to x+3y≥3,x+y≥2, x≥0,y≥0.

Q.14 Maximise Z = 3x+2y


subject to x+2y≤10,3x+y≤15, x≥0,y≥0.

KVS RO DDN S SM XII MATHEMATICS/ 2023-24(VN) Page 125


*KVS RO DEHRADUN STUDENT SUPPORT MATERIAL MATHEMATICS/ XII/2023-24(VN)*

Answer key
1 : A. a constraint
2. C. Feasible solution
3. The feasible region OAB is bounded and the maximum value will occur at a corner point of the
feasible region.
Corner Points are O(0, 0), A (4, 0) and B (0, 4).

Corner Point Value of Z

O(0, 0) 2 (0) + 3(0) = 0

A (4, 0) 2 (4) + 3(0) = 8

B (0, 4) 2 (0) + 3 (4)= 12 ← max

4.O (0, 0), A (4, 0), and B (0, 4) are the corner points of the feasible region. The values of Z at these
points are given below:
Corner point Z = 3x + 4y

O (0, 0) 0

A (4, 0) 12

B (0, 4) 16

Hence, the maximum value of Z is 16 at point B (0, 4)

5.

Corner point Value of Z


A(300, 0) 153000
B(180, 120) 172800 = Maximum
C(0, 240) 162000

KVS RO DDN S SM XII MATHEMATICS/ 2023-24(VN) Page 126


*KVS RO DEHRADUN STUDENT SUPPORT MATERIAL MATHEMATICS/ XII/2023-24(VN)*

6.The shaded region in the above figure is the feasible region ABC. The coordinates of corner point A,
B and C are (0,5), (4,3) and (0,6) respectively.

Corner point Value of Z


(0, 5) 2500
(4, 3) 2300← Minimum
(0, 6) 3000

Hence, the minimum value of Z is 2300 is at the point (4, 3).

7.The feasible region (R) is unbounded. Therefore, a minimum of Z may or may not exist. If it exists, it
will be at the corner point Fig.
Corner Point Value of Z

A (12, 0) 3(12) + 2(0) = 36

B(4, 2) 3(4) + 2(2) = 16

C(1, 5) 3(1) + 2(5) = 13 (smallest)

D(0, 10) 3(0) + 2(10) = 20

Let us graph 3x + 2y < 13. We see that the open half plane determined by 3x + 2y < 13 and R do not have a
common point. So, the smallest value 13 is the minimum value of Z.

8.

Corner point Z = 5x + 10y


A (60, 0) 300 (minimum)
8(120,0) 600
C (60, 30) 600
D (40, 20) 400

Clearly, the minimum value of Z is 300 at the points (60, 0)

KVS RO DDN S SM XII MATHEMATICS/ 2023-24(VN) Page 127


*KVS RO DEHRADUN STUDENT SUPPORT MATERIAL MATHEMATICS/ XII/2023-24(VN)*
9.

Comer points Z = x + 2y
A(0, 50) Z = 0 + 2 × 50 = 100
B(20, 40) Z = 20 + 2 × 40 = 100
C(50, 100) Z = 50 + 2 × 100 = 250
O(0, 200) Z = 0 + 2 × 200 = 400

The maximum value of Z is 400 at 0(0, 200) and the minimum value of Z is 100 at all the points on the
line segment joining A(0, 50) and B(20, 40).

10.

the minimum value of Z is 380 obtained


at the point (2, 4)
Min Z=-12 at (4,0).
235 20 45
Max Z= 19 at (19 , 19).
3 1
Min Z=7 at (2 , 2).
Max Z=18 at (4,3).

Corner point Value of Z


A(0, 8) 560
B(2, 4) 380 = Minimum
C(10, 0) 500

KVS RO DDN S SM XII MATHEMATICS/ 2023-24(VN) Page 128


*KVS RO DEHRADUN STUDENT SUPPORT MATERIAL MATHEMATICS/ XII/2023-24(VN)*

HOTS
Multiple Choice Questions ( 1 Mark each)
Q1 : The point which does not lie in the half plane 2x + 3y – 15 <0 is
(a): (4,2) (b): (2,4) (c): (3,1) (d): (5,1)
Q2: The solution set of the inequality 3x + 5y < 4 is
(a). an open half –plane not containing the origin.
(b). a closed half containing the origin.
(c). the whole XYplane not containing the line 3x + 5y = 4
(d). an open half –plane containing the origin.
Q3: Shaded region gives the feasible region of aLPP on the following graph.

The point at which the objective function given by Z = x + y attains maximum value is
(a) Point A (b) Point O (c) Point B (d ) Point C

Q4: Corner points of a feasible region determined by the system of linear constraints are (0,0), (0,4) and (12,6).
Let Z = 3x - 4y be the objective function. The maximum and minimum values of Z are given by
(a) 12 and -16 (b) -16 and 0 (c) 12 and 0 (d) 36 and -16
Q5: Corner points of a feasible region determined by the system of linear constraints are (0,3), (1,1) and (3,0).
Let Z = px +qy where p, q >0. If Z is minimum at (3,0) and (1,1) then
(a) p = 2q (b) 2p = q (c) p= 3q (d) p = q
Q6: In a linear programming problem, the constraints on the decision variables x and y are
x + 2y≥ 100, 2x - y≤ 0, 2x + y≤ 200, x≥ 0, 𝑦 ≥ 0
The feasible region is
(a). unbounded (b) not in first quadrant
(c) bounded in first quadrant (d) does not exist
Q7: ALPP is given by:
Maximise Z = 2x + 3y, subject to the constraints: x + 2y≤ 10, 2x + y≤ 14, x≥ 0, 𝑦 ≥ 0
Maximum value of Z occurs at
( a) no point (b) a unique point
(c) infinitely many points (d) two points only
Q8: The corner points of aLPP are given by (15, 20), (40, 15) and (2, 72). If the objective function is
Z = 6x + 3y then the difference of Maximum and Minimum values of Z is
(a). 57 (b). 285 (c).78 (d) 135
Q9: The corner points of a feasible region determined by the system of linear constraints are (40,0), (20,30)
(60,20) and (60,0).The objective function is Z = 5x+3y . Compare the quantity in column A and column B
Column A Column B
Maximum of Z 390
Which of the following is correct:
(a). The quantity in column A is greater.
(b). The quantity in column B is greater.
(c). Both the quantities in column A and column B are equal
(d). The relationship between above two quantities cannot be determine on the basis of information supplied.

KVS RO DDN S SM XII MATHEMATICS/ 2023-24(VN) Page 129


*KVS RO DEHRADUN STUDENT SUPPORT MATERIAL MATHEMATICS/ XII/2023-24(VN)*

Q10: In the given graph, shaded region gives the feasible region for LPP. The objective function Z = 4x + y will
be maximum at point

(a). (0,0) (b). (20, 30) (c).(0, 50) (d). (30, 0)

KVS RO DDN S SM XII MATHEMATICS/ 2023-24(VN) Page 130


*KVS RO DEHRADUN STUDENT SUPPORT MATERIAL MATHEMATICS/ XII/2023-24(VN)*

Short Answer Type

Q1: Solve the following LPP graphically:

Minimize: Z = x + y
Subject to the constraints 3x + 2y ≥ 12, x + 3y ≥ 11 and x≥ 0, y≥0
Q2: Solve the following LPP graphically:
Maximize Z= 600x +400y
Subject to the constraints : x + 2y ≤ 12, 2x + y ≤ 12, x + 1.25y ≥ 5, x ≥ 0, y≥ 0
Q3: Solve the following LPP graphically:
Maximize Z = 2.50x + y
Subject to the constraints x + 3y ≤ 12, 3x + y ≤ 12, x ≥ 0, y≥ 0
Q4: Solve the following LPP graphically:
Maximize Z = 2x + 5 y
Subject to the constraints x + 2y ≤ 4, 3x + y≤ 6, x + y ≤ 4, x ≥ 0, y≥ 0
Q5: Solve the following LPP graphically:
Minimize Z = 3x + 5y
Subject to the constraints −2x + y ≤ 4, x + y ≥ 3, x – 2y ≤ 2, x, y ≥ 0

KVS RO DDN S SM XII MATHEMATICS/ 2023-24(VN) Page 131


*KVS RO DEHRADUN STUDENT SUPPORT MATERIAL MATHEMATICS/ XII/2023-24(VN)*
Answer Key:
MCQ
Ans1: (b): (2,4)
Ans2: (d). an open half –plane containing the origin.
Ans3: (c) Point B
Ans4: (a) 12 and -16
Ans5: (b) 2p = q
Ans6: (c) bounded in first quadrant
Ans7: (b) a unique point
Ans8: (d) 135
Ans9: (b). The quantity in column B is greater.
Ans10: (d). (30, 0)
SA
Ans1:

Corner Points Z=x+y Remark


A(0,6) 6
B(2,3) 5 Minimum value
D(11,0) 11

Ans2:

KVS RO DDN S SM XII MATHEMATICS/ 2023-24(VN) Page 132


*KVS RO DEHRADUN STUDENT SUPPORT MATERIAL MATHEMATICS/ XII/2023-24(VN)*

Corner points Z= 600x + 4000y Remark


A(5,0) 3000
B(6,0) 3600
C(4,4) 4000 Maximum Value
D(0,6) 2400
E(0,4) 1600

Ans3: The maximum value of Z is 10.50 at x =3, y=3


46 8 6
Ans4: Maximum Z = when x = and y =
5 5 5
29 8 1
Ans5: The minimum value of Z is at x = and y =
3 3 3

CASE BASED QUESTION


Q1: On solving aLPP Mudit find that the feasible region of the system of linear constraints is given by following
figure

Answer the following questions


(a). Taking the feasible region into the account determine the constrains for the LPP.
(b). If objective function Z = 400x + 300 y, Find the maximum value.

ANSWER

Ans1: Determine the equation of lines making boundaries of feasible region by using two points form.
(a). The constraints are given by:
x + y ≤ 200, y ≥ 4x , x ≥ 20 and y ≥ 0
(b). Maximum value of Z is 64,000 at (40,160)

KVS RO DDN S SM XII MATHEMATICS/ 2023-24(VN) Page 133


*KVS RO DEHRADUN STUDENT SUPPORT MATERIAL MATHEMATICS/ XII/2023-24(VN)*

CLASS TEST
SECTION A (1×2=2)
Q.1 The linear inequalities or equations or restrictions on the variables of a linear programming
problem are called:
A. a constraint B. Decision variables
C. Objective function D. None of the above

Q.2 A set of values of decision variables that satisfies the linear constraints and non-negativity
conditions of an L.P.P. is called its:
A. Unbounded solution B. Optimum solution
C. Feasible solution D. None of these

SECTION B( 2×2=4)
Q.3 Solve the following LPP graphically:
Maximise Z = 2x + 3y, subject to x + y ≤ 4, x ≥ 0, y ≥ 0.

Q.4 What is the maximum value of Z = 3x + 4y subjected to constraints x + y ≤ 4, x ≥ 0 and y ≥ 0?

SECTION C(3×3=9)
Q5. Solve the following LPP graphically:
Maximise Z = 510x + 675y, subject to x + y ≤ 300
2x + 3y ≤ 720, x ≥ 0, y ≥ 0.
Q.6 Solve the following linear programming problem graphically:
Minimise Z = 200 x + 500 y subject to the constraints:
x + 2y ≥ 10; 3x + 4y ≤ 24 ; x ≥ 0; y ≥ 0

Q.7 Determine the minimum value of Z = 3x + 2y (if any), if the feasible region for an LPP is
shown in Fig.
SECTION D(5×2=10)
Q.8 Solve the following LPP graphically:
Minimise Z = 5x + 10y subject to the constraints
x + 2y ≤ 120
x + y ≥ 60,
x – 2y > 0 and x, y ≥ 0

Q.9 Solve the following LPPgraphically :


Maximise and minimise Z = x + 2y subject to the constraints
x + 2 y ≥ 100
2x – y ≤ 0
2x+ y ≤ 200
x, y ≥ 0

Q.10 Solve the following LPP graphically


Minimise Z = 50x + 70y , subject to the constraints:
2x + y ≥ 8 , x + 2y ≥ 10 , x, y ≥ 0

KVS RO DDN S SM XII MATHEMATICS/ 2023-24(VN) Page 134


*KVS RO DEHRADUN STUDENT SUPPORT MATERIAL MATHEMATICS/ XII/2023-24(VN)*

CLASS TEST
Q1: The solution set of the inequality 4x + 5y > 4 is
(a). an open half –plane not containing the origin.
(b). a closed half containing the origin.
(c). the whole XYplane not containing the line 4x + 5y = 4
(d). an open half –plane containing the origin.
Q2: Corner points of a feasible region determined by the system of linear constraints are (0,0), (0,4)
and (12,6). Let Z = 3x - 4y be the objective function. The maximum and minimum values of Z are
given by
(a) 12 and -16 (b) -16 and 0 (c) 12 and 0 (d) 36 and -16
Q3: Corner points of a feasible region determined by the system of linear constraints are (0,3),
(1,1) and (3,0). Let Z = px +qy where p, q >0. If Z is minimum at (3,0) and (1,1) then
(a) p = 2q (b) 2p = q (c) p= 3q (d) p = q
Q4: A LPP is given by:
Maximize Z = 2x + 3y, subject to the constraints: x + 2y≤ 10, 2x + y≤ 14, x≥ 0, 𝑦 ≥ 0
Maximum value of Z occurs at
( a) no point (b) a unique point
(c) infinitely many points (d) two points only
Q5: In the given graph, shaded region gives the feasible region for LPP. The objective function Z =
4x + y will be maximum at point

(a). (0,0) (b). (20, 30) (c).(0, 50) (d). (30, 0)

Q6: In feasible region of aLPP, minimum value of objective function occurs at two corner points
then Z is minimum at
(a) one point (b) two points
(c)infinitely many points (d)none of above

KVS RO DDN S SM XII MATHEMATICS/ 2023-24(VN) Page 135


*KVS RO DEHRADUN STUDENT SUPPORT MATERIAL MATHEMATICS/ XII/2023-24(VN)*

Q7: The corner points of the shaded bounded feasible region of an LPP are (0, 50), (20,40),
(50,100) and (0,200) as shown in the figure. The minimum value of the objective function
Z = x + 2y occurs at

(a). (0, 50) only


(b). (20, 40) only
(c). (0, 50) only and(20, 40) only
(d). At the every point of the line segment joining the points (0, 50) and (20,40).
Q8: Solve the following LPP graphically:
Maximize Z = 2.50x + y
Subject to the constraints x + 3y ≤ 12, 3x + y ≤ 12, x ≥ 0, y≥ 0
Q9: Solve the following LPP graphically:
Maximize Z = 2x + 5 y
Subject to the constraints x + 2y ≤ 4, 3x + y≤ 6, x + y ≤ 4, x ≥ 0, y≥ 0
Q10: Solve the following LPP graphically:
Minimize Z = 3x + 5y
Subject to the constraints −2x + y ≤ 4, x + y ≥ 3, x – 2y ≤ 2, x, y ≥ 0

KVS RO DDN S SM XII MATHEMATICS/ 2023-24(VN) Page 136


*KVS RO DEHRADUN STUDENT SUPPORT MATERIAL MATHEMATICS/ XII/2023-24(VN)*

CHAPTER 13
PROBABILITY
MINIMUM LEARNING LEVEL

Short answer type questions

1. Given P (A) = 0.2, P (B) = 0.3 and P (𝐴 ∩ 𝐵) = 0.1. Find P (A/B)


2. Given P(A) = ½ , P(B) = 1/3 and P(A∩ 𝐵) = 1/6. Are the events A and B independent?
3. A bag contains 5 white, 7 red and 3 black balls. If three balls are drawn one by one without
replacement, find what the probability that none is red is.
4. If A and B are independent events, Find P(B) if P(A  B) = 0.60 and P(A) = 0.35
5. If P (A) = 0.8, P (B) = 0.5 and P (B|A) = 0.4, what is the value of P (A ∩ B)?
(a) 0.32 (b) 0.25 (c) 0.1 (d) 0.5
6. If P(A) = 0.4, P(B) = 0.7 and P(B/A) = 0.6. Find P(A ∪ B).
(a) 0.46 (b) 0.86 (c) 0.76 (d) 0.54

7. The probability of solving the specific problems independently by A and B are 1/2 and 1/3
respectively.
If both try to solve the problem independently, find the probability that exactly one of them solves
the problem.
(a) 1 (b) ½ (c) ⅓ (d) ¼
8. The probability of obtaining an even prime number on each die, when a pair of dice is rolled is
(a) 0 (b) 1/3 (c) 1/12 (d) 1/36

9. If P(A) = 4/5 P( (A ∩ B) = 7/10 find P(B/A)


(a) ) 7/8 (b) 1/8 (c )1/10 (d) 17/30

VERY SHORT ANSWER (2 MARKS EACH)


1. An unbiased coin is tossed 4 times.Find the probability of getting atleast one head
2. Two independent events A and B are given such that P(A) = 0.3, P(B) = 0.6, find P(A and not
B)
3. The probability that atleast one of the two events A and B occurs is 0.6. If A and B occur
simultaneously with probability 0.2, evaluate P(𝐴̅) + P(𝐵̅)
4. An urn contains 10 black and 5 white balls. Two balls are drawn from the urn one after the
other without replacement. What is the probability that both drawn balls are black?

KVS RO DDN S SM XII MATHEMATICS/ 2023-24(VN) Page 137


*KVS RO DEHRADUN STUDENT SUPPORT MATERIAL MATHEMATICS/ XII/2023-24(VN)*

SHORT ANSWER TYPE QUESTIONS (3 MARKS EACH)

1. Evaluate P(A ∪ B), if 2P (A) = P(B) = 5/13 and P(A/ B) = 2/5.


2. Let A and B be two independent events with P(A) = 0.3 and P(B) = 0.4.
Find (i) 𝑃(𝐴 ∩ 𝐵), (ii) 𝑃(𝐴 ∪ 𝐵) (iii) P (𝐴/𝐵) (iv) 𝑃(𝐵/𝐴)
3. A couple has 2 children. Find the probability that both are boys, if it is known that
atleastone of them is a boy.
4. There are 4 cards numbered 1 to 4,one number on one card.Two cards are drawn at random
without replacement.Let X denote the sum of the numbers on the two drawn cards. Find the
probability distribution of X

Long Answer type Questions( 4 marks)

1. A random variable X has the following probability distribution:


X 0 1 2 3 4 5 6 7

0 K 2K 2K 3K 𝐾2 2𝐾 2 7𝐾 2 + 𝐾
P(X)

Find (i) K(ii) P(X< 3) (iii) P( X> 6) (iv) P( 0<X<3)


2. A dice is thrown twice and sum of numbers appearing is observed to be 6. What is the conditional
probability that the number 4 has appeared at least once.
3 1
3. The probability of A hitting a target is and that of B hitting is . They both fire at the target.
7 3
Find the probability that (i) at least one of them will hit the target, (ii) Only one of them will hit
the target.
4. Two integers are selected from integers 1 through 11. If the sum is even, find the probability that
both the numbers are odd.
5. 4 defective apples are accidentally mixed with 16 good ones. Three apples are drawn at random
from the mixed lot. Find the probability distribution of the number of defective apples.
Long Answer type Questions (5 marks)
1. A man is known to speak the truth 3 out of 4 times. He throws a die and reports that it is a
six. Find the probability that it is actually a six.
2. An insurance company insured 3000 scooters, 4000 cars and 5000 trucks. The probabilities of
the accident involving a scooter, a car and a truck are 0.02, 0.03 and 0.04 respectively. One
of the insured vehicles meets with an accident. Find the probability that it is a (i) scooter (ii)
car and (iii) truck

KVS RO DDN S SM XII MATHEMATICS/ 2023-24(VN) Page 138


*KVS RO DEHRADUN STUDENT SUPPORT MATERIAL MATHEMATICS/ XII/2023-24(VN)*

3. A company has two plants to manufacture bicycles. The first plant manufactures 60 % of the
bicycles and the second plant 40 % . Out of that 80 % of the bicycles are rateof standard
quality at the first plant and 90 % of standard quality at the second plant. A bicycle is picked
up at random and found to be standard quality. Find the probability that it comes from the
second plant.
4. Given three identical boxes I,II and III each containing two coins. In box I, both coins are
gold coins, in box II both are silver coins and in box III, there is one gold coin and one silver
coin. A person chooses a box at random and takes out a coin. If the coin is of gold, what is
the probability that the other coin in the box is also of gold?

5. Two cards are drawn simultaneously (without replacement) from a well shuffled pack of 52
cards. Find the probability distribution of the number of aces. Also, find the mean of
distribution.
6. Find the probability distribution of number of heads in two tosses of a coin.

KVS RO DDN S SM XII MATHEMATICS/ 2023-24(VN) Page 139


*KVS RO DEHRADUN STUDENT SUPPORT MATERIAL MATHEMATICS/ XII/2023-24(VN)*

ANSWERS

1 MARK QUESTIONS
1. 1/3 2. Yes 3. 8/65 4. 5/13
5. A 6. D 7. B 8. D 9. 7/8
(2 )MARK QUESTIONS

1. 15/16 2. 0.12 3. 1.2 4. 9/21

3MARK QUESTIONS

1. 11/26
2. (i) 0.12 (ii) 0.58 (iii) 0.3 (iv) 0.4
3. 1/3
4.
X 3 4 5 6 7

P(X) 1/6 1/6 1/3 1/6 1/6

2 MARK QUESTIONS

1. (i) k = 1/10 (ii) 3/10 (iii) 17/100 (iv) 3/10


2. 2/5
3. (i) 13/21 (ii) 10/21
4. 5/11
5. Ans.

6.
X 0 1 2 3
P(X) 28/57 24/57 24/285 1/285

KVS RO DDN S SM XII MATHEMATICS/ 2023-24(VN) Page 140


*KVS RO DEHRADUN STUDENT SUPPORT MATERIAL MATHEMATICS/ XII/2023-24(VN)*

5 MARK QUESTIONS

1. 3/8
2. (i) 3/19, (ii) 6/19 (iii) 10/19
3. 3/7
4. 2/3
5.
X 0 1 2

P(X) 188 32 1
221 221 221

Mean = 34/221

6.
X 0 1 2

P(X) 1 1 1
4 2 4

KVS RO DDN S SM XII MATHEMATICS/ 2023-24(VN) Page 141


*KVS RO DEHRADUN STUDENT SUPPORT MATERIAL MATHEMATICS/ XII/2023-24(VN)*

Higher Order Thinking Skills (HOTS)


MCQ
1.If P (A) = 0.8, P (B) = 0.5 and P (B|A) = 0.4, what is the value of P (A ∩ B)?
A. 0.32 B. 0.25 C. 0.1 D. 0.5
2.If P (A) = 6/11, P (B) = 5/11 and P (A ∪ B) = 7/11, what is the value of P(B|A)?
1 2
A.3 B. .3 C. 1 D. None of the above
3.Find P(E|F), where E: no tail appears, F: no head appears, when two coins are tossed in the air.
1
A. 0 B C. 1 D. None of the above
3
4.If P(A ∩ B) = 70% and P(B) = 85%, then P(A/B) is equal to:
A. 17/14 B. 14/17 C. ⅞ D. ⅛
5.A bag contains 5 red and 3 blue balls.If 3 balls are drawn at random without replacement the
probability of getting exactly one red ball is (a)45/196 (b)135/392 (c)15/56 (d)15/29
6.A flashlight has 8 batteries out of which 3 are dead.If two batteries are selected without replacement
and tested, the probability that both are dead is
(a)33/56 (b)9/64 (c)1/14 (d) 3/28
7. Probability that A speaks truth is 4/5.A coin is tossed .A reports that a head appears.The probability
that actually there was a head is
(a)4/5 (b)1/2 (c)1/5 (d) 2/5
8. A and B are two students. Their chances of solving a problem correctly are 1/3 and ¼ respectively. If
the probability of their making a common error is 1/20 and they obtain the same number , then the
probability of their answer to be correct is
(a)1/12 (b)1/40 (c)13/120 (d) 10/13
9. Given that the events A and B are such that P (A) = 1/2, P (A∪B) = 3/5 and P (B)=p. Find p, if they
are
mutually exclusive
(a)1/12 (b)1/10 (c)1/5 (d) 10/13
10.Given that the events A and B are such that P (A) = 1/2, P (A∪B) = 3/5 and P (B)=p. Find p, if they
are independent
(a)1/12 (b)1/10 (c)1/5 (d) 10/13

VSA(2 marks)

1.Given that A and B are two events such that P (A) = 0.6, P (B) =0.3 and P(A ∩ B) = 0.2, Then find P
(A/B) and P (B/A).
6 5 7
2.If P(A)= 11, P(B)= 11 and P(A ∪ B)= 11 Then find (i) P(A ∩ B) (ii) P (A/B) (iii) P (B/A)
5 2
3.Evaluate P(A ∪ B), if 2P(A) = P(B)=13 and P (A/B)=5
4.Compute P (A/B), if P (B) = 0.5 and P (A∩B) = 0.32
5.An electronic assembly consists of two sub-systems say A and B. From previous testing procedures,
the following probabilities use assumed to be known. P (A fails) = 0.2, P (B fails alone) = 0.15, P (A
and B fail) = 0.15. Evaluate the following probabilities: (i) P (A fails/B has failed) (ii) P
(A fails alone)
7 9 4
6.If P (A) = 13, P (B) = 13and P (A ∩ B) =13, find P (A/B).
7.Three cards are drawn successively, without replacement from a pack of 52 well shuffled cards. What
is the probability that first two cards are kings and third card drawn is an ace?

KVS RO DDN S SM XII MATHEMATICS/ 2023-24(VN) Page 142


*KVS RO DEHRADUN STUDENT SUPPORT MATERIAL MATHEMATICS/ XII/2023-24(VN)*

8.A box of oranges is inspected by examining three randomly selected oranges drawn without
replacement If all the three oranges are good, the box is approved for sale otherwise it is rejected Find
the probability that a box containing 15 oranges out of which 12 are good and 3 are bad ones will be
approved for sale. RT]
9.A bag contains 3 red and 7 black balls. Two balls are selected at random one-by-one without
replacement. If the second selected ball happens to be red, what is the probability that the first selected
ball is also red?
10.A and B are two candidates seeking admission in a college. The probability that A is selected is 0.7
and the probability that exactly one of them is selected is 0.6. Find the probability that B is selected.

SA(3 marks)
1.Let X denotes the number of hours you study during a randomly selected school day. The probability
that X can take the values x, has the following form, where k is some unknown constant.

0.1, if x = 0
kx, if x = 1 or 2
P(X = x) {
k(5 − x), if x = 3 or 4
0, otherwise
(a) Find the value of k.
(b) What is the probability that you study at least two hours? Exactly two hours? At most two
hours?
2.The probability of simultaneous occurrence of at least one of two events A and B is p. If the
probability that exactly one of A, B occurs is q, then prove that P (A′) + P (B′) = 2 – 2p + q.

3.A speaks truth in 75% of the cases, while B in 90% of the cases. In what per cent of cases are they
likely to contradict each other in stating the same fact? Do you think that statement of B is true?

4.Probability of solving specific problem independently by A and B are 1/2 and 1/3 respectively. If both
try to solve the problem independently, find the probability that
(i) The problem is solved
(ii) Exactly one of them solved the problem.
5.One bag contains 4 white and 5 black balls. Another bag contains 6 white and 7 black balls. A ball is
transferred form first bag to the second bag and then a ball is drawn from the second bag. Find the
probability that the ball drawn is white.
6.There are two bags, one of which contains 3 black and 4 white balls, while the other contains 4 black
and 3 white balls. A fair die is cast, if the face 1or 3 turns up, a ball is taken from the first bag, and if
any other face turns up a ball is chosen from the second bag. Find the probability of choosing a black
ball.
7.A bag A contains 4 black and 6 red balls and bag B contains 7 black and 3 red balls. A die is thrown.
If 1 or 2 appears on it, then bag A is chosen, otherwise bag B. if two balls are drawn at random (without
replacement) from the selected bag, find the probability of one of them being red and another black.

8.An urn contains m white and n black balls. A ball is drawn at random and is put back into the urn
along with k balls of the same colour as that of the ball drawn. A ball is again drawn at random. Show
that the probability of drawing a white ball now does not depend on k.

KVS RO DDN S SM XII MATHEMATICS/ 2023-24(VN) Page 143


*KVS RO DEHRADUN STUDENT SUPPORT MATERIAL MATHEMATICS/ XII/2023-24(VN)*

9.A bag contains (2n + l) coins. It is known that n of these coins have a head on both sides whereas the
rest of the coins are fair. A coin is picked up at random from the bag and is tossed. If theprobability that
the toss results in a head is 31/42, determine the value of n.

10.Three bags contain a number of red and white balls as follows:


Bag 1: 3 red balls; Bag II: 2 red balls and 1 white ball; Bag III: 3 white balls
𝐢
The probability that bag I will be chosen and a ball is selected from it is 𝟔, 𝐢 = 𝟏, 𝟐, 𝟑. What is the
probability that
(i) a red ball will be selected? (ii) a white ball will be selected?

LA(5 marks)
1. A shopkeeper sells three types of flower seeds A1, A2 and A3. They are sold as a mixture where the
proportions are 4:4:2 respectively. The germination rates of the three types of seeds are 45%, 60% and
35%. Calculate the probability
(i) of a randomly chosen seed to germinate
(ii) that it will not germinate given that the seed is of type A3,

(iii)that it is of the type A2 given that a randomly chosen seed does not germinate.
2. A letter is known to have come either from TATA NAGAR or from CALCUTTA. On the envelope,
just two consecutive letter TA are visible. Whatis the probability that the letter came from TATA
NAGAR.
3.An item is manufactured by three machines A, B and C. Out of the total number of items
manufactured during a specified period, 50% are manufactured on A, 30% on B and 20% on C. 2% of
the items produced on A and 2% of items produced on B are defective, and 3% of these produced on C
are defective. All the items are stored at one godown. One item is drawn at random and is found to be
defective. What is the probability that it was manufactured on machine A?

4.Often it is known that a truthful person commands, more respecting the society. A man is known to
speak truth 4 out of 5 times. He throws a die and reports that it is a six. Find the probability that it is
actually the six. Do you also agree that the value of truthfulness leads to more respect in the society?

5.Three persons A, B and C apply for job of manager in a private company. Chances of their selection
(A, B and C) are in the ratio 1 : 2 : 4.The probabilities that A, B and C can introduce changes to
improve profits of the company are 0.8, 0.5 and 0.3 respectively. If the change does not take place,find
the probability that it is due to the appointment of C.

6. In a factory which manufactures bolts, machines A, B and C manufacture respectively 30%, 50% and
20% of the bolts. Of their outputs 3, 4 and 1 percent respectively are defective bolts. A bolt is drawn at
random from the product and is found to be defective. Find the probability that this is not manufactured
by machine B.
7.An insurance company insured 2000 scooter drivers, 4000 car drivers and 6000 truck drivers. The
probability of an accident for them is 0.01, 0.03 and 0.15 respectively. One of the insured persons meets
with an accident. What is the probability that he is a scooter driver or a car driver?

KVS RO DDN S SM XII MATHEMATICS/ 2023-24(VN) Page 144


*KVS RO DEHRADUN STUDENT SUPPORT MATERIAL MATHEMATICS/ XII/2023-24(VN)*

8.In a game, a man wins Rs: 5 for getting a number greater than 4 and loses Rs: 1 otherwise, when a fair
die is thrown. The man decided to throw a die thrice but to quit as and when he gets a number greater
than 4. Find the expected value of the amount he wins/loses.

9.In a group of 400 people, 160 are smokers and non-vegetarian, 100 are smokers and vegetarian and
the remaining are non-smokers and vegetarian. The probabilities of getting a special chest disease are
35%, 20% and 10% respectively. A person is chosen from the group at random and is found to be
suffering from the disease. What is the probability that the selected person is a smoker and non-
vegetarian? What value is reflected in this question?

10.In a hockey match, both teams A and B scored same number of goals up to the end of the game, so
as to decide the winner, the referee asked both the captains to throw a die alternately and decided that
the team, whose captain gets a six first, will be declared the winner. If the captain of team A was asked
to start, find their respective probabilities of winning the match and state whether the decision of the
referee was fair or not.

KVS RO DDN S SM XII MATHEMATICS/ 2023-24(VN) Page 145


*KVS RO DEHRADUN STUDENT SUPPORT MATERIAL MATHEMATICS/ XII/2023-24(VN)*
HOTS Answer: MCQ

1 2 3 4 5 6 7 8 9 10
A B A B c d A d b c

VSA
Answer: (1) 2/3, 1/3
6 5 7 4 4⁄ 4 4⁄ 2
Answer: (2) 11
+ 11 − 11 = 11 (ii) 5 11 = 5 (iii) 6 11 = 3
⁄11 ⁄11
P(A∩B) 2 2
Answer:(3) 5⁄ = ⇒P(A ∩ B) = 13 Now, P (A ∪ B) = P (A) + P (B) −P(A ∩ B)⇒ P (A ∪ B) =
13 5
5 5 2 11
26
+ 13 − 13 = 26
16
Answer(4)25
Answer(5) Given that, P (A)= 0.2, P (B) − P(A ∩ B) = 0.15, P(A ∩ B) = 0.5
P(A∩B) 0.15 1
(i) P (A/B) = = 0.30 = 2 = 0.5 (ii) P (A) − P(A ∩ B) = 0.2 – 0.15 = 0.05
P (B)
P(A∩B) P (B−A)9⁄ −4⁄ P (B)−P (A∩B)
5
13 13
Answer:(6) P (A/B) =P (B)
= P (B)
=
9⁄ = 9
P (B)
=
13
4 3 4 2
Answer(7)P(A ∩ B ∩ C ) = P(A)P(B⁄A)P(C⁄A ∩ B) = × × =
52 51 50 5525
12 11 10 44
Answer (8)P(A ∩ B ∩ C ) = P(A)P(B⁄A)P(C⁄A ∩ B) = 15 × 14 × 13 = 91
B 3 2 1
Answer:(9)P(A ∩ B) = P (A)P (A) = 10 × 9 = 15
Answer(10) P (A) = 0.7, P[ (A∩ B) ∪ (A ∩ B) = 0.6 ⇒ P (A) + P (B) – 2 P (A∩B) = 0.6
1
⇒ P (A) + P (B) – 2 P (A) . P (B) = 0.6 Substitute P (A) value and then simplify we will get P (B) =
4

(OR)
P [(A∩ B) ∪ (A ∩ B)] = 0.6 ⇒ P (A∩ B) + P (A ∩ B) = 0.6
1
⇒ P (A) . P (B) + P (A) . P (B)⇒ (0.7) (1- P (B)) + (1- 0.7) . P (B) = 0.6 ⇒ P (B) = 0.25=4

SA
Answer(1)

X 0 1 2 3 4 otherwise

P(X) 0.1 K 2k 2k K 0

(a) k= 0.15 (b) P(X ≥ 2) = 0.75; P(X = 2) = 0.3; P (X ≤ 2)=0.55

Answer(2) Since P (exactly one of A, B occurs) = q (given), we get P (A∪B) – P (A∩B) = q


⇒ p – P (A∩B) = q ⇒ P (A∩B) = p – q
⇒ 1 – P (A′∪B′) = p – q ⇒ P (A′∪B′) = 1 – p + q
⇒ P (A′) + P (B′) – P (A′∩B′) = 1 – p + q
⇒ P (A′) + P (B′) = (1 – p + q) + P (A′ ∩ B′)
⇒ P (A′) + P (B′) = (1 – p + q) + (1 – P (A ∪ B))
⇒ P (A′) + P (B′) = (1 – p + q) + (1 – p)
⇒ P (A′) + P (B′) = 2 – 2p + q.

KVS RO DDN S SM XII MATHEMATICS/ 2023-24(VN) Page 146


*KVS RO DEHRADUN STUDENT SUPPORT MATERIAL MATHEMATICS/ XII/2023-24(VN)*

𝟑 𝟗 𝟑 𝟗 𝟑𝟎
Answer(3): P (E).P (𝐅)+P (𝐄). P (𝐅) = 𝟒 × (𝟏 − 𝟏𝟎) + (𝟏 − 𝟒) × 𝟏𝟎 = 𝟏𝟎𝟎
We think that statement may be false.
𝟏 𝟏 𝟐
Answer(4) (i) P (E∪F)=𝟏 − 𝐏(𝐄 ∩ 𝐅) = 𝟏 − 𝐏(𝐄). 𝐏(𝐅) = 𝟏 − (𝟏 − 𝟐) (𝟏 − 𝟑) = 𝟑
(ii) P[(A∩ 𝐁) ∪ (𝐀 ∩ 𝐁) = P (A) + P (B) – 2 P (A∩B) = P (A) + P (B) – 2 P (A) . P (B)
𝟏 𝟏 𝟏 𝟏 𝟏
= + − 𝟐. . =
𝟐 𝟑 𝟐 𝟑 𝟐

Answer(5):E1 : Event that ball transvered from the first bag is white;
E2 : Event that ball transvered from the first bag is black
A: Event that the ball drawn from the second bag is white
4 7 5 6 29
P (A) = P(E1 )P(A⁄E1 ) + P(E2 )P(A⁄E2 )=9 × 14 + 9 × 14 = 63
Answer(6)E1 : the die shows 1 or 3; E2 : the die shows 2, 4, 5 or 6 A: the ball drawn is black.
1 3 2 4 11
P (A) = P(E1 )P(A⁄E1 ) + P(E2 )P(A⁄E2 )=3 × 7 + 3 × 7 = 21.
Answer(7)𝐄𝟏 : getting 1 or 2 on the die; 𝐄𝟐 : getting any number other than 1 and 2 on the die

A: getting 1 red and 1 black ball from the bag; P (A) = 𝐏(𝐄𝟏 )𝐏(𝐀⁄𝐄𝟏 ) + 𝐏(𝐄𝟐 )𝐏(𝐀⁄𝐄𝟐 )
𝟏 𝟖 𝟐 𝟕 𝟐𝟐
=𝟑 × 𝟏𝟓 + 𝟑 × 𝟏𝟓 = 𝟒𝟓
.

Answer(8)E1 : Event that first ball drawn from the urn is white;
E2 : Event that first ball drawn from the urn is red.
A: Getting a white ball from the urn when ball drawn in first draw is put back along with k balls
of the same colour.
m m+k n m m
P (A) = P(E1 )P(A⁄E1 ) + P(E2 )P(A⁄E2 )=m+n × m+n+k + m+n × m+n+k = m+n.

Answer(9):E1 : taking out a fair coin from the bag;


E2 : taking out a coin having head on both sides.
A: tossed coin results in a head.
n+1 1 n 3n+1 3n+1 31
P (A) = P(E1 )P(A⁄E1 ) + P(E2 )P(A⁄E2 )=2n+1 × 2 + 2n+1 × 1 = 2(2n+1)⇒2(2n+1) = 42
⇒n = 10
Answer(10) E1 : selecting bag I; E2 : selecting bag II;E3 : selecting bag III; A: getting a red ball.
1 3 2 2 3 7
(i) P (A) = P(E1 )P(A⁄E1 ) + P(E2 )P(A⁄E2 ) + P(E3 )P(A⁄E3 )=6 × 3 + 6 × 3 + 6 × 0 = 18
7 11
(ii) P (A) = 1−P (A) = 1 − =
18 18
LA
Answer:(1) 𝐄𝟏 : Event that of flower seeds A1;𝐄𝟐 : Event that of flower seeds A2; 𝐄𝟑 : Event that of
flower seeds A3;
A: Event that the seed germinates; 𝐀: Event that the seed does not germinates.
(i) Required probability ==𝐏(𝐄𝟏 ⁄𝐀) = 𝐏(𝐄𝟏 )𝐏(𝐀⁄𝐄𝟏 ) + 𝐏(𝐄𝟐 )𝐏(𝐀⁄𝐄𝟐 ) + 𝐏(𝐄𝟑 )𝐏(𝐀⁄𝐄𝟑 )
𝟒 𝟒𝟓 𝟒 𝟔𝟎 𝟐 𝟑𝟓 𝟒𝟗𝟎
= 𝟏𝟎 × 𝟏𝟎𝟎 + 𝟏𝟎
× 𝟏𝟎𝟎 + 𝟏𝟎 × 𝟏𝟎𝟎 = 𝟏𝟎𝟎𝟎=0.49
𝟑𝟓 𝟔𝟓
(ii) Required probability ==𝐏(𝐀⁄𝐄𝟑 ) = 𝟏 − 𝐏(𝐀/𝐄𝟑 ) = 𝟏 − 𝟏𝟎𝟎 = 𝟏𝟎𝟎
𝐏(𝐄𝟐 )𝐏(𝐀⁄𝐄𝟏 )
(iii) Required probability =𝐏(𝐄𝟐 ⁄𝐀) = =
𝐏(𝐄𝟏 )𝐏(𝐀⁄𝐄𝟏 )+𝐏(𝐄𝟐 )𝐏(𝐀⁄𝐄𝟐 )
𝐀
𝐏(𝐄𝟐 )[𝟏−𝐏( )]
𝐄𝟑
𝐀 𝐀 𝐀
𝐏(𝐄𝟏 )[𝟏−𝐏( )]+𝐏(𝐄𝟐 )[𝟏−𝐏( )]+𝐏(𝐄𝟑 )[𝟏−𝐏( )]
𝐄𝟏 𝐄𝟐 𝐄𝟑
𝟒 𝟔𝟎
𝟏𝟎
× (𝟏 − 𝟏𝟎𝟎) 𝟏𝟔
= 𝟒 𝟒𝟓 𝟒 𝟔𝟎 𝟐 𝟑𝟓
=
× (𝟏 − 𝟏𝟎𝟎) + × (𝟏 − ) + 𝟏𝟎 × (𝟏 − ) 𝟓𝟏
𝟏𝟎 𝟏𝟎 𝟏𝟎𝟎 𝟏𝟎𝟎

KVS RO DDN S SM XII MATHEMATICS/ 2023-24(VN) Page 147


*KVS RO DEHRADUN STUDENT SUPPORT MATERIAL MATHEMATICS/ XII/2023-24(VN)*

Answer(2) 𝐄𝟏 : Event that letter is from TATA NAGAR; 𝐄𝟐 : Event that letter is from
CALCUTTA;

A: Event that on the envelope two consecutive letters TA are visible.


If letter is from TATA NAGAR, then we choose two consecutive letters from set
𝟐
{TA, AT, TA, AN, NA, AG, GA,AR}. So, 𝐏(𝐀⁄𝐄𝟏 ) = 𝟖
If letter is from CALCUTTA, then we choose two consecutive letters from set
𝟏
{CA, AL, LC, CU, UT, TT, TA}. So, 𝐏(𝐀⁄𝐄𝟐 ) = 𝟖
𝟏 𝟐
𝐏(𝐄𝟏 )𝐏(𝐀⁄𝐄𝟏 ) × 𝟕
𝟐 𝟖
Required probability= 𝐏(𝐄𝟏 ⁄𝐀) = 𝐏(𝐄 =𝟏 𝟐 𝟏 𝟏 = 𝟏𝟏
𝟏 )𝐏(𝐀⁄𝐄𝟏 )+𝐏(𝐄𝟐 )𝐏(𝐀⁄𝐄𝟐 ) × + ×
𝟐 𝟖 𝟐 𝟕

KVS RO DDN S SM XII MATHEMATICS/ 2023-24(VN) Page 148


*KVS RO DEHRADUN STUDENT SUPPORT MATERIAL MATHEMATICS/ XII/2023-24(VN)*
Answer:(3) 𝐄𝟏 : Event that selected item is manufactured by machine A; 𝐄𝟐 : Event that selected
item is manufactured by machine B;
𝐄𝟑 : Event that selected item is manufactured by machine C; A: Event that selected item is defective.
𝟓𝟎 𝟐
𝐏(𝐄𝟏 )𝐏(𝐀⁄𝐄𝟏 ) × 𝟓
𝟏𝟎𝟎 𝟏𝟎𝟎
Required probability ==𝐏(𝐄𝟏 ⁄𝐀) = 𝐏(𝐄 = 𝟓𝟎 𝟐 𝟑𝟎 𝟐 𝟐𝟎 𝟑 = 𝟏𝟏
𝟏 )𝐏(𝐀⁄𝐄𝟏 )+𝐏(𝐄𝟐 )𝐏(𝐀⁄𝐄𝟐 )+𝐏(𝐄𝟑 )𝐏(𝐀⁄𝐄𝟑 ) × + × + ×
𝟏𝟎𝟎 𝟏𝟎𝟎 𝟏𝟎𝟎 𝟏𝟎𝟎 𝟏𝟎𝟎 𝟏𝟎𝟎
Answer:(4) 𝐄𝟏 : Event that ‘6’ occurs; 𝐄𝟐 : Event that ‘6’ does not occurs; A: Event that
the man reports that it is ‘6’.
𝟏 𝟒
𝐏(𝐄𝟏 )𝐏(𝐀⁄𝐄𝟏 ) × 𝟒
𝟔 𝟓
Required probability= 𝐏(𝐄𝟏 ⁄𝐀) = =𝟏 𝟒 𝟓 𝟏 =
𝐏(𝐄𝟏 )𝐏(𝐀⁄𝐄𝟏 )+𝐏(𝐄𝟐 )𝐏(𝐀⁄𝐄𝟐 ) × + × 𝟗
𝟔 𝟓 𝟔 𝟓
Yes, we are agree that the value of truthfulness leads to more respect in the society.
Answer:((5) 𝐄𝟏 : Event that chances of selecting person A; 𝐄𝟐 : Event that chances of selecting
person B; 𝐄𝟑 : Event that chances of selecting person C
A: Event that changes takes place to improve profits. 𝐀: Event that changes does not takes place to
improve profits
𝐏(𝐄𝟑 )𝐏(𝐀⁄𝐄𝟑 )
Required probability =𝐏(𝐄𝟑 ⁄𝐀) = =
𝐏(𝐄𝟏 )𝐏(𝐀⁄𝐄𝟏 )+𝐏(𝐄𝟐 )𝐏(𝐀⁄𝐄𝟐 )+𝐏(𝐄𝟑 )𝐏(𝐀⁄𝐄𝟑 )
𝟒
×(𝟏−𝟎.𝟑)
𝟕
𝟏 𝟐 𝟒 = 𝟎. 𝟕
×(𝟏−𝟎.𝟖)+ ×(𝟏−𝟎.𝟓)+ ×(𝟏−𝟎.𝟑)
𝟕 𝟕 𝟕
Answer:(6)𝐄𝟏 : Event that the bolt is manufactured by the machine A; 𝐄𝟐 : Event that the bolt is
manufactured by the machine B;
𝐄𝟑 : Event that the bolt is manufactured by the machine C; A: Event that the bolt is defective.
P (Bolt is defective and is manufactured by machine B) =𝐏(𝐄𝟐 ⁄𝐀)
𝟓𝟎 𝟒
𝐏(𝐄𝟐 )𝐏(𝐀⁄𝐄𝟐 ) × 𝟐𝟎
𝟏𝟎𝟎 𝟏𝟎𝟎
= 𝐏(𝐄 )𝐏(𝐀⁄ )+𝐏(𝐄
= 𝟑𝟎 𝟑 𝟓𝟎 𝟒 𝟐𝟎 𝟏 = 𝟑𝟏
𝟏 𝐄 𝟏 𝟐 )𝐏(𝐀⁄𝐄𝟐 )+𝐏(𝐄𝟑 )𝐏(𝐀⁄𝐄𝟑 ) × + × + ×
𝟏𝟎𝟎 𝟏𝟎𝟎 𝟏𝟎𝟎 𝟏𝟎𝟎 𝟏𝟎𝟎 𝟏𝟎𝟎
Required probability = P (Bolt is defective and is manufactured by machine B) = 𝟏 − 𝐏(𝐄𝟐 ⁄𝐀) = 𝟏 −
𝟐𝟎 𝟏𝟏
𝟑𝟏
= 𝟑𝟏
Answer: (7) 𝐄𝟏 : Event that selected person is a scooter driver; 𝐄𝟐 : Event that selected person
is a car driver;
𝐄𝟑 : Event that selected person is a truck driver; A: Event that selected person meets with an accident.
Required probability =1−P (the person who meets with accident is a truck driver) =𝟏 − 𝐏(𝐄𝟑 ⁄𝐀)
𝐏(𝐄𝟑 )𝐏(𝐀⁄𝐄𝟑 )
= 𝟏 − 𝐏(𝐄 )𝐏(𝐀⁄ )+𝐏(𝐄
𝟏 𝐄 𝟏 𝟐 )𝐏(𝐀⁄𝐄𝟐 )+𝐏(𝐄𝟑 )𝐏(𝐀⁄𝐄𝟑 )
𝟔𝟎𝟎𝟎
𝟏𝟐𝟎𝟎𝟎
× (𝟎. 𝟏𝟓) 𝟒𝟓 𝟕
=𝟏− 𝟐𝟎𝟎𝟎 𝟒𝟎𝟎𝟎 𝟔𝟎𝟎𝟎
=𝟏− =
× (𝟎. 𝟎𝟏) + × (𝟎. 𝟎𝟑) + × (𝟎. 𝟏𝟓) 𝟓𝟐 𝟓𝟐
𝟏𝟐𝟎𝟎𝟎 𝟏𝟐𝟎𝟎𝟎 𝟏𝟐𝟎𝟎𝟎

𝟒 𝟒 𝟒 𝟔𝟒
Answer:(8) P(X = −3)=P(getting a number ≤4 in all three throws)= 𝟔 . 𝟔 . 𝟔 = 𝟐𝟏𝟔
𝟒 𝟒 𝟐 𝟑𝟐
P(X = 3)=P(getting a number ≤4 in first two throws and a number >4 in the third throw )= 𝟔 . 𝟔 . 𝟔 = 𝟐𝟏𝟔
𝟒 𝟐 𝟖 𝟒𝟖
P(X = 4)=P(getting a number ≤4 in the first throw and a number > 4 in second throws)= 𝟔 . 𝟔 = 𝟑𝟔 𝐨𝐫 𝟐𝟏𝟔
𝟐 𝟏 𝟕𝟐
P(X = 5)=P(getting a number >4 in the first throw)= 𝟔 = 𝟑 𝐨𝐫 𝟐𝟏𝟔

X -3 3 4 5
P 𝟔𝟒 𝟑𝟐 𝟒𝟖 𝟕𝟐
(X) 𝟐𝟏𝟔 𝟐𝟏𝟔 𝟐𝟏𝟔 𝟐𝟏𝟔
𝟏𝟗
Mean = E(X) = 𝟗

KVS RO DDN S SM XII MATHEMATICS/ 2023-24(VN) Page 149


*KVS RO DEHRADUN STUDENT SUPPORT MATERIAL MATHEMATICS/ XII/2023-24(VN)*

Answer(9) 𝐄𝟏 : Event that selected person is smoker and non-vegetarian; 𝐄𝟐 : Event that selected
person is smoker and vegetarian;
𝐄𝟑 : Event that selected person is non-smoker and vegetarian; A: Event that selected person is
suffering from the disease.
𝟏𝟔𝟎 𝟑𝟓
𝐏(𝐄𝟏 )𝐏(𝐀⁄𝐄𝟏 ) × 𝟐𝟖
𝟒𝟎𝟎 𝟏𝟎𝟎
Required probability =𝐏(𝐄𝟏 ⁄𝐀) = 𝐏(𝐄 )𝐏(𝐀⁄ )+𝐏(𝐄
= 𝟏𝟔𝟎 𝟑𝟓 𝟏𝟎𝟎 𝟐𝟎 𝟏𝟒𝟎 𝟏𝟎 = 𝟒𝟓
𝟏 𝐄 𝟏 𝟐 )𝐏(𝐀⁄𝐄𝟐 )+𝐏(𝐄𝟑 )𝐏(𝐀⁄𝐄𝟑 ) × + × + ×
𝟒𝟎𝟎 𝟏𝟎𝟎 𝟒𝟎𝟎 𝟏𝟎𝟎 𝟒𝟎𝟎 𝟏𝟎𝟎

𝟏 𝟓 𝟓 𝟏 𝟓 𝟓 𝟓 𝟓 𝟏 𝟏 𝟏 𝟓 𝟐 𝟏 𝟓 𝟒
Answer(10) P (A wins) = + × × + × × × × + ⋯ ∞ = + .( ) + .( ) + ⋯∞ =
𝟔 𝟔 𝟔 𝟔 𝟔 𝟔 𝟔 𝟔 𝟔 𝟔 𝟔 𝟔 𝟔 𝟔
𝟏
𝟔 𝟔
𝟓 𝟐
= 𝟏𝟏
𝟏−( )
𝟔
𝟔 𝟓
P (B wins) = 𝟏 − 𝟏𝟏 = 𝟏𝟏
The decision of the referee was not fair as whose ever starts throwing the die gets an upper hand.

KVS RO DDN S SM XII MATHEMATICS/ 2023-24(VN) Page 150


*KVS RO DEHRADUN STUDENT SUPPORT MATERIAL MATHEMATICS/ XII/2023-24(VN)*

CASE BASED QUESTIONS


1. In a play zone, Rohan is playing game. It has 12 blue balls, 8 red balls, 10 yellow balls and 5
green balls. If Rohan draws two balls one after the other without replacement, then answer the
following questions.

(i)What is the probability that the first ball is blue and the second ball is green?
(a )5/119 (b) 12/119 (c) 6/119 (d) 7/119
(ii) What is the probability that the first ball is yellow and the second ball is red?
(a) 6/119 (b) 8/119 (c) 24/119 (d) none of these
(iii) What is the probability that both the balls are red?
(a) 4/85 (b) 204/595 (c) 12/119 (d) 64/119
(iv) What is the probability that the first ball is green and the second ball is not yellow?
(a) 10/119 (b) 6/85 (c) 12/119 (d) none of these
(v) What is the probability that both the balls are not blue?
(a) 6/595 (b) 12/85 (c) 15/595 (d) 253/595
2. A doctor is to visit a patient. From the past experience, it is known that the probabilities that he will come by
cab, metro, bike or by other means of transport are respectively 0.3, 0.2, 0.1 and 0.4. The probabilities that he
will be late are 0.25, 0.3, 0.35 and 0.1 if he comes by cab, metro, bike and other means of transport respectively.
Based on the above information, answer the following questions.

(i) When the doctor arrives late, what is the probability that he comes by metro?
(a) 5/4 (b) 2/7 (c) 5/21 (d) 1/6
(ii) When the doctor arrives late, what is the probability that he comes by cab?
(a) 4/21 (b) 1/7 (c) 5/14 (d) 2/21

KVS RO DDN S SM XII MATHEMATICS/ 2023-24(VN) Page 151


*KVS RO DEHRADUN STUDENT SUPPORT MATERIAL MATHEMATICS/ XII/2023-24(VN)*

(iii) When the doctor arrives late, what is the probability that he comes by bike?
(a) 5/21 (b)4/7 (c) 5/6 (d) 1/6

(iv) When the doctor arrives late, what is the probability that he comes by other means of transport?
(a) 6/7 (b) 5/14 (c) 4/21 (d) 2/7
(v) What is the probability that the doctor is late by any means?
(a) 0.2 (b) 0.21 (c) 1.5 (d) 0.35
3. On a holiday, a father gave a puzzle to his son John and his daughter Priya. The probabilityof solving this
specific puzzle independently by John and Divyaare 1/4 and 1/5 respectively. Based on the above information,
answer the following questions.

(i) The chance that both John and Divya solved the puzzle,
(a) 10% (b) 5% (c) 25% (d) 20%
(ii) Probability that puzzle is solved by John but not by Divya, is
(a) ½ (b) 1/5 (c) 3/5 (d)1/3

(iii) Find the probability that puzzle is solved.


(a) ½ (b) 1/5 (c) 2/5 (d) 5/6

(iv) Probability that exactly one of them solved the puzzle, is


(a) 1/30 (b) 1/20 (c) 7/20 (d) 3/20

(v) Probability that none of them solved the puzzle, is


(a) 1/5 (b) 3/5 (c) 2/5 (d) None of these

4. Ajay enrolled himself in an online practice test portal provided by his school for better practice. Out of 5
questions in a set-I, he was able to solve 4 of them and got stuck in the one which is as shown below.

KVS RO DDN S SM XII MATHEMATICS/ 2023-24(VN) Page 152


*KVS RO DEHRADUN STUDENT SUPPORT MATERIAL MATHEMATICS/ XII/2023-24(VN)*

If A and B are independent events, P(A)=0.6 and P(B)=0.8, then answer the following questions:
(i) P(A∩ 𝐵)
(ii) P(AUB)
(iii) P(B|A) OR P(A|B)
(iv) P(not A and not B)

5. Suman was doing a project on a school survey, on the average number of hours spent on study by students
selected at random. At the end of survey, Suman prepared the following report related to the data.
Let X denotes the average number of hours spent on study by students. The probability that X and can take the
values x, has the following form, where k is some unknown constant.
0.2, 𝑖𝑓 𝑥 = 0
𝑘𝑥, 𝑖𝑓 𝑥 = 1 𝑜𝑟 2
𝑃(𝑋 = 𝑥) = {
𝑘(6 − 𝑥), 𝑖𝑓 𝑥 = 3 𝑜𝑟 4
0, 𝑜𝑡ℎ𝑒𝑟𝑤𝑖𝑠𝑒

Based on the above information, answer the following questions:


(i) Find the value of k
(ii) What is the probability that the average study time of students is not more than 1 hour?
(iii) What is the probability that the average study time of students is at least 3 hours?
(iv) What is the probability that the average study time of students is exactly 2 hours?

6. Linear programming is a method for finding the optimal values (maximum or minimum) of quantities subject
to the constraints when relationship is expressed as linear equations or inequations.
Based on the above information, answer the following questions.
(i) The optimal value of the objective function is attained at the points
(a) on X-axis
(b) on Y-axis
(c) which are corner points of the feasible region th
(d) none of these

KVS RO DDN S SM XII MATHEMATICS/ 2023-24(VN) Page 153


*KVS RO DEHRADUN STUDENT SUPPORT MATERIAL MATHEMATICS/ XII/2023-24(VN)*

(ii) The graph of the inequality 3x + 4y < 12 is


(a) half plane that contains the origin
(b) half plane that neither contains the origin nor the points of the line
3x + 4y =12.
(c) whole XOY-plane excluding the points on line 3x + 4y = 12
(d) It contains the origin but not the points on the line 3x + 4y = 12.

(iii) The feasible region for an LPP is shown in the figure. Let Z = 2x + 5y be the objective function.
Maximum of Z occurs at which point.Also find maximum value of Z.

(iv) The corner points of the feasible region determined by the system of linear constraints are (0, 10), (5, 5),
(15, 15), (0, 20). Let Z = px + qy, where p, q > 0. Find the relation between p and q so that the maximum of
Z occurs at both the points (15, 15) and (0, 20) .

7.In a play zone, Aastha is playing crane game. It has 12 blue balls, 8 red balls, 10 yellow balls and 5 green balls.
If Aastha draws two balls one after the other without replacement, then answer the following questions.

(i) What is the probability that the first ball is blue and the second ball is green?

(ii) What is the probability that the first ball is yellow and the second ball is red?

(iii) What is the probability that both the balls are red?

(v) What is the probability that the first ball is green and the second ball is not yellow?

KVS RO DDN S SM XII MATHEMATICS/ 2023-24(VN) Page 154


*KVS RO DEHRADUN STUDENT SUPPORT MATERIAL MATHEMATICS/ XII/2023-24(VN)*

8. Corner points of feasible region for an LPP are (0,3), (5,0), (6,8), (0,8) .Let Z = 4x – 6y be the objective
function.
Based on the above information, answer the following questions:
(i) The minimum value of Z occurs at which point.
(ii) The maximum value of Z occurs at which point.
(iii) Find the corner points of feasible region determined by the system of linear inequalities.

(iv) The feasible solution of LPP belongs to which quadrant?

KVS RO DDN S SM XII MATHEMATICS/ 2023-24(VN) Page 155


*KVS RO DEHRADUN STUDENT SUPPORT MATERIAL MATHEMATICS/ XII/2023-24(VN)*

Answers:

1. (I) a (ii) b (iii) c (iv) d (v) a


2. (i) b (ii) c (iii) d (iv) c (v)b
3. (i) b (ii) b (iii) c (iv) c (v)b
4. 4.(i) 0.48
(ii) 0.92

(iii) 0.8 OR 0.6


(iv)0.08

5. (i) as ∑ 𝑃i = 1 then k = 0.1


(ii) 0.3
(iii) 0.5
(iv)0.2

6.(i) (c)
(ii) (d)
(iii) (4,5), max. value = 33
(iv)q = 3 p

7.(i) P(G∩B) = P(B).P(G|B)=12/35*5/34=6/119


(ii) 8/119
(iii) Let E= drawing first red ball
F= drawing second red ball
P(E)=8/35, P(F) = 7/34
P(F∩E) = 4/85
(iv)P(Y’∩G) =5/35*24/34=12/119

8.(i) (0,8)
(ii) (5,0)
(iii) O(0,0), A(3,0), B(3,2), C(2,3), D(0,3).

KVS RO DDN S SM XII MATHEMATICS/ 2023-24(VN) Page 156


*KVS RO DEHRADUN STUDENT SUPPORT MATERIAL MATHEMATICS/ XII/2023-24(VN)*

CLASS TEST

1 mark questions
1. If P (A) = 0.8, P (B) = 0.5 and P (B|A) = 0.4, what is the value of P (A ∩ B)?
(b) 0.32 (b) 0.25 (c) 0.1 (d) 0.5
2. The probability of obtaining an even prime number on each die, when a pair of dice is rolled is
(a) 0 (b) 1/3 (c) 1/12 (d) 1/36
3 mark questions
3. An urn contains 10 black and 5 white balls. Two balls are drawn from the urn one after the other
without replacement. What is the probability that both drawn balls are black?

4. Evaluate P(A ∪ B), if 2P (A) = P(B) = 5/13 and P(A/ B) = 2/5.


5. A bag contains 5 white, 7 red and 3 black balls. If three balls are drawn one by one without
replacement, find what is the probability that none is red.
6. If P(A) = 0.4, P(B) = 0.7 and P(B/A) = 0.6. Find P(A ∪ B).

3 mark questions
7. Two cards are drawn simultaneously (without replacement) from a well shuffled pack of 52 cards. Find
the probability distribution of the number of aces. Also, find the mean of distribution
8. Given three identical boxes I,II and III each containing two coins. In box I, both coins are gold coins, in
box II both are silver coins and in box III, there is one gold coin and one silver coin. A person chooses a
box at random and takes out a coin. If the coin is of gold, what is the probability that the other coin in
the box is also of gold?
4 mark questions
9. In an office three employees James Sophia and Oliver process incoming copies of a certain form. James
processes 50% of the forms, Sophia processes 20% and Oliver the remaining 30% of the forms. James
has an error rate of 0.06,sophia has an error rate of 0.04 and Oliver has an error rate of 0.03

Based on the above information answer the following questions


(I) Find the probability that Sophia processed the form and committed an error.
(II) Find the total probability of committing an error in processing the form.
(III) The manager of the company wants to do a quality check. During inspection he selects
a form at random from the day’s output of the processed form if the form selected at
random has an error find the probability that the form is not processed by James.
10. Let E be the event of committing an error in processing the form and let
𝐸1 , 𝐸2 𝑎𝑛𝑑 𝐸3 𝑏𝑒 𝑡ℎ𝑒 𝑒𝑣𝑒𝑛𝑡𝑠 that james, Sophia and Oliver processed the form. Find the value of
𝐸
∑31 𝑃( 1 )
𝐸

A random variable X has the following probability distribution:


X 0 1 2 3 4 5 6 7

0 K 2K 2K 3K 𝐾2 2𝐾 2 7𝐾 2 + 𝐾
P(X)

Find (i) K (ii) P(X< 3) (iii) P( X> 6) (iv) P( 0<X<3)

KVS RO DDN S SM XII MATHEMATICS/ 2023-24(VN) Page 157


*KVS RO DEHRADUN STUDENT SUPPORT MATERIAL MATHEMATICS/ XII/2023-24(VN)*

CLASS- TEST
1.If P(A ∩ B) = 70% and P(B) = 85%, then P(A/B) is equal to:
A. 17/14 B. 14/17 C. ⅞ D. ⅛
2.A bag contains 5 red and 3 blue balls .If 3 balls are drawn at random without replacement the probability of
getting exactly one red ball is (a)45/196 (b)135/392 (c)15/56 (d)15/29
3.A flashlight has 8 batteries out of which 3 are dead.If two batteries are selected without replacement and tested,
the probability that both are dead is
(a)33/56 (b)9/64 (c)1/14 (d) 3/28
4. Probability that A speaks truth is 4/5.A coin is tossed .A reports that a head appears.The probability that
actually there was a head is
(a)4/5 (b)1/2 (c)1/5 (d) 2/5
5. A and B are two students. Their chances of solving a problem correctly are 1/3 and 1/4 respectively. If the
probability of their making a common error is 1/20 and they obtain the same number , then the probability of
their answer to be correct is
(a)1/12 (b)1/40 (c)13/120 (d) 10/13
6. Given that the events A and B are such that P (A) = 1/2, P (A∪B) = 3/5 and P (B)=p. Find p, if they are
mutually exclusive
(a)1/12 (b)1/10 (c)1/5 (d) 10/13
7.Given that the events A and B are such that P (A) = 1/2, P (A∪B) = 3/5 and P (B)=p. Find p, if they are
independent
(a)1/12 (b)1/10 (c)1/5 (d) 10/13
8.An urn contains 10 black and 5 white balls. Two balls are drawn from the urn one after the other without
replacement. What is the probability that both drawn balls are black?
1 2
A.3 B. .3 C. 1 D.3/7

SECTION-B ( 4×2=8)
5 2
9..Evaluate P(A ∪ B), if 2P(A) = P(B) = and P (A/B) =
13 5

10.An electronic assembly consists of two sub-systems say A and B. From previous testing procedures, the
following probabilities use assumed to be known. P (A fails) = 0.2, P (B fails alone) = 0.15, P (A and B fail) =
0.15. Evaluate the following probabilities: (i) P (A fails/B has failed) (ii) P (A fails alone)

11.Three cards are drawn successively, without replacement from a pack of 52 well shuffled cards. What is the
probability that first two cards are kings and third card drawn is an ace?

12.A and B are two candidates seeking admission in a college. The probability that A is selected is 0.7 and the
probability that exactly one of them is selected is 0.6. Find the probability that B is selected.

SECTION-C ( 3×3=9)
13. Let X denotes the number of hours you study during a randomly selected school day. The probability that X
can take the values x, has the following form, where k is some unknown constant.
0.1, 𝑖𝑓 𝑥 = 0
𝑘𝑥, 𝑖𝑓 𝑥 = 1 𝑜𝑟 2
𝑃(𝑋 = 𝑥) {
𝑘(5 − 𝑥), 𝑖𝑓 𝑥 = 3 𝑜𝑟 4
0, 𝑜𝑡ℎ𝑒𝑟𝑤𝑖𝑠𝑒
(a) Find the value of k.
(b) What is the probability that you study at least two hours? Exactly two hours? At most two hours?
14.The probability of simultaneous occurrence of at least one of two events A and B is p. If the probability that
exactly one of A, B occurs is q, then prove that P (A′) + P (B′) = 2 – 2p + q.
15A speaks truth in 75% of the cases, while B in 90% of the cases. In what per cent of cases are they likely to
contradict each other in stating the same fact? Do you think that statement of B is true?

KVS RO DDN S SM XII MATHEMATICS/ 2023-24(VN) Page 158


*KVS RO DEHRADUN STUDENT SUPPORT MATERIAL MATHEMATICS/ XII/2023-24(VN)*

OR
Probability of solving specific problem independently by A and B are 1/2 and 1/3 respectively. If both try to
solve the problem independently, find the probability that
(i) The problem is solved
(ii) Exactly one of them solved the problem.

SECTION-C ( 1×5=5)
16. A letter is known to have come either from TATA NAGAR or from CALCUTTA. On the envelope, just two
consecutive letter TA are visible. What is the probability that the letter came from TATA NAGAR.

OR
A card from a pack of 52 playing cards is lost. From the remaining cards of the pack three cards are drawn at
random (without replacement) and are found to be all spades. Find the probability of the lost card being a spade.

KVS RO DDN S SM XII MATHEMATICS/ 2023-24(VN) Page 159


*KVS RO DEHRADUN STUDENT SUPPORT MATERIAL MATHEMATICS/ XII/2023-24(VN)*
BLUE PRINT
Sample Paper Session 2023-24
Class XII
Mathematics (Code-041)
Unit Chapter MCQ SA-I SA-II LA CASE
(1 (2 (3 (5 BASED TOTAL
mark) marks) marks) marks) (4
marks)
Relations and 1 (1) - - 1(5)
Relations and Functions (A&R) (with
Functions OR) 3(8)
(08 marks) Inverse - 1(2) - -
Trigonometric (with
Functions OR)
Matrices 2(2)
Algebra 6(10)
Determinants 3(3) 1(5)
(10 marks)
Continuity and 2(2)
Differentiability
Application of 1(1) 3(6) 1(4)
Calculus Derivatives (A&R) (1 with CB
(35 marks) OR)
Integrals 2(2) 1(2) 2(6) 16(35)
(1 with
OR)
Application of 1(5)
Integrals
Differential 1(1) 2(6)
Equations (1 with
OR)
Vectors and Vector Algebra 3(3) 1(4)
Three- CB 7(14)
Dimensional Three- 2(2) 1(5)
Geometry Dimensional (with
(14 marks) Geometry OR)
Linear Linear 2(2) 1(3) 3(5)
Programming Programming (with
(05 marks) OR)
Probability Probability 1(1) 1(3) 1(4) 3(8)
(08 marks) CB
TOTAL 20(20) 5(10) 6(18) 4(20) 3(12) 38(80)

Time Allowed: 3 Hours


Maximum Marks: 80

KVS RO DDN S SM XII MATHEMATICS/ 2023-24(VN) Page 160


*KVS RO DEHRADUN STUDENT SUPPORT MATERIAL MATHEMATICS/ XII/2023-24(VN)*
Sample Question Paper-1
Class XII
Session 2023-24
Mathematics (Code-041)
Time Allowed: 3 HoursMaximum Marks: 80
_____________________________________________________________________________________
General Instructions:
1. This Question paper contains - five sections A, B, C, D and E. Each section is compulsory. However,
there are internal choices in some questions.
2. Section A has 18 MCQ's and 02 Assertion-Reason based questions of 1 mark each.
3. Section B has 5 Very Short Answer (VSA)-type questions of 2 marks each.
4. Section C has 6 Short Answer (SA)-type questions of 3 marks each.
5. Section D has 4 Long Answer (LA)-type questions of 5 marks each.
6. Section E has 3 source based/case based/passage based/integrated units of assessment (4 marks each)
with sub parts
_____________________________________________________________________________________

SECTION A
(Multiple Choice Questions)
Each question carries 1 mark
1 0
Q1. 𝐼𝑓 [𝑥 1] [ ] = 𝑂, 𝑡ℎ𝑒𝑛 𝑥 =
−2 0

(a) 0(b) 1 (c) 2 (d) -2


7 1
Q2. The inverse of matrix [ ] is
4 −3
7 1 1 3 1 3 1 −3 −1
(a) [ ] (b)25 [ ] (𝑐) [ ] (𝑑) [ ]
−4 −3 4 −7 4 −7 −4 7

Q3. The direction cosine of Z-axis is


1 1 1
(a) 1, 1, 1 (b) , , (c) 0, 0, 0 (d) 1, 0, 0
√3 √3 √3
1−𝑐𝑜𝑠2𝑥
,𝑥 ≠ 0
Q4. The value of 'k' for which the function 𝑓(𝑥) = { 2𝑥2 is continuous at x=0 is
𝑘 ,𝑥 = 0

(a) 0 (b)-1 (c) 1. (d) 2


1
Q5. If 𝑓 ′ (𝑥) = 𝑥 + 𝑥 , 𝑡ℎ𝑒𝑛 𝑓(𝑥) 𝑖𝑠
1
(𝑎) 𝑥² + 𝑙𝑜𝑔 𝐼 𝑥 𝐼 + 𝐶 (𝑏) 𝑥 2 + 𝑙𝑜𝑔 𝐼 𝑥 𝐼 + 𝐶 (𝑐) + 𝑙𝑜𝑔 1𝑥1 + 𝐶 (𝑑) − 𝑙𝑜𝑔 [𝑥] + 𝐶
2

Q6. If m and n, respectively, are the order and the degree of the differential equation
𝑑𝑥 𝑑2 𝑦
𝑦 = 𝑥(𝑑𝑦)3 +𝑑𝑥 2 , then m + n=

(a) 5 (b) 3 (c) 2 (d) 4


Q7. The solution set of the inequality 3x+5y <4 is
(a) an open half-plane not containing the origin.
(b) an open half-plane containing the origin.
(c) the whole XY-plane not containing the line 3x+5y=4.
(d) a closed half plane containing the origin.
Q8. The projection of the vector 7𝑖⃗ + 𝑗⃗ − 4𝑘⃗⃗ 𝑜𝑛 𝑡ℎ𝑒 𝑣𝑒𝑐𝑡𝑜𝑟 2𝑖⃗ + 6𝑗⃗ + 3𝑘⃗⃗ 𝑖𝑠
7 7 8 7
(a) 14 (𝑏) 14 (𝑐) 7 (𝑑) 2

2 𝑥2
Q9. The value of ∫1 𝑥 3 +1
𝑑𝑥 is
1 9 1 2 2
(a) 3
𝑙𝑜𝑔 2 (𝑏) 3 𝑙𝑜𝑔 9 (𝑐)𝑙𝑜𝑔 8 (𝑑)0

KVS RO DDN S SM XII MATHEMATICS/ 2023-24(VN) Page 161


*KVS RO DEHRADUN STUDENT SUPPORT MATERIAL MATHEMATICS/ XII/2023-24(VN)*

Q10. If A and B are nonsingular square matrices of the same order, then 𝐴𝐵𝑇 − 𝐵𝑇 𝐴 𝑖𝑠 𝑎
(a) skew-symmetric matrix (b) symmetric matrix (c) 𝑛𝑢𝑙𝑙 𝑚𝑎𝑡𝑟𝑖𝑥 (d) Nonsingular matrix

Q11. The corner points of the shaded unbounded feasible region of an LPP are (0, 4). (0.6, 1.6) and
(3, 0) as shown in the figure. The minimum value of the objective function Z = 4x + 6y occurs at

(a) (0.6, 1.6) only (b) (3.0) only (c) (0.6, 1.6) and (3,0) (d) at every point of
the line-segment joining the points (0.6, 1.6) and (3,0) only
2 4 2𝑥 4
Q12.𝐼𝑓 | |=| | , 𝑡ℎ𝑒𝑛 𝑡ℎ𝑒 𝑣𝑎𝑙𝑢𝑒 𝑜𝑓 𝑥 𝑖𝑠
5 1 6 𝑥
(a) 1 (𝑏)√3(𝑐) − √3(𝑑)√3,− √3
Q13. If A is a square matrix of order 3 and I A I = 4, then I adj A I =
(a) 4 (b) 9 (c) 64 (d)16
Q 14. A and B are the two independent events such that P(A)=0.3 P(B)=0.6 and P(𝐴′ ∩ 𝐵′ ) is
(a) 0.9 (b) 0.18 (c) 0.28 (d) 0.1
Q15. The general solution of differential equation 𝑦𝑑𝑥 − 𝑥𝑑𝑦 = 0 is
𝑑
(𝑎) 𝑥𝑦 = 𝑐 (𝑏) 𝑥 = 𝑐𝑦 2 (𝑐) 𝑦 = 𝑐𝑥 (𝑑)𝑦 = 𝑐𝑥 2 . Q16. 𝑑𝑥 (𝑥 𝑥 ) 𝑖𝑠 𝑒𝑞𝑢𝑎𝑙 𝑡𝑜
(a) 𝑥 𝑥−1 (b) 𝑥𝑙𝑜𝑔𝑥 (c) 𝑥 𝑥 (1 + 𝑙𝑜𝑔𝑥) (d) x𝑥 𝑥−1
Q17. 𝑇ℎ𝑒 𝑣𝑎𝑙𝑢𝑒 𝑜𝑓 𝐼 𝑎⃗ − 𝑏⃗⃗ 𝐼 𝑖𝑓 𝐼𝑎⃗𝐼 = 2, 𝐼𝑏⃗⃗𝐼 = 5 𝑎𝑛𝑑 𝑎⃗. 𝑏⃗⃗ = 8
(a) 0 (b) 49 (c)√10 (d) √13
⃗⃗ ⃗⃗
Q18. The area of triangle determined by the vectors 𝑖⃗ + 2𝑗⃗ + 3𝑘 𝑎𝑛𝑑 3𝑖⃗ − 2𝑗⃗ + 𝑘 𝑖𝑠
(a) 4√3 (b) 8√3 (c) 36 (d) 18

ASSERTION-REASON BASED QUESTIONS


In the following questions, a statement of assertion (A) is followed by a statement of Reason (R). Choose the
correct answer out of the following choices.
(a) Both A and R are true and R is the correct explanation of A.
(b) Both A and R are true but R is not the correct explanation of A.
(c) A is true but R is false.
(d) A is false but R is true.
1 1
Q19. Assertion (A): The domain of the function 𝑠𝑒𝑐 −1 2𝑥 is(−∞, − 2] ∪ [2 , ∞)
𝜋
Reason (R): 𝑠𝑒𝑐 −1 (−2) = − 4
𝜋
Q20. Assertion (A): The acute angle between the line 𝑟⃗ = 𝑖⃗ + 𝑗⃗ + 2𝑘⃗⃗ + 𝜆(𝑖⃗ − 𝑗⃗)and the x-axis is 4
Reason(R): The acute angle 𝜃 between the lines
𝑟⃗ = 𝑥1 𝑖̂ + 𝑦1 𝑗̂ + 𝑧1 𝑘̂ + 𝜆(𝑎1 𝑖̂ + 𝑏1 𝑗̂ + 𝑐1 𝑘̂)𝑎𝑛𝑑 𝑎1 𝑎2+
𝐼𝑎1 𝑎2 +𝑏1 𝑏2 +𝑐1 𝑐2 𝐼
𝑟⃗ = 𝑥2 𝑖̂ + 𝑦2 𝑗̂ + 𝑧2 𝑘̂ + 𝜆(𝑎2 𝑖̂ + 𝑏2 𝑗̂ + 𝑐2 𝑘̂) is given by 𝑐𝑜𝑠𝜃 =
√𝑎12 +𝑏12 +𝑐12 √𝑎22 +𝑏22 +𝑐22

KVS RO DDN S SM XII MATHEMATICS/ 2023-24(VN) Page 162


*KVS RO DEHRADUN STUDENT SUPPORT MATERIAL MATHEMATICS/ XII/2023-24(VN)*

SECTION B
This section comprises of very short answer type-questions (VSA) of 2 marks each

5
Q21.Find the value of x, when cos−1 (13) = tan−1(𝑥)

OR
n+1
, if n is odd
Show that f: N → N defined by f(n) = { n 2 is not one − one , but onto
, if n is even
2
Q22.A particle moves along the curve 6𝑦 = 𝑥 3 + 2. Find the point at which the y coordinate is changing 8 times
as fast as the 𝑥 𝑐𝑜𝑜𝑟𝑑𝑖𝑛𝑎𝑡𝑒.
Q23. If 𝑎⃗ = 𝑖⃗ − 𝑗⃗ + 7𝑘⃗⃗ and 𝑏⃗⃗ = ⃗⃗⃗⃗
5𝑖 − 𝑗⃗ + 𝜆𝑘⃗⃗, then find the value of λ so that the vectors
𝑎⃗ + 𝑏⃗⃗ and ar𝑎⃗ − 𝑏⃗⃗ are perpendicular.
OR
Find the direction ratio and direction cosines of a line parallel to the line whose equations are
6𝑥 − 12 = 3𝑦 + 9 = 2𝑧 − 2

𝑑2𝑦 1 𝜋
Q24. If 𝑥 = 𝑎(1 − 𝑐𝑜𝑠𝜃), 𝑦 = 𝑎(𝜃 + 𝑠𝑖𝑛𝜃), 𝑝𝑟𝑜𝑣𝑒 𝑡ℎ𝑎𝑡 2 = − 𝑎𝑡 𝜃 =
𝑑 𝑥 𝑎 2
Q25. Find 𝐼𝑥⃗𝐼 𝑖𝑓 (𝑥⃗ − 𝑎⃗). (𝑥⃗ + 𝑎⃗) = 12 where 𝑎⃗ is a unit vector

SECTION C
(This section comprises of short answer type questions (SA) of 3 marks each)
𝑑𝑥
Q26. Find: ∫
√7−6𝑥−𝑥 2

Q27.Probabilities of solving a specific problem independently by A and B are ½ and 1/3 respectively. If both try
to solve the problem independently, find the probability that
(i)the problem is solved
(ii) Exactly one of them solve the problem.

OR
Find the mean number of defective items in a sample of two items drawn one-by-one without replacement from
an urn containing 6 items, which include 2 defective items. Assume that the items are identical in shape and size.
𝜋
Q28. Evaluate: ∫02 𝑙𝑜𝑔𝑡𝑎𝑛𝑥𝑑𝑥
OR
4
Evaluate: ∫0 𝐼𝑥 − 1𝐼 𝑑𝑥

Q29. Solve the differential equation: 𝑦𝑑𝑥 + (𝑥 − 𝑦 3 )𝑑𝑦 = 0


OR
2 2
Solve the differential equation: (𝑥 − 𝑦 ) 𝑑𝑥 + 2𝑥𝑦 𝑑𝑦 = 0

Q30. Solve the following Linear Programming Problem graphically:


𝑀𝑎𝑥𝑖𝑚𝑖𝑧𝑒 𝑍 = 400𝑥 + 300𝑦 𝑠𝑢𝑏𝑗𝑒𝑐𝑡 𝑡𝑜
𝑥 + 𝑦 ≤ 200, 𝑥 ≤ 40, 𝑥 ≥ 20, 𝑦 ≥0
2𝑥
031 Find. ∫ (𝑥 2 +1)(𝑥2 +2) 𝑑𝑥
SECTION D
(This section comprises of long answer-type questions (LA) of 5 marks each)

Q32. Make a rough sketch of the region in the first quadrant enclosed by x-axis, the line 𝑦 = √3𝑥and the circle
𝑥 2 + 𝑦 2 = 16 and find its area using integration.
𝑄33. 𝐷𝑒𝑓𝑖𝑛𝑒 𝑡ℎ𝑒 𝑟𝑒𝑙𝑎𝑡𝑖𝑜𝑛 𝑅 𝑖𝑛 𝑡ℎ𝑒 𝑠𝑒𝑡 𝑁 𝑥 𝑁 𝑎𝑠 𝑓𝑜𝑙𝑙𝑜𝑤𝑠:
𝐹𝑜𝑟 (𝑎, 𝑏), (𝑐, 𝑑)𝜖 𝑁 𝑋 𝑁, (𝑎, 𝑏)𝑅 (𝑐, 𝑑)𝑖𝑓𝑓 𝑎𝑑 = 𝑏𝑐. 𝑃𝑟𝑜𝑣𝑒 𝑡ℎ𝑎𝑡 𝑅 𝑖𝑠 𝑎𝑛 𝑒𝑞𝑢𝑖𝑣𝑎𝑙𝑒𝑛𝑐𝑒 𝑟𝑒𝑙𝑎𝑡𝑖𝑜𝑛 𝑖𝑛 𝑁 𝑥 𝑁.

KVS RO DDN S SM XII MATHEMATICS/ 2023-24(VN) Page 163


*KVS RO DEHRADUN STUDENT SUPPORT MATERIAL MATHEMATICS/ XII/2023-24(VN)*

OR
𝐿𝑒𝑡 𝐴 = 𝑅 − {2} 𝑎𝑛𝑑 𝐵 = 𝑅 − {1}. 𝐼𝑓 𝑓: 𝐴 → 𝐵 𝑖𝑠 𝑎 𝑓𝑢𝑛𝑐𝑡𝑖𝑜𝑛 𝑑𝑒𝑓𝑖𝑛𝑒𝑑 𝑎𝑠
𝑥−1
𝑓(𝑥) = , 𝑠ℎ𝑜𝑤 𝑡ℎ𝑎𝑡 𝑓 𝑖𝑠 𝑏𝑖𝑗𝑒𝑐𝑡𝑖𝑣𝑒.
𝑥−2
Q34.Find the shortest distance between the lines:
4𝑖 − 𝑗⃗ + 𝜆(𝑖⃗ + 2𝑗⃗ − 3𝑘⃗⃗ )
𝑟⃗ = ⃗⃗⃗⃗
and 𝑟⃗ = 𝑖⃗ − 𝑗 + 2𝑘⃗⃗ + 𝜇(2 𝑖⃗ + 4𝑗⃗ − 5𝑘⃗⃗ )
1 −1 1
Q35. If A = [2 1 −3 , find A– 1. Using A– 1, solve the following system of equations:
1 1 1
x + 2y + z = 4, − x + y + z = 0, x − 3y + z = 2
SECTION E
(This section comprises of 3 case-study/passage-based questions of 4 marks each with two sub-parts. First two
case study questions have three sub-parts (i), (ii), (iii) of marks 1, 1, 2 respectively. The third case study question
has two sub-parts of 2 marks each.)

Q36. Case-Study 1: Read the following passage and answer the questions given below.

The relation between the height of the plant (y in cm) with respect to exposure to sunlight is governed by the the
1
equation 𝑦 = 4𝑥 − 𝑥 2 where x is number of days exposed to sunlight.
2
(a) The rate of growth of the plant with respect to sunlight is.
1 1
(i) 4𝑥 − 2 𝑥 2 (ii) 4-x (iii) x-4 (iv)𝑥 − 2 𝑥 2
(b) What is the number of days it will take for the plant to grow to the maximum height?
(i) 4 (ii) 6 (iii) 7 (iv) 10
(c) What is the maximum height of the plant?
(i) 12cm (ii) 10cm (iii) 8cm (iv) 6cm
(d) What is the height of the plant after 2 days?
(i) 4cm (ii) 6cm (iii) 8cm (iv) 10cm
(e) If the height of the plant is 7/2 cm , the number of days it has been exposed to the sun light is
(i) 2 (ii) 3 (iii) 4 (iv) 1

Q37. Case study II:

45cm

24cm

A packaging company got the orders to make open boxes of maximum volume from rectangular sheets of
dimensions 45cm x 24cm. The execution department of company suggested to cut squares of equal side from
all corners of rectangular sheet and folding up the flaps as shown.
If square of side x cm is cut from each corner, then answer the following:

KVS RO DDN S SM XII MATHEMATICS/ 2023-24(VN) Page 164


*KVS RO DEHRADUN STUDENT SUPPORT MATERIAL MATHEMATICS/ XII/2023-24(VN)*

(a) Height of the box is


(i) (45 - x) cm (ii) (24 - x) cm (iii) 2x cm (iv) x cm
(b) Volume of the box is
(i) x(45 - 2x)(24 - 2x) cm3 (ii) x(45 - x)(24 - x) cm3
3
(iii) (45 - 2x)(24 - 2x) cm (iv) (45 - x)(24 - x) cm3
(c) The side of the square for which volume is maximum is
(i) 7 cm (ii) 5 cm (iii) 18 cm (iv)
3cm
(d)Maximum volume of the box is
(i) 2450 cm3 (ii) 1080 cm3 (iii) 800 cm3 (iv) 2450 cm2
(e) Area of base of box is
(i) 2450 cm2 (ii) 1080 cm2 (iii) 490 cm2 (iv) 800cm2
Q38. Case-Study 3: Read the following passage and answer the questions given below.

This Photo by Unknown Author is


There are two antiaircraft guns, named as licensed
A and B. The
under CCprobabilities
BY-SA that the shell fired from them hits an
airplane are 0.3 and 0.2 respectively. Both of them fired one shell at an airplane at the same time. (i) What is the
probability that the shell fired from exactly one of them hit the plane? (ii) If it is known that the shell fired from
exactly one of them hit the plane, then what is the probability that it was fired from B?

**********************************************************************************

KVS RO DDN S SM XII MATHEMATICS/ 2023-24(VN) Page 165


*KVS RO DEHRADUN STUDENT SUPPORT MATERIAL MATHEMATICS/ XII/2023-24(VN)*

SQP- 1
MATHEMATICS
CLASS XII
MARKING SCHEME
1.c 1

2.b 1

3.d 1
4.c 1

5.b 1

6.b 1
7.b 1
8.c 1
9.a 1

10.a 1

11.d 1
12.d 1
13.d 1
14.c 1
15.c 1
16.c 1
17.d 1
18.a 1

19.c 1

20.a
21.
𝟏𝟐 1
𝐭𝐚𝐧−𝟏 ( ) = 𝐭𝐚𝐧−𝟏(𝒙)
𝟓
𝟏𝟐 1
𝐱=
𝟓
𝐎𝐑
1
one-one
1
Not onto

𝑑𝑦 𝑑𝑥 1
22.6 = 3𝑥 2
𝑑𝑡 𝑑𝑡 2

𝑑𝑦 𝑑𝑥 1
=8
𝑑𝑡 𝑑𝑡 1
2
−31
Finding (4,11) and(-4, )_
3

KVS RO DDN S SM XII MATHEMATICS/ 2023-24(VN) Page 166


*KVS RO DEHRADUN STUDENT SUPPORT MATERIAL MATHEMATICS/ XII/2023-24(VN)*

1111
23.𝒂⃗⃗ + 𝒃⃗⃗ = 𝟔𝒊⃗ − 𝟐𝒋⃗ + (𝟕 + 𝝀)𝒌⃗⃗ 2222
⃗⃗ = −𝟒𝒊⃗ + (𝟕 − 𝝀)𝒌
⃗⃗ − 𝒃
𝒂 ⃗⃗
Perpendicular if (𝒂 ⃗⃗
⃗⃗ + 𝒃). (𝒂 ⃗⃗
⃗⃗ − 𝒃) = 𝟎
𝝀 = ±𝟓

𝑑𝑦 𝜃
24. Finding = 𝑐𝑜𝑡 1
𝑑𝑥 2
𝜋 1
Proving at 𝜃 =
2
25. correct dot product 1
1
Finding I x I=√13
𝒅𝒙 2
26. finding 𝐼 = ∫ 𝟐 𝟐
√𝟒 −(𝒙+𝟑)
𝒙+𝟑 1
𝑰 = 𝐬𝐢𝐧−𝟏 ( )+𝒄
𝟒
27. A and B are independent P(A∩ 𝐵)=P(A)P(B) 1
1
(i) P(A∪ 𝐵)=P(A)+P(B)-P(A)P(B) 2
1 1 1 2
= + − =
2 3 6 3
1 2 1 1 1 1
(ii) P(A)P(𝐵) + P(B)P(𝐴̅) = x + 𝑋 =
̅ 1
2 3 3 2 2 2
OR

1
2
1
2

𝜋
28. 2𝐼 = ∫0 (𝑙𝑜𝑔 𝑡𝑎𝑛𝑥 + log 𝑐𝑜𝑡𝑥 )𝑑𝑥
2
2
𝜋 1
=∫0 log 1𝑑𝑥 = 0
2

OR
1 4 1
𝐼 = ∫ (1 − 𝑥)𝑑𝑥 + ∫ (𝑥 − 1)𝑑𝑥 1
0 1
1
Integration
=5

KVS RO DDN S SM XII MATHEMATICS/ 2023-24(VN) Page 167


*KVS RO DEHRADUN STUDENT SUPPORT MATERIAL MATHEMATICS/ XII/2023-24(VN)*
1
𝑑𝑥 𝑥
29 + = 𝑦2
𝑑𝑦 𝑦
1 1
∫𝑦 𝑑𝑦=𝑦
IF=𝑒 1
𝑦4
Solving 𝑥𝑦 = + 𝐶
4
OR 1
𝑑𝑦 𝑦 2 − 𝑥 2
= 1
𝑑𝑥 2𝑥𝑦
2𝑣 𝑑𝑥 1
∫ 2 𝑑𝑣 = − ∫
𝑣 +1 𝑥
2
𝑦
𝑥 ( 2 + 1) = 𝐶
𝑥
30.

31 substitution 𝑥 2 = 𝑡 2𝑥𝑑𝑥 = 𝑑𝑡 1
1 1
Using partial fraction and doing 𝐼 = ∫( − )𝑑𝑡 1
𝑡+1 𝑡+2
2 2 1
I = log(𝑥 + 1) − log(𝑥 + 2) + 𝐶

32 fig. 1
Finding point of intersection of line and 1
circle(√3, 1)
2 1
√3
∆= + ∫ √4 − 𝑥 2 𝑑𝑥
2 √3 2
√3 𝜋 √3
= + −
2 3 2

KVS RO DDN S SM XII MATHEMATICS/ 2023-24(VN) Page 168


*KVS RO DEHRADUN STUDENT SUPPORT MATERIAL MATHEMATICS/ XII/2023-24(VN)*
1
33.

1
1
2

1
2

34. finding ⃗⃗⃗⃗⃗


𝑎2 − 𝑎 ⃗⃗⃗⃗⃗1 = −3𝑖 + 2𝑘 1
2
𝑏1 𝑋𝑏2 = 2𝑖 − 𝑗
(𝑎2 − 𝑎1). (𝑏1𝑋𝑏2) 6 1+1
𝑑=| |=
|𝑏1𝑋𝑏2| √5
35. |𝐴| = 10 ≠ 0 1
𝟒 𝟐 𝟐
2
𝑎𝑑𝑗𝐴 = [−𝟓 𝟎 𝟓] 𝒂𝒏𝒅 𝒇𝒊𝒏𝒅𝒊𝒏𝒈 𝑨−𝟏
𝟏 −𝟐 𝟑
−𝟏
𝑵𝒐𝒘 𝑺𝒚𝒔𝒕𝒆𝒎 𝒐𝒇 𝒆𝒒𝒖𝒂𝒕𝒊𝒐𝒏 𝑨𝑻 𝑿 = 𝑩, 𝑿 = (𝑨𝑻 ) 𝑩 1
𝟗 𝟐 𝟕
𝑭𝒊𝒏𝒅𝒊𝒏𝒈 𝒙 = , 𝒚 = , 𝒛 = 1
𝟓 𝟓 𝟓
36.(a)ii (b)i (c)iii (d)ii (e)iv
37. (a)iv (b)I (c)ii (d)i (e)iii
38.

KVS RO DDN S SM XII MATHEMATICS/ 2023-24(VN) Page 169


*KVS RO DEHRADUN STUDENT SUPPORT MATERIAL MATHEMATICS/ XII/2023-24(VN)*

Sample Question Paper-2


Class XII
Session 2023-24
Mathematics (Code-041)
Time Allowed: 3 Hours Maximum Marks: 80

General Instructions:
1. This Question paper contains - five sections A, B, C, D and E. Each section is compulsory. However,
there are internal choices in some questions.
2. Section A has 18 MCQ’s and 02 Assertion-Reason based questions of 1 mark each.
3. Section B has 5 Very Short Answer (VSA)-type questions of 2 marks each.
4. Section C has 6 Short Answer (SA)-type questions of 3 marks each.
5. Section D has 4 Long Answer (LA)-type questions of 5 marks each.
6. Section E has 3 source based/case based/passage based/integrated units of assessment (4 marks each)
with sub parts.

S.NO SECTION A
. Each question carries 1 mark M

1. If A is square matrix such that 𝐴2 = 𝐼, then (𝐴 − 𝐼)3 + (𝐴 + 𝐼)3 − 7𝐴 is equal to 1


(A) A (B) I-A (C) I+A (D) 3A
2. If A is a square matrix such that 𝐴2 = 𝐼, then 𝐴−1 is equal to: 1
(A) 2A (B) O (C) A (D) A+I
3. If 𝑎⃗ = 𝑖̂ − 2𝑗̂ + 3𝑘̂and 𝑏⃗⃗ be such that |𝑎⃗ − 𝑏⃗⃗| = √7 and |𝑏⃗⃗| =
√2
, then 𝑎⃗ . 𝑏⃗⃗ = |𝑏⃗⃗|2 then𝑏⃗⃗ 1
3
is
(A) √3 (B)√7(C) 3 (D)7
4. 𝑙𝑜𝑔(1+3𝑥)−𝑙𝑜𝑔(1−2𝑥) 1
𝑖𝑓 𝑥 ≠ 0
𝑇ℎ𝑒 𝑓𝑢𝑛𝑐𝑡𝑖𝑜𝑛 𝑓(𝑥) = { 𝑥 is continuous at 𝑥 = 0, then the value
𝑘, 𝑖𝑓 𝑥 = 0
of 𝑘 is
(A)1 (B) 5 (C) -1 (D) 2
5. 𝑑2 𝑦 1
If If𝑦 2 = 𝑎𝑥 2 + 𝑏 then 𝑑𝑥 2 is
𝑎𝑏 𝑥3 𝑎𝑏 𝑎𝑏
(A) 𝑥 3 (B) 𝑎𝑏 (C) 𝑦 2 (D) 𝑦 3
6. 𝑑𝑦 3/2 𝑑3 𝑦 1
The sum of order and degree of the differential equation (1 + 5 𝑑𝑥 ) = 10 𝑑𝑥 3 is
(A) 2 (B) 3 (C) 5 (D) 4

7. Objective function of a LPP is 1


(A) constant graph (B) a function to be optimized
(C) inequality (D) quadratic equation
8. The scalar projection of 𝑎⃗ = −2𝑖̂ +3𝑗̂ +𝑘̂ on 𝑏⃗⃗ =𝑖̂ +2𝑗̂-𝑘̂ is 1
√6 √3 2 √5
(A) (B) (C) (D)
2 2 √6 2
9. 𝑠𝑒𝑐 2 (𝑠𝑖𝑛−1 𝑥)
∫ 𝑑𝑥 𝑖𝑠 𝑒𝑞𝑢𝑎𝑙𝑠 𝑡𝑜 1
√1 − 𝑥 2
(A) sin(𝑡𝑎𝑛−1 𝑥) + 𝐶 ( B) tan(𝑠𝑒𝑐 −1 𝑥) + 𝐶
(C)tan(𝑠𝑖𝑛−1 𝑥) + 𝐶 (D) − tan(𝑐𝑜𝑠 −1 𝑥) + 𝐶
10. The maximum value of Z = 3x+ 4y subject to constraints x+ y≤4, x≥0, y≥0 is 1
(A) 16 (B) 12 (C) 0 (D) not defined

KVS RO DDN S SM XII MATHEMATICS/ 2023-24(VN) Page 170


*KVS RO DEHRADUN STUDENT SUPPORT MATERIAL MATHEMATICS/ XII/2023-24(VN)*

11. The number of all possible matrices of order 3 × 3 with each entry 0 or 1 is: 1
(A) 27 (B) 81 (C) 18 (D) 512
12. The area of a triangle with vertices (-3, 0), (3, 0) and (0, k) is 9 sq. units. then k is equals to 1
(A) 9 (B) 3 (C) -9 (D) 6

13. If A is an invertible matrix of order 2, then det(𝐴−1 ) is equal to 1


1
(A) det(𝐴) (B)det(𝐴) (C)1 (D) 0
14. The probability distribution of a discrete random variable X is given below, the value of k is 1
X 2 3 4 5
P(X) 5 7 9 11
𝑘 𝑘 𝑘 𝑘
(A) 8 (B) 32 (C) 16 (D) 48
15. If two lines 1−𝑥 = 𝑦−2 = 𝑧−3 and𝑥−1 = 𝑦 − 1 = 6−𝑧are perpendicular, then the value of 𝜆 is 1
3 2𝜆 2 3𝜆 5
−10 10 −10 10
(A) 7 (B) 7 (C) 11 (D) 11
16. The direction cosines of the line that makes equal angles with the three axes in space are 1
1 1
(A) ± (B)± (C) ±√2(D) ±√3
√2 √3
17. The value of 𝜆 for which two vectors 𝑎⃗ =3𝑖̂ -𝑗̂ +2𝑘̂ on 𝑏⃗⃗ =−𝑖̂ +𝑗̂-𝑘̂ are perpendicular 1
(A) 2 (B)3 (C) −2 (D) -3
18. ∫ √1 + 𝑐𝑜𝑠 𝑥 𝑑𝑥 is equal to 1
𝑥 𝑥 𝑥 1 𝑥
(A) 2 𝑠𝑖𝑛 (2) + 𝐶 (B) √2 𝑠𝑖𝑛 (2) + 𝐶 (C) 2√2 𝑠𝑖𝑛 (2) + 𝐶 (D) 2 𝑠𝑖𝑛 (2) +
𝐶
ASSERTION-REASON BASED QUESTIONS
In the following questions, a statement of assertion (A) is followed by a statement of
Reason (R). Choose the correct answer out of the following choices.
(a) Both A and R are true and R is the correct explanation of A.
(b) Both A and R are true but R is not the correct explanation of A.
(c) A is true but R is false.
(d) A is false but R is true.
19. Assertion(A): If 𝑋 = {0,1,2) and the function 𝑓: 𝑋 → 𝑌 defined by 𝑓(𝑥) = 𝑥 2 − 2 is 1
surjective then 𝑌 = {−2, −1,0}
Reason (R ) :If 𝑓: 𝑋 → 𝑌 is surjective then 𝑓(𝑥) = 𝑦.

20. Assertion (A):𝑓(𝑥) = 𝑙𝑜𝑔(𝑐𝑜𝑠 𝑥), 𝑥 > 0 is strictly increasing in (0, 𝜋). 1
Reason(R): If 𝑓 ′ (𝑥) ≥ 0 , then 𝑓(𝑥) is strictly increasing function.
SECTION B
This section comprises of very short answer type-questions (VSA) of 2 marks each
21. Find the value of 𝑠𝑖𝑛 [𝜋 − 𝑠𝑖𝑛−1 (−√3)]. 2
6 2
OR
Is the greatest integer function f : R → R be defined by f(x) = [x] one-one and onto? Justify.
22. The side of equilateral triangle is increasing at the rate 2 cm/sec at what rate area will 2
increase
when the side is 20 cm.
OR
The volume of a cube is increasing at the rate of 9 cm3/s. How fast is its surface area is
increasing when the length of an edge is 10 cm.?
23. Find the least value of a such that the function 𝑓 given by 𝑓(𝑥) = 𝑥 2 + 𝑎𝑥 + 1 is strictly 2
increasing on (1 ,2).

KVS RO DDN S SM XII MATHEMATICS/ 2023-24(VN) Page 171


*KVS RO DEHRADUN STUDENT SUPPORT MATERIAL MATHEMATICS/ XII/2023-24(VN)*

24. Find:𝑑𝑦 if 𝑦 = 𝑥 𝑠𝑖𝑛 𝑥 . 2


𝑑𝑥
25. Evaluate:∫1 𝑥(1 − 𝑥)𝑛 𝑑𝑥. 2
0
SECTION C
(This section comprises of short answer type questions (SA) of 3 marks each)
26. Evaluate ∫ 𝑥 2 +1 3
𝑑𝑥
(𝑥 +4)(𝑥 2 +25)
2

27. A bag contains 4 red and 4 black balls, another bag contains 2 red and 6 black balls. One of 3
the two bags is selected at random and a ball is drawn from the bag which is found to be
red.Find the probability that the ball is drawn from the first bag.
28. Evaluate 3
𝜋⁄
4

𝐼 = ∫ log(1 + tan 𝑥) 𝑑𝑥
0
OR
Evaluate:
𝜋
4 sin 𝑥 + cos 𝑥
𝐼=∫ 𝑑𝑥
0 9 + 16 sin 2𝑥

29. Solve the differential equation: 3


𝑑𝑦 𝜋
− 3𝑦 cot 𝑥 = sin 2𝑥,given that 𝑦 = 2 when 𝑥 = 2
𝑑𝑥
OR
Solve the differential equation:
𝑥𝑑𝑦 − 𝑦𝑑𝑥 = √𝑥 2 + 𝑦 2 𝑑𝑥
30. Solve the following LPP graphically: 3
Minimize: Z = x + y
Subject to the constraints 3x + 2y ≥ 12, x + 3y ≥ 11 and x ≥ 0, y ≥ 0.
OR
Solve the following Linear Programming Problem graphically:
Maximize:𝑧 = −𝑥 + 2𝑦
Subject to constraint:𝑥 ≥ 3, 𝑥 + 𝑦 ≥ 5, 𝑥 + 2𝑦 ≥ 6, 𝑦 ≥ 0
31. If (𝑥 2 − 𝑦 2 ) = 𝑐(𝑥 2 + 𝑦 2 )2 ,where c is parameter, then show that (𝑥 3 − 3𝑥𝑦 2 )𝑑𝑥 = 3
(𝑦 3 − 3𝑦𝑥 2 )𝑑𝑦.

SECTION D
(This section comprises of long answer-type questions (LA) of 5 marks each)
32. Using integration, find the area of the region in the first quadrant enclosed by the line x + y = 5
2, the parabola 𝑦 2 = x and the x-axis.
33. 𝑥−2 5
Let 𝐴 = 𝑅 − {3}, 𝐵 = 𝑅 − {1}. If 𝑓: 𝐴 → 𝐵 be defined by 𝑓(𝑥) = 𝑥−3 ∀ 𝑥 ∈ 𝐴. Then show
that the function 𝑓 is one and onto.
OR
Let 𝑁 denote the set of all natural numbers and 𝑅 be the relation on 𝑁 × 𝑁 defined by:
(𝑎, 𝑏)𝑅(𝑐, 𝑑) is 𝑎𝑑(𝑏 + 𝑐) = 𝑏𝑐(𝑎 + 𝑑). Show that R is an equivalence relation
34. Two bikers are running at the speed more than allowed on the road along the lines 𝑟⃗ = (𝑖̂ +𝑗̂ - 5
𝑘̂) + 𝜆(3𝑖̂ -𝑗̂) and 𝑟⃗ = (4𝑖̂ -𝑘̂) + 𝜇(2𝑖̂ +3𝑘̂).Find the Shortest distance between them at any
moment. Also check whether they meet to an accident or not.
OR
𝑥 𝑦−1 𝑧−2
Find the image of the point (1, 6, 3) in the line 1 = 2 = 3
35. 1 −2 0 7 2 −6 5
Find the product [2 1 3 [−2 1 −3 and using it solve the system of equations 𝑥 −
0 −2 1 −4 2 5
2𝑦 = 10, 2𝑥 + 𝑦 + 3𝑧 = 8, −2𝑦 + 𝑧 = 7

KVS RO DDN S SM XII MATHEMATICS/ 2023-24(VN) Page 172


*KVS RO DEHRADUN STUDENT SUPPORT MATERIAL MATHEMATICS/ XII/2023-24(VN)*

SECTION E
This section comprises of 3 case study/passage-based questions of 4 mark each with two sub-
parts. First two case study questions have three subparts (i), (ii), (iii) of marks 1, 2, 1
respectively. The third case study question has two subparts of 2 marks each.
36 Case-Study 1: Read the following passage and answer the questions given below.
An architect designs a building for a multi-national Design of Floor
company. The floor consists of a rectangular region Building
with semicircular ends having a perimeter of 200m as
shown
below:

Based on the above information answer the following:


(i)If x and y represents the length and breadth of the rectangular region, then what is the 1
relation between the variables.
(ii)What is the area of the rectangular region expressed as a function of x . 1
(iii)What is the maximum value of area A. 2
OR
The CEO of the multi-national company is interested in maximizing the area of the whole
floor including the semi-circular ends. For this to happen what should be the valve of x.
37 Case-Study 2: Read the following passage and answer the questions given below.
An insurance company believes that people can be
divided into two classes: those who are accident prone
and those who are not. The company’s statistics show
that an accident-prone person will have an accident at
sometime within a fixed one-year period with
probability 0.6, whereas this probability is 0.2 for a
person who is not accident prone. The company knows
that 20 percent of the population is accident prone.

Basedonthe above informationanswerthefollowing:


(i) what is the probability that a new policyholder will have an accident within a year of 2
purchasing a policy?
(ii) Suppose that a new policyholder has an accident within a year of purchasing a policy. 2
What is the probability that he or she is accident prone?

KVS RO DDN S SM XII MATHEMATICS/ 2023-24(VN) Page 173


*KVS RO DEHRADUN STUDENT SUPPORT MATERIAL MATHEMATICS/ XII/2023-24(VN)*

38
Case-Study 3: Read the following passage and answer the questions given below.
Solar panel have to be installed carefully so that tilt of the roof, and the direction to sun,
produce the largest possible electric power in the solar panel. A surveyor uses his instrument to
determine the coordinate of the four corners of a roof where Solar panels are to be mounted. In
the picture, suppose the points are labelled counter clockwise from the roof corner nearest to the
camera in units of meters A(6,8,4) , B(21,8,4), C(21,16,10) and D(6,16,10).
Based on the above information, Answer the following questions.

1
(i) What are the components to the two-edge vector defined by ⃗⃗⃗⃗⃗⃗
𝐴𝐵 𝑎𝑛𝑑 ⃗⃗⃗⃗⃗⃗
𝐴𝐷.
⃗⃗⃗⃗⃗⃗
(ii) Write unit vector in the direction of vector 𝐴𝐷. 1
⃗⃗ perpendicular to both ⃗⃗⃗⃗⃗⃗
(iii) Write the vector 𝑁 𝐴𝐵 𝑎𝑛𝑑 ⃗⃗⃗⃗⃗⃗
𝐴𝐷 . 2
OR
1 6 1
The sun located along the unit vector 𝑆⃗ = 2 𝑖̂ -7 𝑗̂ +7 𝑘̂ flow of solar energy is given by
the vector 𝐹⃗ = 910 𝑆⃗. What scalar product of 𝐹⃗ 𝑎𝑛𝑑 𝑁 ⃗⃗ .

KVS RO DDN S SM XII MATHEMATICS/ 2023-24(VN) Page 174


*KVS RO DEHRADUN STUDENT SUPPORT MATERIAL MATHEMATICS/ XII/2023-24(VN)*

Marking Scheme
Sample Paper-2
Class XII
Mathematics (Code -041)
Session 2023-24

SECTION A
(Multiple Choice Questions)
1 A 1 mark
2 C 1 mark
3 B 1 mark
4 B 1 mark
5 D 1 mark
6 C 1 mark
7 B 1 mark
8 A 1 mark
9 C 1 mark
10 A 1 mark
11 D 1 mark
12 B 1 mark
13 B 1 mark
14 B 1 mark
15 A 1 mark
16 B 1 mark
17 C 1 mark
18 C 1 mark
19 A Both A and R are true and R is the correct explanation of A. 1 mark
20 D A is false but R is true. 1 mark

SECTION B
21 𝜋 −√3
𝑠𝑖𝑛 [ − 𝑠𝑖𝑛−1 ( )]
6 2
𝜋 𝜋
1
= sin ( 6 − (− 3 )) ½
𝜋
= sin 2
½
=1
OR
For showing one-one
For showing onto
1
1
22 Let x is the side of equilateral triangle
√3 2 ½
𝐴= 𝑥
4
𝑑𝐴 √3 𝑑𝑥 1
Or = (2𝑥)
𝑑𝑡 4 𝑑𝑡
𝑑𝐴
Or ( 𝑑𝑡 ) 𝑎𝑡 𝑥 = 20 𝑖𝑠 20√3𝑐𝑚2 /𝑠𝑒𝑐 ½
OR
Let each edge of cube is x cm
𝑉 = 𝑥3
𝑑𝑉 𝑑𝑥 𝑑𝑥 𝑑𝑥 3
= 3𝑥 2 ⇒ 9 = 3 × 100 × ⇒ = 1
𝑑𝑡 𝑑𝑡 𝑑𝑡 𝑑𝑡 100 1

KVS RO DDN S SM XII MATHEMATICS/ 2023-24(VN) Page 175


*KVS RO DEHRADUN STUDENT SUPPORT MATERIAL MATHEMATICS/ XII/2023-24(VN)*
𝑑𝑆 𝑑𝑥
Now 𝑆 = 6𝑥 2 ⇒ = 12𝑥 𝑑𝑡 ⇒ (𝑑𝑆/𝑑𝑡)𝑎𝑡 𝑥 = 10 𝑖𝑠 3.6 𝑐𝑚2 /𝑠
𝑑𝑡

′ ½
𝑓(𝑥) = 𝑥 2 + 𝑎𝑥 + 1 ⇒ 𝑓 (𝑥) = 2𝑥 + 𝑎
23
Given f(x) is strictly increasing in (1,2) or f(x) is increasing
in [1,2]
𝑓 ′ (𝑥) ≥ 0 ⇒ 2𝑥 + 𝑎 ≥ 0 ⇒ 𝑎 ≥ −2 1
Therefore least value of a is -2. ½
24 log 𝑦 = sin 𝑥 log 𝑥 ½
Diff w r to x
1 𝑑𝑦 sin 𝑥
= + cos 𝑥 log 𝑥 1
𝑦 𝑑𝑥 𝑥
𝑑𝑦 sin 𝑥
= 𝑥 sin 𝑥 ( + cos 𝑥 log 𝑥) ½
𝑑𝑥 𝑥
25 1
∫ 𝑥(1 − 𝑥)𝑛 𝑑𝑥
0 1
1 𝑎 𝑎
𝐼 = ∫0 (1 − 𝑥)𝑥 𝑛 𝑑𝑥using property ∫0 𝑓(𝑥)𝑑𝑥 = ∫0 𝑓(𝑎 − 𝑥)𝑑𝑥
1 1 1
1 1 𝑥 𝑛+1 𝑥 𝑛+2 1
𝐼 = ∫0 𝑥 𝑛 𝑑𝑥 − ∫0 𝑥 𝑛+1 𝑑𝑥 ⇒ 𝐼 = [ 𝑛+1 ] − [ 𝑛+2 ] =(𝑛+1)(𝑛+2)
0 0

SECTION C
6 𝑥 2 +1
Evaluate ∫ (𝑥 2 +4)(𝑥 2 +25) 𝑑𝑥
𝑥 2 +1 𝑦+1 𝐴 𝐵
Let 𝑥 2 = 𝑦 𝑛𝑜𝑤 𝑙𝑒𝑡 = (𝑦+4)(𝑦+25) = 𝑦+4 + 𝑦+25
(𝑥 2 +4)(𝑥 2 +25) 1
−1 8
On solving 𝐴 = 𝑎𝑛𝑑 𝐵 = 7
7
𝑥 2 +1 −1 𝑑𝑥 8 𝑑𝑥 ½
Now 𝐼 = ∫ (𝑥 2 +4)(𝑥 2 +25) 𝑑𝑥 = 7 ∫ (𝑥 2 +4 + 7 ∫ (𝑥 2 +25)
−1 1 𝑥 8 1 𝑥 1
= × 𝑡𝑎𝑛−1 + × 𝑡𝑎𝑛−1 + 𝐶
7 2 2 7 5 5 ½
−1 −1
𝑥 8 −1
𝑥
= 𝑡𝑎𝑛 + 𝑡𝑎𝑛 +𝐶
14 2 35 5
27 Let
𝐸1 =Bag I is selected
𝐸2 = Bag II is selected
𝐴 = 𝐴 𝑏𝑎𝑙𝑙 𝑑𝑟𝑤𝑎𝑛 𝑖𝑠 𝑜𝑓 𝑅𝑒𝑑 𝑐𝑜𝑙𝑜𝑢𝑟
𝑃(𝐸1 )𝑃(𝐴/𝐸1 ) 1
𝑃(𝐸1 /𝐴) =
𝑃(𝐸1 )𝑃(𝐴/𝐸1 ) + 𝑃(𝐸2 )𝑃(𝐴/𝐸2 )
1 4 1
( )
𝑃(𝐸1 /𝐴) = 1 4 2 8 1 2
( ) + 2 (8) ½
2 8
2
𝑃(𝐸1 /𝐴) = ½
3
28 𝜋⁄
4

𝐼 = ∫ log(1 + tan 𝑥) 𝑑𝑥
0
𝜋⁄ 𝜋 𝑎 𝑎
𝐼= ∫0 4 log(1 + tan (4 − 𝑥) 𝑑𝑥using property ∫0 𝑓(𝑥)𝑑𝑥 = ∫0 𝑓(𝑎 − 𝑥)𝑑𝑥 1

KVS RO DDN S SM XII MATHEMATICS/ 2023-24(VN) Page 176


*KVS RO DEHRADUN STUDENT SUPPORT MATERIAL MATHEMATICS/ XII/2023-24(VN)*
𝜋⁄
4
1 − tan 𝑥 ½
I = ∫ log (1 + ) 𝑑𝑥
1 + tan 𝑥
0

𝜋⁄
4 ½
2
𝐼 = ∫ log ( ) 𝑑𝑥
1 + tan 𝑥
0
𝜋⁄ ½
4
𝜋
𝐼 = ∫ log 2 𝑑𝑥 − 𝐼 ⇒ 2𝐼 = log
4
0
𝜋 log 2
𝐼=
8
OR 1
𝜋
sin 𝑥 + cos 𝑥
4
𝐼=∫ 𝑑𝑥 ½
0 9 + 16 sin 2𝑥
𝜋
4 sin 𝑥 + cos 𝑥
𝐼=∫ 2
𝑑𝑥
0 25 − 16(sin 𝑥 − cos 𝑥) 1
Let sin 𝑥 − cos 𝑥 = 𝑡 ⇒ (cos 𝑥 + sin 𝑥)𝑑𝑥 = 𝑑𝑡
0
1 1 0 1 1 5 + 4𝑡 0
𝐼=∫ 𝑑𝑥 = ∫ 𝑑𝑥 = [𝑙𝑜𝑔 | |] ½
−1 25 − 16𝑡
2 16 −1 5 2 2 40 5 − 4𝑡 −1
(4) − 𝑡
1
𝐼= log 9
40

29 𝑑𝑦
− 3𝑦 cot 𝑥 = sin 2𝑥,
𝑑𝑥
1
𝑃 = −3 cot 𝑥 𝑄 = sin 2𝑥 𝑎𝑛𝑑 𝐼𝐹 = 𝑒 ∫ −3 cot 𝑥 𝑑𝑥 = 1
𝑠𝑖𝑛3 𝑥
Solution is 𝑦 × 𝐼𝐹 = ∫(𝑄 × 𝐼𝐹)𝑑𝑥
𝑦 sin 2𝑥 𝑦
=∫ 𝑑𝑥 ⇒ = −2𝑐𝑜𝑠𝑒𝑐 𝑥 + 𝐶 1
3
𝑠𝑖𝑛 𝑥 3
𝑠𝑖𝑛 𝑥 𝑠𝑖𝑛3 𝑥
𝜋
𝑖𝑓 𝑦 = 2 𝑎𝑡 𝑥 = ⇒ 𝐶 = 4 1
2
Therefore 𝑦 = 4𝑠𝑖𝑛3 𝑥 − 2𝑠𝑖𝑛2 𝑥
OR
𝑥𝑑𝑦 − 𝑦𝑑𝑥 = √𝑥 2 + 𝑦 2 𝑑𝑥
𝑑𝑦 𝑦 𝑦 2 𝑦 𝑑𝑦 𝑑𝑣 1
= + √1 + ( ) 𝑛𝑜𝑤 𝑙𝑒𝑡 = 𝑣 ⇒ =𝑣+𝑥
𝑑𝑥 𝑥 𝑥 𝑥 𝑑𝑥 𝑑𝑥
𝑑𝑣
Now 𝑣 + 𝑥 𝑑𝑥 = 𝑣 + √1 + 𝑣 2 ½
1 1
∫ 𝑑𝑥 = ∫ 𝑑𝑣 ½
𝑥 √1 + 𝑣 2
log 𝑥 = 𝑙𝑜𝑔 |𝑣 + √1 + 𝑣 2 | + log 𝐶 ⇒ 𝐶𝑥 2 = 𝑦 + √𝑥 2 + 𝑦 2
1

30 Corner Points Z=x+y Remark


A(0,6) 6
B(2,3) 5 Minimum value
D(11,0) 11
½× 2 = 1 mark for drawing of each constraint

KVS RO DDN S SM XII MATHEMATICS/ 2023-24(VN) Page 177


*KVS RO DEHRADUN STUDENT SUPPORT MATERIAL MATHEMATICS/ XII/2023-24(VN)*
½ marks for plotting correct graph for feasible region
1 mark for correct corner points and corresponding value of z.
½ for correct answer

Corner Points Z = -x+2y Remark


A(3,2) 1 May or may not be the
maximum value as the
feasible region is
unbounded
B(4,1) -2
D(6,0) -6
½× 2 = 1 mark for drawing of each constraint
½ marks for plotting correct graph for feasible region
1 mark for correct corner points and corresponding value of z.
½ for correct answer
31 (𝑥 2 − 𝑦 2 ) ½
(𝑥 2 − 𝑦 2 ) = 𝑐(𝑥 2 + 𝑦 2 )2 ⇒ 𝑐 = 2
(𝑥 + 𝑦 2 )2
d.w.r.to x to both sides
𝑑𝑦 𝑑𝑦 1
(𝑥 2 + 𝑦 2 )2 (2𝑥 − 2𝑦 ) − (𝑥 2 − 𝑦 2 )2(𝑥 2 + 𝑦 2 ) (2𝑥 + 2𝑦 )
𝑑𝑥 𝑑𝑥
0=
(𝑥 2 + 𝑦 2 )4
On simplifying 1
𝑑𝑦(𝑥 2 𝑦 + 𝑦 3 + 2𝑥 2 𝑦 − 2𝑦 3 ) = 𝑑𝑥(𝑥 3 + 𝑥𝑦 2 − 2𝑥 3 + 2𝑥𝑦 2 )
Or (𝑥 3 − 3𝑥𝑦 2 )𝑑𝑥 = (𝑦 3 − 3𝑦𝑥 2 )𝑑𝑦. ½

SECTION D
(This section comprises of long answer-type questions (LA) of 5 marks each)

32 Drawing correct figure 1

Solution: Solving x + y = 2 and y 2 =


x simultaneously, we get the points
of intersection as (1, 1) and (4, -2) 1
The required area=area of shaded
region 1
1 2
𝐴 = ∫ √𝑥 𝑑𝑥 + ∫ (2 − 𝑥)𝑑𝑥
0 1
1 2 1
2 𝑥2 2
A= [𝑥 3/2 ] + [2𝑥 − ] =3 +
3 0 2 1
1 7 1
= 6 𝑢𝑛𝑖𝑡
2
33 Injectivity(one -one):𝑙𝑒𝑡 𝑓(𝑥1 ) = 𝑓(𝑥2 ) 1
𝑥1 − 2 𝑥2 − 2
⇒ =
𝑥1 − 3 𝑥2 − 3
1
⇒ (𝑥1 − 2)(𝑥2 − 3) = (𝑥2 − 2)(𝑥1 − 3)
⇒ on solving 𝑥1 = 𝑥2 ½
Hence the function is one-one.
Subjectivity(on to):
𝑥−2
𝑦=
𝑥−3
⇒ 𝑦(𝑥 − 3) = 𝑥 − 2 1
⇒ 𝑦𝑥 − 3𝑦 = 𝑥 − 2
⇒ 𝑥(𝑦 − 1) = 3𝑦 − 2
3𝑦 − 2
⇒𝑥=
𝑦−1

KVS RO DDN S SM XII MATHEMATICS/ 2023-24(VN) Page 178


*KVS RO DEHRADUN STUDENT SUPPORT MATERIAL MATHEMATICS/ XII/2023-24(VN)*

Now show 𝑓(𝑥) = 𝑦 1


Therefore, the function is on to.
½
OR

We have : (a, b) R (c, d) ⇒ad (b + c) = bc (a + d) on N.


Reflexive: {a, b) R (a, b) ⇒ab(b + a) = ba (a + b)
⇒ab(a + b) = ab (a + b), which is true.
Therefore R is reflexive.
Symmetric: (a, b) R (c, d) 1
⇒ad(b + c) = be (a + d)
⇒bc(a + d) = ad (b + c)
⇒cb(d + a) = da (c + b)
[∵bc = cb and a + d = d + a; etc. ∀a, b, c, d ∈N] ⇒(cb) R (a,b).]
Therefore R is symmetric
Transitive: Let (a , b) R (c, d) and (c, d) R ( e, f)
∴ad (b + c) = bc (a + d)
and cf(d + e) = de(c + f) 2
⇒be(a+f) = af(b+e)
⇒af(b+e) = be(a
⇒(a, b) R (e,f).
Therefore R is transitive.
Hence, R is an equivalence relation. 2
34 𝑟⃗ = (𝑖̂ +𝑗̂ -𝑘̂ ) + 𝜆(3𝑖̂ -𝑗̂) and 𝑟⃗ = (4𝑖̂ -𝑘̂) + 𝜇(2𝑖̂ +3𝑘̂ ) 1

⃗⃗⃗⃗2 = −3𝑖ˆ − 9𝑗ˆ + 2𝑘ˆ


⃗⃗⃗⃗1 × 𝑏
𝑏
1
𝑎2 − 𝑎
⃗⃗⃗⃗⃗ ⃗⃗⃗⃗⃗1 = 3𝑖̂ − 𝑗ˆ
1
|(𝑎
⃗⃗⃗⃗⃗2 − 𝑎⃗⃗⃗⃗⃗). ⃗⃗⃗⃗ ⃗⃗⃗⃗
1 (𝑏1 × 𝑏2 )|
𝑆𝐷 =
⃗⃗⃗⃗⃗⃗1 × 𝑏
|𝑏 ⃗⃗⃗⃗2 |
1
For substituting values
For correct answer =0
They meet an accident if drive with higher speed.
OR 1
𝑥−0 𝑦−1 𝑧−2
= = =𝜆
1 2 3
𝑥 = 𝜆, 𝑦 = 2𝜆 + 1, 𝑧 = 3𝜆 + 2
direction ratios of perpendicular drawn from (1,6,3) are
(1- 𝜆, 5 − 2𝜆, 1 − 3𝜆) 1

(1 − 𝜆)1 + (5 − 2𝜆)2 + (1 − 3𝜆)3 = 0 1


𝜆=1
The foot of perpendicular from (1,6,3) is (1,3,5)
Image is (1,0,7) 1

1
1

KVS RO DDN S SM XII MATHEMATICS/ 2023-24(VN) Page 179


*KVS RO DEHRADUN STUDENT SUPPORT MATERIAL MATHEMATICS/ XII/2023-24(VN)*

35 1 −2 0 7 2 −6 11 0 0
[2 1 3 [−2 1 −3 =[ 0 11 0
0 −2 1 −4 2 5 0 0 11 1

1 −2 0 7 2 −6
𝐴𝐵 = 11 𝐼 where A=[2 1 3 𝑎𝑛𝑑 𝐵 = [−2 1 −3
0 −2 1 −4 2 5

−1
1 7 2 −6 1
𝐴 = [−2 1 −3
11
−4 2 5
½
Now 𝐴𝑋 = 𝐶 or 𝑋 = 𝐴−1 𝐶

1 −2 0 𝑥 10
[2 1 3 [𝑦 = [8 ½
0 −2 1 𝑧 7
½
𝑥 1 7 2 −6 10
[𝑦 = [−2 1 −3 [8
11
𝑧 −4 2 5 7 ½

𝑥 4
[𝑦 = [−3 1
𝑧 1

SECTION E
36 (i) 2𝑥 + 𝜋𝑦 = 200 1
200−2𝑥 200𝑥−2𝑥 2 1
(ii) 𝐴 = 𝑥𝑦 = 𝑥 × ( 𝜋 ) = 𝜋
𝑑𝐴 200−4𝑥
(iii) 𝑑𝑥
= 𝜋
=0
100
𝑜𝑟 𝑥 = 50 𝑎𝑛𝑑 𝑦 = 2
𝜋
100 5000
𝐴 = 𝑥𝑦 = 50 × = sq m
𝜋 𝜋
𝑂𝑅
x=50 m
37 Let 𝐸1 = 𝑇ℎ𝑒 𝑝𝑜𝑙𝑖𝑐𝑦 ℎ𝑜𝑙𝑑𝑒𝑟 𝑖𝑠 𝑎𝑐𝑐𝑖𝑑𝑒𝑛𝑡 𝑝𝑟𝑜𝑛𝑒
𝐸2 = 𝑇ℎ𝑒 𝑝𝑜𝑙𝑖𝑐𝑦 ℎ𝑜𝑙𝑑𝑒𝑟 𝑖𝑠 𝑛𝑜𝑡 𝑎𝑐𝑐𝑖𝑑𝑒𝑛𝑡 𝑝𝑟𝑜𝑛𝑒 2
A= The new policy holder has an accident within a year of purchasing a policy
𝐴 𝐴
(i) 𝑃(𝐴) = 𝑃(𝐸1 )𝑃 (𝐸 ) + 𝑃(𝐸2 )𝑃 (𝐸 )
1 2
20 6 80 2 7
= × + × =
100 10 100 10 25
(𝑖𝑖) using Bayes, theorem
𝐴 20 6
𝐸1 𝑃(𝐸1 )𝑃 (𝐸 ) × 10
1 100
𝑃( ) = 𝐴 𝐴
= 7
𝐴 𝑃(𝐸1 )𝑃 (𝐸 ) + 𝑃(𝐸2 )𝑃 (𝐸 )
1 25
2
2

3
=7

38 i. 15,0,0 and 0,8 6 1


1
ii. 10
(8𝑗̂ +6𝑘̂ )
1
iii. −90𝑖̂ +120𝑗̂ 𝑂𝑅
85800
2

KVS RO DDN S SM XII MATHEMATICS/ 2023-24(VN) Page 180


*KVS RO DEHRADUN STUDENT SUPPORT MATERIAL MATHEMATICS/ XII/2023-24(VN)*
SAMPLE QUESTION PAPER-3
Class:-XII
Session 2023-24
Mathematics (Code-041)
Time: 3 hours Maximum marks: 80
General Instructions:
1. This Question paper contains - five sections A, B, C, D, and E. Each section is compulsory. However, there are
internal choices in some questions.
2. Section A has 18 MCQs and 02 Assertion-Reason-based questions of 1 mark each.
3. Section B has 5 Very Short Answer (VSA)-type questions of 2 marks each.
4. Section C has 6 Short Answer (SA)-type questions of 3 marks each.
5. Section D has 4 Long Answer (LA)-type questions of 5 marks each.
6. Section E has 3 source-based/case-based/passage-based/integrated units of assessment of 4 marks each with
sub-parts.
Q. Question Marks
No. Allotted
Section –A (Multiple Choice Questions)
1 The position vector of the point which divides the join of points with position vectors 𝑎⃗ + 1
⃗⃗ ⃗⃗⃗⃗⃗ ⃗⃗
𝑏 and 2𝑎 − 𝑏 in the ratio 1:2 is
1 ⃗⃗⃗⃗⃗ 1
5 1
(a) 3 (3𝑎⃗ + 2𝑏⃗⃗) (b) 𝑎⃗ (c) 3 𝑎 − 3 𝑏⃗⃗ (d) 3 (4𝑎⃗ + 𝑏⃗⃗)
2 If 𝐴 = [𝑎𝑖𝑗 ] is a square matrix of order 3 such that 𝑎𝑖𝑗 = 𝑖 2 − 𝑗 2 , then A is 1
(a) A skew-symmetric matrix (b) a symmetric matrix
(c ) a scalar matrix (d) an identity matrix
3 If A and B are square matrices of the same order, then matrix (𝐴𝐵𝑇 − 𝐵𝐴𝑇 ) is 1
(a) A null matrix (b) a symmetric matrix
(c ) A skew-symmetric matrix (d) a unit matrix
4 If A and B are square matrices of order 3 X 3 such that |𝐴| = 5 𝑎𝑛𝑑 |𝐵| = 1
3 𝑡ℎ𝑒𝑛 |3𝐴𝐵|𝑖𝑠 𝑒𝑞𝑢𝑎𝑙 𝑡𝑜
(a) 135 (b) 45 (c) 405 (d) None of the above
5 1 1 1 1 √3 √3 1
𝑇ℎ𝑒 𝑚𝑎𝑥𝑖𝑚𝑢𝑚 𝑣𝑎𝑙𝑢𝑒 𝑜𝑓 ∆= | 1 1 + sin 𝜃 1| 𝑖𝑠 (𝑎) (𝑏) (𝑐)√2(𝑑)
2 2 4
1 + cos 𝜃 1 1
6 2 3 2 1
If |𝑥 𝑥 𝑥 | + 3 = 0, then the value of x is
4 9 1
(a) 4 (b) 0 (c) -1 (d) 1
7 1−sin 𝑥 1
∫ 𝑒 𝑥 (1−cos 𝑥) 𝑑𝑥 is equal to
𝑥 𝑥
(a) −𝑒 𝑥 tan + 𝐶 (b) −𝑒 𝑥 cot + 𝐶
2 2
1 𝑥 1 𝑥
(c ) − 2 𝑒 𝑥 tan 2 + 𝐶 (d) − 2 𝑒 𝑥 cot 2 + 𝐶
3
8 1
𝑑𝑦 2 2 𝑑2 𝑦
The sum of degree and order of the differential equation [1 + (𝑑𝑥 ) ] = (𝑑𝑥 2 ) is
(a) 1 (b) 2 (c ) 3 (d) 4

9 𝑑𝑦 1
The integrating factor of the differential equation 𝑡𝑎𝑛𝑥 𝑑𝑥 − 𝑦 = 2𝑥 2 is
1
(a) 𝑒 −𝑥 (b) 𝑒 −𝑦 (c) 𝑥
(d) cosecx
10 If |𝑎⃗| = 3 𝑎𝑛𝑑 − 1 ≤ 𝑘 ≤ 2, 𝑡ℎ𝑒𝑛 |𝑘𝑎⃗| 𝑙𝑖𝑒𝑠 𝑖𝑛 𝑡ℎ𝑒 𝑖𝑛𝑡𝑒𝑟𝑣𝑎𝑙 1
(a) [0, 6] (b)[-3, 6] (c) [3, 6] (d) [1, 2]

11 For real numbers x and y, define x R y iff x-y+√2 is an irrational number. Then the 1
relation R is
(a) Reflexive (b) symmetric (c) transitive (d) None of the above

KVS RO DDN S SM XII MATHEMATICS/ 2023-24(VN) Page 181


*KVS RO DEHRADUN STUDENT SUPPORT MATERIAL MATHEMATICS/ XII/2023-24(VN)*

12 ABCD is rhombus whose diagonals intersect at E. Then, ⃗⃗⃗⃗⃗⃗


𝐸𝐴 + ⃗⃗⃗⃗⃗⃗
𝐸𝐵 + 𝐸𝐶⃗⃗⃗⃗⃗⃗ + 𝐸𝐷
⃗⃗⃗⃗⃗⃗ equals 1

(a) 0⃗ ⃗⃗⃗⃗⃗⃗
(b) 𝐴𝐷 ⃗⃗⃗⃗⃗⃗
(c) 2 𝐵𝐶 (d) 2𝐴𝐷 ⃗⃗⃗⃗⃗⃗
13 If 𝑎⃗ and 𝑏⃗⃗ are non-zero vectors such that |𝑎⃗ + 𝑏⃗⃗| = |𝑎⃗ − 𝑏⃗⃗|, then 1
(a) 𝑎⃗ ∥ 𝑏⃗⃗ (b) 𝑎⃗ ⊥ 𝑏⃗⃗ (c) 𝑎⃗ = 𝑏⃗⃗ (d) None of the above

14 The direction cosines of a line are equal then direction cosine are 1
1 1
(𝑎)0 (𝑏)√3(𝑐)2 (𝑑) 𝑜𝑟 −
√3 √3
15 Corner points of the feasible region determined by the system of linear constraints are: (0, 1
10), (5, 5), (15, 15) and (0, 20). Let z = px - qy, where p, q > 0. Condition on p and q so
that the maximum value of z occurs at both the points (15, 15) and (0, 20) is
(a) P = 2q (b) 2p = q (c) q = -3p (d) p = q

16 Which of the following sets is convex? 1


(𝑎){(𝑥, 𝑦): 𝑥 2 + 𝑦 2 ≥ 1}(𝑏){(𝑥, 𝑦): 𝑦 2 ≥ 𝑥}
(𝑐){(𝑥, 𝑦): 3𝑥 2 + 4𝑦 2 ≥ 5}(𝑑){(𝑥, 𝑦): 𝑥 2 + 𝑦 2 ≤ 1}

𝜋
17 𝑎 sin (𝑥 + 1), 𝑥≤0 1
2
If 𝑓(𝑥) = {tan 𝑥−sin 𝑥 is continuous at x = 0, then ‘a’ equals to
, 𝑥>0
𝑥3
1 1 1 1
(𝑎) (𝑏) (𝑐) (𝑑)
2 3 4 6
18 Let 𝑓(𝑥) = |sin 𝑥|. Then, 1
(a) 𝑓 is differentiable everywhere
(b) 𝑓 is continuous everywhere but not differentiable at 𝑥 = 𝑛𝜋, 𝑛𝜖𝑍
𝜋
(c) 𝑓 is continuous everywhere but not differentiable at 𝑥 = (2𝑛 + 1) 2 , 𝑛𝜖𝑍
(d) 𝑁𝑜𝑛𝑒 𝑜𝑓 𝑡ℎ𝑒𝑠𝑒
ASSERTION-REASON BASED QUESTIONS
In the following questions, a statement of Assertion (A) is followed by a statement of
Reason (R). Choose the correct answer out of the following choices.
(a) Both (A) and (R) are true and (R) is the correct explanation of (A).
(b) Both (A) and (R) are true but (R) is not the correct explanation of (A).
(c) (A) is true but (R) is false.
(d) (A) is false but (R) is true.

19 ASSERTION (A): The value of sin−1 (sin 10) 𝑖𝑠 3𝜋 − 10. 1


REASON (R): the value of 𝑐𝑜𝑠 −1(cos 10) = 4𝜋 − 10

20 ASSERTION (A): the critical points of 𝑓(𝑥) = |𝑥 − 1| + |𝑥 + 1|are square roots of 1


unity.
REASON (R): 𝑥 = 𝛼 is a critical point of f(x), if either 𝑓′(𝛼) does not exist or 𝑓 ′ (𝛼) = 0.

Section B(Very short answer type questions)


21 −1 cos 𝑥 2
Express tan (1−sin 𝑥) in simplest form.
22 𝑒 𝑥 𝑑𝑥 2
Find∫ (𝑒 𝑥 −1)2 (𝑒 𝑥 +2)
23 Each side of an equilateral triangle is increasing at the rate of 8 cm/sec. find the rate of 2
increase of its area when side is 2 cm.
24 Show that the function f(x) = 𝑥 3 − 3𝑥 2 + 4𝑥 + 5 is strictly increasing function. 2
Or
Find the interval in which the function 𝑓(𝑥) = 𝑥 2 − 5𝑥 + 3 is strictly decreasing.
25 𝑑𝑦 𝜋 2
If 𝑦 = log √tan 𝑥 , 𝑡ℎ𝑒𝑛 𝑓𝑖𝑛𝑑 𝑎𝑡 𝑥 =
𝑑𝑥 4

KVS RO DDN S SM XII MATHEMATICS/ 2023-24(VN) Page 182


*KVS RO DEHRADUN STUDENT SUPPORT MATERIAL MATHEMATICS/ XII/2023-24(VN)*

Section C
26 𝑑𝑦 3
If𝑥𝑦+𝑦𝑥+𝑥𝑥=𝑎𝑎,find 𝑑𝑥

27 𝜋⁄3 1 3
E𝑣𝑎𝑙𝑢𝑎𝑡𝑒 ∫𝜋⁄6 1+√tan 𝑥
𝑑𝑥.
28 3
Evaluate: ∫ √1 + 3𝑥 − 𝑥 2 𝑑𝑥
OR
1 𝜋𝑥
Evaluate:∫0 (𝑥𝑒 𝑥 + sin ) 𝑑𝑥
4

29 Solve the differential equation: 𝑦𝑑𝑥 + (𝑥 − 𝑦 2 )𝑑𝑦 = 0 3


Or
Solve the differential equation: 𝑥𝑑𝑦 − 𝑦𝑑𝑥 = √𝑥 2 + 𝑦 2 𝑑𝑥
30 Solvethefollowingproblemgraphically: 3
Maximize𝑧=6𝑥+3𝑦,
subjecttotheconstraints:
4𝑥+𝑦≥80, 𝑥+5𝑦≥115,3x+2y≤150,x ≥0,y≥0.
Or
Solvethefollowingproblemgraphically:
Minimize𝑧=-3x + 4y,
subjecttotheconstraints:
𝑥 + 2𝑦 ≤ 8, 3𝑥 + 2𝑦 ≤ 12, 𝑥 ≥ 0, 𝑦 ≥ 0
31 Of the students in a college, it is known that 60% reside in hostel and 40% are day 3
scholars (not residing in hostel). Previous yearresults report that 30% of all students
who reside in hostel attainA grade and 20% of day scholars attain A grade in their
annual examination. At the end of the year, one student is chosen
atrandomfromthecollege andthatstudenthas anAgrade,what is
theprobabilitythatthestudent ishostlier?
Section D
32 Show that the relation R defined by (a,b) R (c,d) if and only if a + d = b + c on the set N × N 5
is an equivalence relation.

33 Solve the following system of equations by matrix method 5


3x - 2y + 3z = 8
2x + y – z = 1
4x - 3y + 2z = 4

34 𝑥2 𝑦2 5
Find the area of the region bounded by the ellipse 16 + 9 = 1.
OR
Using integration find the area bounded by the curves y2 =9x ,x = 2,x=4
and the x-axis in first quadrant.

35 Findtheshortestdistance betweenthelineswhose vectorequationsare 5


𝑟⃗=(1−2𝑡)𝑖̂+(1−𝑡)𝑗̂+(𝑡)𝑘̂and
𝑟⃗=(2+3𝑠)𝑖̂+(1−5𝑠)𝑗̂+(2𝑠−1) 𝑘̂
OR
Findthe equation ofalinethrough A(5,−3, -2)and through the intersection of the line lines:
𝑥−2 𝑦−3 𝑧−4 𝑥−4 𝑦−2 𝑧+3
1
= 5 = 4 and 3 = 4 = −3

KVS RO DDN S SM XII MATHEMATICS/ 2023-24(VN) Page 183


*KVS RO DEHRADUN STUDENT SUPPORT MATERIAL MATHEMATICS/ XII/2023-24(VN)*

Section E
36 One day, a sangeetMahotsav is to be organised in an open area of Rajasthan. In recent years, it
has rained only 6 days each year. Also, it is given that when it actually rains, the weatherman
correctly forecasts rain 80% of the time. When it doesn’t rain, he incorrectly forecasts rain 20%
1
of the time. If leap year is considered, then answer the following questions.
1
(i) Find the probability that the weatherman predict rain. 2
(ii) Find The probability that it will rain on the chosen day, if weatherman predict rain
for that day,

37 In the concept of Vector Algebra, Aditi took three vectors 𝑎⃗=2𝑖̂+2𝑗̂+3𝑘̂, 𝑏⃗⃗=-𝑖̂+2𝑗̂+𝑘̂ and 𝑐⃗=3𝑖̂+𝑗̂
in such a way that 𝑎⃗+λ𝑏⃗⃗ is perpendicular to the vector 𝑐⃗. Now she is curious to find out the
answers of some questions. Help her in finding them. 1

(i) What is the vector 𝑑⃗, if she defines 𝑑⃗=𝑎⃗+λ𝑏⃗⃗? 2


(ii) What is the value of λ? 1
Or
Aditi wants to find unit vector along vector 𝑎⃗ + 𝑏⃗⃗ − 2𝑐⃗.
(iii) Find the unit vector in direction of 𝑎⃗+λ𝑏⃗⃗
38 In order to set a rainwater harvesting system, a tank to collect rain water is to dug. The tank
should have a square base and a capacity 250 𝑚3 . The cost of land is 5000 𝑝𝑒𝑟 𝑚2 and cost of
digging increase with depth and for the whole tank, it is Rs 40000 ℎ2 , where h is the depth of
2
the tank in meters and x is the side of the square base of the tank in meters.
Based on the above information answer the following questions: 2
(i) Find the total cost C of digging the tank in term of x.
𝑑𝐶
(ii) Find 𝑑𝑥.

(iii) Find the value of x for which cost C is minimum.


Or
Check whether the cost function C expressed in the term of x is increasing or not,
where x greater than 0.

KVS RO DDN S SM XII MATHEMATICS/ 2023-24(VN) Page 184


*KVS RO DEHRADUN STUDENT SUPPORT MATERIAL MATHEMATICS/ XII/2023-24(VN)*
MARKING SCHEME
SAMPLE QUESTION PAPER-3
Class:-XII
Session 2023-24
Mathematics (Code-041)

Q. No. Question Marks


Alloted
Section –A(Multiple Choice Questions)
1
1 (d) 3
(4𝑎⃗+ 𝑏⃗⃗) 1
2 (b) A skew-symmetric matrix 1
[𝐻𝑖𝑛𝑡: 𝑎𝑖𝑗 = 𝑖 2 − 𝑗 2 ⇒ 𝑎𝑖𝑗 = 0 𝑎𝑛𝑑 𝑎𝑖𝑗 = 𝑖 2 − 𝑗 2 = −(𝑗 2 − 𝑖 2 ) = −𝑎𝑗𝑖 ]
3 (c ) A skew-symmetric matrix 1
3 |𝐴||𝐵|
4 hint: AB is square matrix of order 3 X 3, So |3𝐴𝐵| = 3 = 27 × 5 × 3 = 405] 1
5 1 1 1 1 1 1
(𝑎) [Hint: ∆= − sin 𝜃 cos 𝜃 = − sin 2𝜃 , 𝑛𝑜𝑤 − 1 ≤ sin 2𝜃 ≤ 1 ⇒ ≥ − sin 2𝜃 ≥ −
2 2 2 2 2
6 (c) -1 [ Hint: 3(x-0)+3=0⇒ x = -1 ] 1
𝑥
7 (b) −𝑒 𝑥 cot 2 + 𝐶 1
8 (d) 4 1
9 (d) cosecx 1
10 (a) [0, 6] 1
11 Reflexive 1
12 ⃗⃗
(𝑎)0 1
13 (b) 𝑎⃗ ⊥ 𝑏⃗⃗ 1
14 1 1 1
(𝑑)𝑘 = 𝑜𝑟 −
√3 √3
15 (c) q = -3p 1
16 (𝑑){(𝑥, 𝑦): 𝑥 2 + 𝑦 2 ≤ 1} 1
17 1 1
(𝑎)
2
18 (𝑏)𝑓 is continuous everywhere but not differentiable at 𝑥 = 𝑛𝜋, 𝑛𝜖𝑍. 1
19 (b) Both (A) and (R) are true but (R) is not the correct explanation of (A). 1
20 (a) Both (A) and (R) are true and (R) is the correct explanation of (A). 1
Section B

21 cos 𝑥 1
tan−1 ( )
1 − sin 𝑥
𝜋 𝜋 𝑥 𝜋 𝑥
sin ( 2 − 𝑥) 2 sin ( 4 − 2) cos ( 4 − 2)
= tan−1 ( 𝜋
) = tan−1
( 𝜋 𝑥
) 1
1 − cos ( 2 − 𝑥) 2 sin2 ( 4 − 2)
𝜋 𝑥 𝜋 𝜋 𝑥 𝜋 𝑥
= tan−1 (cot ( − )) = tan−1 (tan ( − ( − ))) = +
4 2 2 4 2 4 2

KVS RO DDN S SM XII MATHEMATICS/ 2023-24(VN) Page 185


*KVS RO DEHRADUN STUDENT SUPPORT MATERIAL MATHEMATICS/ XII/2023-24(VN)*

22 𝑒 𝑥 𝑑𝑥
I=∫ (𝑒 𝑥 −1)2 (𝑒 𝑥 +2)

Put 𝑒 𝑥 =t
1
𝑑𝑡
I=∫ (𝑡−1)2 (𝑡+2)

1
=1/9[log|𝑡 + 2|-log|𝑡 − 1|]-3(𝑡−1)+c

𝑒 𝑥 +2 1
1/9log|𝑒 𝑥 −1|-3(𝑒 𝑥 −1)+c
1
23 √3
A= 4 𝑥 2 ⇒
𝑑𝐴
=
√3 𝑥 𝑑𝑥
. 𝑑𝑡 =
√3
. 2.8 = 8√3𝑐𝑚 /𝑠𝑒𝑐 2 1+1
𝑑𝑡 2 2

24 𝑓 ′ (𝑥) = 3𝑥 2 − 6𝑥 + 4 = 3(𝑥 − 1)2 + 1 > 0 1+1


Or
5 5 1+1
𝑓 ′ (𝑥) = 2𝑥 − 5 = 0 ⇒ 𝑥 = : 𝑓𝑜𝑟 𝑥 ∈ (−∞, ) , 𝑓 ′ (𝑥) < 0,
2 2
5
𝑓(𝑥) 𝑖𝑓 𝑠𝑡𝑟𝑖𝑐𝑡𝑙𝑦 𝑑𝑒𝑐𝑟𝑒𝑎𝑠𝑖𝑛𝑔 𝑖𝑛 (−∞, )
2
25 1 𝑑𝑦 1 𝜋 𝑑𝑦 1 +1
𝑦 = log tan 𝑥 ⇒ = ⇒ 𝐴𝑡 𝑥 = , =1
2 𝑑𝑥 sin 2𝑥 4 𝑑𝑥
Section C
26 Letx =u andyx=v xx=w
y

du/dx=𝑦𝑥𝑦−1+(dy/dx)𝑥𝑦log𝑥 1
dv/dx 1
=𝑥𝑦𝑥−1 (dy/dx)+𝑦𝑥log 𝑦
𝑑w=𝑥𝑥(1+ log𝑥)
𝑑𝑥 1

𝑑𝑦/dx= −[𝑦𝑥𝑦−1+𝑦𝑥log𝑦+𝑥𝑥(1+log𝑥)] / 𝑥𝑦log𝑥+𝑥𝑦𝑥−1


=
27 𝑏 𝑏 1
Use property ∫𝑎 𝑓(𝑥)𝑑𝑥 = ∫𝑎 𝑓(𝑎 + 𝑏 − 𝑥)𝑑𝑥 and

simplify. 1

Ans𝜋/12 1
28 1
∫ √1 + 3𝑥 − 𝑥 2 dx = ∫ √−(−1 − 3𝑥 + 𝑥 2 ) dx
2 1
√13 3 2
= ∫ √( 2
) − (𝑥 − 2) dx
2𝑥−3 13 2𝑥−3 1
= 4
√1 + 3𝑥 − 𝑥 2 + 8
sin−1 13 +𝐶

Or
1 𝜋𝑥 1 1 𝜋𝑥
∫0 (𝑥𝑒 𝑥+ sin 4 ) 𝑑𝑥 =∫0 (𝑥𝑒 𝑥 )𝑑𝑥 + ∫0 (sin 4 ) 𝑑𝑥 ½
4 𝜋 1
=[𝑥𝑒 𝑥 − 𝑒 𝑥 ]10 − 𝜋 [cos 4 𝑥]
0 1½
4 2√2 1
=1 + 𝜋 − 𝜋

KVS RO DDN S SM XII MATHEMATICS/ 2023-24(VN) Page 186


*KVS RO DEHRADUN STUDENT SUPPORT MATERIAL MATHEMATICS/ XII/2023-24(VN)*

29 𝑦𝑑𝑥+(𝑥−𝑦2)𝑑𝑦=0 1
Reducingthegivendifferentialequationtotheform
dx/dy +Px =Q
𝒅𝒙 𝒙
weget,𝒅𝒚 + 𝒚 = y
𝑑𝑦
∫𝑦
IF = 𝑒 = 𝑒 𝑙𝑜𝑔𝑦 =y
Thegeneralsolutionisgivenby
1
𝑥.𝐼𝐹=ƒ𝑄.𝐼𝐹𝑑𝑦⟹𝑥𝑦=ƒ𝑦2𝑑𝑦
𝑦3
⟹𝑥𝑦= +𝐶,whichistherequiredgeneralsolution
3
OR
𝑥𝑑𝑦−𝑦𝑑𝑥=√𝑥2+𝑦2𝑑𝑥
ItisaHomogeneousEquationas
𝑑𝑦 √𝑥2+𝑦2+𝑦 𝑦 𝑦 𝑦
= =√1+( )2+ =𝑓().
𝑑𝑥 𝑥 𝑥 𝑥 𝑥
Put𝑦=𝑣𝑥
𝑑𝑣
𝑣+𝑥 =√1 + 𝑣2+𝑣
𝑑𝑥
Separatingvariables,weget
𝑑𝑣 𝑑𝑥
=
√1+𝑣2 𝑥
Integrating,weget𝑙𝑜𝑔|𝑣+√1+𝑣2|=𝑙𝑜𝑔|𝑥|+𝑙𝑜𝑔𝐾,𝐾>0
𝑙𝑜𝑔|𝑦+√𝑥2+𝑦2|=𝑙𝑜𝑔𝑥2𝐾

⇒𝑦+√𝑥2+𝑦2=±𝐾𝑥2
⇒𝑦+√𝑥2+𝑦2=𝐶𝑥2,whichistherequiredgeneralsolution
30 For the required graphs and shading feasible region correctly 1+
Corner points: (2, 72) , (15, 20) (40, 15) ½
Value of Z = 228 , 150, 285 1
Maximum value is 285 at (40, 15) ½
Or
For the required graphs and shading feasible region correctly 1+
½
Corner points: (0, 4), (2, 3), (4, 0) and (0, 0)
Corresponding Values of Z: 0, 6, -12, 0 1
Minimum value of Z is -12 at (4, 0) ½

31 P(E1) = 𝟔𝟎/ 𝟏𝟎𝟎 P(E2) = 𝟒𝟎 /𝟏𝟎𝟎 P(A/E1) = 𝟑𝟎/ 𝟏𝟎𝟎 P(A/E2) = 𝟐𝟎


/𝟏𝟎𝟎 1
By Bayes’ Theorem P(E1/A) = 1
60 1
(100) ∗ (30/100) 9
60 30 40 20 = 13
∗ + 100
∗ 100 100 100

Section D
32 For proving reflexive, symmetric and transitive For conclusion 1.5+
1.5+
2

KVS RO DDN S SM XII MATHEMATICS/ 2023-24(VN) Page 187


*KVS RO DEHRADUN STUDENT SUPPORT MATERIAL MATHEMATICS/ XII/2023-24(VN)*

33 The given system of equations can be written as AX=B 1

|𝐴|=-17≠0 1

−1 – 5 − 1 2
A-1=-1/17[ −8 – 6 9 ]
−10 1 7
1
-1
X= A B

X=1,Y=2,Z=3

34 For correct figure 2

4 𝑥2 1
Area=4 ∫0 3 √1 − dx
16

=12πSq.units

OR

For correct figure and intersection points 2


4
Area=∫2 3 √𝑥dx 2

=(16-4√2)Sq.units 1

35 ⃗𝑎⃗⃗12− ⃗𝑎⃗⃗⃗1⃗=ı̂− 𝑘̂ 1
⃗⃗⃗⃗⃗⃗⃗⃗⃗⃗⃗⃗𝑏⃗⃗⃗1⃗X⃗𝑏2⃗=3ı̂− 𝑗̂− 7𝑘̂ 2
1
|⃗𝑏⃗⃗1⃗X⃗𝑏2⃗|=√59 1
Shortestdistancebetweentheline:10
√59
OR
1

KVS RO DDN S SM XII MATHEMATICS/ 2023-24(VN) Page 188


*KVS RO DEHRADUN STUDENT SUPPORT MATERIAL MATHEMATICS/ XII/2023-24(VN)*

Section E
36 Let E be the event that it rains on chosen 1
day, F be the event that it does not rain on chosen
1
day and A be the event the weatherman predict rain.
6 360 2
Then we have, P(E) = 366, P(F) =366,
8 2
P(A | E) = 10and P(A | F)= 10

(i) P(A) = P(E) P(A | E) +P(F) P(A | F)


6 8 360 2 768 64
= 366 × 10 + 366 × 10 = 3660 = 305
𝐸 𝑃(𝐸) 𝑃(𝐴 | 𝐸)
(ii) 𝑃 (𝐴) = 𝑃(𝐸) 𝑃(𝐴 | 𝐸) +𝑃(𝐹) 𝑃(𝐴 | 𝐹)
6 8
× 1
366 10
= 6 8 360 2 =
× + × 16
366 10 366 10

37 (i) 𝑑⃗ = (2 − 𝜆)𝑖̂ + (2 + 2𝜆)𝑗̂ + (3 + 𝜆)𝑘̂ 1


(ii) 𝜆=8
Or
3𝑖̂ + 4𝑘̂ 2
5
(iii) For correct answer 1

38 2502
(i) 𝐶 = 5000𝑥 2 + 40000 × 𝑥4 1
𝑑𝐶 2502
(ii) = 10000𝑥 − 160000 × 5 1
𝑑𝑥 𝑥
(iii) x=10 1
Or 1
C is increasing in [10, ∞)

KVS RO DDN S SM XII MATHEMATICS/ 2023-24(VN) Page 189


*KVS RO DEHRADUN STUDENT SUPPORT MATERIAL MATHEMATICS/ XII/2023-24(VN)*

SAMPLE QUESTION PAPER-4

Class XII
Mathematics (Code-041)
Time Allowed: 3 Hours Maximum Marks: 80
General Instructions:
1. This Question paper contains - five SECTIONS A, B, C, D and E. Each section is compulsory.
However, there are internal choices in some questions.
2. Section A has 18 MCQ’s and 02 Assertion-Reason based questions of 1 mark each.
3. Section B has 5 Very Short Answer (VSA)-type questions of 2 marks each.
4. Section C has 6 Short Answer (SA)-type questions of 3 marks each.
5. Section D has 4 Long Answer (LA)-type questions of 5 marks each.
6. Section E has 3 source based/case based/passage based/integrated units of assessment (4 marks
each) with sub parts.

SECTION – A
(Multiple Choice Questions)
Each questions carries 1 mark
2 5 (𝑎𝑑𝑗𝐴)′
1. For matrix 𝐴 = [ ], is equal to
−11 7
−2 −5 7 5 7 11 7 −5
a. [ ] b. [ ]c. [ ] d. [ ]
11 −7 11 2 −5 2 11 2
2𝑥 5 6 −2
2. If | |= | | , then the value of 𝑥 is :
8 𝑥 7 3
a. 3 b.  6 c. – 6 d. – 3
1−cos 𝑘𝑥
,𝑥 ≠ 0
3. The value of k ( k < 0 ) for which the function f defined as 𝑓(𝑥) = {1𝑥 sin 𝑥
2
,𝑥 = 0
is continuous at x = 0 is
1 1
a.  1 b. – 1 c. ± 2 d. 2
1−𝑥 2 𝑑𝑦
4. If 𝑦 = log (1+𝑥2 ) , 𝑡ℎ𝑒𝑛 𝑑𝑥 is equal to
4𝑥 3 −4𝑥 1 4𝑥
a. 1−𝑥 4
b. 1−𝑥 4
c. 4−𝑥 4
d. 4 −𝑥 4
5. The sides of an equilateral triangle are increasing at the rate of 2 cm/sec . The rate at which the area
increases, when side is 10 cm is :
10
a. 10 cm2/sec b. √3 cm2/sec c. 10√3 cm2/sec d. 3
cm2/sec

𝝅
𝟒+𝟑 𝐬𝐢𝐧 𝒙
6. The value of ∫𝟎𝟐 𝐥𝐨𝐠 (𝟒+𝟑 𝐜𝐨𝐬 𝒙) 𝒅𝒙 is
a. 2 b. -1 c. 0 d. -2
7. The area enclosed by the circle x2 + y2 = 2 is equal to
a. 4sq units b. 2√2𝑠𝑞𝑢𝑛𝑖𝑡𝑠 c. 4𝜋 2 𝑠𝑞𝑢𝑛𝑖𝑡𝑠 d. 2𝑠𝑞𝑢𝑛𝑖𝑡𝑠
𝑑2 𝑦 𝑑𝑦 2 𝑑2 𝑦
8. The degree of the differential equation 𝑑𝑥 2 + 3 (𝑑𝑥 ) = 𝑥 2 log (𝑑𝑥 2 ) is
a. 1 b. 2 c. 3 d. Not defined
𝑑𝑦
9. The solution of the differential equation 2𝑥 𝑑𝑥 − 𝑦 = 3 represents a family of
a. straight lines b. circles c. parabolas d. ellipses
⃗⃗
10. The angle between two vectors 𝑎⃗𝑎𝑛𝑑𝑏 with magnitudes √3 and 4 respectively and
𝑎⃗ . 𝑏⃗⃗ = 2√3 is
𝜋 𝜋 𝜋 5𝜋
a. 6 b. 3 c. 2 d. 2

11. The interval in which the function f given by 𝑓(𝑥) = 𝑥 2 𝑒 −𝑥 is strictly increasing, is
a. ( -  ,  ) b. ( -  , 0 ) c. ( 2 ,  ) d. ( 0 , 2 )

KVS RO DDN S SM XII MATHEMATICS/ 2023-24(VN) Page 190


*KVS RO DEHRADUN STUDENT SUPPORT MATERIAL MATHEMATICS/ XII/2023-24(VN)*

12. Maximize Z = 4x + 6y, subject to 3x + 2y ≤ 12, x + y ≥ 4, x, y ≥ 0.


a. 16 at (4, 0) b. 24 at (0, 4) c. 24 at (6, 0) d. 36 at (0, 6)
13. Of all the points of the feasible region for maximum or minimum of objective
function the points
a. Inside the feasible region b. At the boundary line of the feasible region
c. Vertex point of the boundary of the feasible region d. None of these
14. The probability of obtaining an even prime number on each die, when a pair of dice
is rolled is
1 1 1
a. 0 b. 3 c. 12 d. 36
𝜋 𝜋
15. If a line makes an angle 3 and 4 with x – axis and y – axis respectively , then the angle
made by the line with z – axis is
𝜋 𝜋 𝜋 15 𝜋
a. 2 b. 3 c. 4 d. 12
𝑥−3 𝑦 𝑧
16. The shortest distance between the line = = and the y –axis is
3 0 −4
1 12
a. 5
b. 1 c. 0 d. 5
𝑥3
17. ∫ 𝑥 2 𝑒 𝑑𝑥 is equal to
1 𝑥3 1 4 1 3 1 2
a. 𝑒 + 𝐶 b. 3 𝑒 𝑥 + 𝐶 c. 𝑒𝑥 + 𝐶 d. 𝑒𝑥 + 𝐶
3 2 2
18. If A and B are symmetric matrices of the same order , then ( AB’ – BA’) is a
a. Skew symmetric matrix b. Null matrix c. Symmetric matrix d. None of these

ASSERTION-REASON BASED QUESTIONS


In the following questions, a statement of assertion (A) is followed by a statement of
Reason (R). Choose the correct answer out of the following choices.
(a) Both A and R are true and R is the correct explanation of A.
(b) Both A and R are true but R is not the correct explanation of A.
(c) A is true but R is false.
(d) A is false but R is true.
1
19. Assertion(A) :cos−1 𝑥+sin−1 𝑥 = 0 𝑡ℎ𝑒𝑛 𝑥 =
√2
𝜋
Reason ( R ) :cos −1 𝑥 + sin−1 𝑥 = 2
𝜋
20. Assertion (A):𝑓(𝑥) = 𝑙𝑜𝑔(𝑐𝑜𝑠 𝑥), 𝑥 > 0 is strictly increasing in ( 2 , 𝜋).

Reason(R) :If 𝑓 ′ (𝑥) ≥ 0 , then 𝑓(𝑥) is strictly increasing function.


SECTION – B
(This section comprises of very short answer type-questions (VSA) of 2 marks each )
3𝜋
21. Evaluate : sin−1 (sin 4 ) + cos −1(cos 𝜋) + tan−1 (1)
OR
Find the value of cot −1 (−1) + cosec −1(−√2)
2 3
22. If 𝐴 = | | , 𝑤𝑟𝑖𝑡𝑒 𝐴−1 𝑖𝑛 𝑡𝑒𝑟𝑚𝑠 𝑜𝑓 𝐴.
5 −2
𝑑2 𝑦
23. If y = x + tanx , then prove that 𝑐𝑜𝑠 2 𝑥 𝑑𝑥 2 − 2𝑦 + 2𝑥 = 0
24. The volume of a cube is increasing at a constant rate .Prove that the increase in total surface
area varies inversely as the length of the edge of the cube.
25. Find the area of the parallelogram whose diagonals are determined by the vectors are
2𝑖̂ − 𝑗̂ + 𝑘̂ 𝑎𝑛𝑑 3𝑖̂ + 4𝑗̂ − 𝑘̂.
OR
Find the direction ratios and direction cosines of a line parallel to the line whose equation
4−𝑥 𝑦 1−𝑧
is = =
2 6 3

KVS RO DDN S SM XII MATHEMATICS/ 2023-24(VN) Page 191


*KVS RO DEHRADUN STUDENT SUPPORT MATERIAL MATHEMATICS/ XII/2023-24(VN)*

SECTION – C
(This section comprises of Short answer type-questions (SA) of 3 marks each )
4
26. Evaluate:∫1 (|𝑥 − 1| + |𝑥 − 2| + |𝑥 − 4|)𝑑𝑥

OR
𝜋
𝑥 sin 𝑥 cos 𝑥
Evaluate:∫ 2
0 𝑠𝑖𝑛4 𝑥+𝑐𝑜𝑠4 𝑥
𝑑𝑥

27. Evaluate:∫ 𝑒 2𝑥 . sin 3𝑥 𝑑𝑥

28. Solve the following Linear Programming Problem graphically:


𝑀𝑎𝑥𝑖𝑚𝑖𝑠𝑒𝑍 = 400𝑥 + 350𝑦
𝑆𝑢𝑏𝑗𝑒𝑐𝑡𝑡𝑜𝑥 + 𝑦 ≤ 12; 𝑥 − 2𝑦 ≤ 0; 2𝑥 + 𝑦 ≥ 10; 𝑥 ≥ 2, 𝑦 ≥ 0
8
29. Find∫ 𝑑𝑥
(𝑥−2)(𝑥 2 +4)
30. A bag contains 4 red and 4 black balls, another bag contains 2 red and 6 black balls. One of
the two bags is selected at random and a ball is drawn from the bag which is found to be red.
Find the probability that the ball is drawn from the first bag.

OR
A bag contains two coins, one biased and the other unbiased. When tossed, the biased coin
has a 60% chance of showing heads. One of the coins is selected at random and on tossing it
shows tails. What is the probability it was an unbiased coin?
𝑑𝑦
31. Solve the differential equation:(1 + 𝑥 2 ) 𝑑𝑥 + 𝑦 = 𝑡𝑎𝑛−1 𝑥
OR
𝑦 dy 𝑦
Find the general solution of the differential equation𝑥 cos ( ) = y cos ( ) + x
𝑥 dx 𝑥

SECTION – D
(This section comprises ofLong answer type-questions (LA) of 5 marks each )
1 2 5
32. Find 𝐴−1 = [1 −1 1 . Hence solve the equations
2 3 −1
𝑥 + 2𝑦 + 5𝑧 = 10. 𝑥 − 𝑦 − 𝑧 = −2, 2𝑥 + 3𝑦 − 𝑧 = −11
𝑥−1 𝑦−1 𝑧 𝑥+1 𝑦−2 z
33Find the shortest distance between the lines = = and = =
2 3 1 5 3 2

OR
𝑥−1 𝑦−2 𝑧+3
Find the shortest distance of the point (3,2, −1) from the line = =
6 2 3
34. Find the area lying in the first quadrant and bounded by the circle 𝑥 2 + 𝑦 2 = 4 and the
lines x = 0 and x = 2
35. Let A = {x  Z : 0  x  12}. Show that R = {(a,b) : a , b  A , |𝑎 − 𝑏| is divisible by 4 }
is an equivalence relation. Find the set of all elements related to 1. Also write the equivalence
class [ 2 ].
OR
𝑥−2
Let A = 𝑅 − {3} , 𝐵 = 𝑅 − {1} . 𝐼𝑓 𝑓: 𝐴  𝐵 𝑏𝑒 𝑑𝑒𝑓𝑖𝑛𝑒𝑑 𝑏𝑦 𝑓(𝑥) = 𝑥−3 ,  𝑥  𝐴. Then,
show that f is bijective.

KVS RO DDN S SM XII MATHEMATICS/ 2023-24(VN) Page 192


*KVS RO DEHRADUN STUDENT SUPPORT MATERIAL MATHEMATICS/ XII/2023-24(VN)*

SECTION – E
This section comprises of 3case-study/passage based questions of 4 marks each with two
sub-parts. First two case study questions have three sub-parts (i), (ii), (iii) of marks 1,1,2
respectively. The third case study has two sub-parts of 2 marks each.

36. Read the following passage and answer the questions given below:

A rectangular hall is to be developed for a meeting of farmers in an agriculture college to aware them for new
technique in cultivation. It is given that the floor has a fixed perimeter P as shown.

i. If x and y represents the length and breadth of the rectangular region , then find the relation between the
variables to represent its perimeter ‘ P ‘.
ii. Express the area of the rectangular region ‘ A ‘ as a function of x .
iii. Principal of agriculture college is interested in maximizing the area of floor ‘ A ‘ . For this to
happen, What should be the value of x ?
OR
What will be the maximum value of area of floor ‘ A ‘ ?

37. Read the following passage and answer the questions given below:

The equation of motion of missile are x= 3t , y = - 4t , z = t , where the time ‘ t ‘ is given in seconds and the
distance is measured in kilometers.
i. What is the path of the missile?
ii. At what distance will the missile be from the starting point ( 0 , 0 , 0 )in 5 seconds?
iii. If the position of missile at a certain instant of time is ( 5 , -8 , 10 ), then what will be the height of
the missile from the ground? (The ground is considered as the xy- plane).

OR
𝑥−1 𝑦−1 𝑧−1 𝑥−2 𝑦−3 𝑧−5
For what value of k are the lines = = 𝑎𝑛𝑑 = = perpendicular?
2 3 𝑘 −2 −1 7

KVS RO DDN S SM XII MATHEMATICS/ 2023-24(VN) Page 193


*KVS RO DEHRADUN STUDENT SUPPORT MATERIAL MATHEMATICS/ XII/2023-24(VN)*
.

38. Mahindra Tractors is India’s leading farm equipment manufacturer. It is the largest tractor selling

factory in the world.


This factory has two machines A and B. Past record shows that machine A produced 60% and
machine B produced 40% of the output ( tractors). Further 2% of the tractors produced by machine A
and 1% produced by machine B were defective . All the tractors are put into one big store hall and
one tractor is chosen at random.
On the above information answer the following questions :

i. Find the total probability of chosen tractor (at random) is defective.


ii. If in random choosing , chosen tractor is defective , then find the probability that the chosen tractor is
produced by machine ‘A’.

******************

KVS RO DDN S SM XII MATHEMATICS/ 2023-24(VN) Page 194


*KVS RO DEHRADUN STUDENT SUPPORT MATERIAL MATHEMATICS/ XII/2023-24(VN)*

MARKING SCHEME

Sample Paper-4
Q.No. Solution Marks
SECTION - A
1. 7 11 1
c. [ ]
−5 2
2. b.  6 1
3. b. – 1 1
4. −4𝑥 1
b. 1−𝑥 4
5. c. 10√3 cm2/sec 1
6. c. 0 1
7. d. 2𝑠𝑞𝑢𝑛𝑖𝑡𝑠 1
8. d. Not defined 1
9. c. parabolas 1
𝜋 1
10. b. 3
11. d. ( 0 , 2 ) 1
12. d. 36 at (0, 6) 1
13. c. Vertex point of the boundary of the feasible region 1
14. 1 1
d. 36

𝜋 1
15. b. 3
16. 12 1
d. 5
17. 1 𝑥3 1
a. 𝑒 + 𝐶
3
18. a. Skew symmetric matrix 1
19. a. Both A and R are true and R is the correct explanation of A. 1

20. c. A is true but R is false. 1

SECTION - B
21. 3𝜋
sin−1 (sin ) + cos −1 (cos 𝜋) + tan−1 (1)
4
𝜋 𝜋 1
= sin (sin (𝜋 − )) + cos −1 (cos 𝜋) + tan−1 (tan )
−1
4 4
𝜋 𝜋
= sin−1 (sin ( )) + cos−1(cos 𝜋) + tan−1 (tan )
4 4
𝜋 𝜋
= +𝜋+
4 4
3𝜋 1
=
2
OR

𝜋 𝜋 1
cot −1 (−1) + cosec −1(−√2) = (𝜋 − ) + (− ) 1
𝜋
4 4
=2

KVS RO DDN S SM XII MATHEMATICS/ 2023-24(VN) Page 195


*KVS RO DEHRADUN STUDENT SUPPORT MATERIAL MATHEMATICS/ XII/2023-24(VN)*

|𝐴| = |2
22. 3 1
| = −19 ≠ 0
5 −2 ½
𝐶11 = −2 , 𝐶12 = −5 , 𝐶21 = −3 , 𝐶22 = 2
−2 −3
𝑎𝑑𝑗 𝐴 = | |
−5 2
1 2 3 1 ½
𝐴−1 = [ ]= 𝐴
19 5 −2 19

23. 𝑑𝑦
= 1 + 𝑠𝑒𝑐 2 𝑥
𝑑𝑥
𝑑2 𝑦
=2𝑠𝑒𝑐 2 𝑥𝑡𝑎𝑛𝑥 1
𝑑𝑥 2
2 𝑑2 𝑦
𝑐𝑜𝑠 𝑥 𝑑𝑥 2 =2𝑡𝑎𝑛𝑥 = 2(𝑦 − 𝑥)= 2y-2x

𝑑2 𝑦
𝑐𝑜𝑠 2 𝑥 𝑑𝑥 2 - 2y+2x=0 1

24. Let V be the volume, S be the total surface area and a be the edge of the cube .
𝑑𝑉 1/2
=𝑘
𝑑𝑡
V=𝑎3 S= 6𝑎2
𝑑𝑆 𝑑𝑎 1/2
= 12a 𝑑𝑡
𝑑𝑡
𝑑𝑎 𝑘
= 1/2
𝑑𝑡 3𝑎2

𝑑𝑆 4𝑘 1/2
=
𝑑𝑡 𝑎

25. ⃗⃗⃗⃗⃗⃗⃗⃗⃗⃗⃗⃗⃗⃗⃗⃗⃗⃗⃗⃗⃗⃗⃗⃗⃗⃗⃗⃗⃗⃗⃗⃗⃗
1 ½
𝐴⃗ = |𝑑 ⃗⃗⃗⃗⃗ × ⃗⃗⃗⃗⃗
𝑑2 |
2 1 ½
⃗⃗⃗⃗⃗
𝑑1 × ⃗⃗⃗⃗⃗ 𝑑2 = −3𝑖̂ + 5𝑗̂+11𝑘̂
1
Getting ⃗⃗⃗⃗⃗⃗ |𝐴|= √155 1
2

OR
𝑥−4 𝑦−0 𝑧−1 1/2
= =
−2 6 −3
1/2
Dr’s are - 2, 6,-3

−2 6 −3 1
𝐷𝑖𝑟𝑒𝑐𝑡𝑖𝑜𝑛 𝐶𝑜𝑠𝑖𝑛𝑒𝑠 𝑎𝑟𝑒 , ,
7 7 7

SECTION – C

26. 4
∫ (|𝑥 − 1| + |𝑥 − 2| + |𝑥 − 4|)𝑑𝑥
1
2 4
= ∫1 {(𝑥 − 1) − (𝑥 − 2) − (𝑥 − 4)}𝑑𝑥 + ∫2 {(𝑥 − 1) + (𝑥 − 2) − (𝑥 − 4)}𝑑𝑥 1

2 4 1
2 4 𝑥2 𝑥2
= ∫1 (5 − 𝑥)𝑑𝑥 + ∫2 (1 + 𝑥)𝑑𝑥 = [5𝑥 − ] + [𝑥 + ]
2 1 2 2

1
= (10 − 2) − (5 − ) + (4 + 8) − (2 + 2)
2 1
9 9 𝟐𝟑
= 8 − 2 + 12 − 4 = 16 − 2 = 𝟐

KVS RO DDN S SM XII MATHEMATICS/ 2023-24(VN) Page 196


*KVS RO DEHRADUN STUDENT SUPPORT MATERIAL MATHEMATICS/ XII/2023-24(VN)*

𝑶𝑹
𝜋
2 𝑥 sin 𝑥 cos 𝑥 𝑑𝑥
𝐼=∫
0 𝑠𝑖𝑛4 𝑥 + 𝑐𝑜𝑠 4 𝑥

𝜋 𝜋 𝜋 𝜋
2 ( 2 − 𝑥) sin ( 2 − 𝑥) cos ( 2 − 𝑥) 𝑑𝑥
𝐼=∫ 𝜋 𝜋
0 𝑠𝑖𝑛4 ( − 𝑥) + 𝑐𝑜𝑠 4 ( − 𝑥)
2 2
𝜋 (𝜋−𝑥) cos 𝑥 sin 𝑥𝑑𝑥 1
2
𝐼 = ∫02 𝑐𝑜𝑠4 𝑥+𝑠𝑖𝑛4 𝑥

𝜋
𝜋 2 sin 𝑥 cos 𝑥 𝑑𝑥
2𝐼 = ∫
2 0 𝑠𝑖𝑛4 𝑥 + 𝑐𝑜𝑠 4 𝑥
𝜋
𝜋 2 tan 𝑥 sec 2 𝑥 𝑑𝑥
= ∫ 1
2 0 𝑡𝑎𝑛4 𝑥 + 1
2
𝑡 = 𝑡𝑎𝑛 𝑥
𝜋 ∞ 𝑑𝑡
= 4 ∫0 𝑡 2 +1 [ 𝑑𝑡 = 2 tan 𝑥 𝑠𝑒𝑐 2 𝑥 𝑑𝑥 ]
𝜋
𝑥 =0⟶𝑡 =0&𝑥 = 2 ⟶𝑡 =∞
𝜋 −1 ∞
𝜋 𝜋 𝜋2 1
2𝐼 = [tan 𝑡]0 = ( − 0) =
4 4 2 8
𝝅 𝟐
𝑰 = 𝟏𝟔

27. 𝑑(𝑒 2𝑥 )
𝐼 = ∫ 𝒆𝟐𝒙 . sin 3𝑥 𝑑𝑥 = 𝑒 2𝑥 ∫ sin 3𝑥 𝑑𝑥 − ∫( ∫ sin 3𝑥 𝑑𝑥)𝑑𝑥
𝑑𝑥

𝑒 2𝑥 cos 3𝑥 2 1
=− 3
+ 3 ∫ 𝒆𝟐𝒙 . cos 3𝑥 𝑑𝑥
2𝑥
𝑒 cos 3𝑥 2 𝑒 2𝑥 sin 3𝑥 2
𝐼=− + { − ∫ 𝒆𝟐𝒙 . sin 3𝑥 𝑑𝑥}
3 3 3 3

−1 2𝑥 2 4
= 𝑒 cos 3𝑥 + 𝑒 2𝑥 . sin 3𝑥 − 𝐼
3 9 9 1
13 𝑒 2𝑥
𝐼= (−3 cos 3𝑥 + 2 sin 3𝑥)
9 9
𝒆𝟐𝒙 1
𝑰 = 𝟏𝟑 (𝟐 𝐬𝐢𝐧 𝟑𝒙 − 𝟑 𝐜𝐨𝐬 𝟑𝒙)

28.

1
1
2

Feasible Region & Corner Points


Corner P(4,2) Q(2,6) R(2,10) S(8,4)
Points
1
Z 2300 2900 4300 4600
=400x+350y
1/2
Zmax = 4600 at x = 8 & y = 4

KVS RO DDN S SM XII MATHEMATICS/ 2023-24(VN) Page 197


*KVS RO DEHRADUN STUDENT SUPPORT MATERIAL MATHEMATICS/ XII/2023-24(VN)*

29. 8 𝐴 𝐵𝑥+𝐶 ½
Let (𝑥−2)(𝑥 2 +4) = 𝑥−2 + 𝑥 2 +4

On Solving 𝐴 = 1, 𝐵 = −1, 𝐶 = −2 1

8 𝑑𝑥 𝑑𝑥 𝑥 2 ½
∫ (𝑥−2)(𝑥 2 +4) = ∫ 𝑥−2 − ∫ 𝑥 2 +4 − ∫ 𝑥 2 +4

1 𝑥
= log |𝑥 − 2| − 2 log|𝑥 2 + 4| − tan−1 2 + 𝐶 1

30.
Let
𝐸1 =Bag I is selected
𝐸2 = Bag II is selected
𝐴 = 𝐴 𝑐𝑜𝑖𝑛 𝑠ℎ𝑜𝑤𝑠 𝑎 𝑡𝑎𝑖𝑙 1
𝑃(𝐸1 )𝑃(𝐴/𝐸1 )
𝑃(𝐸1 /𝐴) = 1
𝑃(𝐸1 )𝑃(𝐴/𝐸1 ) + 𝑃(𝐸2 )𝑃(𝐴/𝐸2 )
1 4
( )
𝑃(𝐸1 /𝐴) = 1 4 2 8 1 2
( ) + 2 (8 ) ½
2 8
2
𝑃(𝐸1 /𝐴) = ½
3
OR
Let
𝐸1 =Bag I is selected(biased coin)
𝐸2 = Bag II is selected(unbiased coin)
𝐴 = 𝐴 𝑟𝑒𝑑 𝑏𝑎𝑙𝑙 𝑖𝑠 𝑑𝑟𝑎𝑤𝑛
𝑃(𝐸2 )𝑃(𝐴/𝐸1 ) 1
𝑃(𝐸2 /𝐴) =
𝑃(𝐸1 )𝑃(𝐴/𝐸1 ) + 𝑃(𝐸2 )𝑃(𝐴/𝐸2 )
1 1 1
( )
𝑃(𝐸1 /𝐴) = 1 402 2 1 1
( ) + 2 (2)
2 100 ½
5
𝑃(𝐸1 /𝐴) =
9 ½
31. 𝑑𝑦 ½
(1 + 𝑥 2 ) + 𝑦 = 𝑡𝑎𝑛−1 𝑥
𝑑𝑥
𝑑𝑦 𝑦 𝑡𝑎𝑛−1 𝑥 ½
+ =
𝑑𝑥 (1 + 𝑥 2 ) (1 + 𝑥 2 )
½
The above differential equation is a linear differential equation
1
Here p= (1+𝑥 2) ½
−1 𝑥
𝐼𝐹 = 𝑒 ∫ 𝑝𝑑𝑥 = 𝑒 𝑡𝑎𝑛 ½
−1 𝑥 𝑡𝑎𝑛−1 𝑥 −1 𝑥
y 𝑒 𝑡𝑎𝑛 = ∫ (1+𝑥 2) 𝑒 𝑡𝑎𝑛 𝑑𝑥
1
let 𝑡𝑎𝑛−1 𝑥 = 𝑡 ⇒ (1+𝑥 2) 𝑑𝑥 = 𝑑𝑡
−1 𝑥 ½
y 𝑒 𝑡𝑎𝑛 = ∫ 𝑡 𝑒 𝑡 𝑑𝑡 = 𝑒 𝑡 (𝑡 − 1)

KVS RO DDN S SM XII MATHEMATICS/ 2023-24(VN) Page 198


*KVS RO DEHRADUN STUDENT SUPPORT MATERIAL MATHEMATICS/ XII/2023-24(VN)*

−1 −1
y 𝑒 𝑡𝑎𝑛 𝑥 = 𝑒 𝑡𝑎𝑛 𝑥 (𝑡𝑎𝑛−1 𝑥 − 1)+C
𝐶
y = (𝑡𝑎𝑛−1 𝑥 − 1)+ 𝑡𝑎𝑛−1𝑥
𝑒
OR
𝑦 dy 𝑦
𝑥 cos ( ) = y cos ( ) + x
𝑥 dx 𝑥
𝑦
dy y cos( ) 𝑥 𝑦 𝑦
𝑥
= 𝑦 + 𝑦 = 𝑥 + sec 𝑥
dx 𝑥 cos( ) 𝑥 cos( )
𝑥 𝑥 ½
𝑑𝑦 𝑑𝑣
Substitute, 𝑦 = 𝑣𝑥 ⟶ 𝑑𝑥 = 𝑣 + 𝑥 𝑑𝑥

𝑑𝑣 𝑑𝑥 1
𝑣+𝑥 = 𝑣 + sec 𝑣 ⟹ ∫ cos 𝑣 𝑑𝑣 = ∫
𝑑𝑥 𝑥 1
sin 𝑣 = log|𝑥| + 𝐶 1/2
𝒚
𝐬𝐢𝐧 (𝒙) = 𝐥𝐨𝐠|𝒙| + 𝑪

SECTION - D
32. 1 2 5 𝑥 10 1
[1 −1 −1 [𝑦 = [−2
2 3 −1 𝑧 −11
½
|𝐴| = 27

4 17 3
adj(𝐴) = [−1 −11 6
5 1 −3
1
1
A−1 = adj 𝐴
|A|
½

1 4 17 3
= [−1 −11 6
27
5 1 −3 ½
𝑋 = 𝐴−1 𝐵 ½
𝑥 1 4 17 3 10
[𝑦 = [−1 −11 6 [−2 ½
27
𝑧 5 1 −3 −11

𝑥 −1
[𝑦 = [−2 ½
𝑧 3

KVS RO DDN S SM XII MATHEMATICS/ 2023-24(VN) Page 199


*KVS RO DEHRADUN STUDENT SUPPORT MATERIAL MATHEMATICS/ XII/2023-24(VN)*

33. 𝑥−1 𝑦−1 𝑧−0 𝑥+1 𝑦−2 𝑧−0 1


= = 𝑎𝑛𝑑 = =
2 3 1 5 3 2

𝑏 ⃗⃗⃗⃗2 = 5𝑖ˆ + 𝑗ˆ − 13𝑘ˆ


⃗⃗⃗⃗1 × 𝑏 1

𝑎2 − 𝑎
⃗⃗⃗⃗⃗ ˆ + 𝑗ˆ
⃗⃗⃗⃗⃗1 = −2𝑖 1

|(𝑎
⃗⃗⃗⃗⃗2 − 𝑎
⃗⃗⃗⃗⃗) ⃗⃗⃗⃗ ⃗⃗⃗⃗
1 × (𝑏1 × 𝑏2 )|
𝑆𝐷 = 1
⃗⃗⃗⃗⃗⃗1 × 𝑏
|𝑏 ⃗⃗⃗⃗2 |

For substituting values


9 1
For correct answer =
√195
OR
Shortest distance between a point and a line = Shortest distance between given line
and line parallel to it, passing through the given point.
|(𝑎
⃗⃗⃗⃗⃗2 − ⃗⃗⃗⃗⃗)
𝑎1 × (𝑏 ⃗⃗⃗⃗1 )|
𝑆ℎ𝑜𝑟𝑡𝑒𝑠𝑡 𝑑𝑖𝑠𝑡𝑎𝑛𝑐𝑒 𝑏𝑒𝑡𝑤𝑒𝑒𝑛 𝑎 𝑝𝑜𝑖𝑛𝑡 𝑎𝑛𝑑 𝑎 𝑙𝑖𝑛𝑒 = | |
|𝑏⃗⃗⃗⃗1 |
𝑎2 = 3𝑖̂ + 2𝑗̂ − 𝑘̂, ⃗⃗⃗⃗⃗⃗⃗
⃗⃗⃗⃗⃗ 𝑎1 = 𝑖̂ + 2𝑗̂ − 3𝑘̂ 𝑎𝑛𝑑 ⃗⃗⃗⃗⃗⃗⃗
𝑏1 = 6𝑖̂ + 2𝑗̂ + 3𝑘̂ 1

𝑎2 − ⃗⃗⃗⃗⃗
⃗⃗⃗⃗⃗ 𝑎1 = (3𝑖̂ + 2𝑗̂ − 𝑘̂) − (𝑖̂ + 2𝑗̂ − 3𝑘̂) = 2𝑖̂ + 2𝑘̂
𝑖̂ 𝑗̂ 𝑘̂
(𝑎 ⃗⃗⃗⃗⃗2 − ⃗⃗⃗⃗⃗)
𝑎1 × (𝑏 ⃗⃗⃗⃗1 ) = |2 0 2| = −4𝑖̂ + 6𝑗̂ + 4𝑘̂ 1
6 2 3
|(𝑎 ⃗⃗⃗⃗⃗2 − ⃗⃗⃗⃗⃗)
𝑎1 × (𝑏 ⃗⃗⃗⃗1 )| = √16 + 36 + 16 = √68 = 2√17 𝑎𝑛𝑑
|𝑏⃗⃗⃗⃗1 | = √36 + 4 + 9 = √49 = 7 1
|(𝑎
⃗⃗⃗⃗⃗2 − ⃗⃗⃗⃗⃗)
𝑎1 × (𝑏 ⃗⃗⃗⃗1 )|
𝑆ℎ𝑜𝑟𝑡𝑒𝑠𝑡 𝑑𝑖𝑠𝑡𝑎𝑛𝑐𝑒 𝑏𝑒𝑡𝑤𝑒𝑒𝑛 𝑎 𝑝𝑜𝑖𝑛𝑡 𝑎𝑛𝑑 𝑎 𝑙𝑖𝑛𝑒 = | |
|𝑏⃗⃗⃗⃗1 | 1
2√17
=
7
1
34. Drawing correct figure 1
Finding limits for integration 1
2
𝐴 = ∫ √4 − 𝑥 2 𝑑𝑥 1
0
𝑥 4 𝑥 1
A=[2 √4 − 𝑥 2 + 𝑠𝑖𝑛−1 2] 20
2
1
𝐴 = 𝜋 𝑠𝑞 𝑢𝑛𝑖𝑡𝑠

KVS RO DDN S SM XII MATHEMATICS/ 2023-24(VN) Page 200


*KVS RO DEHRADUN STUDENT SUPPORT MATERIAL MATHEMATICS/ XII/2023-24(VN)*

35. Reflexive : |𝑎 − 𝑎| = 0 , which is divisible by 4


 (a , a)  R  a A
 R is Reflexive. ½
Symmetric : Let ( a , b )  R
|𝑎 − 𝑏| is divisible by 4
|𝑏 − 𝑎| is divisible by 4
(b,a)R 1
 R is Symmetric.
Trasnsitive : Let ( a , b ) , ( b , c )  R
|𝑎 − 𝑏| and |𝑏 − 𝑐| are divisible by 4 1
 a – b =  4m , b – c =  4n , where m , n  Z
On adding , We get a – c =  4 ( m + n )
 ( a – c ) is divisible by 4
|𝑎 − 𝑐| is divisible by 4
(a,c)R 1
½
 R is Transitive . ½
Hence , R is an equivalence relation in A. ½
Set of elements related to 1 is { 1 , 5 , 9 }
Equivalence class [ 2 ] = { 2 , 6 , 10 }.

OR
For one – one 2
Let f(x1) = f(x2)
x1 = x2
So, f(x) is onto function
For onto 2
𝑥−2
Let 𝑦 = 𝑥−3
3𝑦−2
𝑥 = 𝑦−1
 𝐴 ,  𝑦  𝐵 [ 𝑐𝑜𝑑𝑜𝑚𝑎𝑖𝑛 ] 1
So, f(x) is onto function.
Hence, f(x) is one – one and onto function
 f(x) is bijective.

SECTION – E
36. i.. Perimeter of rectangle : P = 2( x + y ) 1
ii. 𝐴 = 𝑥𝑦
𝑃 − 2𝑥
𝐴 = 𝑥. ( )
2
2
𝑃𝑥 − 2𝑥
𝐴=
2

KVS RO DDN S SM XII MATHEMATICS/ 2023-24(VN) Page 201


*KVS RO DEHRADUN STUDENT SUPPORT MATERIAL MATHEMATICS/ XII/2023-24(VN)*

𝑃𝑥−2𝑥 2 1
iii. 𝐴 =
2
𝑑𝐴 𝑃−4𝑥
 =
𝑑𝑥 2
For maximum or minimum value of x 1
𝑑𝐴 𝑃 − 4𝑥
= 0  =0
𝑑𝑥 2
𝑃
x=4
1
𝑑2 𝐴 𝑃
Also, ( 𝑎𝑡 x = ) ] = −2
𝑑𝑥 2 4
OR
𝑃 𝑃−2𝑥
Putting x = 4 in 𝑦 = 2
𝑃
y=4 1
 A = xy
𝑃 𝑃
𝐴 = 4 . 4
𝑃2
 A = 16 1

37. I. Straight Line 1


ii. After 5 sec. position of the missile be x=3x5=15 , y = -4x5 = -20 , z= 5
Point is ( 15 , -20,5)
Its distance from origin ( 0,0,0) = √650 km 1
iii. Given position of the missile at a time is (5,-8,10)
Height of the missile from the ground 1
= Distance between the points ( 5,-8,10) and ( 5,-8,0)
= 10 km 1

OR

Given lines are perpendicular if 2 x-2 + 3 x-1+7 x k = 0 1


k=1 1

38. Let E1 = Production of tractor by machine A


E2 = Production of tractor by machine B
A = Choosing of defective tractors
P(E1) = 60/100 = 3/5
P(E2)= 40/100 = 2/5
P (A/E1) = 2/100 = 1/50
P(A/E2) = 1/100
i. Required Total Probability P(A) = P(E1). P (A/E1)+ P(E2). P(A/E2)= 2/125 2
ii. By Baye’s Theorem
P(E1/A)= 3/4 2

*****************************************************

KVS RO DDN S SM XII MATHEMATICS/ 2023-24(VN) Page 202

You might also like